Clinical Integration Exam 9 Combined

Ace your homework & exams now with Quizwiz!

What cytokines do patients with Ulcerative colitis demonstrate greater expression of?

IL-5 and IL-13 are characteristically associated with a TH2 response

What do Th1 cells produce?

INF-gamma and IL-2 these cytokines influence the development of cell-mediated immune responses of both CD8 T C cells and other CD4 T Helper cells

What is the goal of surveillance biopsies?

Identify dysplastic epithelium which is a precursor to colitisassociated carcinoma

What is Negative selection?

If a lymphocyte that has not completed its maturation interacts strongly with a self antigen, displayed as a peptide bound to a self major histocompatibility complex (MHC) molecule, that lymphocyte receives signals that trigger apoptosis Thus, the self-reactive cell dies before it can become functionally competent

Fecal fat analysis

If fat malabsorption is suspected, the simplest qualitative method for detecting fat in stool is microscopic examination with Sudan staining of a drop of stool. Sensitivity is limited, but the test is quick and easy, and it correlates well with the quantitative measurement of fecal fat when moderate to severe steatorrhea is present. To quantify fat, stool is collected for three consecutive days while the patient is on a diet containing 100 g of fat per day and the specimen is analyzed for fat content. Normal fat excretion should not exceed 6 g/day. Although this test is cumbersome and nonspecific, it offers an accurate quantification of fecal fat excretion provided fat consumption is appropriate

How does Volvulus or twisting of the bowel occur?

If the duodenojejunal flexure or the cecum does not end up in its usual site, the origin of the small bowel mesentery shortens, which permits twisting of the small bowel around the axis of the superior mesenteric artery

What do both Basophils (circulating) and mast cells (tissue-resident) bind? What happens after they bind?

IgE antibody display this IgE on the cell surface, and maintain histamine-containing granules that are released when exogenous antigen binds to and cross-links the IgE Basophils and mast cells are important in allergic responses

What is the final branch arising from the right side of the SMA?

Ileocolic artery passes downward and to the right toward the right iliac fossa where it divides into superior and inferior branches The superior branch passes upward along the ascending colon to anastomose with the right colic artery.

What does the absence of Bowel sounds suggest?

Ileus

Why is Mycophenolate Mofetil (MMF) increasingly used in the treatment of ?

Immune-mediated diseases because of its high selectivity and profound effects on lymphocytes

Immunological tolerance: significance and mechanisms

Immunological tolerance is a lack of response to antigens that is induced by exposure of lymphocytes to these antigens. When lymphocytes with receptors for a particular antigen encounter this antigen, any of several outcomes is possible. The lymphocytes may be activated to proliferate and to differentiate into effector and memory cells, leading to a productive immune response; antigens that elicit such a response are said to be immunogenic. The lymphocytes may be functionally inactivated or killed, resulting in tolerance; antigens that induce tolerance are said to be tolerogenic. In some situations, the antigenspecific lymphocytes may not react in any way; this phenomenon has been called immunological ignorance, implying that the lymphocytes simply ignore the presence of the antigen. Normally, microbes are immunogenic and self antigens are tolerogenic. The choice between lymphocyte activation and tolerance is determined largely by the nature of the antigen and the additional signals present when the antigen is displayed to the immune system. In fact, the same antigen may be administered in different ways to induce an immune response or tolerance. This experimental observation has been exploited to analyze what factors determine whether activation or tolerance develops as a consequence of encounter with an antigen. The phenomenon of immunological tolerance is important for several reasons. First, as we stated at the outset, self antigens normally induce tolerance, and failure of self-tolerance is the underlying cause of autoimmune diseases. Second, if we learn how to induce tolerance in lymphocytes specific for a particular antigen, we may be able to use this knowledge to prevent or control unwanted immune reactions. Strategies for inducing tolerance are being tested to treat allergic and autoimmune diseases and to prevent the rejection of organ transplants. The same strategies may be valuable in gene therapy to prevent immune responses against the products of newly expressed genes or vectors and even for stem cell transplantation if the stem cell donor is genetically different from the recipient. Immunological tolerance to different self antigens may be induced when developing lymphocytes encounter these antigens in the generative (central) lymphoid organs, a process called central tolerance, or when mature lymphocytes encounter self antigens in peripheral (secondary) lymphoid organs or peripheral tissues, called peripheral tolerance (Fig. 9-1). Central tolerance is a mechanism of tolerance only to self antigens that are present in the generative lymphoid organs—namely, the bone marrow and thymus. Tolerance to self antigens that are not present in these organs must be induced and maintained by peripheral mechanisms. We have only limited knowledge of how many and which self antigens induce central or peripheral tolerance or are ignored by the immune system. With this brief background, we proceed to a discussion of the mechanisms of immunological tolerance and how the failure of each mechanism may result in autoimmunity. Tolerance in T cells, particularly CD4+ helper T lymphocytes, is discussed first because many of the mechanisms of self-tolerance were defined by studies of these cells. In addition, CD4+ helper T cells orchestrate virtually all immune responses to protein antigens, so tolerance in these cells may be enough to prevent both cell-mediated and humoral immune responses against self proteins. Conversely, failure of tolerance in helper T cells may result in autoimmunity manifested by T cell-mediated attack against self antigens or by the production of autoantibodies against self proteins.

Summary

Immunological tolerance is specific unresponsiveness to an antigen induced by exposure of lymphocytes to that antigen. All individuals are tolerant of (unresponsive to) their own (self) antigens. Tolerance against antigens may be induced by administering that antigen in particular ways, and this strategy may be useful for treating immunologic diseases and for preventing the rejection of transplants. Central tolerance is induced in immature lymphocytes that encounter antigens in the generative lymphoid organs. Peripheral tolerance results from the recognition of antigens by mature lymphocytes in peripheral tissue Central tolerance of T cells is the result of high-affinity recognition of antigens in the thymus. Some of these self-reactive T cells die (negative selection), thus eliminating the potentially most dangerous T cells, which express high-affinity receptors for self antigens. Other T cells of the CD4 lineage develop into regulatory T cells that suppress self reactivity in the periphery Peripheral tolerance in T cells is induced by multiple mechanisms. Anergy (functional inactivation) results from the recognition of antigens without costimulators (second signals). The mechanisms of anergy include a block in TCR signaling and engagement of inhibitory receptors such as CTLA-4 and PD-1. Self- reactive regulatory T cells suppress potentially pathogenic T cells. Deletion (death by apoptosis) may occur when T cells encounter self antigens. In B lymphocytes, central tolerance occurs when immature cells recognize self antigens in the bone marrow. Some of the cells change their receptors (receptor editing), and others die by apoptosis (negative selection, or deletion). Peripheral tolerance is induced when mature B cells recognize self antigens without T cell help, which results in anergy and death of the B cells, or engagement of inhibitory receptors. Autoimmune diseases result from a failure of self-tolerance. Multiple factors contribute to autoimmunity, including the inheritance of susceptibility genes and environmental triggers such as infections Many genes contribute to the development of autoimmunity. The strongest associations are between HLA genes and various T cell-dependent autoimmune diseases. Infections predispose to autoimmunity by causing inflammation and stimulating the expression of costimulators or because of crossreactions between microbial and self antigens

What agents are the mainstay of treatment for IBD?

Immunosuppresive agents

What does the slow release of 6-MP from AZA favor?

Immunosuppression Although AZA prolongs organ graft survival, this drug is less efficacious than mycophenolate mofetil in improving the long-term survival of kidney allografts

What is a way of preventing acute cellular rejection?

Immunosuppression of T cells is is highly effective at preventing or limiting activation of the recipient immune system by the transplanted organ, thereby preventing acute cellular rejection

What do patients with autoimmune diseases and patients who received transplanted tissues or organs require therapy with?

Immunosuppressive drugs

What are the resultant defects in all phases of absorption?

Impaired gastric churning, premature emptying, and impaired mixing (in the jejunum) of food with bile and pancreatic enzymes The impaired mixing is a consequence of anatomic changes (gastrojejunostomy bypassing the duodenum) and reduced production of pancreatic enzymes (because cholecystokinin and secretin release is blunted when gastric contents bypass the duodenum) stasis may lead to bacterial overgrowth in the afferent loop with changes in the bile acids needed for fat absorption Another example of manifold mechanisms is diabetes mellitus, which may lead to delayed gastric emptying, abnormal intestinal motility, bacterial overgrowth, and pancreatic exocrine insufficiency.

Identification of the bleeding source

In 80% to 90% of cases, acute GI hemorrhage resolves spontaneously without recurrence. Nevertheless, it is prudent to localize the bleeding source, especially in those with significant bleeding or comorbidities. Proper identification allows for direct treatment if the bleeding does not spontaneously resolve and for recognition of those patient who are at risk for further bleeding. For example, in a patient with a bleeding gastric or duodenal ulcer, acid suppression with an intravenous proton pump inhibitor may maximize clot stability and enhance platelet aggregation. Proton pump inhibitors, in combination with appropriate endoscopic management, decrease the risks for ulcer rebleeding, need for urgent surgery, and death. Direct visualization of the bleeding site by endoscopy can alter patient management. Classification systems, such as the Forrest Ulcer Description or the Rockall Scoring System, rely heavily on endoscopic criteria for rebleeding risk stratification. Various stigmata of hemorrhage may be identified within the ulcer crater. Stigmata that carry a high risk for rebleeding include active bleeding (Forrest 1) and the visible presence of a pigmented protuberance (artery) within the ulcer crater (Forrest 2). The patient with a "clean base" ulcer exhibiting no such stigmata has an excellent prognosis for cessation of bleeding. Patients found to have high-risk stigmata are likely (~50%) to have continued or recurrent bleeding. In such patients, the site of bleeding may be treated by injection therapy with vasoconstrictors or saline, thermal therapy by electrocautery, or mechanical therapy by placement of endoscopic clips. These endoscopic therapies decrease rates of rebleeding, mortality, need for transfusion, need for surgery, and length of hospital stay. Thermal or mechanical therapy, applied alone or in combination with injection therapy, is more effective than injection therapy alone. Recently developed alternatives, such as hemostatic sprays and cyanoacrylate compounds, offer additional modalities that can be used in such cases. An overall approach to the patient with acute GI bleeding is outlined in Figure 33-2. Historical points and objective findings often enable localization of the bleeding site to the upper GI tract (proximal to the ligament of Treitz) or to the lower GI tract (distal to that point). For the patient with melena or hematemesis, the upper GI tract should be examined first. Patients with hematochezia more commonly have lower GI bleeding, but when the pace of bleeding is brisk, an upper GI tract lesion may manifest with hematochezia. Placement of a nasogastric tube with aspiration of contents is a reasonable first step. The absence of blood does not by itself rule out the presence of an upper GI source because blood from a duodenal bulb ulcer may not flow back into the stomach to allow sampling by the nasogastric tube. In general, in patients with acute GI hemorrhage who have significant blood loss, an upper endoscopy should be the initial step in the evaluation. Once the lower GI tract has been identified as the source of bleeding, sigmoidoscopy or colonoscopy is the test of choice. In cases of lower GI bleeding in which the pace of bleeding is so brisk as to preclude endoscopic visualization of the colon and rectum, scintigraphic erythrocyte scans using technetium-99m (99mTc)-labeled sulfur colloid or pertechnetate can localize the bleeding site if the rate of blood loss exceeds 0.5 mL/minute. Although the bleeding site identified by scintigraphic examination may not be accurate, it will direct the visceral arteriographic search while minimizing the amount of dye used. The recent description of capsule endoscopy followed by directed push or balloon enteroscopy has provided a possible endoscopic means of delineating and controlling bleeding lesions in the small bowel. There is no role for barium studies in the evaluation of acute GI hemorrhage.

What form is MPA usually administered in?

In a Sodium salt or in its prodrug form called- Mycophenolate Mofetil (MMF) both have a higher oral bioavailability than MPA

imaging studies

In patients with malabsorption, barium studies of the small bowel are usually nonspecific. Occasionally, however, distinct anatomic changes are seen, such as in jejunal diverticulosis, lymphoma, Crohn's disease, strictures, or enteric fistulas. Also, there may be a distinctive barium pattern of thin-walled, dilated loops suggestive of celiac disease. CT and magnetic resonance enterography provide a more detailed imaging of the small intestine and are more sensitive in identifying abnormalities such as active bowel inflammation, mesenteric stranding and edema, strictures, fibrofatty proliferation of the mesentery, and fistula formation. Wireless capsule endoscopy is a noninvasive method that permits direct visualization of the small bowel mucosa and can provide a more detailed evaluation of small bowel disease compared with radiographic studies. However, capsule endoscopy should be avoided in patients in whom a stricture is suspected because of the risk of retention. The detection of mucosal lesions by the capsule endoscopy can often be followed by deep enteroscopy (double-balloon endoscopy, single-balloon endoscopy, or spiral enteroscopy), allowing for tissue biopsy, tattoo placement before surgery, balloon dilatation, and foreign body retrieval

Secretion of cyotkines and expression of cytokine receptors

In response to antigen and costimulators, T lymphocytes, especially CD4+ T cells, rapidly secrete the cytokine IL-2. Cytokines are a large group of proteins that function as mediators of immunity and inflammation. We have already discussed cytokines in innate immune responses, which are produced mainly by dendritic cells and macrophages (see Chapter 2). In adaptive immunity, cytokines are secreted by T cells, mainly CD4+ cells. Because most of these cytokines are produced by effector T cells and serve diverse roles in host defense, we describe them in Chapter 6 when we discuss the effector mechanisms of cell-mediated immunity IL-2 is produced within 1 to 2 hours after activation of CD4+ T cells. Activation also increases the expression of the high-affinity IL-2 receptor, thus rapidly enhancing the ability of the T cells to bind and respond to IL-2 (Fig. 5-11). The receptor for IL-2 is a three-chain molecule. Naive T cells express two signaling chains but do not express the α chain (CD25) that enables the receptor to bind IL-2 with high affinity. Within hours after activation by antigens and costimulators, the T cells produce the third chain of the receptor, and now the complete IL-2 receptor is able to bind IL-2 strongly. Thus, IL-2 produced by antigen-stimulated T cells preferentially binds to and acts on the same T cells, an example of autocrine cytokine action. The principal functions of IL-2 are to stimulate the survival and proliferation of T cells, resulting in an increase in the number of the antigen-specific T cells; because of these actions, IL-2 was originally called T cell growth factor. IL-2 also is essential for the maintenance of regulatory T cells and thus for controlling immune responses, as we discuss in Chapter 9. CD8+ T lymphocytes that recognize antigen and costimulators do not appear to secrete large amounts of IL-2, but these lymphocytes proliferate prodigiously during immune responses. Antigen recognition and costimulation may be able to drive the proliferation of CD8+ T cells, or IL-2 may be provided by CD4+ helper T cells.

How do Ladd's bands develop?

In some patients, the cecum ends up in the midabdomen From the cecum and the right side of the colon a series of peritoneal folds (Ladd's bands) develop that extend to the right undersurface of the liver and compress the duodenum. A small bowel volvulus may then occur as well as duodenal obstruction

chronic abdominal pain

In the evaluation of chronic abdominal pain, it can be challenging to distinguish between organic pain resulting from a specific pathologic process and functional pain. The location and characteristics of pain, as already discussed, serve as important guides, as do other accompanying symptoms. The presence of postprandial nausea and vomiting suggests chronic peptic ulcer, disorders of gastric emptying, or outlet obstruction. Documentation of weight loss mandates the search for an organic cause, such as inflammatory bowel disease or celiac disease. If anorexia accompanies weight loss, particularly in elderly patients, cancer must be excluded. If no cancer can be found and all objective tests are normal, the possibility of chronic depression must be entertained. The most frequent causes of chronic abdominal pain are functional. Dyspepsia is characterized by chronic intermittent epigastric discomfort, sometimes accompanied by nausea or bloating. These symptoms are not always relieved by acid suppression and may be the result of an underlying motor disorder. Furthermore, when Helicobacter pylori is found in a patient with dyspeptic symptoms, its eradication may not necessarily lead to the resolution of symptoms. Controversy exists regarding the most effective strategy for the treatment of dyspepsia when H. pylori organisms are found in the absence of peptic ulcer disease. Irritable bowel syndrome (IBS) is a very common disorder. Estimates are that 15% of Americans suffer from IBS on a regular basis and that 40% to 50% of referrals to gastroenterologists are related to IBS. The syndrome consists of abdominal distention, flatulence, and disordered bowel function. The abdominal pain of IBS tends to be in the left lower quadrant, but it can be located elsewhere or be more generalized. Any patient with weight loss, anemia, nocturnal symptoms, steatorrhea, or onset of symptoms after age 50 years should be evaluated carefully for organic disease because these symptoms are not associated with IBS. The Rome criteria, developed for research studies, may be helpful in the diagnosis of IBS. These criteria include pain that is associated with change in bowel habits, relieved with defecation, or accompanied by distention or bloating. Patients are reassured, counseled, and treated with anticholinergic agents and stool softeners. Although serotonin (5-HT) agonists such as alosetron and tegaserod showed promise initially, they have been relegated to limited use due to unacceptable side effects. Linaclotide is a new agent for chronic constipation and IBS with constipation (IBSC). This medication causes increased secretion of chloride and bicarbonate into the intestinal lumen via a cyclic guanosine monophosphate (cGMP) pathway. This pathway also may be responsible for relief of visceral pain in patients with IBS-C. The more challenging clinical problem is functional abdominal pain syndrome. This term describes a condition in which the pain has been present for months or years. The complaints of pain often are not related to eating, defecation, or menses, unlike other causes of chronic pain. The patient is most likely to be a woman who has undergone numerous examinations and diagnostic studies with negative findings and, in many cases, surgical operations without any relief. Lengthy or repeated diagnostic work-ups are counterproductive and only convince the patient that one more test is what is needed to determine the source of the pain. The physician must establish that organic disease is not present and must also realize that the pain is real. These patients are not malingerers despite the fact that the pain does not fit any familiar pattern. Depression may be the result rather than the cause of the pain. Management of chronic abdominal pain is demanding and requires as much tact, diplomacy, and compassion as scientific knowledge. An effort should be made to inquire about social factors, including history of physical and sexual abuse, particularly in women. Psychiatric evaluation may be necessary, but the suggestion for such a consultation may be interpreted by the patient as evidence that the physician believes "the pain is in my head." Referral to a competent pain management specialist is helpful in some cases. This approach offers the possibility of providing relief with nerve blocks if the pain is localized or with other pain-relieving devices. If this approach fails, referral to a psychologist or psychiatrist may be acceptable to the patient.

What is the treatment for diverticular disease?

In the first instance, patients will be treated with antibiotic therapy; however, a surgical resection may be necessary if symptoms persist.

When is acute rejection usually seen?

In the initial months after transplantation

Where do naive T cells recognize antigens? what does that intiiate?

In the peripheral lymphoid organs (secondary lymphoid organs) initiates proliferation of the T cells and their differentiation into effector and memory cells, and the effector cells perform their functions when they are activated by the same antigens in peripheral tissues or lymphoid organs

What is Molecular mimicry? What can it lead to?

In which epitopes from infectious agents are similar to self-antigens can also lead to a breakdown of tolerance and may be the mechanism underlying poststreptococcal glomerulonephritis

What does a functional obstruction occur due to?

Inability of the bowel to peristalse has a number of causes, and most frequently is a postsurgical state due to excessive intraoperative bowel handling. Other causes may well include abnormality of electrolytes (e.g., sodium and potassium) rendering the bowel paralyzed until correction has occurred.

What are the polyclonal activators of T cells?

Include antibodies specific for the TCR or associated CD3 proteins, polymeric carbohydrate-binding proteins such as phytohemagglutinin (PHA), and certain microbial proteins, including staphylococcal enterotoxins, which are called superantigens.

What is Malrotation ?

Incomplete rotation and fixation of the midgut after it has passed from the umbilical sac and returned to the abdominal coelom

What doe vasodilation following tissue injury lead to?

Increased blood flow to the site of injury causing the clinical signs of redness(rubor) and warmth (calor)

recruitment of cells

Increased vascular permeability also allows cells in the blood to enter the interstitium. Cellular migration out o the blood is not random; rather, leukocyte recruitment is orchestrated to optimize clearing o the in ection and local repair o the injured tissue (Fig. 42-6). At the onset o an inf ammatory response, the endothelial cells at the site o injury are activated to express adhesion molecules that bind speci c receptors expressed by leukocytes. For example, intercellular adhesion molecules ( ICAMs ) and vascular cell adhesion molecules ( VCAMs ) expressed by the vascular endothelium bind integrins expressed on the cell sur ace o leukocytes. This interaction causes the leukocytes, which normally roll along the sur ace o the endothelium by means o loose, transient binding interactions, to adhere tightly to the activated endothelium at the site o injury. The adherent leukocytes then bind other endothelial cell receptors that promote transmigration (diapedesis) o the leukocytes rom the vasculature into the interstitium. Speci city o the immune response is achieved according to the pattern o adhesion molecules expressed by the activated endothelium and by the various types o leukocytes; or example, neutrophils dominate the early inf ammatory response, while monocytes predominate a ter 24 hours.

What do the terminal MAP kinases in the pathway - d extracellular signal-regulated kinase (ERK) and c-Jun amino-terminal (N-terminal) kinase (JNK) induce?

Induce the expression of a protein called c-Fos and the phosphorylation of another protein called c-Jun c-Fos and phosphorylated c-Jun combine to form the transcription factor activating protein 1 (AP-1), which enhances the transcription of several T cell genes.

Intro

Inf ammation and the immune system are closely intertwined. Inf ammation is composed o a complex web o responses to tissue injury and in ection, characterized by the classic signs o rubor (redness), calor (heat), tumor (swelling), dolor (pain), and functio laesa (loss o unction). The immune system includes the cells and soluble actors, such as antibodies and complement proteins, which mediate the inf ammatory response; these cells and actors both eliminate the inciting inf ammatory stimulus and initiate immunologic memory A normal inf ammatory response is an acute process that resolves a ter removal o the inciting stimulus. Diseases o inf ammation and immunity can occur due to inappropriate inf ammation or when the normal inf ammatory response progresses to chronic inf ammation, either because o a longterm inappropriate response to a stimulus (e.g., allergies) or because the o ending agent is not removed (e.g., chronic in ection, transplantation, and autoimmunity). Two pharmacologic strategies are used to target the pathophysiology o immune diseases. The rst involves modi cation o the signaling mediators o the inf ammatory process or suppression o components o the immune system. This is the rationale or drugs that modulate eicosanoid pathways (Chapter 43, Pharmacology o Eicosanoids), histamine (Chapter 44, Histamine Pharmacology), and cells o the immune system (Chapter 45, Pharmacology o Hematopoiesis and Immunomodulation, and Chapter 46, Pharmacology o Immunosuppression). This approach is still in its in ancy, because it depends on understanding the molecular events in the relevant pathway, but it promises to yield multiple new drugs in the oreseeable uture. The second pharmacologic approach, used in diseases such as peptic ulcer disease (Chapter 47, Integrative Inf ammation Pharmacology: Peptic Ulcer Disease), asthma (Chapter 48, Integrative Inf ammation Pharmacology: Asthma), and gout (Chapter 49, Integrative Inf ammation Pharmacology: Gout), involves modi cation o the underlying pathophysiologic stimulus, thus removing the impetus or inf ammation. The di erence between these two approaches is, at times, indistinct and will continue to overlap as the pathophysiology o chronic inf ammatory disease is better understood at the molecular level This chapter provides su cient background on the physiology o inf ammation and the immune system to understand the subsequent chapters in this section o the textbook. The treatment is necessarily brie , with an emphasis on pharmacologically relevant targets o the inf ammatory response. The chapter is organized in our parts. First, a general overview o the immune system is presented. Second, the molecular signals that mediate cellular communication and inf ammation are introduced. Third, the immune and inf ammatory cells and signaling molecules are discussed in the context o an integrated inf ammatory response. Fourth, chronic inf ammation, a pathologic state that is o ten associated with autoimmunity, is presented. For a more comprehensive presentation o this rapidly changing subject, see "Suggested Reading" at the end o this chapter.

How can infections lead to the occurrence of autoimmune diseases?

Infections may activate self-reactive lymphocytes, thereby triggering the development of autoimmune diseases

role of infections and other environmental influencfes

Infections may activate self-reactive lymphocytes, thereby triggering the development of autoimmune diseases. Clinicians have recognized for many years that the clinical manifestations of autoimmunity sometimes are preceded by infectious prodromes. This association between infections and autoimmune tissue injury has been formally established in animal models. Infections may contribute to autoimmunity in several ways (Fig. 9-13): • An infection of a tissue may induce a local innate immune response, which may lead to increased production of costimulators and cytokines by tissue APCs. These activated tissue APCs may be able to stimulate self-reactive T cells that encounter self antigens in the tissue. In other words, infection may break T cell tolerance and promote the activation of selfreactive lymphocytes. • Some infectious microbes may produce peptide antigens that are similar to, and cross- react with, self antigens. Immune responses to these microbial peptides may result in an immune attack against self antigens. Such crossreactions between microbial and self antigens are termed molecular mimicry. Although the contribution of molecular mimicry to autoimmunity has fascinated immunologists, its actual significance in the development of most autoimmune diseases remains unknown. In some rare disorders, antibodies produced against a microbial protein bind to self proteins. In rheumatic fever, antibodies against streptococci cross-react with a myocardial antigen and cause heart disease. • The innate response to infections may alter the chemical structure of self antigens. For example, some periodontal bacterial infections are associated with rheumatoid arthritis. It is postulated that the acute and chronic inflammatory responses to these bacteria lead to enzymatic conversion of arginines to citrullines in self proteins, and the citrullinated proteins are recognized as nonself and elicit adaptive immune responses. • Infections also may injure tissues and release antigens that normally are sequestered from the immune system. For example, some sequestered antigens (e.g., in testis and eye) normally are not seen by the immune system and are ignored. Release of these antigens (e.g., by trauma or infection) may initiate an autoimmune reaction against the tissue. • The abundance and composition of normal commensal microbes in the gut, skin, and other sites (the microbiome) may also influence the health of the immune system and the maintenance of self-tolerance. This possibility has generated a great deal of interest, but normal variations in the microbiome of humans related to environmental exposure and diet make it difficult to define the relationship between particular microbes and the development of autoimmune diseases Paradoxically, some infections appear to confer protection from autoimmune diseases. This conclusion is based on epidemiologic data and limited experimental studies. The basis of this protective effect of infections is unknown. Several other environmental and host factors may contribute to autoimmunity. Many autoimmune diseases are more common in women than in men, but how gender might affect immunological tolerance or lymphocyte activation remains unknown. Exposure to sunlight is a trigger for the development of the autoimmune disease systemic lupus erythematosus (SLE), in which autoantibodies are produced against self nucleic acids and nucleoproteins. It is postulated that these nuclear antigens may be released from cells that die by apoptosis as a consequence of exposure to ultraviolet radiation in sunlight.

What is the first branch of the Superior mesenteric artery? What does it divide into? What does it supply?

Inferior pancreaticoduodenal artery It divides immediately into anterior and posterior branches, which ascend on the corresponding sides of the head of the pancreas This arterial network supplies the head and uncinate process of the pancreas and the duodenum.

What is a characteristic finding on cross-sectional imaging in Crohn's disease?

Infiltration of the mesentery with fat, commonly known as Creeping fat

Intro

Inflammatory bowel disease (IBD) comprises two disorders: ulcerative colitis and Crohn's disease. The diagnosis of IBD is based on review of clinical, endoscopic, radiologic, and histologic data. Despite the chronic nature of these two diseases and the fact that their causes have not yet been defined, new and emerging targeted anti-inflammatory treatments hold great promise in helping to reduce morbidity and improve the quality of life of individuals with IBD.

What does an elevated white blood cell count indicate?

Inflammatory disease extremely high values are typical of acute intestinal ischemia

Genetic factors

Inherited risk for most autoimmune diseases is attributable to multiple gene loci, of which the largest contribution is made by MHC genes. If an autoimmune disease develops in one of two twins, the same disease is more likely to develop in the other twin than in an unrelated member of the general population. Furthermore, this increased incidence is greater among monozygotic (identical) twins than among dizygotic twins. These findings prove the importance of genetics in the susceptibly to autoimmunity. Genome-wide association studies have revealed some of the common variations (polymorphisms) of genes that may contribute to different autoimmune diseases. Emerging results suggest that different polymorphisms are more frequent (predisposing) or less frequent (protective) in patients than in healthy controls. Their importance is reinforced by the finding that many of these polymorphisms may affect immune responses, and the same genetic polymorphisms are associated with different autoimmune diseases. However, these polymorphisms are frequently present in healthy individuals, and the individual contribution of each of these genes to the development of autoimmunity is very small. In rare cases, autoimmunity- associated genes are variants (mutations) that are essentially nonexistent in healthy individuals, rather than commonly detected polymorphisms. Such rare variants can have a large impact on the development of autoimmunity. Many autoimmune diseases in humans and inbred animals are linked to particular MHC alleles (Fig. 9-11). The association between human leukocyte antigen (HLA) alleles and autoimmune diseases in humans was recognized many years ago and was one of the first indications that T cells played an important role in these disorders (because the only known function of MHC molecules is to present peptide antigens to T cells). The incidence of a particular autoimmune disease often is greater among individuals who inherit a particular HLA allele(s) than in the general population. This increased incidence is called the odds ratio or relative risk of an HLA-disease association; the same nomenclature is applicable to the association of any gene with any disease. It is important to point out that although an HLA allele may increase the risk of developing a particular autoimmune disease, the HLA allele is not, by itself, the cause of the disease. In fact, the disease never develops in the vast majority of people who inherit an HLA allele that does confer increased risk of the disease. Despite the clear association of MHC alleles with several autoimmune diseases, how these alleles contribute to the development of the diseases remains unknown. Some hypotheses are that particular MHC alleles may be especially effective at presenting pathogenic self peptides to autoreactive T cells, or they are inefficient at displaying certain self antigens in the thymus, leading to defective negative selection of T cells. Polymorphisms in non-HLA genes are associated with various autoimmune diseases and may contribute to failure of self-tolerance or abnormal activation of lymphocytes (Fig. 9-12, A). Many such disease-associated genetic variants have been described: • Polymorphisms in the gene encoding the tyrosine phosphatase PTPN22 (protein tyrosine phosphatase N22) may lead to uncontrolled activation of both B and T cells and are associated with numerous autoimmune diseases, including rheumatoid arthritis, systemic lupus erythematosus, and type 1 diabetes mellitus. • Variants of the innate immune cytoplasmic microbial sensor NOD-2 that cause reduced resistance to intestinal microbes are associated with Crohn's disease, an inflammatory bowel disease, in some ethnic populations. • Other polymorphisms associated with multiple autoimmune diseases include genes encoding the IL-2 receptor α chain (CD25), believed to influence the balance of effector and regulatory T cells; the receptor for the cytokine IL-23, which promotes the development of proinflammatory Th17 cells; and CTLA-4, a key inhibitory receptor in T cells discussed earlier. It is hoped that elucidation of these genetic associations will reveal pathogenic mechanisms or provide new ideas for better prediction and treatment. Some rare autoimmune disorders are mendelian in origin, caused by mutations in single genes that have high penetrance and lead to autoimmunity in most or all individuals who inherit these mutations. These genes, alluded to earlier, include AIRE, FOXP3, FAS, and CTLA4 see (Fig. 9-12, B). Mutations in these genes have been valuable for identifying key molecules and pathways involved in self-tolerance. These mendelian forms of autoimmunity are exceedingly rare, however, and common autoimmune diseases are not caused by mutations in any of these known genes.

inhibitory receptors of T cells

Inhibitory receptors are critical for limiting and terminating immune responses. Two important inhibitory receptors—CTLA-4 and PD-1—are structurally related to CD28. . CTLA-4, like CD28, recognizes B7-1 and B7-2 on APCs, and PD-1 recognizes different but structurally related ligands on many cell types. Both CTLA-4 and PD-1 are induced in activated T cells, and function to terminate responses of these cells. CTLA-4 also plays an important role in the suppressive function of regulatory T cells (see Chapter 9). Because these inhibitory receptors evolved to prevent immune responses against self antigens, genetic deletion or blockade of these molecules in mice and humans results in systemic autoimmune disease. CTLA-4 and PD-1 are also involved in inhibiting responses to some tumors and chronic viral infections. These discoveries are the basis for the use of antibodies that block CTLA -4 or PD-1 to enhance immune responses to tumors in patients with cancer (see Chapter 10). The function of these inhibitory receptors is discussed in more detail in Chapter 9, in the context of maintaining unresponsiveness to self antigens.

What effect does Interaction of CTLA-4 with B7-1 or B7-2 have on the ?

Inhibits T cell proliferation This appears to be a physiologic mechanism for self limitation of the immune response.

What is the function of Cyclosprine

Inhibits calcineurin's phosphatase activity thus suppresses the NFAT-dependent production of cytokines by T cells This agent is widely used as an immunosuppressive drug to prevent graft rejection

activation of the innate immune response

Innate immune cells respond to common determinants that are present on many invading agents ( or example, lipopolysaccharide [LPS] in the outer membrane o Gram-negative bacteria). In this role, innate immune cells use pattern recognition to phagocytose a class o in ectious agents rather than a speci c in ectious agent. In contrast, adaptive immune cells, as discussed below, mount a speci c response to the three-dimensional con ormation o a particular antigen, re erred to as an epitope . From a teleological perspective, innate immunity provides a broad gating unction, attempting to counteract harm ul e ects o oreign invaders in a rapid manner and to determine whether an in ectious agent should be urther attacked by adaptive immunity, while adaptive immunity provides a specialized response that is speci c to the particular invading in ectious agent. In an individual, innate immune cells respond in the same way and to the same extent to repeated in ections with the same agent In contrast, adaptive immune cells mount a aster and more intense response upon reexposure to the infectious agent The pattern recognition unction o innate immune cells is mediated by multiple mechanisms. One important mechanism involves Toll-like receptors (TLRs) . TLRs are transmembrane proteins that bind to shared microbial components such as LPS expressed by Gram-negative bacteria, mannans expressed by ungi, and double-stranded RNA expressed by viral pathogens. Ten TLRs are expressed in humans, and each has a characteristic immune cell distribution and set o pathogen-associated ligands. For example, TLR4 is expressed by antigen-presenting cells and it binds to LPS. Binding o TLRs to their ligands activates an intracellular signaling cascade that converges on the expression o proinf ammatory cytokines, leading to urther immune cell recruitment and activation o the inf ammatory response. A undamental role o innate immunity is to provide an immediate "alarm" that recruits elements o adaptive immunity. This alarm indicates that a molecular structure associated with a pathogen has been detected and serves as an early warning system that the adaptive immune system should initiate an immune response against the pathogen-associated antigens encountered in the context o a TLR agonist or another innate immune signal. Several pharmacologic agents are being investigated as modulators o TLR signaling. Imiquimod , discussed in Chapter 38, Pharmacology o Viral In ections, may unction as a TLR agonist.

Cells of what immune system are the first responders to an offending agent?

Innate immune system

What is abdominal viscera innervated by?

Innervated by both extrinsic and intrinsic components of the nervous system: - Extrinsic innervation involves receiving motor impulses from, and sending sensory information to, the central nervous system. - Intrinsic innervation involves the regulation of digestive tract activities by a generally self-sufficient network of sensory and motor neurons (the enteric nervous system).

What is Mycophenolic acid (MPA) an inhibitor of?

Inosine monophosphate dehydrogenase (IMPDH) ^ is the the rate limiting step in the formation of Guanosine

What is a byproduct of PLC-gamma mediated PIP2 breakdown? What is its function?

Inositol 1,4,5-triphosphate (IP3), binds to IP3 receptors on the endoplasmic reticulum (ER) membrane and stimulates release of Ca2+ from the ER, thereby raising the cytosolic Ca2+ concentration.

What process is critical for the ability of T cells to bind to APCs displaying microbial antigens?

Integrin-mediated adhesion Integrins also play an important role in directing the migration of effector T cells and other leukocytes from the circulation to sites of infection.

What family do the most important adhesion molecules belong to?

Integrins

How is Indeterminante colitis different from Ulcerative colitis?

Interdeteminate colitis do not involve the small bowel and have colonic disease in a continuous pattern that would typically indicate ulcerative colitis.

What is the MOA of Alkylating agents?

Interfere with DNA replication and gene expression by alkylation of DNA.

What does the TH1 cytokine profile contain elevated levels of in Crohn's disease?

Interferon-γ (INF gamma), IL-2, IL-12, and tumor necrosis factor-α (TNF-α),

IL-1 cyotokine and IL-1 receptor inhibitors

Interleukin-1 (IL-1) is an ancient cytokine, expressed in both vertebrates and invertebrates, that serves as a bridge between innate and adaptive immunity. Two orms o IL-1, IL-1 and IL-1 , are encoded on di erent genes. In humans, IL-1 has primarily an immune role, while IL-1 may be involved in maintenance o epithelial cell unction. Human genetic data and studies with IL-1 antagonists point to a nonredundant role or IL-1 as an in ammatory mediator. Herea ter, there ore, we use the term IL-1 to re er to IL-1 beta Most IL-1 is generated by activated mononuclear cells. IL-1 stimulates IL-6 production, enhances adhesion molecule expression, and stimulates cell proli eration. Modulation o IL-1 activity in vivo is accomplished in part by an endogenous IL-1 receptor antagonist (IL-1ra). Anakinra , a recombinant orm o IL-1ra, is approved or use in rheumatoid arthritis. Anakinra has modest e ects on pain and swelling but signif cantly reduces bony erosions, possibly because it decreases osteoclast production and blocks IL-1-induced metalloproteinase release rom synovial cells. Several rare syndromes are mediated in part by increased levels o IL-1. Collectively, these syndromes are termed cryopyrin-associated periodic syndromes (CAPS). Anakinra is approved or use in the CAPS syndrome neonatal-onset multisystem in ammatory disease, and the CAPS syndromes Muckle-Wells syndrome and Hibernian ever have also been treated e ectively with anakinra. Anakinra may cause neutropenia and increase susceptibility to infection . The rapid clearance o this small recombinant peptide and its competitive binding mechanism may explain the need or daily injections o the drug to achieve efficacy. Rilonacept is a recombinant, soluble IL-1 receptor Fc usion protein that is approved or use in CAPS. Rilonacept binds both IL-1 and IL-1 and also endogenous IL-1ra. Possibly because o its binding to the endogenous receptor antagonist, once-weekly injections o rilonacept are su f cient or e f cacy in the treatment o CAPS. Canakinumab is a human IgG1 monoclonal antibody directed against IL-1 that is approved or use in CAPS and systemic juvenile idiopathic arthritis. Perhaps because o its specif city or IL-1 , canakinumab may be administered just once monthly and still exhibit ull e f cacy. No studies have been conducted to compare the e f cacy o the three anti-IL1 therapies

IL-17 cytokine and IL-17 receptor inhibitors

Interleukin-17 (IL-17 , also known as IL-17A ) is a cytokine that is mainly produced by IL-23-induced T H 17 cells. IL-17 plays a central role in in ammation by stimulating the production o key in ammatory mediators, including IL-6, TNF- , and IL-1 . Keratinocytes, f broblasts, macrophages, and neutrophils are among the cell types that respond to IL-17-induced proin ammatory cytokine production. IL-17 also stimulates the proli eration and survival o neutrophils, T cells, and B cells. Ixekizumab and secukinumab are two ully human monoclonal antibodies directed against IL-17A that are in late-phase clinical trials or the management o psoriasis and rheumatoid arthritis IL-17 exerts its proin ammatory e ect by signaling through two receptor subtypes, IL-17 receptor A (IL-17RA) and IL-17 receptor C (IL-17RC). Brodalumab is an antiIL-17 receptor monoclonal antibody that acts as a competitive inhibitor o the IL-17RA subunit. This drug is under active investigation or the treatment o psoriasis, rheumatoid arthritis, and Crohn's disease

What is the most valuable test in establishing the diagnosis of Celiac disease?

Intestinal biopsy

What is Celiac disease characterized by?

Intestinal mucosal injury resulting from gluten-related immunologic damage in persons genetically predisposed to this condition.

What is Radiographic examination with an abdominal film important for revealing?

Intra-abdominal gas pattern an upright film that includes the diaphragm or a left lateral decubitus film may identify intra-abdominal air suggesting perforation of a hollow viscus.

Is the cecum Intraperitoneal or retroperitoneal?

Intraperitoneal because of its mobility, even though it normally is not suspended in the peritoneal cavity by a mesentery

What drug is used as a rescue medicine and as a bridge treatment to immunomodulators or biologic agents?

Intravenous cylosporine

What is the treatment of Obstruction?

Intravenous replacement of fluid and electrolytes, analgesia, and relief of obstruction The passage of a nasogastric tube allows aspiration of fluid from the stomach In many instances, small bowel obstruction, typically secondary to adhesions, will settle with nonoperative management. Large bowel obstruction may require an urgent operation to remove the obstructing lesion, or a temporary bypass procedure (e.g., defunctioning colostomy)

Fever with diarrhea usually indicates what type of organisms? examples?

Invasive organisms such as Salmonella, Shigella, Campylobacter, certain viruses, Entamoeba histolytica, or C. difficile.

Inflammatory bowel disease

Inflammatory bowel disease (IBD) is a chronic condition resulting from inappropriate mucosal immune activation. The two disorders that comprise IBD are ulcerative colitis and Crohn disease. Descriptions of ulcerative colitis and Crohn disease date back to antiquity and at least the sixteenth century, respectively, but it took modern microbiologic techniques to exclude conventional infectious etiologies for these diseases. As will be discussed later, however, the luminal microbiota likely play a role in the pathogenesis of IBD. The distinction between ulcerative colitis and Crohn disease is based, in large part, on the distribution of affected sites (Fig. 17-32) and the morphologic expression of disease (Table 17-9) at those sites. Ulcerative colitis is limited to the colon and rectum and extends only into the mucosa and submucosa. In contrast, Crohn disease, which has also been referred to as regional enteritis (because of frequent ileal involvement) may involve any area of the GI tract and is typically transmural.

What may develop in individuals with Crohns disease?

Iron-deficiency anemia while extensive small bowel disease may result in serum protein loss and hypoalbuminemia, generalized nutrient malabsorption, or malabsorption of vitamin B12 and bile salts Fibrosing strictures, particularly of the terminal ileum, are common and require surgical resection

What does chronic rejection eventually lead to?

Irreversible organ failure other contributing nonimmune factors include ishchemia-reperfusion injury and infection No effective treatment regimens are currently available to eliminate chronic rejection

Irritable bowel syndrome

Irritable bowel syndrome (IBS) is characterized by chronic, relapsing abdominal pain, bloating, and changes in bowel habits. Despite very real symptoms, the gross and microscopic evaluation is normal in most IBS patients. Thus, the diagnosis depends on clinical symptoms and functional testing. It should be recognized that IBS is a syndrome, and that multiple illnesses are represented under this global descriptor. IBS is currently divided into several subtypes, as defined by successive revisions of the Rome criteria.

What is Functional diarrhea defined by?

Is defined by the Rome III criteria as continuous or recurrent passage of loose or watery stools without abdominal pain or discomfort, occurring in at least 75% of stools for at least 3 months.

What can happen if the bowel continues to distend compromising the blood supply within the bowel wall?

Ischemia and perforation

Where does the right gastric artery ascend and what does it supply?

It courses to the left and ascends along the lesser curvature of the stomach in the lesser omentum, supplies adjacent areas of the stomach

Describe the path of the abdominal aorta

It descends through the abdomen, anterior to the vertebral bodies, and by the time it ends at the level of vertebra LIV it is slightly to the left of midline

What does PD-1 have that helps deliver the inhibitory signals?

It has an immunoreceptor tyrosine-based inhibitory motif (ITIM) typical of receptors that deliver inhibitory signal PD-1 terminates responses of T cells to self antigens and also to chronic infections, notably virus infections

Ostomies

It is occasionally necessary to surgically externalize bowel to the anterior abdominal wall. Externalization of bowel plays an important role in patient management. These extraanatomical bypass procedures use our anatomical knowledge and in many instances are life saving

Where does the Splenic artery travel?

It travels in the splenorenal ligament and divides into numerous branches, which enter the hilum of the spleen.

What is Tofacitinib?

JAK inhibitor, already approved for rheumatoid arthritis, is now being investigated for Crohn's disease

Where are most dietary lipids absorbed?

Jejunum along with the fat-soluble vitamins A, D, E, and K. It is recommended that dietary fat account for no more than 35% of calories because higher levels are associated with increased risk of cardiac disease, obesity, and some cancers.

Describe the Vasa recta supplying the Jejunum vs the Vasa recta supplying the Ileum

Jejunum: are usually long and close together, forming narrow windows visible in the mesentery Ileum: are generally short and far apart, forming low broad windows.

What are the cell types that respond to IL-17 induced proinflammatory cytokine?

Keratinocytes, Fibroblasts, macrophages, and neutrophils are among the cell types

Antigen recognition by a T cell increases the affinity of that cells?

LFA-1 Therefore, once a T cell sees antigen, it increases the strength of its binding to the APC presenting that antigen, providing a positive feedback loop

What is the major T cell integrin involved in binding to APCs?

LFA-1 leukocyte function-associated antigen 1 whose ligand on APCs is called intercellular adhesion molecule 1 (ICAM-1).

LFA-3

LFA-3 (also called CD58 ) is the counter-receptor or CD2, an antigen expressed at high levels on the sur ace o memory e ector T cells. Interaction o CD2 on T cells with LFA-3 on antigen-presenting cells promotes increased T-cell proli eration and enhanced T-cell-dependent cytotoxicity. Because the memory e ector T-cell population is increased in patients with psoriasis, a pharmacologic agent that disrupts the CD2- LFA-3 interaction was tested or use in psoriasis Alefacept is an LFA-3/Fc usion protein that interrupts CD2-LFA-3 signaling by binding to T-cell CD2 and thereby inhibits T-cell activation. Additionally, the Fc portion o ale acept may activate NK cells to deplete the immune system o memory e ector T cells. Clinically, ale acept signif cantly decreases the severity o chronic plaque psoriasis. Because CD2 is expressed on other adaptive immune cells, however, administration o ale acept also causes a dosedependent reduction in CD4 and CD8 T-cell populations. Patients taking ale acept may have an increased risk o serious in ection and an increased risk o malignancy, primarily skin cancer. Ale acept was voluntarily withdrawn rom the market by the manu acturer in 2011.

Pathophysiologic mechanisms in Malabsorption

LUMINAL PHASE MUCOSAL PHASE TRANSPORT PHASE Reduced nutrient availability Extensive mucosal loss (resection or infarction) Vascular conditions (vasculitis; atheroma) Cofactor deficiency (pernicious anemia; gastric surgery) Diffuse mucosal disease (celiac disease) Lymphatic conditions (lymphangiectasia; irradiation; nodal tumor, cavitation, or infiltrations) Nutrient consumption (bacterial overgrowth) Crohn's disease; irradiation; infection; infiltrations; drugs: alcohol, colchicine, neomycin, iron salts Impaired fat solubilization Brush border hydrolase deficiency (lactase deficiency) Reduced bile salt synthesis (hepatocellular disease) Transport defects (Hartnup cystinuria; vitamin B12 and folate uptake) Impaired bile salt secretion (chronic cholestasis) Epithelial processing (abetalipoproteinemia) Bile salt inactivation (bacterial overgrowth) Impaired cholecystokinin release (mucosal disease) Increased bile salt losses (terminal ileal disease or resection) Defective nutrient hydrolysis Lipase inactivation (Zollinger-Ellison syndrome) Enzyme deficiency (pancreatic insufficiency or cancer) Improper mixing or rapid transit (resection; bypass; hyperthyroidism)

What is the most common example of carbohydrate malabsorption? tests to measure?

Lactose intolerance specific tests include the oral lactose tolerance test, but measurement of breath hydrogen is more sensitive and more specific.

What is the most common malabsorptive process? How can it be diangosed?

Lactose intolerance Diagnosed by lactose breath testing or the more simple lactose avoidance trial Celiac disease and tropical sprue can result in a wide spectrum and severity of symptoms, from iron deficiency to calcium and magnesium deficiency, fat-soluble vitamin deficiencies, and weight loss

What is the Celiac plexus? What ganglia are associated with the Celiac plexus?

Large accumulation of nerve fibers and ganglia associated with the roots of the celiac trunk and superior mesenteric artery immediately below the aortic hiatus of the diaphragm 2 celiac ganglia, 1 mesenteric ganglion, and 2 aorticorenal ganglia.

What is the arterial supply of the descending colon?

Left colic artery from the inferior mesenteric artery.

What does the right gastric artery anastamose with?

Left gastric artery

What is the smallest branch of the Celiac trunk?

Left gastric artery It ascends to the cardioesophageal junction and sends esophageal branches upward to the abdominal part of the esophagus Some of these branches continue through the esophageal hiatus of the diaphragm and anastomose with esophageal branches from the thoracic aorta

What does the right gastro-omental artery anastamose with?

Left gastro-omental artery from the splenic artery. right gastro-omental artery sends branches to both surfaces of the stomach and additional branches descend into the greater omentum

What does the hindgut include? What is it supplied by?

Left one-third of the transverse colon, descending colon, sigmoid colon, rectum, and upper part of the anal canal Inferior mesenteric artery

graft versus host disease

Leukemia, primary immunodef ciency, and other conditions can be treated with bone marrow or peripheral stem cell transplantation. In this procedure, hematopoietic and immune unction is restored a ter the patient's bone marrow has been eradicated by aggressive chemotherapy and/or radiation therapy. GVHD is a major complication o allogeneic bone marrow or stem cell transplantation. GVHD is an alloimmune in ammatory reaction that occurs when transplanted immune cells attack the cells o the recipient. The severity o GVHD ranges rom mild to li e-threatening and typically involves the skin (rash), gastrointestinal tract (diarrhea), lungs (pneumonitis), and liver (veno-occlusive disease). GVHD can o ten be ameliorated by removing T cells rom the donor bone marrow be ore transplantation. Mild to moderate GVHD can also be benef cial when donor immune cells attack recipient tumor cells that have survived the aggressive chemotherapy and radiation therapy. (In the case o leukemia, this is called the graft-versus-leukemia effect , or GVL .) There ore, although removing donor T cells rom the "gra t" reduces the risk o GVHD, this may not be the best approach or marrow transplants used in antineoplastic therapy

What are inhibitory T cells critical for ?

Limiting and terminating immune responses

What is the treatment for Microscopic colitis?

Loperamide and cholestyramine may be useful in mild disease Budesonide, a poorly absorbed steroid, may be used, although relapses are frequent after weaning

Where does the abdominal pain of Inflammatory bowel syndrome (IBS) tend to be located?

Lower left Quadrant but it can be located elsewhere or be more generalized. Any patient with weight loss, anemia, nocturnal symptoms, steatorrhea, or onset of symptoms after age 50 years should be evaluated carefully for organic disease because these symptoms are not associated with IBS.

What do SIgmoid artery (branches of IMA) supply? Anastamoses?

Lowest part of the descending colon and the sigmoid colon anastomose superiorly with branches from the left colic artery inferiorly with branches from the superior rectal artery

What has MMF been frequently used in the initial therapy of?

Lupus nephritis There have also been isolated reports of successful treatment of myasthenia gravis, psoriasis, autoimmune hemolytic anemia, and inflammatory bowel disease with MMF.

lymphatics

Lymphatic drainage of the abdominal part of the gastrointestinal tract, as low as the inferior part of the rectum, as well as the spleen, pancreas, gallbladder, and liver, is through vessels and nodes that eventually end in large collections of pre-aortic lymph nodes at the origins of the three anterior branches of the abdominal aorta, which supply these structures. These collections are therefore referred to as the celiac, superior mesenteric, and inferior mesenteric groups of pre-aortic lymph nodes. Lymph from viscera is supplied by three routes ■ The celiac trunk (i.e., structures that are part of the abdominal foregut) drains to pre-aortic nodes near the origin of the celiac trunk (Fig. 4.134)—these celiac nodes also receive lymph from the superior mesenteric and inferior mesenteric groups of pre-aortic nodes, and lymph from the celiac nodes enters the cisterna chyli. ■ The superior mesenteric artery (i.e., structures that are part of the abdominal midgut) drains to pre-aortic nodes near the origin of the superior mesenteric artery (Fig. 4.134)—these superior mesenteric nodes also receive lymph from the inferior mesenteric groups of pre-aortic nodes, and lymph from the superior mesenteric nodes drains to the celiac nodes. ■ The inferior mesenteric artery (i.e., structures that are part of the abdominal hindgut) drains to pre-aortic nodes near the origin of the inferior mesenteric artery (Fig. 4.134), and lymph from the inferior mesenteric nodes drains to the superior mesenteric nodes

What do MPA and MFA both act primarily on? What factors contribute to this selectivity?

Lymphocytes (B cells and T cells) 1- Lymphocytes are dependent on the de novo pathway of purine synthesis, whereas most other tissues rely heavily on the salvage pathway. Because IMPDH is required for fde novo synthesis of guanosine nucleotides but not for the salvage pathway, MPA selectively affects cells such as lymphocytes that rely on de novo purine synthesis. 2- IMPDH is expressed in two isoforms: type I and type II. MPA preferentially inhibits type II IMPDH, the isoform expressed mainly in lymphocytes Together, these factors confer on MPA and MMF selectivity against T and B cells, with relatively low toxicity to other cells.

What cells produce IL-6? What cells express IL-6?

Lymphocytes, monocytes, fibroblasts, keratinocytes, endothelial cells, mesangial cells, and adipocytes all produce IL-6 while cells of hematopoietic origin express the IL-6 receptor

What is the difference between Collagenous colitis and lymphocytic colitis?

Lymphocytic colitis is histologically similar to Collagenous, but: 1- Subepithelial collagen layer is of normal thickness 2- The increase in intraepithelial lymphocytes is greater, frequently exceeding one T lymphocyte per five colonocytes

Where is the proliferation and differentiation of Naive T cells confined to?

Lymphoid organs where dendritic cells (which express abundant costimulators) display antigens, but the functions of effector T cells may be reactivated by any host cell displaying microbial peptides bound to MHC molecules, not just dendritic cells

All nucleated cells express what class of MHC protein?

MHC I the repertoire of protein fragments displayed by MHC class I proteins on a cell provides a fingerprint or all the proteins expressed within that cell

What does the T cell receptor recognize?

MHC associated peptide antigens

What is the inherited risk for most autoimmune diseases attributable to multiple gene loci of which the largest contribution is made by what genes?

MHC genes

What do CD4 or CD8 coreceptors on the T cells recognize?

MHC molecules on the APC also help the TCR complex to deliver activating signas

major histocompatability complex

MHC proteins are transmembrane proteins that bind and display on their sur ace proteolytically degraded protein ragments and, in some cases, glycolipid antigens. There are two classes o MHC proteins: MHC class I and MHC class II. MHC class I proteins primarily display ragments o cytosolic proteins (Fig. 42-2). All nucleated cells express MHC class I proteins; the repertoire o protein ragments displayed by MHC class I proteins on a cell provides a ngerprint or all the proteins expressed within that cell. I a cell is expressing a recognizable pattern o proteins, then it will not be attacked by the immune system. However, i oreign (e.g., viral) proteins are being generated within the cell, then proteolytic ragments o those viral proteins will be displayed on MHC class I proteins at the sur ace o the cell, and the immune system will recognize that cell as virally in ected. Antigens presented by MHC class I proteins are recognized by T cells bearing the cell sur ace protein CD8. (The designation "CD" stands or cluster of differentiation or cluster designation and is a system or naming an ever-growing list o cell-associated antigens. Each antigen must be de ned by at least two di erent monoclonal antibodies in order to earn the CD designation. CD antigens now number in the hundreds and are present on leukocytes and other cell types MHC class II proteins display protein ragments derived rom endocytic vesicles. In contrast to class I proteins, which are expressed on all nucleated cells, MHC class II proteins are expressed mostly on antigen-presenting cells (e.g., macrophages and dendritic cells), although some other cell types can be induced to express MHC class II proteins. Endocytic vesicles contain antigenic protein ragments derived rom in ectious agents a ter phagocytosis and proteolytic processing o those agents. There ore, the protein ragments expressed on MHC class II proteins generally identi y extracellular oreign agents (e.g., bacteria). As discussed below, T cells bearing the cell sur ace protein CD4 recognize antigens presented by MHC class II proteins. In the process, these T cells stimulate the antigen-presenting cells to produce soluble actors called cytokines and chemokines , which, in turn, aid the T cells in responding to the antigen. In general, then, protein fragments bound to MHC class I identify infected cells , whereas fragments bound to MHC class II identify infectious agents . However, because o the phenomenon o cross-presentation, some proteins generated in the cytosol can be presented by MHC class II to CD4 T cells, and some phagocytosed antigens can be presented by MHC class I to CD8 t cells

What have clinical studies comparing MMF and AZA shown?

MMF to be more efficacious in preventing acute rejection of kidney transplants Animal models show that chronic rejection is also reduced more effectively in recipients treated with MMF than in those treated with AZA or cyclosporine. The efficacy of MMF in treating chronic rejection may be related to its inhibition of both the lymphocyte and the smooth muscle cell proliferation characteristic of chronic rejection.

MOA and USE: abatacept

MOA: CTLA-4 analogue fused to IgG1 constant region; prevents delivery of a costimulatory signal and T cells develops anergy or undergoes apoptosis USE: rheumatoid arthritis and juvenile idopathic arthritis

MOA and USE: infliximab

MOA: Tumor necrosis factor inhibitor (anti-TNF antibody) USE: Rheumatoid arthritis, ankylosing spondylitis, psoriatic arthritis, Crohn's disease, plaque psoriasis

MOA and USE: etanercept

MOA: Tumor necrosis factor inhibitor (soluble TNF receptor dimer) USE: Rheumatoid arthritis, juvenile idiopathic arthritis, plaque psoriasis, psoriatic arthritis, ankylosing spondylitis

MOA and Use: Mycophenolate Mofetil (MMF)

MOA: antimetabolite, cytotoxic agent, *inhibits inosine monophosphate dehydrogenase (IMPDH) the rate-limiting enzyme in the formation of guanosine* USE: solid organ transplantation; used in immune-mediated disease because of high selectivity and profound effect on lymphocytes

MOA and USE: cyclosporine

MOA: binds to cylophilin, and the resulting complex inhibits the phosphatase activity of calcineurine, a cell-signaling protein that mediates T cell activation inhibits production of IL-2 by activated T cells USE: solid organ transplantation, keratoconjunctivitis sicca (topical cyclosporine)

MOA and USE: tacrolimus

MOA: binds to the FK-binding protein (FKBP), and the tracolimus-FKBP complex inhibits calcineurin USE: organ transplantation, atopic dermatitis (topical tacrolimus)

MOA and USE: rituximab

MOA: chimeric , partially humanized anti-CD20 monoclonal antibody USE: cronic lymphocytic leukemia, B-cell non-Hodgkin's lymphoma, rheumatoid arthritis, granulomatosis with polyangiitis and microscopic polyangiitis

MOA and USE: brentuximab vedotin

MOA: chimeric human monoclonal antibody-drug conjugate directed against CD30 USE: Hodgkin's lymphoma, anaplastic large cell lymphoma

MOA and Use: leflunomide

MOA: cytotoxic agent; inhibitor of dihydroorotate dehydrogenase (DHOD) leading to inhibition of pyrimidine synthesis USE: rheumatoid arthritis

MOA and USE: methotrexate

MOA: folate analogue that competitively inhibits dihydrofolate reductase (DHFR) and prevents regeneration of tetrahydrofolate from dihydrofolate USE: many tumor types: carcinomas of breast, lung, head, and neck; acute lymphoblastic leukemia; choriocarcinomaautoimmune disease including psoriasis and rheumatoid arthritis

MOA and USE: belimumab

MOA: human monoclonal antibody directed against B-lymphocyte stimulator (BLyS) cytokine; Blocks BLyS function resulting in B cell apoptosis USE: systemic lupus erythematosus

MOA and USE: eculizumab

MOA: humanized monoclonal antibody against C5 USE: Paroxysmal nocturnal hemoglobinuria, atypical hemolytic uremic syndrome

MOA and USE: Ipilimunab

MOA: humanized monoclonal antibody against CTLA-4; release checkpoint pathway-mediated inhibition of the anti-tumor response USE: unresectable or metastatic melanoma

MOA and Use: azathioprine

MOA: inhibits purine metabolism; inhibits IMPDH and other synthetic enzymes, thereby interfering with AMP and GMP synthesis USE: immunosuppression in renal transplantation; rheumatoid arthritis; used for immunosuppression of autoimmune diseases

MOA and Use: Prednisone

MOA: mimic cortisol function by acting as agnoists at the glucocorticoid receptor USE: inflammatory conditions in many different organs; autoimmune diseases

MOA and USE: natalizumab

MOA: monoclonal antibody against alpha-4 integrin which inhibits immune cell interaction with cells expression VCAM-1 and MAdCAM-1 USE: Multiple Sclerosis; relapsing form Crohn's Disease

MOA and USE: anakinra

MOA: recombinant IL-1 receptor antagonist USE: rheumatoid arthritis, cryopyrin-associated periodic syndromes including neonatal-onset multisystem inflammatory disease

What does interaction of T helper cell CD40l with macrophage CD40 promote?

Macrophage expression of B7-1 and B7-2 This pathway thus provides a positive feedback mechanism whereby activated T cells can promote further expansion of activated T cells In addition, the increased expression of B7-1 and B7-2 on macrophages is important or promoting CD8 T cell activation

What do patients with Subtotal gastrectomy or Bariatric surgery often experience?

Malabsorption

Diagnosis

Malabsorption can be caused by a large number of disorders, and some of the more common of which are listed in Table 33-4. The cause of malabsorption can often be determined by a very detailed patient history. However, because the clinical symptoms are varied, more specific assays of albumin, cobalamin, iron, cholesterol, calcium, folic acid, and prothrombin time are useful to support the diagnosis of malabsorption. These tests are helpful in assessing the severity of malabsorption, but they are not specific for the differential diagnosis. Many tests are available in the work-up of malabsorption; those that have been most useful clinically are discussed in th

What are possible causes of Malabsorption?

Malabsorption can be congenital, caused by membrane transport defects of small bowel enterocytes, or acquired due to extensive damage or resection of the small bowel resulting in decreased absorptive area. Celiac disease, Crohn's disease, and short bowel syndrome after resection can be causes

What is the difference between Maldigestion and Malabsorption?

Maldigestion refers to defective hydrolysis of nutrients Malabsorption refers to impaired mucosal absorption In clinical practice, however, malabsorption refers to all aspects of impaired nutrient assimilation.

How can maldigestion occur?

Maldigestion resulting from lack of pancreatic enzymes (chronic pancreatitis or, occasionally, pancreatic tumor) and lack of bile salts for fat absorption also can occur.

Malrotation and midgut volvulus

Malrotation is incomplete rotation and fixation of the midgut after it has passed from the umbilical sac and returned to the abdominal coelom (Figs. 4.93 and 4.94). The proximal attachment of the small bowel mesentery begins at the suspensory muscle of duodenum (ligament of Treitz), which determines the position of the duodenojejunal junction. The mesentery of the small bowel ends at the level of the ileocecal junction in the right lower quadrant. This long line of fixation of the mesentery prevents accidental twists of the gut. If the duodenojejunal flexure or the cecum does not end up in its usual site, the origin of the small bowel mesentery shortens, which permits twisting of the small bowel around the axis of the superior mesenteric artery. Twisting of the bowel, in general, is termed volvulus. Volvulus of the small bowel may lead to a reduction of blood flow and infarction In some patients, the cecum ends up in the midabdomen. From the cecum and the right side of the colon a series of peritoneal folds (Ladd's bands) develop that extend to the right undersurface of the liver and compress the duodenum. A small bowel volvulus may then occur as well as duodenal obstruction. Emergency surgery may be necessary to divide the bands.

What is TNF-alpha central to? What secretes TNF alpha?

Many aspects of the inflammatory response Macrophages, mast cells, and activated Th cells (especially Th1 cells)

What happens once an infection is cleared adn the stimuli for the lymphocyte disappears?

Many of the cells that had proliferated in response to antigen are deprived of these survival signals and as a result these cells die by apoptosis The response subsides within 1 or 2 weeks after the infection is eradicated, and the only sign that a T cell-mediated immune response had occurred is the pool of surviving memory lymphocytes.

What can Anastomotic connections between arteries supplying the colon can result in the formation of?

Marginal artery courses along the ascending, transverse, and descending parts of the large bowel

What prevents hyperacute rejection?

Matching of blood types between donor and recipients therefore drug therapy for hyperacute rejection is typically not necessary Hyperacute rejection also occurs in xenotransplantation (i.e., organ transplantation between species, such as a pig heart transplanted into a human recipient), due to the presence of preformed human antibodies that react against antigenic proteins and carbohydrates expressed by the donor species

Peripheral B cell tolerance

Mature B lymphocytes that encounter self antigens in peripheral lymphoid tissues become incapable of responding to that antigen (Fig. 9-9). According to one hypothesis, if B cells recognize an antigen and do not receive T cell help (because helper T cells have been eliminated or are tolerant), the B cells become anergic because of a block in signaling from the antigen receptor. Anergic B cells may leave lymphoid follicles and are subsequently excluded from the follicles. These excluded B cells may die because they do not receive necessary survival stimuli. B cells that recognize self antigens in the periphery may also undergo apoptosis, or inhibitory receptors on the B cells may be engaged, thus preventing activation. As mentioned earlier, regulatory T cells may also contribute to B cell tolerance During somatic hypermutation of Ig genes in germinal centers (discussed in Chapter 7), some antigen receptors may be generated that are capable of recognizing self antigens. B cells expressing these autoreactive receptors die either because there are no follicular helper T cells to rescue them or because germinal center B cells express high levels of Fas and are killed by FasL-expressing T cells. The autoimmune disease that results from FAS mutations may in part be caused by survival of these self-reactive germinal center B cells

Where do people with appendicular problems (eg: appendicitis) describe their pain near?

McBurnery's point

What is Hyperacute rejection mediated by? When does it usually occur?

Mediated by preformed recipient antibodies against donor antigens Because these antibodies are present at the time of organ implantation, hyperacute rejection occurs almost immediately after reperfusion of the transplanted organ.

Some of the progeny of the T cells that have proliferated in response to antigen develop into ............ T cells, which are long-lived and functionally inactive, circulate for months?

Memory T cells

What are Eicosanoids?

Metabolites of arachidonic acid, a fatty acid component of phospholipids in the inner leaflet of the plasma membrane of many cell types Multiple biochemical reactions ensue, resulting in the formation of prostaglandins, leukotrienes, and other eicosanoids.

What drug can be used in the induction and maintenance in active Crohn's disease?

Methotrexate

Methotrexate

Methotrexate is a olate analogue used since the 1950s to treat malignancies. Since that time, methotrexate has also become an extremely versatile drug in treating a wide variety o immune-mediated diseases, including rheumatoid arthritis and psoriasis. In addition, methotrexate is used or the prevention o GVHD. The precise mechanism by which methotrexate exerts its anti-in ammatory e ect is uncertain. Although methotrexate inhibits dihydro olate reductase, the combination o methotrexate and low-dose olate is as e ective as methotrexate alone in the treatment o rheumatoid arthritis. (High-dose olinic acid does inter ere with the e f cacy o methotrexate, however.) Methotrexate may also act as an anti-in ammatory agent by increasing adenosine levels. Adenosine is a potent endogenous anti-in ammatory mediator that inhibits neutrophil adhesion, phagocytosis, and superoxide generation. Methotrexate also causes apoptosis o activated CD4 and CD8 T cells but not o resting T cells. Other immunosuppressive agents, including 5- uorouracil, 6-mercaptopurine, and mycophenolic acid, also promote apoptosis o activated T cells. Methotrexate may be such a versatile drug because o its combined antineutrophil, antiT-cell, and antihumoral e ects.

Microscopic colitis

Microscopic colitis encompasses two entities, collagenous colitis and lymphocytic colitis. These idiopathic diseases both present with chronic, nonbloody, watery diarrhea without weight loss. Radiologic and endoscopic studies are typically normal. Collagenous colitis, which occurs primarily in middle-aged and older women, is characterized by the presence of a dense subepithelial collagen layer, increased numbers of intraepithelial lymphocytes, and a mixed inflammatory infiltrate within the lamina propria (Fig. 17-39B). Lymphocytic colitis is histologically similar, but the subepithelial collagen layer is of normal thickness and the increase in intraepithelial lymphocytes is greater, frequently exceeding one T lymphocyte per five colonocytes (Fig. 17-39C). Lym phocytic colitis shows a strong association with celiac disease and autoimmune diseases, including Graves disease, rheumatoid arthritis, and autoimmune or lymphocytic gastritis

What artery is the first of the 3 branches from the right side of the main trunk of the SMA? anastamoses?

Middle colic artery emerges from beneath the pancreas, the middle colic artery enters the transverse mesocolon and divides into right and left branches right branch anastamoses with right colic artery left branch anastamoses with the left colic artery

What are corticosteroids indicated for the treatment of?

Moderate or severe disease in Ulcerative Colitis patients for whom treatment with 5-ASA has failed

What tests are useful to support the diagnosis of malabsorption as well as the severity of malabsorption?

More specific assays of albumin, cobalamin, iron, cholesterol, calcium, folic acid, and prothrombin time These tests are helpful in assessing the severity of malabsorption, but they are not specific for the differential diagnosis.

acute diarrhea

Most cases of acute diarrhea are caused by viral or bacterial infections and are self-limited, resolving without specific therapy. Acute diarrhea in Western countries is common and occurs at a rate of about one episode per person per year. It tends to occur in outbreaks involving food or waterborne contamination or recurrently in specific groups, such as people who care for infants, small children, travelers, and immunocompromised individuals Most of the deaths associated with acute diarrhea occur in elderly persons because of the physiologic changes of aging that include abnormalities in water homeostasis and decreased thirst perception. Because of volume depletion, elderly patients are at increased risk for falls due to orthostatic hypotension, electrolyte disturbances, and delirium Most cases of acute diarrhea are caused by viral infection; studies of acute diarrhea show positive bacterial cultures in only 1.5% to 5.6% of cases. Viruses commonly causing acute infectious diarrhea include noroviruses, rotaviruses, and adenoviruses. The symptoms typically last approximately 48 hours and clear spontaneously. Bacterial causes of acute infectious diarrhea include Salmonella, Campylobacter, Shigella, enterotoxigenic Escheria coli, and Clostridium difficile. These are the likely causes of most severe cases of acute diarrhea. Among patients with diarrhea lasting longer than 3 days and high outputs, a bacterial cause was found in 87% of cases. Protozoa are infrequent causes of acute diarrhea. Because of the short duration of symptoms, good prognosis, and high frequency of viral etiologies found in acute diarrhea, clinical investigation is not needed and is not cost-effective in most cases. The indications warranting a complete evaluation involve clinical signs of severe illness: profuse watery diarrhea and hypovolemia; frequent passage of small-volume, bloody stools containing mucus; bloody diarrhea; fever greater than 101° F; illness lasting longer than 48 hours; severe abdominal pain; hospitalization; recent use of antibiotics; age greater than 70 years; immunocompromise; and systemic illness and diarrhea in pregnancy (listeriosis).

digestion and absorption of carbohydrates

Most dietary carbohydrates consist of starch (a glucose polymer) and the disaccharides sucrose and lactose, but only monosaccharides are absorbed. Salivary and pancreatic amylases release oligosaccharides from starch. The final hydrolysis to glucose monomers occurs at the brush border and includes disaccharide hydrolysis by sucrase and lactase. Glucose and galactose are actively transported in conjunction with sodium, whereas fructose absorption occurs by facilitated diffusion. About one half of dietary energy is derived from carbohydrate, with a nutritional goal of 55% and an increased component of insoluble fiber (i.e., that which is indigestible by mammalian enzymes but variably broken down by colonic bacteria.)

What are the clinical features of Colonic diverticula?

Most individuals with diverticular disease remain asymptomatic throughout their lives about 20% of individuals with diverticuli develop manifestations of diverticular disease, such as intermittent cramping, continuous lower abdominal discomfort, constipation, distention, or a sensation of never being able to completely empty the rectum Even when diverticulitis occurs, it most often resolves spontaneously and relatively few patients require surgical intervention.

evaluation of acute diarrhea

Most patients with acute diarrhea do not require a detailed evaluation because their illness is neither severe nor prolonged. There is significant cost involved in laboratory testing, cultures, and procedures, and because a viral infection is responsible for most cases, the results will be unrevealing. These patients typically have watery, nonbloody diarrhea and are not systemically ill. Many patients with traveler's diarrhea have large-volume watery diarrhea due to enterotoxigenic E. coli. Some patients who have severity indicators in their history or examination findings, or who are elderly or immunocompromised, should have a laboratory evaluation. Patients with bloody, frequent, small-volume stools should be evaluated for potential bacterial causes. The same is true for patients who have systemic symptoms (e.g., abdominal pain, fever), fecal leukocytes, an elevated fecal lactoferrin level (a marker for leukocytes), or occult blood in the stool. Stool cultures should be done to look for Shigella, Samonella, Campylobacter, and enterohemorrhagic E coli. Identification of Listeria, Yersinia, and Vibrio may require additional testing. Stool should be evaluated for ova and parasites if cultures are negative and if there is persistent diarrhea or a history HIV/AIDS. Giardia and Cryptosporidum immunoassays and staining for Microsporidium are also indicated in these individuals and in patients with a possible exposure history. Because of the risk of communityacquired C. difficile infection, testing should be done for the presence of this organism even without a history of antibiotic use Finally, if there are no definitive results from the stool studies, endoscopic imaging with either flexible sigmoidoscopy or colonoscopy may be required to evaluate for IBD, ischemic colitis, or cytomegalovirus colitis in immunocompromised patients. Despite an extensive evaluation, no cause can be determined in 20% to 40% of cases.

What does the Enteric nervous system consist of?

Motor and sensory neurons in two interconnected plexuses in the walls of the gastrointestinal tract.

What does the d-Xylose test serve as an indicator of? what is it used to determine?

Mucosal absorption in the proximal small bowel whether defects in the epithelium of the intestine are responsible for malabsorption

pathology

Mucosal biopsies in IBD reveal acute and chronic inflammation with infiltration by plasma cells, neutrophils, lymphocytes, and eosinophils; focal ulcerations; crypt architectural distortion; and crypt abscesses (Figs. 37-1 and 37-2). In Crohn's disease, the inflammation is transmural and more commonly focal. Granulomas are found in 25% to 30% of histologic specimens in Crohn's disease, but not in UC. The presence of granulomas is not required but can assist in making the diagnosis of Crohn's disease in the right clinical setting (Fig. 37-3). Granulomas are not diagnostic because they can be found in many other diseases, such as Beçhet's disease, tuberculosis, Yersinia infection, and lymphoma.

Mucosal biopsy specimen

Mucosal biopsy specimen demonstrates crypt branching and a crypt abscess characteristic of ulcerative colitis (hematoxylin and eosin stain).

Mucosal Phase

Mucosal disease is a more common cause of malabsorption. It can result from diffuse small intestinal diseases such as celiac disease or Crohn's disease or from a decrease in surface area (e.g., after surgical resection for small bowel infarction). The net effect is a smaller effective mucosal surface and a relative loss of mucosal absorption. Selective defects in an otherwise normal intestine may result in specific entities such as lactase deficiency or abetalipoproteinemia

causes of abdominal pain

Multiple intra-abdominal and extra-abdominal disorders can produce abdominal pain. Distinguishing acute from chronic symptoms is helpful. The approach varies with each specific cause, but acute abdominal pain usually demands prompt intervention.

What does the initiation of T cell response require?

Multiple receptors on the T cells recognizing ligands on Antigen presenting cells (APCs)

Mycophenolic acid and Mycophenolate mofetil

Mycophenolic acid (MPA) is an inhibitor o inosine monophosphate dehydrogenase (IMPDH), the rate-limiting enzyme in the ormation o guanosine (see Fig. 39-3). Because MPA has low oral bioavailability, it is usually administered as a sodium salt or in its prodrug orm, mycophenolate mo etil (MMF) , both o which have greater oral bioavailability (Fig. 46-3). MMF is increasingly used in the treatment o immune-mediated disease because o its high selectivity and pro ound e ect on lymphocytes MPA and MMF both act primarily on lymphocytes. Two main actors contribute to this selectivity. First, as discussed in Chapter 39, lymphocytes are dependent on the de novo pathway o purine synthesis, whereas most other tissues rely heavily on the salvage pathway. Because IMPDH is required or de novo synthesis o guanosine nucleotides but not or the salvage pathway, MPA selectively a ects cells such as lymphocytes that rely on de novo purine synthesis. Second, IMPDH is expressed in two iso orms: type I and type II. MPA pre erentially inhibits type II IMPDH, the iso orm expressed mainly in lymphocytes. Together, these actors con er on MPA and MMF selectivity against T and B cells, with relatively low toxicity to other cells Inhibition o IMPDH by MPA reduces intracellular guanosine levels and increases intracellular adenosine levels, with many downstream e ects on lymphocyte activation and activity. MPA has a cytostatic e ect on lymphocytes and it can also induce apoptosis o activated T cells, leading to the elimination o reactive clones o proli erating cells. Because guanosine is required or some glycosylation reactions, the reduction in guanosine nucleotides leads to decreased expression o adhesion molecules that are required or recruitment o several immune cell types to sites o in ammation. Furthermore, because guanosine is a precursor o tetrahydrobiopterin (BH4), which regulates inducible nitric oxide synthase (iNOS), the reduction in guanosine levels leads to decreased NO production by neutrophils. Endothelial NOS (eNOS), which controls vascular tone and is regulated by Ca 2 and calmodulin, is not a ected by changes in guanosine levels, again demonstrating the considerable selectivity of MPA. As noted above, clinical studies comparing MMF and AZA have shown MMF to be more e f cacious in preventing acute rejection o kidney transplants. Animal models show that chronic rejection is also reduced more e ectively in recipients treated with MMF than in those treated with AZA or cyclosporine. The e f cacy o MMF in treating chronic rejection may be related to its inhibition o both the lymphocyte and the smooth muscle cell proli eration characteristic o chronic rejection. MMF is also e f cacious in the treatment o autoimmune disease. In rheumatoid arthritis, levels o rheumatoid actor, immunoglobulin, and T cells are all reduced by treatment with MMF. MMF is requently used in the initial therapy o lupus nephritis. There have also been isolated reports o success ul treatment o myasthenia gravis, psoriasis, autoimmune hemolytic anemia, and in ammatory bowel disease with MMF. The most common adverse e ect o MMF is gastrointestinal discom ort, which is dose-dependent and can include nausea, diarrhea, so t stools, anorexia, and vomiting

What gene is most strongly associated with Crohn disease? What does it encode?

NOD2 (nucleotide oligomerization binding domain 2) an intracellular protein that binds to bacterial peptidoglycans and activates signaling events, including the NF-κB pathway fewer than 10% of individuals carrying risk associated NOD2 variants develop disease

Why don't Naive T cells migrate into the sites of infection or tissue injury?

Naive T cells do not express ligands for E- or P-selectin and do not express receptors for chemokines produced at inflammatory sites Therefore, naive T cells do not migrate into sites of infection or tissue injury.

Phases of T cell responses

Naive T lymphocytes recognize antigens in the peripheral (secondary) lymphoid organs, which initiates proliferation of the T cells and their differentiation into effector and memory cells, and the effector cells perform their functions when they are activated by the same antigens in peripheral tissues or lymphoid organs (Fig. 5-2). Naive T cells express antigen receptors and co-receptors that function in recognizing cells harboring microbes, but these cells are incapable of performing the effector functions required for eliminating the microbes. Differentiated effector cells are capable of performing these functions, which they do in lymphoid organs and in peripheral, nonlymphoid tissues. In this chapter we focus on the responses of naive T cells to antigens. The development of effector T lymphocytes and their functions in cell-mediated immunity are described in Chapter 6, and the roles of helper T cells in antibody responses in Chapter 7. The responses of naive T lymphocytes to cell-associated microbial antigens consist of a series of sequential steps that result in an increase in the number of antigen-specific T cells and the conversion of naive T cells to effector and memory cells (Fig. 5-3). • One of the earliest responses is the secretion of cytokines and increased expression of receptors for various cytokines. • Some cytokines stimulate the proliferation of the antigen-activated T cells, resulting in a rapid increase in the number of antigen-specific lymphocytes, a process called clonal expansion. • The activated lymphocytes undergo the process of differentiation, which results in the conversion of naive T cells into a population of effector T cells which function to eliminate microbes. • Many of the effector T cells leave the lymphoid organs, enter the circulation, and migrate to any site of infection, where they can eradicate the infection. Some effector T cells may remain in the lymph node, where they function to eradicate infected cells at that site or provide signals to B cells that promote antibody responses against the microbes. • Some of the progeny of the T cells that have proliferated in response to antigen develop into memory T cells, which are long-lived and functionally inactive, circulate for months or years, and are ready to respond rapidly to repeated exposure to the same microbe. • As effector T cells eliminate the infectious agent, the stimuli that triggered T cell expansion and differentiation also are eliminated. As a result, most of the cells in the greatly expanded clone of antigen-specific lymphocytes die, returning the system to a resting state, with only memory cells remaining from the immune response. This sequence of events is common to both CD4+ and CD8+ T lymphocytes, although there are important differences in the properties and effector functions of CD4+ and CD8+ cells, as discussed in Chapter 6. Naive and effector T cells have different patterns of circulation and migration through tissues, which are critical for their different roles in immune responses. As discussed in previous chapters, naive T lymphocytes constantly recirculate through peripheral lymphoid organs searching for foreign protein antigens. The antigens of microbes are transported from the portals of entry of the microbes to the same regions of peripheral lymphoid organs through which naive T cells recirculate. In these organs, the antigens are processed and displayed by MHC molecules on dendritic cells, the antigenpresenting cells (APCs) that are the most efficient stimulators of naive T cells (see Chapter 3). When a T cell recognizes antigen, it is transiently arrested on the dendritic cell and it initiates an activation program. Following activation and differentiation, the cells may leave the lymphoid organ and migrate preferentially to the inflamed tissue, the original source of the antigen. The control of this directed migration is discussed later in this chapter. With this overview, we proceed to a description of the stimuli required for T cell activation and regulation. We then describe the biochemical signals that are generated by antigen recognition and the biologic responses of the lymphocytes.

What is the appendix?

Narrow, hollow, blind-ended tube connected to the cecum has large aggregations of lymphoid tissue in its walls and is suspended from the terminal ileum by the mesoappendix

What can Tacrolimus cause?

Nephrotoxicity Can also cause new-onset diabetes mellitus in post-transplant patients

What are the adverse effects associated with the use of Cyclosporine?

Nephrotoxicity, hypertension, infection, neurotoxicity, and hepatotoxicity

What does the Aortic plexus consist of? What is the major ganglion in the aortic plexus?

Nerve fibers and associated ganglia on the anterior and lateral surfaces of the abdominal aorta extending from just below the origin of the superior mesenteric artery to the bifurcation of the aorta into the two common iliac arteries Inferior mesenteric ganglion at the root of the inferior mesenteric artery

What is the most abundant cell type of the immune system and is considered first responders? What are they primarily responsible for?

Neutrophils Are phagocytic cells primarily responsible for defense against bacterial infection These cells envelop invading bacteria in phagocytic vesicles and destroy the bacteria within these vesicles using enzymes such as myeloperoxidase

IL-12/IL-23p40 cytokine inhibitors

New biological therapies or the treatment o T-cell-mediated diseases include antibodies to IL-12 and IL-23. IL-12 and IL-23 are cytokines involved in natural killer cell activation and CD4 T-cell di erentiation and activation. IL-12, a heterodimer composed o p40 and p35 subunits, directs the di erentiation o naïve T cells into T H 1 cells, which secrete IL-2, IFN- , and TNF- . IL-23 is also a heterodimer that has the same p40 subunit covalently linked to a p19 subunit. IL-23 directs the di erentiation o naïve T cells into T H 17 cells, which secrete IL-17 and IL-22. Ustekinumab is a high-a f nity human IgG1 monoclonal antibody that binds to the p40 subunit shared by IL-12 and IL-23. Ustekinumab is approved or use in psoriasis and psoriatic arthritis and is in clinical trials or the treatment o Crohn's disease, sarcoidosis, and multiple sclerosis. Adverse e ects include an increased risk of infection

PD-1 blockade

Nivolumab and pembrolizumab are recombinant, humanized IgG4 monoclonal antibodies that bind to PD-1 and block the interaction o PD-1 with its ligands PD-L1 and PD-L2. By inhibiting the inhibition o T-cell proli eration and cytokine production that results rom binding o PD-1 to PD-L1 and PD-L2, blockade o PD-1 by nivolumab and pembrolizumab releases PD-1 pathway-mediated inhibition o the immune response, including the anti-tumor immune response. In appropriate mouse tumor models, blocking PD-1 activity results in decreased tumor growth. Nivolumab and pembrolizumab are indicated or the treatment o unresectable or metastatic melanoma with disease progression ollowing treatment with ipilimumab and, i the tumor is positive or a B-RAF V600 mutation, a B-RAF inhibitor. These therapies are also approved or metastatic non-small cell lung cancers that express PD-L1 and show disease progression on or a ter platinum-containing chemotherapy. For non-small cell lung cancer with sensitizing EGFR mutations or ALK rearrangements, disease progression on therapies targeted or these genomic tumor aberrations should be demonstrated prior to treatment with nivolumab or pembrolizumab

Is there a role for Barium studies in the evaluation of acute GI hemorrhage?

No

Do patients with Arteriosclerosis always suffer symptoms? If no then why not?

No because anastomoses between the right, middle, and left colic arteries gradually enlarge, forming a continuous marginal artery The distal large bowel therefore becomes supplied by this enlarged marginal artery (marginal artery of Drummond), which replaces the blood supply of the inferior mesenteric artery

Is Endothelial NOS (eNOS) which controls vascular tone and is regulated by Ca2+ and Calmodulin affected by changes in Guanosine levels?

No this demonstrates the considerable selectivity of MPA

What is Wireless capsule endoscopy?

Noninvasive method that permits direct visualization of the small bowel mucosa and can provide a more detailed evaluation of small bowel disease compared with radiographic studies

What does Belimumab block? What does that result in?

Normal functioning of BLys B-cell apoptosis

When Heliobacter pylori is found in a patient with dyspeptic symptoms will its eradication always lead to resolution of the symptoms?

Not necessarily

What does the splenic artery give off as it passes along the superior border of the pancreas?

Numerous small branches to supply the neck, body, and tail of the pancreas

monoclonal antibodies -OKT3

OKT3 (muromonab-CD3, anti-CD3) is a mouse monoclonal antibody directed against human CD3, one o the cell sur ace signaling molecules important or activation o the T-cell receptor. CD3 is specif cally expressed on T cells (both CD4 and CD8 cells). Treatment with OKT3 depletes the available pool o T cells via antibody-mediated activation o complement and clearance o immune cells. Because OKT3 targets all T cells, OKT3 treatment can result in pro ound immunosuppression. Also, because OKT3 binds to CD3, and CD3 is important or T-cell activation, OKT3 therapy can broadly activate T cells, resulting in the cytokine release syndrome. Another limitation is that OKT3 is a mouse antihuman antibody (see above). OKT3 was approved or use in acute renal transplant rejection but was subsequently (in 2010) withdrawn voluntarily rom the market by the manu acturer

What may a patient who is withering in bed and unable to find a comfortable position be suffering from?

Obstruction

What does the presence of hyperactive, high pitched sounds suggest?

Obstruction Multiple bruits alert the examiner to the possibility of significant vascular disease, suggesting ischemia

What does Abdominal distention indicate?

Obstruction or Ascites

What do Salivary and Pancreatic amylases release?

Oligosaccharides from Starch

What can be seen on a traditional small bowel radiography?

On traditional small bowel radiography, segments of edematous bowel appear thickened next to uninvolved mucosa, a characteristic pattern referred to as cobblestoning. Tight, long strictures in the small bowel can be identified and are called a string sign .rosssectional imaging with computed tomographic (CT) enterography and magnetic resonance enterography has replaced traditional small bowel radiography

chemotaxis

Once the cells o the immune system cross the endothelial barrier, they migrate through the interstitium to the speci c site o injury or in ection. Immune cell targeting is accomplished by the process o chemotaxis , or chemical signaling. Inf ammatory mediators released at the site o injury, such as N- ormyl peptides derived rom bacterial proteins or endogenous mediators such as C5a and leukotriene B 4 (LTB 4 ), create a chemical gradient to which the leukocytes respond, allowing them to crawl pre erentially toward the site o the inf ammatory reaction.

Colitis associated neoplasia

One of the most feared long-term complications of ulcerative colitis and colonic Crohn disease is the development of neoplasia. The risk of dysplasia is related to several factors: • Duration of the disease. Risk increases sharply 8 to 10 years after disease onset. • Extent of the disease. Patients with pancolitis are at greater risk than those with only left-sided disease. • Nature of the inflammatory response. Greater frequency and severity of active inflammation (characterized by the presence of neutrophils) confers increased risk. To facilitate early detection of neoplasia, patients are typically enrolled in surveillance programs approximately 8 years after diagnosis of IBD. The major exception to this is patients with IBD and primary sclerosing cholangitis, who have an even greater risk of developing cancer and are generally enrolled for surveillance at the time of diagnosis. Surveillance requires regular and extensive mucosal biopsies, making it a costly practice. Research efforts are therefore focused on discovery of molecular markers of dysplasia The goal of surveillance biopsies is to identify dysplastic epithelium, which is a precursor to colitisassociated carcinoma. Dysplasia can develop in flat areas of mucosa that are not grossly recognized as abnormal. Thus, advanced endoscopic imaging techniques including chromoendoscopy and confocal endoscopy are beginning to be used to increase the sensitivity of detection. IBDassociated dysplasia is classified histologically as low grade or high grade (Fig. 17-38A, B) and may be multifocal. High-grade dysplasia may be associated with invasive carcinoma at the same site (Fig. 17-38C) or elsewhere in the colon and, therefore, often prompts colectomy. Lowgrade dysplasia may be treated with colectomy or followed closely, depending on a variety of factors including patient age and the number of dysplastic foci present. Colonic adenomas (discussed later) also occur in IBD patients, and in some cases these may be difficult to differentiate from a polypoid focus of IBD-associated dysplasia

Intro

One of the remarkable properties of the normal immune system is that it can react to an enormous variety of microbes but does not react against the individual's own (self) antigens. This unresponsiveness to self antigens, also called immunological tolerance, is maintained despite the fact that the molecular mechanisms by which lymphocyte receptor specificities are generated are not biased to exclude receptors for self antigens. In other words, lymphocytes with the ability to recognize self antigens are constantly being generated during the normal process of lymphocyte maturation. Furthermore, many self antigens have ready access to the immune system, so unresponsiveness to these antigens cannot be maintained simply by concealing them from lymphocytes. It follows that there must exist mechanisms that prevent immune responses to self antigens. These mechanisms are responsible for one of the cardinal features of the immune system—namely, its ability to discriminate between self and nonself (usually microbial) antigens. If these mechanisms fail, the immune system may attack the individual's own cells and tissues. Such reactions are called autoimmunity, and the diseases they cause are called autoimmune diseases. In addition to tolerating the presence of self antigens, the immune system has to coexist with many commensal microbes that live within their human hosts, often in a state of symbiosis, and the immune system of a pregnant female has to accept the presence of a fetus that expresses antigens derived from the father. Unresponsiveness to commensal microbes and the fetus is maintained by many of the same mechanisms involved in unresponsiveness to self.

Foreign cells must be coated with what before they can be ingested(phagocytosed) by leukocytes?

Opsonin opsonins are molecular adaptors that coat foreign surfaces and signal leukocytes that a particle should be attacked.

What does pain location often indicate?

Organ responsible for the problem eg: Epigastric pain is usually typical of peptic ulcer or dyspepsia, whereas right upper quadrant pain eg: upper right shoulder or back is more suggestive of cholecystitis and other biliary disorders

How is the osmotic gap calculated?

Osmotic gap Plasma osmolality- [2 x (fecal[Na+] + fecal [K+] The osmotic gap is not calculated by directly measuring stool osmolality because it increases with time in the specimen container.

other agents

Other agents for treatment of IBD include antibiotics, probiotics, antidiarrheals, bile salt resin binders, and nutritional support. Antibiotics are used primarily in patients with Crohn's disease who have perianal or fistulizing disease. In colonic Crohn's disease, antibiotics can be used in combination with immunosuppressive drugs as an alternate strategy. The role of antibiotics in UC is unclear, and further studies are required. However, intravenous antibiotics are used in the initial treatment of severe, toxic, or fulminant colitis Probiotics are viable nonpathogenic organisms that, after ingestion, may prevent or treat intestinal diseases and have been explored in the treatment of IBD. Multiple studies have found them to be effective in treating pouchitis after ileal pouch-anal anastomosis. One probiotic mixture, termed VSL#3, includes four strains of Lactobacillus (L. paracasei, L. plantarum, L. acidophilus, and L. delbrueckii subsp bulgaricus), three strains of Bifidobacteria (B. longum, B. breve, and B. infantis), and one strain of Streptococcus (S. thermophiles). It has been shown in randomized clinical trials to be a safe and effective adjunct modality for achieving clinical response and remission in patients with mild to moderately active UC. So far, none of the probiotics has been shown to be effective in induction or maintenance of remission in patients with Crohn's disease Antidiarrheal agents and bile salt resin binders can be used as adjuncts for management of diarrhea in patients with IBD, but antidiarrheal agents should be used cautiously during exacerbations of colitis because they can precipitate toxic megacolon. The main role of antidiarrheal medications involves controlling diarrhea in patients who have undergone previous resections. Patients with Crohn's disease who have had less than 100 cm of terminal ileum removed can develop a bile salt malabsorptive state, during which bile salts enter the colon and cause a secretory diarrhea. Bile salt resin binders such as cholestyramine are an effective treatment in these cases. When patients have undergone one or more extensive resections amounting to more than 100 cm of ileum, the bile salt pool is depleted and fat malabsorption develops. These patients may require a low-fat diet supplemented with medium-chain triglycerides and antidiarrheal agents, but bile salt resin binders should not be used Nutritional support is an important adjunctive aspect in the management of IBD. However, the role of nutrition as a primary treatment has been limited to patients with small bowel Crohn's disease. These patients may achieve and maintain remission with total parenteral nutrition or elemental diets after prolonged periods (at least 4 weeks). Many patients with Crohn's disease or UC experience weight loss during exacerbations of their illness and need caloric supplements. Vitamins and minerals can be given orally as a multivitamin with folic acid. Vitamin B12 should be supplemented parenterally in patients who have extensive ileal disease or an ileal resection. Patients taking corticosteroids require supplemental calcium and vitamin D, and individuals with extensive small bowel involvement can also develop malabsorption of fat-soluble vitamins (A, D, E, and K), iron, and, rarely, trace minerals. Lactose-free diets and lowfiber diets may be necessary in patients with active disease or strictures

What is the ability of Macrophages to envelop and destroy pathogens enhanced by?

Other components of the immune system including: 1- Antibodies and complement (which mediate opsonization) 2- Cytokines (which enhance killing ability) 3- macrophages produce cytokines like TNF-alpha that modify immune responses

What does the dyregulation of the homeostatic balance or immune tolerance result in, in IBD?

Overactivation of the immune system

What does Severe ulcerative colitis with more than 10 bloody stools per day usually indicate?

Pancolitis severe cramping pain, fever, leukocytosis, and anemia, which often requires transfusion, also there can be rapid weight loss leading to malnutrition approximately one third of these patients have rectosigmoid disease, one third have left-sided disease, and the remainder have pancolitis. Fulminant disease is present in about 10% of patients at presentation

What test indicates the presence of Chronic pancreatitis?

Pancreatic calcifications seen on abdominal films or computed tomography (CT) scans Magnetic resonance cholangiopancreatography (MRCP) and endoscopic retrograde cholangiopancreatography (ERCP) can help outline abnormal duct anatomy and may supplement CT scanning for diagnostic purposes to evaluate the sequelae of chronic pancreatitis.

What type of innervation does the Vagus nerve provide and to where?

Parasympatethetic innervation to the foregut and midgut

parasympathetic innervation

Parasympathetic innervation of the abdominal part of the gastrointestinal tract and of the spleen, pancreas, gallbladder, and liver is from two sources—the vagus nerves [X] and the pelvic splanchnic nerves

Where do Sacral splanchnic nerves pass from and to?

Pass from the sacral part of the sympathetic trunk or associated ganglia enter the inferior hypogastric plexus, which is an extension of the prevertebral plexus into the pelvis

What may recognition of self antigens trigger?

Pathways of apoptosis that result in elimination (deletion) of the self-reactive lymphocytes

What are antibiotics used in the treatment of?

Patients with Crohn's disease who have perianal or fistulizing disease In colonic Crohn's disease, antibiotics can be used in combination with immunosuppressive drugs as an alternate strategy Intravenous antibiotics are used in the initial treatment of severe, toxic, or fulminant colitis.

intro

Patients with autoimmune disease and patients who have received transplanted tissues or organs typically require therapy with immunosuppressive drugs. Immunosuppressive agents have been in use or more than 50 years, beginning with corticosteroids, antimetabolites, and alkylating agents. These early agents assisted in the treatment o previously incurable conditions, but their lack o specif city led to many serious adverse e ects. Over the past 20 years, the f eld o immunosuppression has shi ted to specif c inhibitors o immunity that a ect distinct immune pathways. This shi t is important both because o the greater e f cacy and reduced toxicity o these agents and because, as the mechanisms o these agents are discovered, insights are gained into the operation o the immune system

What are the only splanchnic nerves that carry parasympathetic fibers?

Pelvic splanchnic nerves

What can occur in patients with severe or fulminant colitis?

Perforation especially those taking corticosteroids, and in the setting of toxic megacolon.

Ingestion of what type of toxins typically cause diarrhea within 6 hours?

Performed bacterial toxins Staphylococcus aureus, Bacillus cereus, or Clostridium perfringens

peripheral t lymphocyte tolerance

Peripheral tolerance is induced when mature T cells recognize self antigens in peripheral tissues, leading to functional inactivation (anergy) or death, or when the self-reactive lymphocytes are suppressed by regulatory T cells (Fig. 9-3). Each of these mechanisms of peripheral T cell tolerance is described in this section. Peripheral tolerance is clearly important for preventing T cell responses to self antigens that are not present in the thymus, and it also may provide backup mechanisms for preventing autoimmunity in situations where central tolerance is incomplete Antigen recognition without adequate costimulation results in T cell anergy or death, or makes T cells sensitive to suppression by regulatory T cells. As noted in previous chapters, naive T lymphocytes need at least two signals to induce their proliferation and differentiation into effector and memory cells: Signal 1 is always antigen, and signal 2 is provided by costimulators that are expressed on antigen-presenting cells (APCs), typically as part of the innate immune response to microbes (or to damaged host cells) (see Chapter 5, Fig. 5-6). It is believed that dendritic cells in normal uninfected tissues and peripheral lymphoid organs are in a resting (or immature) state, in which they express little or no costimulators, such as B7 proteins (see Chapter 5). These dendritic cells may constantly process and display the self antigens that are present in the tissues. T lymphocytes with receptors for the self antigens are able to recognize the antigens and thus receive signals from their antigen receptors (signal 1), but the T cells do not receive strong costimulation because there is no accompanying innate immune response. The presence or absence of costimulation is a major factor determining whether T cells are activated or tolerized. Some examples illustrating this concept are discussed below.

What does pain or percussion of the abdomen indicate?

Peritoneal reaction, as does severe rebound tenderness.

What may a patient who is lying with lower extremities flexed and avoiding any motions be suffering from?

Peritonitis because movement makes peritoneal pain worse

What do CD4 helper T cells activate through the action of plasma membrane proteins and secreted cytokines?

Phagocytes and B lymphocytes

pharmacologic classes and agents

Pharmacologic suppression o the immune system utilizes eight mechanistic approaches 1. Inhibition o gene expression to modulate inflammatory responses 2. Depletion o expanding lymphocyte populations with cytotoxic agents 3. Inhibition o lymphocyte signaling to block activation o lymphocytes and expansion o lymphocyte populations 4. Neutralization o cytokines and cytokine receptors essential or mediating the immune response 5. Depletion o specif c immune cells, usually via cellspecific antibodies 6. Blockade o costimulation to induce anergy 7. Blockade o cell adhesion to prevent migration and homing o in ammatory cells 8. Inhibition o innate immunity, including complement activation

What plays a key role in the exit of T cells from Lymph nodes?

Phospholipid sphinogsine 1-phosphate (S1P) The levels of S1P are higher in the blood and lymph than inside lymph nodes S1P binds to and thereby reduces expression of its receptor, which keeps the expression of the receptor on circulating naive T cells low.

What is the function of phosphatidylinositol-3 (PI-3) kinase?

Phosphorylates the membrane phospholipid PIP2 to generate PIP3

What gene is implicated in IBD?

Polymorphisms in the NOD2 gene (previously known as CARD15), located on chromosome 16 were the first definitive genetic risk factors identified for Crohn's disease. - Homozygous mutations of the NOD2 gene are associated with a greater than 20-fold increase in susceptibility for Crohn's disease -Defects in the NOD2 protein appear to result in abnormal intestinal immune responses to bacterial cell wall components -These gene mutations are estimated to account for 15% to 25% of the cases of Crohn's disease and are linked predominantly to fibrostenotic terminal ileal disease

The Venous drainage of the spleen, pancreas, gallbladder, and abdominal part of the gastrointestinal tract, except for the inferior part of the rectum is through what?

Portal system of the veins which deliver blood from these structures to the liver Once blood passes through the hepatic sinusoids, it passes through progressively larger veins until it enters the hepatic veins,which return the venous blood to the inferior vena cava just inferior to the diaphragm.

What does the Gastroduodenal artery give rise to?

Posterior superior pancreaticoduodenal artery near the upper border of the superior part of the duodenum may give off the supraduodenal artery After these branch the gastroduodenal artery continues descending posterior to the superior part of the duodenum

What is the appendix attached to?

Posteromedial wall of the cecum just inferior to the end of the ileum

Where does the Enteric system receive inpt from?

Postganglionic sympathetic and preganglionic parasympathetic neurons that modifies its activities

What is the most commonly used oral Corticosteroid?

Prednisone Patients who do not improve after 1 week of oral treatment and those with more severe disease are best treated in the hospital with intravenous corticosteroids, such as intravenous hydrocortisone (300 mg/day), or methylprednisolone (which can be given either by continuous infusion or in three divided doses).

What is one of the hallmarks of Chronic inflammation as compared to acute inflammation?

Predominance of Macrophages while in acute inflammation neutrophils is predominant

What do the abdominal prevertebral plexus branches contain?

Preganglionic parasympathetic fibers and visceral afferent fibers, which are distributed with the other components of the prevertebral plexus along the branches of the abdominal aorta

What do pelvic splanchnic nerves carry? Where do they enter?

Preganglionic parasympathetic fibers from S2 to S4 spinal cord levels inferior hypogastric plexus in the pelvis Some of these fibers move upward into the inferior mesenteric part of the prevertebral plexus in the abdomen Once there, these fibers are distributed with branches of the inferior mesenteric artery and provide parasympathetic innervation to the hindgut.

Where does Ulcerative colitis occur? Common symptoms?

Primarily in the Colon Typically, if the inflammation is limited to the rectum or rectosigmoid region, the symptoms are relatively mild Urgency, mild cramping, and a sensation of incomplete evacuation (tenesmus) are commonly reported, and constipation may also be a complaint More serious symptoms are often associated with greater involvement of the colon (left-sided colitis or sometimes pancolitis). Patients have frequent loose, bloody stools (up to 10 per day) with mucus Mild anemia and leukocytosis, mild-to-moderate cramping abdominal pain, and low-grade fever may be present. weight loss is not common in mild disease

combination procedure

Probably the most popular procedure currently in the United States is gastric bypass surgery. This procedure involves stapling the proximal stomach and joining a loop of small bowel to the small gastric remnant. The procedure is usually performed by fashioning a Roux-en-Y loop with alimentary and pancreaticobiliary limbs The other type of the procedure, sleeve gastrectomy, is increasing in popularity because it can be used in patients deemed to be at high risk for gastric bypass surgery. It involves reduction of the gastric lumen by removing a large portion of the stomach along the greater curvature Any overweight patient undergoing surgery faces significant risk and increased morbidity, with mortality rates from 1% to 5%

What is the function of antigen presenting cells (APCs)?

Process the macromolecules (especially proteins) of an invading agent to display the processed fragments on the surface of the APC In this form, the fragments serve as molecular fingerprints used by cells of the adaptive immune system to recognize the invading agent

What does Episcleritis produce?

Produces burning eyes and scleral injection without vision deficits is treated with topical steroids.

What are examples of antigens that are not self but are produced by cells or tissues that have to be tolerated by the immune system?

Products of commensal microbes

Since CD20 is expressed on the surface of all mature B cells what does administration of Rituximab cause?

Profound depletion of circulating B cells

The overall effect of glucocorticoid administration is?

Profoundly anti-inflammatory and immunosuppressive explaining the use of glucocorticoids or the treatment of numerous inlammatory diseases such as rheumatoid arthritis and transplant rejection

What protein antigens are displayed by MHC II molecules?

Protein antigens are ingested by APCs from the extracellular milieu into vesicles, these antigens are processed into peptides that are displayed by class II MHC molecule

Any disorder of the liver (including cirrhosis) can decrease the production of what proteins in particular? What can it prevent?

Proteins involved in the clotting cascade prevent adequate blood clotting

What do people with long lasting Ulcerative colitis have?

Pseudopolyps which represent islands of normal tissue in regions of previous ulceration

What does the portal vein divide into on approaching the liver?

RIght and left branches which enter the liver parenchyma

How is the Small intestine initially evaluated?

Radiographically The patient may ingest barium, which is followed through the length of the small intestine To distend the small bowel and provide greater mucosal detail, an enteroclysis tube may be placed with its distal tip near the ligament of Treitz, allowing more forceful administration of barium and air

What are the range of severity of Graft vs host disease? What areas does it involve?

Ranges from mild to life-threatening and typically involves the skin (rash), gastrointestinal tract (diarrhea), lungs (pneumonitis), and liver (veno-occlusive disease).

What has Natalizumab been linked to?

Rare cases of progressive multifocal leukoencephalopathy caused by the human JC virus

What are the potential side effects of anti-TNF agents?

Reactivation of latent tuberculosis and other serious infections Other rare but serious complications include nonHodgkin's lymphoma, exacerbation of congestive heart failure, abnormal complete blood count (CBC) and liver function test results, and demyelinating disease

What is the ultimate cure in Diversion colitis?

Reanastomosis of the diverted segment enemas containing short-chain fatty acids, a product of bacterial digestion in the colon and an important energy source for colonic epithelial cells, can promote mucosal recovery in some cases

What occurs in acute humoral rejection?

Recipient B cells become sensitized to donor antigens in the transplanted organ and produce antibodies against these alloantigens after a period of 7-10 day The antibody response is typically directed against endothelial cells and is thus also known as acute vascular rejection Acute humoral rejection (like acute cellular rejection) can be prevented by immunosuppression

What is the activation of CD8 T cells stimulated by? What does it require ?

Recognition of MHC I peptides Costimulation and helper T cells

Deletion: apoptosis of mature lymphocytes

Recognition of self antigens may trigger pathways of apoptosis that result in elimination (deletion) of the self-reactive lymphocytes (Fig. 9-6). There are two likely mechanisms of death of mature T lymphocytes induced by self antigens: • Antigen recognition induces the production of pro-apoptotic proteins in T cells that induce cell death by causing mitochondrial proteins to leak out and activate caspases, cytosolic enzymes that induce apoptosis. In normal immune responses, the activity of these pro-apoptotic proteins is counteracted by anti-apoptotic proteins that are induced by costimulation and by growth factors produced during the responses. However, self antigens, which are recognized without strong costimulation, do not stimulate production of anti-apoptotic proteins, and the relative deficiency of survival signals induces death of the cells that recognize these antigens. • Recognition of self antigens may lead to the coexpression of death receptors and their ligands. This ligand-receptor interaction generates signals through the death receptor that culminate in the activation of caspases and apoptosis. The best-defined death receptor- ligand pair involved in self-tolerance is a protein called Fas (CD95), which is expressed on many cell types, and Fas ligand (FasL), which is expressed mainly on activated T cells. The Fas pathway may also be involved in death of some B cells in germinal centers, discussed later. Evidence from genetic studies supports the role of apoptosis in self-tolerance. Eliminating the mitochondrial pathway of apoptosis in mice results in a failure of deletion of self-reactive T cells in the thymus and also in peripheral tissues. Mice with mutations in the fas and fasl genes and children with mutations in FAS all develop autoimmune diseases with lymphocyte accumulation. Children with mutations in the genes encoding caspase-8 or -10, which are downstream of FAS signaling, also have similar autoimmune diseases. The human diseases, collectively called the autoimmune lymphoproliferative syndrome, are rare and are the only known examples of defects in apoptosis causing a complex autoimmune phenotype From this discussion of the mechanisms of T cell tolerance, it should be clear that self antigens differ from foreign microbial antigens in several ways, which contribute to the choice between tolerance induced by the former and activation by the latter (Fig. 9-7). • Self antigens are present in the thymus, where they induce deletion and generate regulatory T cells; by contrast, most microbial antigens tend to be excluded from the thymus, because they are typically captured from their sites of entry and transported into peripheral lymphoid organs (see Chapter 3). • Self antigens are displayed by resting APCs in the absence of innate immunity and second signals, thus favoring the induction of T cell anergy or death, or suppression by regulatory T cells. By contrast, microbes elicit innate immune reactions, leading to the expression of costimulators and cytokines that promote T cell proliferation and differentiation into effector cells. • Self antigens are present throughout life and may therefore cause prolonged or repeated TCR engagement, again promoting anergy, apoptosis, and the development of regulatory T cells.

What is the of the antigen receptors and co-receptors expressed by naive T cells?

Recognizing cells on microbes but these cells are incapable of performing the effector functions required for eliminating the microbes

What is Anakinra? What is it approved for use in?

Recombinant form of IL-1ra recombinant IL-1 receptor agonist approved for use in Rheumatoid arthritis

What is Ipilimumab?

Recombinant humanized IgG1 monoclonal antibody binds to CTLA-4 blocks the interaction of CTLA-4 with its ligands B7-1 and B7-2

What is the function of IL-17?

Recruit neutrophils and amplify the immune response

What extends from the sigmoid colon?

Rectum rectosigmoid junction is usually described as being at the level of vertebra S3 or at the end of the sigmoid mesocolon because the rectum is a retroperitoneal structure

Where does the Inferior mesenteric vein drain blood from?

Rectum, sigmoid colon, descending colon, and splenic flexure

What are the criteria for IBS?

Recurrent abdominal pain or discomfort occurring on at least 3 days per month in the last 3 months and associated with two or more of following according to Rome Criteria III: (1) improvement of pain with bowel movement (2) onset associated with a change in frequency of stool` (3) onset associated with a change in form or consistency of stool.

What does inhibition of IMPDH by Mycophenolic acid (MPA) reduce? what does it increase?

Reduces intracellular Guanosine levels Increases intracellular Adenosine levels

What can Volvulus of the small intestine lead to?

Reduction of blood flow and infarction

Helper T cells (Th) are primarily the ......... of adaptive immunity?

Regulators

What subset of T cells maintain peripheral tolerance?

Regulatory T cells (T reg) the best characterized o which are CD4 CD25, elaborate inhibitory cytokines in response to self-antigens and thereby limit the immune response to these antigens Pharmacologic induction of T reg cells may have applications in transplantation and autoimmune diseases, including type 1 diabetes

immune suppression by regulatory t cells

Regulatory T cells develop in the thymus or peripheral tissues on recognition of self antigens and suppress the activation of potentially harmful lymphocytes specific for these self antigens (Fig. 9-5). The majority of self-reactive regulatory T cells probably develop in the thymus (see Fig. 9-2), but they may also arise in peripheral lymphoid organs. Most regulatory T cells are CD4+ and express high levels of CD25, the α chain of the interleukin-2 (IL-2) receptor. They also express a transcription factor called FoxP3, which is required for the development and function of the cells. Mutations of the gene encoding FoxP3 in humans or in mice cause a systemic, multiorgan autoimmune disease, demonstrating the importance of FoxP3+ regulatory T cells for the maintenance of self-tolerance. The human disease is known by the acronym IPEX, for immune dysregulation, polyendocrinopathy, enteropathy, X-linked syndrome. The severe autoimmunity seen in FoxP3-deficient mice and humans is the best evidence for the importance of regulatory T cells in maintaining self-tolerance. The survival and function of regulatory T cells are dependent on the cytokine IL-2. This role of IL-2 accounts for the severe autoimmune disease that develops in mice in which IL-2 or IL-2 receptor genes are deleted. Recall that we introduced IL-2 in Chapter 5 as a cytokine made by antigen-activated T cells that stimulates proliferation of these cells. Thus, IL-2 is an example of a cytokine that serves two opposite roles: it promotes immune responses by stimulating T cell proliferation, and it inhibits immune responses by maintaining functional regulatory T cells. Numerous clinical trials are testing the ability of IL-2 to promote regulation and control harmful immune reactions, such as graft rejection and inflammation in autoimmune diseases. The cytokine transforming growth factor β (TGF-β) also plays a role in the generation of regulatory T cells, perhaps by stimulating expression of the FoxP3 transcription factor. Many cell types can produce TGF-β, but the source of TGF-β for inducing regulatory T cells in the thymus or peripheral tissues is not defined. Regulatory T cells may suppress immune responses by several mechanisms. • Some regulatory cells produce cytokines (e.g., IL-10, TGF-β) that inhibit the activation of lymphocytes, dendritic cells, and macrophages. • Regulatory cells express CTLA-4, which, as discussed earlier, may block or remove B7 molecules made by APCs and make these APCs incapable of providing costimulation via CD28 and activating T cells. • Regulatory T cells, by virtue of the high level of expression of the IL-2 receptor, may bind and consume this essential T cell growth factor, thus reducing its availability for responding T cells. The great interest in regulatory T cells has in part been driven by the hypothesis that the underlying abnormality in some autoimmune diseases in humans is defective regulatory T cell function or the resistance of pathogenic T cells to regulation. However, the importance of defects in regulatory T cells in common human autoimmune diseases is not established, perhaps because it has proved difficult to identify regulatory T cells specific for self antigens in humans. There is also growing interest in cellular therapy with regulatory T cells to treat graft- versus-host disease, graft rejection, and autoimmune disorders.

What type of disorder is Ulcerative colitis? What are the clinical features ?

Relapsing disorder characterized by attacks of bloody diarrhea with stringy, mucoid material, lower abdominal pain, and cramps that are temporarily relieved by defecation These symptoms may persist for days, weeks, or months before they subside The factors that trigger ulcerative colitis are not known, but infectious enteritis precedes disease onset in some cases

What is Natalizumab approved for the treatment of?

Relapsing multiple sclerosis

What can chronic inflammation cause over time?

Relentless tissue destruction Promising treatments for chronic inflammation could include cytokine inhibitors that neutralize mediators of the signaling cascades that perpetuate chronic inflammation.

What can ameliorate (make better) Graft versus host disease?

Removing T cells from the donor bone marrow be ore transplantation although removing donor T cells from the graft reduces the risk of GVHD, this may not be the best approach or marrow transplants used in antineoplastic therapy

What is Antithymocyte globulin (ATG) approved for the use in the prevention or treatment of?

Renal transplant rejection and the equine-derived material is also approved for treatment of aplastic anemia

What are the goals of Resuscitation?

Restore the normal circulatory volume and to prevent complications from red blood cell loss, such as cardiac, pulmonary, renal, or neurologic consequences. 1- Initially, at least two large-bore intravenous catheters are used to administer isotonic solutions (e.g., lactated Ringer's solution, 0.9% NaCl), and blood products if indicate. 2- If the patient is in shock, central venous access should be established 3- Although the amount of blood to be infused must be individually determined in each case, recent randomized trials and a retrospective review suggest that use of a lower hemoglobin threshold of 7 g/dL, rather than a more liberal level of 9 g/dL, results in improved mortality rates, lower total transfusion requirements, and lower rates of rebleeding in both peptic ulcer bleeding and variceal bleeding in patients in whom early endoscopy (<5 hours) is available.

predominantly restrictive procedures

Restrictive procedures involve placing a band or stapling in or around the stomach to decrease the size of the organ. This reduction produces an earlier feeling of satiety and prevents the patient from overeating

What parts of the colon are retroperitoneal and which parts are intraperitoneal?

Retroperitoneal: Ascending and descending segments (secondarily) Intraperitoneal: Transverse and sigmoid segments are

What is Leflunomide currently approved for the treatment of?

Rheumatoid arthritis has also shown significant efficacy in the treatment of other immune diseases, including systemic lupus erythematosus and myasthenia gravis

What are agents that block B7: CD28 interactions used int he treatment of?

Rheumatoid arthritis, other inflammatory diseases, and graft rejection

What are immediately lateral to the ascending and descending colon?

Right and left paracolic gutters are formed between the lateral margins of the ascending and descending colon and the posterolateral abdominal wall and are gutters through which material can pass from one region of the peritoneal cavity to another

Where do most people with acute appendicitis have localized tenderness?

Right groin

Anti CD20 monoclonal antibody

Rituximab is a chimeric, partially humanized anti-CD20 monoclonal antibody. CD20 is expressed on the sur ace o all mature B cells, and administration o rituximab causes pro ound depletion o circulating B cells. Originally approved or the treatment o CD20 non-Hodgkin's lymphoma (see Chapter 40), rituximab has also been approved or use in rheumatoid arthritis re ractory to TNF inhibitors, granulomatosis with polyangiitis, and microscopic polyangiitis. Several additional anti-CD20 antibodies are in clinical development; ofatumumab and obinutuzumab are ully human anti-CD20 monoclonal antibodies that recognize an epitope distinct rom that bound by rituximab. O atumumab and obinutuzumab are approved or use in chronic lymphocytic leukemia.

What criteria may be helpful in diagnosis of Inflammatory bowel disease?

Rome Criteria These criteria include pain that is associated with change in bowel habits, relieved with defecation, or accompanied by distention or bloating serotonin (5-HT) agonists such as alosetron and tegaserod showed promise initially, they have been relegated to limited use due to unacceptable side effects

What is Mnemonic for the approach gastrointestinal bleeding?

SET 1- Stabilization 2- Evaluation (Endoscopy) 3- Treatment

Signs associated with malabsorption syndromes

SIGNS ASSOCIATED SYNDROMES GASTROINTESTINAL Mass Crohn's disease, lymphoma, tuberculosis, glands Distention Intestinal obstruction, gas, ascites, pseudocyst (pancreatic), motility disorder Steatorrheic stool Mucosal disease, bacterial overgrowth, pancreatic insufficiency, infective or inflammatory, drug induced EXTRAINTESTINAL Skin Nonspecific Pigmentation, thinning, inelasticity, reduced subcutaneous fat Specific Blisters (dermatitis herpetiformis), erythema nodosum (Crohn's disease), petechiae (vitamin K deficiency), edema (hypoproteinemia) Hair Alopecia Gluten sensitivity Loss or thinning Generalized inanition, hypothyroidism, gluten sensitivity Eyes Conjunctivitis, episcleritis Crohn's disease, Behçet's syndrome Paleness Severe anemia Mouth Aphthous ulcers Crohn's disease, gluten sensitivity, Behçet's syndrome Glossitis Deficiencies of vitamin B12, iron, folate, niacin Angular cheilosis Deficiencies of vitamin B12, iron, folate, B complex Dental hypoplasia (pitting, dystrophy) Gluten sensitivity Hands Raynaud's phenomenon Scleroderma Finger clubbing Crohn's disease, lymphoma Koilonychia Iron deficiency Leukonychia Inanition Musculoskeletal Monoarthropathy and polyarthropathy Crohn's disease, gluten sensitivity, Whipple's disease, Behçet's syndrome Back pain (osteomalacia, osteoporosis, sacroiliitis) Crohn's disease, malnutrition, gluten sensitivity Muscle weakness (low potassium, magnesium, vitamin D, generalized inanition) Diffuse mucosal disease, bacterial overgrowth, lymphoma Nervous System Peripheral neuropathy (weakness, paresthesias, numbness) Vitamin B12 deficiency Cerebral (seizures, dementia, intracerebral calcification, meningitis, pseudotumor, cranial nerve palsies) Whipple's disease, gluten sensitivity, diffuse lymphoma

Extaintestinal manifestations of Inflammatory bowel disease

SKIN Pyoderma gangrenosum Erythema nodosum Sweet's syndrome HEPATOBILIARY Primary sclerosing cholangitis Cholelithiasis Autoimmune hepatitis MUSCULOSKELETAL Seronegative arthritis Ankylosing spondylitis Sacroiliitis OCULAR Uveitis Episcleritis MISCELLANEOUS Hypercoagulable state Autoimmune hemolytic anemia Amyloidosis

Common sources of Acute Gastrointestinal Hemorrhage

SOURCE ASSOCIATED CLINICAL FEATURES TREATMENTS UPPER GASTROINTESTINAL TRACT Esophagitis Heartburn, dysphagia, odynophagia Medication* Antireflux surgery or procedures Esophageal cancer Progressive dysphagia, weight loss Chemoradiotherapy, surgery Palliative endoscopy procedures Gastritis, gastric ulcer Aspirin, NSAID use Withdraw NSAIDs Duodenitis, duodenal ulcer Abdominal pain, dyspepsia, Helicobacter pylori infection Medication† Endoscopic therapy for acute bleeding Gastric cancer Early satiety, weight loss, abdominal pain Surgery, chemotherapy Esophagogastric varices History of CLD Stigmata of CLD on examination Variceal banding, sclerotherapy Vasopressin, octreotide TIPS or decompressive surgery Mallory-Weiss tear History of retching before hematemesis Supportive (usually self-limited) Endoscopic therapy LOWER GASTROINTESTINAL TRACT Infection History of exposure, diarrhea, fever Supportive, antibiotics Inflammatory bowel diseases History of colitis, diarrhea, abdominal pain, fever Steroids, 5-ASA, immunotherapy Surgery if unresponsive to medication Diverticula Painless hematochezia Supportive Surgery for recurrent disease Angiodysplasia Painless hematochezia Often in ascending colon Commonly involves stomach and small bowel as well Endoscopic therapy Supportive Surgery for localized disease Colon cancer Change in bowel habit, anemia, weight loss Surgery Colon polyp Usually asymptomatic Endoscopic or surgical removal Ischemic colitis Typically elderly patients History of vascular disease May produce abdominal pain Supportive (self-limited) Meckel's diverticulum Painless hematochezia in young patient Located at distal ileum Surgery Hemorrhoids Rectal bleeding associated with bowel movement Supportive Surgery, bandin

Representative Examples of Autoimmune Diseases, Categorized by Type of Tissue Damage

SYNDROME AUTOANTIGEN CONSEQUENCE Acute rheumatic ever Streptococcal cell wall antigens that cross react with cardiac muscle Arthritis, myocarditis Autoimmune hemolytic anemia Rh blood group antigens Destruction of erythrocytes Goodpasture's syndrome Renal glomerular basement membrane collagen type IV Glomerulonephritis, pulmonary hemorrhage Immune thrombocytopenic purpura Platelet GPIIb-IIIa Excessive bleeding Pemphigus vulgaris Epidermal cadherin Blistering o skin Immune-Complex Disease SYNDROME AUTOANTIGEN CONSEQUENCE Mixed essential cryoglobulinemia Rheumatoid actor IgG complexes Systemic vasculitis Systemic lupus erythematosus DNA, histones, ribosomes, snRNP, scRNP Glomerulonephritis, vasculitis, arthritis T-Cell-Mediated Disease SYNDROME AUTOANTIGEN CONSEQUENCE Experimental autoimmune encephalitis, multiple sclerosis Myelin basic protein, proteolipid protein, myelin oligodendrocyte glycoprotein Brain invasion by CD4 T cells, several CNS deficits Rheumatoid arthritis Unknown—possible synovial joint antigens Joint inflammation and destruction Type 1 diabetes mellitus Pancreatic -cell antigens -Cell destruction, insulin-dependent diabetes mellitus

What are some common bacteria that cause acute diarrhea?

Salmonella, Campylobacter, Shigella, enterotoxigenic Escheria coli, and Clostridium difficile These are the likely causes of most severe cases of acute diarrhea. Among patients with diarrhea lasting longer than 3 days and high outputs, a bacterial cause was found in 87% of cases.

What does a person develop as the liver progressively fails? What does it produce?

Salt and water retention Skin and subcutaneous edema Fluid (ascites) is also retained in the peritoneal cavity, which can hold many liters.

What test should be used in Lower GI bleeding in which the pace of bleeding is so brisk as to preclude endoscopic visualization of the colon and rectum?

Scintigraphic erythrocyte scans using technetium-99m (99mTc)-labeled sulfur colloid or pertechnetate can localize the bleeding site if the rate of blood loss exceeds 0.5 mL/minut

B lymphocyte tolerance

Self polysaccharides, lipids, and nucleic acids are T-independent antigens that are not recognized by T cells. These antigens must induce tolerance in B lymphocytes to prevent autoantibody production. Self proteins may not elicit autoantibody responses because of tolerance in helper T cells and in B cells. It is suspected that diseases associated with autoantibody production, such as systemic lupus erythematosus, are caused by defective tolerance in both B lymphocytes and helper T cells

What can Chronic Gatrointestinal bleeding manifest as?

Self-limited, recurrent episodes of melena or hematochezia, but usually without the degree of hemodynamic compromise discussed earlier Some patients have no overt evidence of blood loss but rather have persistent anemia and persistent occult blood loss. The evaluation of this condition differs from that of acute GI hemorrhage; the pace of the evaluation is less urgent, and the likely causes of bleeding differ from those associated with acute GI bleeding

Autoimmune diseases usually occur due to breakdown of what?

Self-tolerance- both Central and peripheral Central tolerance refers to the specific clonal deletion of autoreactive T and B cells during their development from precursor cells in the thymus (T cells) and bone marrow (B cells).

What do patients with severe cirrhosis of the liver have a significant risk of?

Serious bleeding even from small cuts; in addition, when varices rupture, there is a danger of rapid exsanguination (raining a person, animal, or organ of blood.)

What type of studies may be useful in indeterminante colitis? Why?

Serologic studies because perinuclear anti-neutrophil cytoplasmic antibodies (p-ANCA) are found in 75% of individuals with ulcerative colitis but only 11% with Crohn disease In contrast, ulcerative colitis patients tend to lack antibodies to Saccharomyces cerevisiae, which are often present in those with Crohn disease

Conclusion

Several approaches are available or the pharmacologic suppression o adaptive immunity, ranging rom the relatively low-selectivity approaches represented by glucocorticoids and cytotoxic agents to the more selective approaches represented by cell-signaling inhibitors and antibody therapies. Glucocorticoids induce pro ound suppression o the in ammatory response and immune system but cause many adverse e ects, most o which are due to drug e ects on cells outside the immune system. Glucocorticoid receptor modulators are being sought that retain the anti-in ammatory e ects o glucocorticoids but have less severe adverse e ects on metabolism and bone mineral homeostasis. Cytotoxic agents target DNA replication; although immune cells are highly susceptible to these drugs, so, too, are other normal cells such as those in the gastrointestinal epithelium. The cytotoxic agent mycophenolate mo etil is highly selective, both because lymphocytes depend on de novo purine synthesis and because mycophenolic acid pre erentially targets the inosine monophosphate dehydrogenase isoenzyme expressed in lymphocytes. Lymphocyte-signaling inhibitors —such as cyclosporine, tacrolimus, sirolimus, and everolimus, which target intracellular signal transduction pathways necessary or T-cell activation—are also reasonably selective. Many new inhibitors o intracellular signaling in lymphocytes are under investigation; inhibition o the Janus kinase amily appears particularly promising. Cytokine inhibitors interrupt soluble signals mediating immune-cell activation. TNF inhibitors— such as etanercept, in iximab, and adalimumab—represent an expanding class o drugs. Promising new targets include cytokine pathways associated with T H 17 cells, which are targeted by the IL-17A inhibitors ixekizumab and secukinumab and the IL-17 receptor antagonist brodalumab, among others. The concept o preventing immune-cell activation has been extended to the blockade o costimulation represented by abatacept and belatacept. Speci c depletion o B cells is a well-established therapy or lymphomas and rheumatoid arthritis: belimumab, a f rst-in-its-class antibody against a crucial B-cell survival actor, is used in the treatment o systemic lupus erythematosus. Speci c depletion o T cells may be benef cial in organ transplantation: antithymocyte globulin is directed against T-cell-specif c epitopes. Several antibody therapeutics and small molecules are available that block immune-cell adhesion and homing, and more such agents are under development. Immune checkpoint inhibitors are an exciting new class o anticancer therapies.

What is a drawback/side effect associated with Natalizumab use?

Several patients treated with natalizumab developed progressive multifocal leukoencephalopathy (PML) a rare demyelinating disorder caused by infection with JC virus Natalizumab was subsequently reapproved for use in the treatment of multiple sclerosis and Crohn's disease.

Describe Somatic pain

Sharp and localized

What are the side effects associated with use of 5-ASA?

Side effects, including headache, nausea, and skin reactions, require discontinuation of sulfasalazine in about 30% of patients. Reversible oligospermia may occur rare serious side effects include pleuropericarditis, pancreatitis, agranulocytosis, interstitial nephritis, and hemolytic anemia

What does the large intestine enter the pelvic cavity as? What does it continue on the posterior wall as? What does it terminate as?

Sigmoid colon Rectum Anal canal

What is the final segment of the colon? Where does it occur from and to?

Sigmoid colon begins above the pelvic inlet and extends to the level of vertebra S3, where it is continuous with the rectum

What is the blood supply of the Sigmoid colon?

Sigmoidal arteries from the inferior mesenteric arteries

What is the preferred tests of choice once the lower GI tract has been identified as the source of bleeding?

Sigmoidoscopy or Colonoscopy

What does B-lymphocyte stimulator (BLyS) binding to normal B cells activate?

Signaling cascades that stimulate cell survival and cell differentiation into antibody and autoantibody-producing cells

Clinical presentation

Significant GI bleeding typically manifests with some combination of weakness, dizziness, lightheadedness, shortness of breath, postural changes in blood pressure or pulse, cramping abdominal pain, and diarrhea. The characteristics of the bleeding may help to localize its source to the upper or lower GI tract. Patients with acute bleeding commonly have one of the following symptoms at presentation. 1. Hematemesis: The patient vomits bright red blood or material that resembles coffee grounds, representing partially digested blood. After exclusion of swallowed blood from the nasopharynx or the respiratory tract (hemoptysis), the source of bleeding is likely to be proximal to the ligament of Treitz. 2. Melena: Black, tarry, usually foul-smelling stools are most often a manifestation of upper GI bleeding; however, a small bowel or proximal colonic source of bleeding may on occasion lead to melenic stools. Volumes as little as 50 to 100 mL of blood in the stomach can result in melena. 3. Hematochezia: The passage of bright-red blood or maroon stools per rectum frequently indicates a lower GI source of bleeding. However, 10% to 15% of patients with acute severe hematochezia have an upper GI source of brisk bleeding. This group of patients commonly displays signs of hemodynamic instability.

Jejunostomy

Similarly the jejunum is brought to the anterior abdominal wall and fixed. The jejunostomy is used as a site where a feeding tube is placed through the anterior abdominal wall into the proximal efferent small bowel

mTOR inhibitors

Sirolimus , also known as rapamycin , is a macrocyclic triene isolated rom the soil bacterium Streptomyces hygroscopicus . Although they are structurally similar and are both used to prevent and treat organ rejection, tacrolimus and sirolimus have di erent mechanisms o action. Both bind to FKBP, but the sirolimus-FKBP complex does not inhibit calcineurin; instead, it blocks the IL-2 receptor signaling required or T-cell proli eration (Fig. 46-6). Sirolimus-FKBP binds to and inhibits molecular target o rapamycin (mTOR), a serine-threonine kinase that phosphorylates p70 S6 kinase and PHAS-1 (among other substrates). p70 S6 kinase and PHAS-1 regulate translation, the ormer by phosphorylating proteins (including the ribosomal S6 protein) involved in protein synthesis and the latter by inhibiting the activity o a actor (eIF4E) required or translation. By inhibiting mTOR, sirolimus-FKBP inhibits protein synthesis and arrests cell division in the G1 phase (Fig. 46-6 Major adverse e ects o sirolimus include hypertension, interstitial lung disease, and leukopenia. Notably, however, the nephrotoxicity associated with CsA and tacrolimus is not observed with sirolimus. Everolimus and zotarolimus are mTOR inhibitors that are structurally related to sirolimus. Everolimus is approved or prevention o kidney transplant rejection and treatment o renal cell carcinoma, hormone-receptor positive, HER2negative breast cancer, progressive neuroendocrine tumors o pancreatic origin, and renal angiomyolipoma and tuberous sclerosis complex, while zotarolimus is used only in drugeluting stents. Recent studies have shown that sirolimus, everolimus, and zotarolimus inhibit mTOR complex 1 but are relatively weak inhibitors o mTOR complex 2; newer drugs are being developed to inhibit both complexes. Sirolimus- , everolimus- , and zotarolimus-eluting stents have been approved or use in the treatment o coronary artery disease. In this unique drug delivery system, the mTOR inhibitor elutes rom stents during the f rst ew weeks a ter stent placement, locally inhibiting proli eration o coronary artery smooth muscle cells and thereby reducing the rate o in-stent restenosis that results rom neointimal proli eration o vascular smooth muscle cells

What forms the first life of defense against any infection?

Skin and other barrier tissues Once an offending agent penetrates these barriers, the immune system mounts a response.

What procedure can be performed for people who are at high risk for gastric bypass surgery? What does it involve?

Sleeve gastrectomy reduction of the gastric lumen by removing a large portion of the stomach along the greater curvature Any overweight patient undergoing surgery faces significant risk and increased morbidity, with mortality rates from 1% to 5%.

Biopsy of what is the key diagnostic test for diseases that affect the cellular phase of absorption?

Small intestinal mucosa

Where does most dietary absorption occur? exceptions?

Small intestine e.g: Vitamin B12 and cholesterol are absorbed only in the terminal ileum Diseases associated with diffuse mucosal involvement, such as celiac disease, can lead to impaired absorption of many nutrients, whereas diseases affecting only the terminal ileum can lead to decreased vitamin B12 absorption.

What is a strong exogenous risk factor for Crohns disease?

Smoking in some cases, disease onset is associated with initiation of smoking Unfortunately, smoking cessation does not result in disease remission.

What is the transport of most amino acids dependent on?

Sodium dependent takes place in the proximal small bowel Dietary requirements for amino acid nitrogen are met with about 15% of calories from protein.

How is immune diversity created ?

Somatic gene recombination and other processes for generating diversity provide a mechanism or generating a specif c response to an infection By recombination, immunoglobulin and T-cell receptor genes semi-randomly create millions of modular three-dimensional protein structures, re erred to as variable regions Recombined variable regions may undergo somatic hypermutation to create additional diversity that, in the aggregate, can recognize almost any structure

What is Molecular mimicry?

Some infectious microbes may produce peptide antigens that are similar to, and cross- react with, self antigens. Immune responses to these microbial peptides may result in an immune attack against self antigens rheumatic fever, antibodies against streptococci cross-react with a myocardial antigen and cause heart disease

differentiation of Naive t cells into effector cells

Some of the progeny of antigen-stimulated, proliferating T cells differentiate into effector cells whose function is to eradicate infections. This process of differentiation is the result of changes in gene expression, such as the activation of genes encoding cytokines (in CD4+ T cells) or cytotoxic proteins (in CD8+ CTLs). It begins in concert with clonal expansion, and differentiated effector cells appear within 3 or 4 days after exposure to microbes. Effector cells of the CD4+ lineage acquire the capacity to produce different sets of cytokines. The subsets of T cells that are distinguished by their cytokine profiles are named Th1, Th2, and Th17 (Fig. 5-13). Many of these cells leave the peripheral lymphoid organs and migrate to sites of infection, where their cytokines recruit other leukocytes that destroy the infectious agents. The development and functions of these effector cells are described in Chapter 6, when we discuss cellmediated immunity. Other differentiated CD4+ T cells remain in the lymphoid organs and migrate into lymphoid follicles, where they help B lymphocytes to produce antibodies (see Chapter 7). Effector cells of the CD8+ lineage acquire the ability to kill infected cells; their development and function are also described in Chapter 6. CD4+ helper T cells activate phagocytes and B lymphocytes through the action of plasma membrane proteins and by secreted cytokines (Fig. 5-14). The most important cell surface protein involved in the effector function of CD4+ T cells is CD40 ligand, a member of a large family of proteins structurally related to the cytokine tumor necrosis factor (TNF). The CD40L gene is transcribed in CD4+ T cells in response to antigen recognition and costimulation, and so CD40L is expressed on activated helper T cells (see Fig. 5-9). It binds to its receptor, CD40, which is expressed mainly on macrophages, B lymphocytes, and dendritic cells. Engagement of CD40 stimulates these cells, and thus CD40L is an important participant in the activation of macrophages and B lymphocytes by helper T cells (see Chapters 6 and 7). The interaction of CD40L on T cells with CD40 on dendritic cells increases the expression of costimulators on these APCs and the production of T cell-stimulating cytokines, thus providing a positive feedback (amplification) mechanism for APC-induced T cell activation

What occurs in Anergy?

Some self antigens, such as soluble proteins, may be recognized in the bone marrow with low avidity. B cells specific for these antigens survive, but antigen receptor expression is reduced, and the cells become functionally unresponsive (anergic)

What is the largest branch of the Celiac trunk?

Splenic artery takes a tortuous course to the left along the superior border of the pancreas

What is the portal vein formed by the union of?

Splenic vein and the superior mesenteric vein posterior to the neck of the pancreas at the level of vertebra L2

What is the first goal of management for someone with GI bleeding?

Stabilize the patient and determine the severity of blood loss 1- If the systolic blood pressure drops more than 10 mm Hg or the pulse increases more than 10 beats per minute as the patient changes position from supine to standing, it is likely the patient has lost at least 800 mL (15%) of circulating blood volume 2-. Hypotension, tachycardia, tachypnea, and mental status changes in the setting of acute GI hemorrhage suggest the loss of at least 1500 mL (30%) of circulating blood volume.

What does Gastric bypass surgery invovle?

Stapling the proximal stomach and joining a loop of small bowel to the small gastric remnant The procedure is usually performed by fashioning a Roux-en-Y loop with alimentary and pancreaticobiliary limbs.

What do most dietary carbohydrates consist of? What is the only type of carbohydrate that is absorbed?

Starch (a glucose polymer) and the disaccharides sucrose and lactose only Monosaccharides are absorbed

Where does digestion of proteins start?

Starts in the stomach with pepsins secreted by the gastric mucosa, but most of the hydrolysis is accomplished by pancreatic enzymes in the proximal small bowel.

What are the principal functions of IL-2?

Stimulate the survival and proliferation of T cells resulting in an increase in the number of the antigen-specific T cells; because of these actions, IL-2 was originally called T cell growth factor. IL-2 also is essential for the maintenance of regulatory T cells and thus for controlling immune responses

What does IL-1 stimulate production of? What does it enhance? What does it stimulate?

Stimulates IL-6 production Enhances Adhesion molecule expression Stimulates cell proliferation

What does abdominal pain result from?

Stimulation of receptors specific for thermal, mechanical or chemical stimuli Once these receptors are excited, pain impulses travel through sympathetic fibers.

How can chronic secretory diarrhea be evaluated?

Stool cultures should be done Imaging of the small bowel and colon should be considered, and appropriate testing for hormones and other secretagogues should be based on the history and findings

Patients who present with bloody, frequent, small-volume stools as well as systemic symptoms should be evaluated for what?

Stool cultures should be done to look for Shigella, Samonella, Campylobacter, and enterohemorrhagic E coli. if there are no definitive results from the stool studies, endoscopic imaging with either flexible sigmoidoscopy or colonoscopy may be required to evaluate for IBD, ischemic colitis, or cytomegalovirus colitis in immunocompromised patients.

What is the only treatment for Celiac disease?

Strict, lifelong adherence to a gluten-free diet . The long-term prognosis is excellent for patients who adhere to the diet, although there may be a slight increase in the incidence of malignancies, particularly lymphoma.

Treatment

Strict, lifelong adherence to a gluten-free diet is the only treatment for celiac disease. Specific nutritional supplementation should be provided to correct deficiencies, particularly those of iron, vitamins, and calcium. A clinical response may be seen within a few weeks. Patients should be monitored to ensure adequate response and proper adherence to the diet. The long-term prognosis is excellent for patients who adhere to the diet, although there may be a slight increase in the incidence of malignancies, particularly lymphoma

What does FIbrosis lead to the formation of?

Strictures of the bile ducts which in turn may lead to recurrent cholangitis (with fever, right upper quadrant pain, and jaundice) and progression to cirrhosis.

What are Antimetabolites?

Structural analogues of natural metabolites that inhibit essential pathways involving these metabolites

What is the simplest quantitative method for detecting fat in stool in microscopic examination if fat malabsorption is suspected?

Sudan staining of a drop of stool To quantify fat, stool is collected for three consecutive days while the patient is on a diet containing 100 g of fat per day and the specimen is analyzed for fat content Normal fat excretion should not exceed 6 g/day Although this test is cumbersome and nonspecific, it offers an accurate quantification of fecal fat excretion provided fat consumption is appropriate

What is Acute abdomen caused by?

Sudden inflammation, perforation, obstruction, or infarction of an intraabdominal organ

The watershed area between what 2 arteries at the splenic flexure is extremely vulnerable to ischemia?

Superior mesenteric artery (SMA) and Inferior mesenteric artery (IMA)

What is the terminal branch of the IMA? Anastamoses?

Superior rectal artery branches from the middle rectal arteries (from the internal iliac artery) and the inferior rectal arteries (from the internal pudendal artery)

What does the inferior mesenteric vein begin as?

Superior rectal vein then ascends receiving tributaries from the sigmoid veins and the left colic vein

Where does the colon extend from? what does it consist of?

Superiorly from the cecum consists of the ascending, transverse, descending, and sigmoid colon

What does the Inferior pancreaticoduodenal artery anastamose with?

Superiorly, these arteries anastomose with anterior and posterior superior pancreaticoduodenal arteries

What do Glucocorticoids have the ability to supress in pharmacologic doses?

Suppress the activation and function of innate and adaptive immune cells has effects on all cells of the body

What is Bariatric surgery?

Surgery for obesity also known as weight loss surgery

What is Ostemoies?

Surgical cutting of the bone It is occasionally necessary to surgically externalize bowel to the anterior abdominal wall. Externalization of bowel plays an important role in patient management. These extraanatomical bypass procedures use our anatomical knowledge and in many instances are life saving.

surgical management

Surgical intervention is indicated for patients with severe complications such as obstruction, perforation, massive gastrointestinal hemorrhage, or toxic megacolon not responsive to medical treatment. The other main indication for surgical treatment is the presence of dysplasia or cancer. For patients with UC, regardless of the extent of disease, the entire colon must be removed. Historically, the initial operation for UC was a total proctocolectomy and Brooke ileostomy, but ileal pouch-anal anastomosis has become the procedure of choice in most patients. In this operation, the colon is removed and the small bowel is constructed into a reservoir (ileal pouch) that is anastomosed to the anus, allowing defecation through the anus. Complications include the development of pouchitis, fecal incontinence, reduced fertility, and need for reoperation. Surgery is not curative in Crohn's disease. Many surgical procedures in patients with Crohn's disease are performed to manage complications of the disease, including segmental resection, stricturoplasty, fistulectomy, and abscess drainage.

other causes of chronic colitis- diversion colitis

Surgical treatment of ulcerative colitis, Hirschsprung disease and other intestinal disorders sometimes require creation of a temporary or permanent ostomy and a blind distal segment of colon, from which the normal fecal flow is diverted. Colitis can develop within the diverted segment, particularly in ulcerative colitis patients. Besides mucosal erythema and friability, the most striking feature of diversion colitis is the development of numerous mucosal lymphoid follicles (Fig. 17-39A). Increased numbers of lamina propria lymphocytes, monocytes, macrophages, and plasma cells may also be present. In severe cases the histopathology may resemble IBD and include crypt abscesses, mucosal architectural distortion, or, rarely, granulomas. The mechanisms responsible for diversion colitis are not well understood, but changes in the luminal microbiota and diversion of the fecal stream that provides nutrients to colonic epithelial cells have been proposed. Consistent with this, enemas containing short-chain fatty acids, a product of bacterial digestion in the colon and an important energy source for colonic epithelial cells, can promote mucosal recovery in some cases. The ultimate cure is reanastomosis of the diverted segment.

To facilitate early detection of neoplasia patients are typically enrolled in what? Exception?

Surveillance programs approximately 8 years after diagnosis of IBD. The major exception to this is patients with IBD and primary sclerosing cholangitis, who have an even greater risk of developing cancer and are generally enrolled for surveillance at the time of diagnosis Surveillance requires regular and extensive mucosal biopsies, making it a costly practice. Research efforts are therefore focused on discovery of molecular markers of dysplasia.

Where does the proximal attachment of the small bowel mesentery begin?

Suspensory muscle of duodenum (ligament of Treitz)

What determines the position of the duodenal junction?

Suspensory muscle of duodenum (ligament of Treitz) The mesentery of the small bowel ends at the level of the ileocecal junction in the lower right quadrant. This long line of fixation of the mesentery prevents accidental twists of the gut.

What can Micorbial superantigens cause?

Systemic inflammatory disease by inducing excessive cytokine release from many T cells.

What autoimmune disease can exposure to sunlight trigger the development of?

Systemic lupus erythematosus (SLE) in which autoantibodies are produced against self nucleic acids and nucleoproteins it is is postulated that these nuclear antigens may be released from cells that die by apoptosis as a consequence of exposure to ultraviolet radiation in sunlight.

What recognition serves as the first or initiating signal for T cell activation?

T cell receptor for antigen (the TCR) and the CD4 or CD8 coreceptor together recognize complexes of peptide antigens and MHC molecules on APCs

Antigen recognition activates several biochemical mechanisms that lead to what?

T cell responses including the activation of enzymes such as kinases, recruitment of adaptor proteins, and production of active transcription factors

What is the net result of the molecular interactions between T cells and endothelial cells?

T cells migrate out of the blood vessels to the site of infection

Mechanisms of apoptosis by T lymphocytes

T cells respond to antigen presented by normal antigen-presenting cells (APCs) by secreting interleukin-2 (IL-2), expressing anti-apoptotic (pro-survival) proteins, and undergoing proliferation and differentiation. The anti-apoptotic proteins prevent the release of mediators of apoptosis from mitochondria. Self antigen recognition by T cells without costimulation may lead to relative deficiency of intracellular anti-apoptotic proteins, and the excess of pro-apoptotic proteins causes cell death by inducing release of mediators of apoptosis from mitochondria (death by the mitochondrial [intrinsic] pathway of apoptosis). Alternatively, self antigen recognition may lead to expression of death receptors and their ligands, such as Fas and Fas ligand (FasL), on lymphocytes, and engagement of the death receptor leads to apoptosis of the cells by the death receptor (extrinsic) pathway.

What cells are activated in Crohn disease?

T helper cells the response is polarized to the TH1 response

Clonal expansion

T lymphocytes activated by antigen and costimulation begin to proliferate within 1 or 2 days, resulting in expansion of antigen-specific clones (Fig. 5-12). This expansion quickly provides a large pool of antigen-specific lymphocytes from which effector cells can be generated to combat infection. The magnitude of clonal expansion is remarkable, especially for CD8+ T cells. Before infection, the frequency of CD8+ T cells specific for any one microbial protein antigen is about 1 in 105 or 1 in 106 lymphocytes in the body. At the peak of some viral infections, possibly within a week after the infection, as many as 10% to 20% of all the lymphocytes in the lymphoid organs may be specific for that virus. This means that the numbers of cells in antigen-specific clones have increased by more than 10,000-fold, with an estimated doubling time of about 6 hours. Several features of this clonal expansion are surprising. First, this enormous expansion of T cells specific for a microbe is not accompanied by a detectable increase in bystander cells that do not recognize that microbe. Second, even in infections with complex microbes that contain many protein antigens, a majority of the expanded clones are specific for only a few, often less than five, immunodominant peptides of that microbe The expansion of CD4+ T cells appears to be 100-fold to 1000-fold less than that of CD8+ cells. This difference may reflect differences in the functions of the two types of T cells. CD8+ CTLs are effector cells that kill infected cells by direct contact, and many CTLs may be needed to kill large numbers of infected cells. By contrast, each CD4+ effector cell secretes cytokines that activate many other effector cells, so a relatively small number of cytokine producers may be sufficient

Summary

T lymphocytes are the cells of cell-mediated immunity, the arm of the adaptive immune system that combats intracellular microbes, which may be microbes that are ingested by phagocytes and live within these cells or microbes that infect nonphagocytic cells. T lymphocytes also mediate defense against some extracellular microbes and help B lymphocytes to produce antibodies The responses of T lymphocytes consist of sequential phases: recognition of cell-associated microbes by naive T cells, expansion of the antigen-specific clones by proliferation, and differentiation of some of the progeny into effector cells and memory cells T cells use their antigen receptors to recognize peptide antigens displayed by MHC molecules on antigen-presenting cells (APCs), which accounts for the specificity of the ensuing response, and also recognize polymorphic residues of the MHC molecules, accounting for the MHC restriction of T cell responses. Antigen recognition by the T cell receptor (TCR) triggers signals that are delivered to the interior of the cells by molecules associated with the TCR (CD3 and ζ chains) and by the coreceptors CD4 and CD8, which recognize class II and class I MHC molecules, respectively The binding of T cells to APCs is enhanced by adhesion molecules, notably the integrins, whose affinity for their ligands is increased by antigen recognition by the TCR. APCs exposed to microbes or to cytokines produced as part of the innate immune reactions to microbes express costimulators that bind to receptors on T cells and deliver necessary second signals for T cell activation. The biochemical signals triggered in T cells by antigen recognition and costimulation result in the activation of various transcription factors that stimulate the expression of genes encoding cytokines, cytokine receptors, and other molecules involved in T cell responses In response to antigen recognition and costimulation, T cells secrete cytokines that induce proliferation of the antigen-stimulated T cells and mediate the effector functions of T cells T cells proliferate following activation by antigen and costimulators, resulting in expansion of the antigen-specific clones. The survival and proliferation of activated T cells are driven by the growth factor interleukin-2. Some of the T cells differentiate into effector cells that are responsible for eradicating infections. CD4+ effector cells produce surface molecules, notably CD40L, and secrete various cytokines that activate other leukocytes to destroy microbes. CD8+ effector cells are able to kill infected cells. Other activated T cells differentiate into memory cells, which survive even after the antigen is eliminated and are capable of rapid responses to subsequent encounter with the antigen Naive T cells migrate to peripheral lymphoid organs, mainly lymph nodes draining sites of microbe entry, whereas many of the effector T cells generated in lymphoid organs are able to migrate to any site of infection. The pathways of migration of naive and effector T cells are controlled by adhesion molecules and chemokines. The migration of T cells is independent of antigen, but cells that recognize microbial antigens in tissues are retained at these sites.

Intro

T lymphocytes perform multiple functions in defending against infections by various kinds of microbes. A major role for T lymphocytes is in cellmediated immunity, which provides defense against infections by intracellular microbes. In several types of infections, microbes may find a haven inside cells, from where they must be eliminated by cell-mediated immune responses (Fig. 5-1). • Many microbes are ingested by phagocytes as part of the early defense mechanisms of innate immunity, but some of these microbes have evolved to resist the microbicidal activities of phagocytes. Many pathogenic intracellular bacteria and protozoa are able to survive, and even replicate, in the vesicles of phagocytes. In such infections, T cells stimulate the ability of macrophages to kill the ingested microbes. • Some microbes, notably viruses, are able to infect and replicate inside a wide variety of cells, and parts of the life cycles of the viruses take place in the cytosol. These infected cells often do not possess intrinsic mechanisms for destroying the microbes, especially in the cytosol. Even some phagocytosed microbes within macrophages can escape into the cytosol and evade the microbicidal mechanisms of the vesicular compartment. T cells kill the infected cells, thus eliminating the reservoir of infection. In addition to cell-mediated immunity, T lymphocytes also play important roles in defense against microbes that replicate outside cells, including several types of bacteria, fungi, and helminthic parasites. Some T cells induce inflammatory responses rich in activated leukocytes that are particularly efficient at killing extracellular microbes. We discuss these T cell subsets and their functions in Chapter 6. Other populations of T cells help B cells to produce antibodies as part of humoral immune responses (see Chapter 7 Most of the functions of T lymphocytes— activation of phagocytes, killing of infected cells, and help for B cells—require that the T lymphocytes interact with other cells, which may be phagocytes, infected host cells, or B lymphocytes. Furthermore, the initiation of T cell responses requires that the cells recognize antigens displayed by dendritic cells, which capture antigens and concentrate them in lymphoid organs. Thus, T lymphocytes work by communicating with other cells. Recall that the specificity of T cells for peptides displayed by major histocompatibility complex (MHC) molecules ensures that the T cells can see and respond only to antigens associated with other cells (see Chapters 3 and 4). This chapter discusses the way in which T lymphocytes are activated by recognition of cell-associated antigens and other stimuli. We address the following questions: • What signals are needed to activate T lymphocytes, and what cellular receptors are used to sense and respond to these signals? • How are the few naive T cells specific for any microbe converted into the large number of effector T cells that have specialized functions and the ability to eliminate diverse microbes? • What molecules are produced by T lymphocytes that mediate their communications with other cells, such as macrophages, B lymphocytes, and other leukocytes? After describing here how T cells recognize and respond to the antigens of cell-associated microbes, in Chapter 6 we discuss how these T cells function to eliminate the microbes.

What is the PI-3 kinase/Akt pathway triggered by?

TCR but also by CD28 and IL-2 receptor Closely linked to the Akt pathway is mTOR (mammalian target of rapamycin), a serine-threonine kinase that is involved in stimulating protein translation and promoting cell survival and growth. A drug that binds to and inactivates mTOR—rapamycin—is used to treat graft rejection.

What also plays a role in the generation of regulatory T cells?

TGF-beta Transforming growth factor beta perhaps by stimulating expression of the FoxP3 transcription factor.

What proinflammatory cytokines play a role in in IBD pathogenesis?

TNF, interferon-γ and IL-13,

What is the MOA of Etanercept?

TNF-alpha inhibitor developed for the treatment of rheumatoid arthritis

Classification of diarrhea

TYPE PRESENTATION CAUSE Acute (duration <2 wk) Usually mild; wide range of symptoms Most commonly infectious Chronic (duration >4 wk) Secretory Large volume, watery stool No change with fasting Cholera, neuroendocrine tumors, drugs, nonosmotic laxatives, bile salts, bacterial toxins Osmotic Watery stool Stops with fasting Carbohydrate malabsorption laxatives (Mg, PO4) Inflammatory Bloody, mucosy diarrhea Frequent, urgent small stools Infectious colitis IBD, invasive bacteria Motility Soft to watery stools IBS Bacterial overgrowth Hyperthyroidism Scleroderma Steatorrhea Greasy, malodorous Malabsorption: celiac disease, short bowel syndrome, maldigestion, fistula

What is an example of a drug that is more potent that Cyclosporine?

Tacrolimus (FK506) Tacrolimus is a macrocyclic triene isolated from the soil bacterium Streptomyces tsukubaensis

Crohn disease may occur in any area of the GI tract but most commonly occurs where?

Terminal ileum, ileocecal valve, and cecum.

What are the subsets of T cell effectors from CD4 T cells?

Th1, Th2, and Th17 Many of these cells leave the peripheral lymphoid organs and migrate to sites of infection, where their cytokines recruit other leukocytes that destroy the infectious agents.

What T cell most likely contributes to IBD pathogenesis?

Th17 T cells polymorphisms of the IL-23 receptor, which is involved in the development and maintenance of TH17 cells, confer marked reductions in the risk of both Crohn disease and ulcerative colitis.

Mature B lymphocytes that encounter self antigens in peripheral lymphoid tissues become incapable of responding to what?

That antigen

What does Central tolerance ensure?

That the majority of immature autoreactive T and B cells do not develop into self-reactive clones

recognition of MHC associated peptides

The T cell receptor for antigen (the TCR) and the CD4 or CD8 coreceptor together recognize complexes of peptide antigens and MHC molecules on APCs, and this recognition provides the initiating, or first, signal for T cell activation (Fig. 5-5). The TCRs expressed on all CD4+ and CD8+ T cells consist of an α chain and a β chain, both of which participate in antigen recognition (see Chapter 4, Fig. 4-7). (A small subset of T cells expresses TCRs composed of γ and δ chains, which do not recognize MHC-associated peptide antigens.) The TCR of a T cell specific for a foreign (e.g., microbial) peptide recognizes the displayed peptide and simultaneously recognizes residues of the MHC molecule located around the peptidebinding cleft. Every mature MHC-restricted T cell expresses either CD4 or CD8, both of which are called coreceptors because they bind to the same MHC molecules that the TCR binds and are required for initiation of signaling from the TCR complex. At the time when the TCR is recognizing the peptide-MHC complex, CD4 or CD8 recognizes the class II or class I MHC molecule, respectively, at a site separate from the peptidebinding cleft. As discussed in Chapter 3, when protein antigens are ingested by APCs from the extracellular milieu into vesicles, these antigens are processed into peptides that are displayed by class II MHC molecules. In contrast, protein antigens present in the cytosol are processed by proteasomes into peptides displayed by class I MHC molecules. Thus, CD4+ and CD8+ T cells recognize antigens from different cellular compartments. The TCR and its coreceptor need to be engaged simultaneously to initiate the T cell response, and multiple TCRs likely need to be triggered for T cell activation to occur. Once these conditions are achieved, the T cell begins its activation program. The biochemical signals that lead to T cell activation are triggered by a set of proteins linked to the TCR that are part of the TCR complex and by the CD4 or CD8 coreceptor (see Fig. 5-5). In lymphocytes, antigen recognition and subsequent signaling are performed by different sets of molecules. The TCR αβ heterodimer recognizes antigens, but it is not able to transmit biochemical signals to the interior of the cell. The TCR is noncovalently associated with a complex of transmembrane signaling molecules including three CD3 proteins and a protein called the ζ chain. The TCR, CD3, and ζ chain make up the TCR complex. Although the α and β TCRs must vary among T cell clones in order to recognize diverse antigens, the signaling functions of TCRs are the same in all clones, and therefore the CD3 and ζ proteins are invariant among different T cells. The mechanisms of signal transduction by these proteins of the TCR complex are discussed later in the chapter. T cells can also be activated experimentally by molecules that bind to the TCRs of many or all clones of T cells, regardless of the peptide-MHC specificity of the TCR. These polyclonal activators of T cells include antibodies specific for the TCR or associated CD3 proteins, polymeric carbohydrate-binding proteins such as phytohemagglutinin (PHA), and certain microbial proteins, including staphylococcal enterotoxins, which are called superantigens. Polyclonal activators often are used as experimental tools to study T cell responses, and in clinical settings to test for T cell function or to prepare metaphase spreads for karyotyping (analyzing chromosomes). Microbial superantigens may cause systemic inflammatory disease by inducing excessive cytokine release from many T cells.

What can happen when T cells recognize antigens without costimulations?

The TCR complex may lose its ability to transmit activating signals

arterial supply

The abdominal aorta begins at the aortic hiatus of the diaphragm, anterior to the lower border of vertebra TXII (Fig. 4.121). It descends through the abdomen, anterior to the vertebral bodies, and by the time it ends at the level of vertebra LIV it is slightly to the left of midline. The terminal branches of the abdominal aorta are the two common iliac arteries

anterior branches of the abdominal aorta

The abdominal aorta has anterior, lateral, and posterior branches as it passes through the abdominal cavity. The three anterior branches supply the gastrointestinal viscera: the celiac trunk and the superior mesenteric and inferior mesenteric arteries The primitive gut tube can be divided into foregut, midgut, and hindgut regions. The boundaries of these regions are directly related to the areas of distribution of the three anterior branches of the abdominal aorta ■ The foregut begins with the abdominal esophagus and ends just inferior to the major duodenal papilla, midway along the descending part of the duodenum. It includes the abdominal esophagus, stomach duodenum (superior to the major papilla), liver, pancreas, and gallbladder. The spleen also develops in relation to the foregut region. The foregut is supplied by the celiac trunk ■ The midgut begins just inferior to the major duodenal papilla, in the descending part of the duodenum, and ends at the junction between the proximal two-thirds and distal one-third of the transverse colon. It includes the duodenum (inferior to the major duodenal papilla), jejunum, ileum, cecum, appendix, ascending colon, and right two-thirds of the transverse colon. The midgut is supplied by the superior mesenteric artery (Fig. 4.122). ■ The hindgut begins just before the left colic flexure (the junction between the proximal two-thirds and distal one-third of the transverse colon) and ends midway through the anal canal. It includes the left one-third of the transverse colon, descending colon, sigmoid colon, rectum, and upper part of the anal canal. The hindgut is supplied by the inferior mesenteric artery

vascular supply to GI system

The abdominal parts of the gastrointestinal system are supplied mainly by the celiac trunk and the superior mesenteric and inferior mesenteric arteries (Fig. 4.129): ■ The celiac trunk supplies the lower esophagus, stomach, superior part of the duodenum, and proximal half of the descending part of the duodenum. ■ The superior mesenteric artery supplies the rest of the duodenum, the jejunum, the ileum, the ascending colon, and the proximal two-thirds of the transverse colon. ■ The inferior mesenteric artery supplies the rest of the transverse colon, the descending colon, the sigmoid colon, and most of the rectum. Along the descending part of the duodenum there is a potential watershed area between the celiac trunk blood supply and the superior mesenteric arterial blood supply. It is unusual for this area to become ischemic, whereas the watershed area between the superior mesenteric artery and the inferior mesenteric artery, at the splenic flexure, is extremely vulnerable to ischemia. In certain disease states, the region of the splenic flexure of the colon can become ischemic. When this occurs, the mucosa sloughs off, rendering the patient susceptible to infection and perforation of the large bowel, which then requires urgent surgical attention Arteriosclerosis may occur throughout the abdominal aorta and at the openings of the celiac trunk and the superior mesenteric and inferior mesenteric arteries. Not infrequently, the inferior mesenteric artery becomes occluded. Interestingly, many of these patients do not suffer any complications, because anastomoses between the right, middle, and left colic arteries gradually enlarge, forming a continuous marginal artery. The distal large bowel therefore becomes supplied by this enlarged marginal artery (marginal artery of Drummond), which replaces the blood supply of the inferior mesenteric artery If the openings of the celiac trunk and superior mesenteric artery become narrowed, the blood supply to the gut is diminished. After a heavy meal, the oxygen demand of the bowel therefore outstrips the limited supply of blood through the stenosed vessels, resulting in severe pain and discomfort (mesenteric angina). Patients with this condition tend not to eat because of the pain and rapidly lose weight. The diagnosis is determined by aortic angiography, and the stenoses of the celiac trunk and superior mesenteric artery are best appreciated in the lateral view.

abdominal prevertebral plexus and ganglia

The abdominal prevertebral plexus is a collection of nerve fibers that surrounds the abdominal aorta and is continuous onto its major branches. Scattered throughout the length of the abdominal prevertebral plexus are cell bodies of postganglionic sympathetic fibers. Some of these cell bodies are organized into distinct ganglia, while others are more random in their distribution. The ganglia are usually associated with specific branches of the abdominal aorta and named after these branches The three major divisions of the abdominal prevertebral plexus and associated ganglia are the celiac, aortic, and superior hypogastric plexuses ■ The celiac plexus is the large accumulation of nerve fibers and ganglia associated with the roots of the celiac trunk and superior mesenteric artery immediately below the aortic hiatus of the diaphragm. Ganglia associated with the celiac plexus include two celiac ganglia, a single superior mesenteric ganglion, and two aorticorenal ganglia. ■ The aortic plexus consists of nerve fibers and associated ganglia on the anterior and lateral surfaces of the abdominal aorta extending from just below the origin of the superior mesenteric artery to the bifurcation of the aorta into the two common iliac arteries. The major ganglion in this plexus is the inferior mesenteric ganglion at the root of the inferior mesenteric artery. ■ The superior hypogastric plexus contains numerous small ganglia and is the final part of the abdominal prevertebral plexus before the prevertebral plexus continues into the pelvic cavity. Each of these major plexuses gives origin to a number of secondary plexuses, which may also contain small ganglia. These plexuses are usually named after the vessels with which they are associated. For example, the celiac plexus is usually described as giving origin to the superior mesenteric plexus and the renal plexus, as well as other plexuses that extend out along the various branches of the celiac trunk. Similarly, the aortic plexus has secondary plexuses consisting of the inferior mesenteric plexus, the spermatic plexus, and the external iliac plexus Inferiorly, the superior hypogastric plexus divides into the hypogastric nerves, which descend into the pelvis and contribute to the formation of the inferior hypogastric or pelvic plexus (Fig. 4.137). The abdominal prevertebral plexus receives: ■ preganglionic parasympathetic and visceral afferent fibers from the vagus nerves [X], ■ preganglionic sympathetic and visceral afferent fibers from the thoracic and lumbar splanchnic nerves, and ■ preganglionic parasympathetic fibers from the pelvic splanchnic nerves

stimuli for activation of CD8 T cells

The activation of CD8+ T cells is stimulated by recognition of class I MHC-associated peptides and requires costimulation and helper T cells. The responses of CD8+ T cells may differ in several ways from responses of CD4+ T lymphocytes: • The initiation of CD8+ T cell activation often requires cytosolic antigen from one cell (e.g.,virus-infected or tumor cells) to be cross-presented by dendritic cells (see Fig. 3-16, Chapter 3). • The differentiation of naive CD8+ T cells into fully active cytotoxic T lymphocytes (CTLs), and, even more, into memory cells, may require the concomitant activation of CD4+ helper T cells (Fig. 5-8). When virus-infected cells are ingested by dendritic cells, the APC may present viral antigens from the cytosol in complex with class I MHC molecules and from vesicles in complex with class II MHC molecules. Thus, both CD8+ T cells and CD4+ T cells specific for viral antigens are activated near one another. The CD4+ T cells may produce cytokines or membrane molecules that help to activate the CD8+ T cells. This requirement for helper T cells in CD8+ T cell responses is the likely explanation for the defective CTL responses to many viruses in patients infected with the human immunodeficiency virus (HIV), which kills CD4+ but not CD8+ T cells. CTL responses to some viruses do not appear to require help from CD4+ T cells. Now that we have described the stimuli required to activate naive T lymphocytes, we next consider the biochemical pathways triggered by antigen recognition and other stimuli

5-Aminosalicyclic acid

The aminosalicylates are given either orally or topically (suppository and enema). They are safe and effective for treatment (i.e., induction of remission) of mild to moderate UC and for maintenance of remission (Level of evidence III, A). The efficacy of the 5-aminosalicylic acid (5-ASA) agents in induction or maintenance of remission in Crohn's disease has not been clearly shown in studies, although they are commonly used off-label for this purpose (level of evidence III, A for induction and III, B for maintenance). This class of anti-inflammatory medications includes sulfasalazine (Azulfidine) at a dose of 4 to 6 g/day in divided doses. This drug consists of 5-ASA linked to a sulfapyridine moiety; the 5-ASA is released after bacterial lysis of an azo bond in the colon. Side effects, including headache, nausea, and skin reactions, require discontinuation of sulfasalazine in about 30% of patients. Reversible oligospermia may occur, and rare serious side effects include pleuropericarditis, pancreatitis, agranulocytosis, interstitial nephritis, and hemolytic anemia. Patients who take sulfasalazine need folic acid supplementation. Derivatives of oral 5-ASA compounds include mesalamine (Pentasa, 4 g/day in divided doses; Delzicol, 2.4 g/day in divided doses; Asacol HD, 2.4 to 4.8 g/day in divided doses; Lialda, 2.4 to 4.8 g once daily; Apriso, 1.5 g once a day), olsalazine (Dipentum, 1 to 2 g/day in divided doses), and balsalazide (Colazal, 6.75 g/day in divided doses; Giazo 3.3 g/day in divided doses). Topical forms of mesalamine (Canasa suppositories, 1000 mg once daily; Rowasa enemas, 4 g once nightly) are commonly used because of a more favorable side-effect profile. In addition to their use in the primary treatment of IBD, several studies suggest that long-term use of 5-ASA medications may reduce the risk for colorectal cancer in patients with UC.

In Brentuximab vedotin what is the anti-CD30 antibody linked to?

The antimitotic drug monomethyl auristatin E (MMAE) through a valine-citrulline dipeptide This linker dipeptide is enzymatically cleaved a ter endocytosis at the target site to release MMAE into the cytoplasm. MMAE prevents microtubule polymerization, causing cell cycle arrest in the G2 to M phase and subsequent apoptosis in CD30-expressing cells

Diarrhea- defintiion

The average number of bowel movements for the normal adult can range from three per day to three per week. Diarrhea can be defined as increased frequency of stools with decreased consistency and increased volume, but the subjective nature of these complaints and lack of fixed normal values make the definition difficult. The 1997 position statement by the American Gastroenterological Association on chronic diarrhea defines diarrhea as the production of loose stools with or without increased stool frequency. There are also definitions that include fecal weight, which in the United States averages less than 200 g/24 hours but varies according to the type of diet. The 2003 guidelines for the investigation of chronic diarrhea define diarrhea as the abnormal passage of loose or liquid stool more than three times a day or a stool volume greater than 200 g/day or both. Because of the subjective nature of the complaints, a detailed history is needed to arrive at a diagnosis (Table 33-6). The duration of symptoms is useful in the differential diagnosis of diarrhea and is a part of the necessary history. Acute diarrhea is limited to 2 weeks or less and is usually infectious in origin. Persistent diarrhea persists for longer than 14 days, and chronic diarrhea is considered to last longer than 4 weeks. As time goes on, is etiology is less likely to be infectious.

What is the cause of IBD?a

The cause of IBD remains unknown, but it is believed that a combination of genetic, immunologic, infectious, and environmental factors plays a role. In addition, state-of-the-art research points toward a relationship between the human microbiome and dysfunction of the immune system in patients with IBD

Cecum and appendix

The cecum is the first part of the large intestine (Fig. 4.82). It is inferior to the ileocecal opening and in the right iliac fossa. It is generally considered to be an intraperitoneal structure because of its mobility, even though it normally is not suspended in the peritoneal cavity by a mesentery The cecum is continuous with the ascending colon at the entrance of the ileum and is usually in contact with the anterior abdominal wall. It may cross the pelvic brim to lie in the true pelvis. The appendix is attached to the posteromedial wall of the cecum, just inferior to the end of the ileum ( The appendix is a narrow, hollow, blind-ended tube connected to the cecum. It has large aggregations of lymphoid tissue in its walls and is suspended from the terminal ileum by the mesoappendix (Fig. 4.83), which contains the appendicular vessels. Its point of attachment to the cecum is consistent with the highly visible free taeniae leading directly to the base of the appendix, but the location of the rest of the appendix varies considerably (Fig. 4.84). It may be: ■ posterior to the cecum or the lower ascending colon, or both, in a retrocecal or retrocolic position; ■ suspended over the pelvic brim in a pelvic or descending position; ■ below the cecum in a subcecal location; or ■ anterior to the terminal ileum, possibly contacting the body wall, in a pre-ileal position or posterior to the terminal ileum in a postileal position. The surface projection of the base of the appendix is at the junction of the lateral and middle one-third of a line from the anterior superior iliac spine to the umbilicus (McBurney's point). People with appendicular problems may describe pain near this location. The arterial supply to the cecum and appendix (Fig. 4.85) includes: ■ the anterior cecal artery from the ileocolic artery (from the superior mesenteric artery), ■ the posterior cecal artery from the ileocolic artery (from the superior mesenteric artery), and ■ the appendicular artery from the ileocolic artery (from the superior mesenteric artery).

celiac trunk

The celiac trunk is the anterior branch of the abdominal aorta supplying the foregut. It arises from the abdominal aorta immediately below the aortic hiatus of the diaphragm (Fig. 4.123), anterior to the upper part of vertebra LI. It immediately divides into the left gastric, splenic, and common hepatic arteries

What does the immune system include?

The cells and soluble factors, such as antibodies and complement proteins, which mediate the inflammatory response these cells and actors both eliminate the inciting inflammatory stimulus and initiate immunologic memory

the inflammatory response

The cells and soluble mediators o the immune system interact with one another to generate the inf ammatory response , which typically occurs in our phases. First, the vasculature around a site o injury reacts to recruit cells o the immune system. Second, circulating immune cells migrate rom these vessels into the injured tissues, and the mechanisms o innate and adaptive immunity (see above) serve to neutralize and remove the inciting stimulus. Next, the process o repair and tissue healing ensues and the acute inf ammatory process is terminated. I the process o acute inf ammation is not halted but continues to smolder, chronic inf ammation can occur.

What may the innate immune response to infections alter?

The chemical structure of self antigens eg: some periodontal bacterial infections are associated with rheumatoid arthritis It is postulated that the acute and chronic inflammatory responses to these bacteria lead to enzymatic conversion of arginines to citrullines in self proteins, and the citrullinated proteins are recognized as non self and elicit adaptive immune responses

crohns disease- clinical features

The clinical manifestations of Crohn disease are extremely variable. In most patients disease begins with intermittent attacks of relatively mild diarrhea, fever, and abdominal pain. Approximately 20% of patients present acutely with right lower quadrant pain, fever, and bloody diarrhea that may mimic acute appendicitis or bowel perforation. Periods of active disease are typically interrupted by asymptomatic periods that last for weeks to many months. Disease re-activation can be associated with a variety of external triggers, including physical or emotional stress, specific dietary items, and cigarette smoking. The latter is a strong exogenous risk factor for development of Crohn disease and, in some cases, disease onset is associated with initiation of smoking. Unfortunately, smoking cessation does not result in disease remission. Iron-deficiency anemia may develop in individuals with colonic disease, while extensive small bowel disease may result in serum protein loss and hypoalbuminemia, generalized nutrient malabsorption, or malabsorption of vitamin B12 and bile salts. Fibrosing strictures, particularly of the terminal ileum, are common and require surgical resection. Disease often recurs at the site of anastomosis, and as many as 40% of patients require additional resections within 10 years. Fistulae develop between loops of bowel and may also involve the urinary bladder, vagina, and abdominal or perianal skin. Perforations and peritoneal abscesses are common. Anti-TNF antibodies have revolutionized treatment of Crohn disease, and other biologic therapies are becoming available. Extraintestinal manifestations of Crohn disease include uveitis, migratory polyarthritis, sacroiliitis, ankylosing spondylitis, erythema nodosum, and clubbing of the fingertips, any of which may develop before intestinal disease is recognized. Pericholangitis and primary sclerosing cholangitis occur in Crohn disease with a higher frequency than in those without Crohn disease, but are even more common in those who have ulcerative colitis (see below and Chapter 18). As discussed later, risk of colonic adenocarcinoma is increased in patients with long-standing IBD affecting the colon.

Clinical presentation

The clinical manifestations of malabsorption are usually nonspecific, particularly in the early stages. A change in bowel movements, usually with diarrhea, and weight loss despite adequate food intake may occur in more severe cases. Usually, however, patients have relatively mild symptoms such as bloating and flatulence. Clinical manifestations related to a specific micronutrient deficiency can occur. For example, iron deficiency anemia may be the only manifestation of celiac disease in some patients. Muscle wasting and edema result from protein malabsorption. Nutritional anemia, caused by deficiencies of iron, folate, and vitamin B12, contributes to fatigue. Bleeding tendency (e.g., ecchymosis) may be attributed to prolonged prothrombin time resulting from vitamin K deficiency related to fat malabsorption. Bulky, oily stools are the hallmark of steatorrhea resulting from fat malabsorption, whereas bloating (abdominal distention) and soft diarrheal movements occur as a result of carbohydrate malabsorption

Colon

The colon extends superiorly from the cecum and consists of the ascending, transverse, descending, and sigmoid colon (Fig. 4.88). Its ascending and descending segments are (secondarily) retroperitoneal and its transverse and sigmoid segments are intraperitoneal At the junction of the ascending and transverse colon is the right colic flexure, which is just inferior to the right lobe of the liver (Fig. 4.89). A similar, but more acute bend (the left colic flexure) occurs at the junction of the transverse and descending colon. This bend is just inferior to the spleen, is higher and more posterior than the right colic flexure, and is attached to the diaphragm by the phrenicocolic ligament. Immediately lateral to the ascending and descending colon are the right and left paracolic gutters (Fig. 4.88). These depressions are formed between the lateral margins of the ascending and descending colon and the posterolateral abdominal wall and are gutters through which material can pass from one region of the peritoneal cavity to another. Because major vessels and lymphatics are on the medial or posteromedial sides of the ascending and descending colon, a relatively blood-free mobilization of the ascending and descending colon is possible by cutting the peritoneum along these lateral paracolic gutters. The final segment of the colon (the sigmoid colon) begins above the pelvic inlet and extends to the level of vertebra SIII, where it is continuous with the rectum (Fig. 4.88). This S-shaped structure is quite mobile except at its beginning, where it continues from the descending colon, and at its end, where it continues as the rectum. Between these points, it is suspended by the sigmoid mesocolon The arterial supply to the ascending colon (Fig. 4.90) includes: ■ the colic branch from the ileocolic artery (from the superior mesenteric artery) , ■ the anterior cecal artery from the ileocolic artery (from the superior mesenteric artery), ■ the posterior cecal artery from the ileocolic artery (from the superior mesenteric artery), and ■ the right colic artery from the superior mesenteric artery The arterial supply to the transverse colon (Fig. 4.90) includes: ■ the right colic artery from the superior mesenteric artery, ■ the middle colic artery from the superior mesenteric artery, and ■ the left colic artery from the inferior mesenteric artery he arterial supply to the descending colon (Fig. 4.90) includes the left colic artery from the inferior mesenteric artery. The arterial supply to the sigmoid colon (Fig. 4.90) includes sigmoidal arteries from the inferior mesenteric artery. Anastomotic connections between arteries supplying the colon can result in a marginal artery that courses along the ascending, transverse, and descending parts of the large bowel

What does Inflammatory bowel disease result from?

The combined effects of alterations in host interactions with intestinal microbiota, intestinal epithelial dysfunction, aberrant mucosal immune responses, and altered composition of the gut microbiome

common hepatic artery

The common hepatic artery is a medium-sized branch of the celiac trunk that runs to the right and divides into its two terminal branches, the hepatic artery proper and the gastroduodenal artery The hepatic artery proper ascends toward the liver in the free edge of the lesser omentum. It runs to the left of the bile duct and anterior to the portal vein, and divides into the right and left hepatic arteries near the porta hepatis (Fig. 4.125). As the right hepatic artery nears the liver, it gives off the cystic artery to the gallbladder. The right gastric artery often originates from the hepatic artery proper but it can also arise from the common hepatic artery or from the left hepatic, gastroduodenal, or supraduodenal arteries. It courses to the left and ascends along the lesser curvature of the stomach in the lesser omentum, supplies adjacent areas of the stomach, and anastomoses with the left gastric artery The gastroduodenal artery may give off the supraduodenal artery and does give off the posterior superior pancreaticoduodenal artery near the upper border of the superior part of the duodenum. After these branch the gastroduodenal artery continues descending posterior to the superior part of the duodenum. Reaching the lower border of the superior part of the duodenum, the gastroduodenal artery divides into its terminal branches, the right gastro-omental artery and the anterior superior pancreaticoduodenal artery The right gastro-omental artery passes to the left, along the greater curvature of the stomach, eventually anastomosing with the left gastro-omental artery from the splenic artery. The right gastro-omental artery sends branches to both surfaces of the stomach and additional branches descend into the greater omentum The anterior superior pancreaticoduodenal artery descends and, along with the posterior superior pancreaticoduodenal artery, supplies the head of the pancreas and the duodenum (Fig. 4.124). These vessels eventually anastomose with the anterior and posterior branches of the inferior pancreaticoduodenal artery

inhibition of complement activation

The complement system mediates multiple innate immune responses (see Chapter 42). Recognition o oreign proteins or carbohydrates leads to sequential activation o complement proteins and eventual assembly o the membrane attack complex , a multiprotein structure that can cause cell lysis. Patients with paroxysmal nocturnal hemoglobinuria (PNH) have acquired de ects in complement regulatory proteins, leading to inappropriate activation o complement and complementmediated lysis o erythrocytes. Eculizumab is a humanized monoclonal antibody directed against C5, a complement protein that mediates late steps in complement activation and triggers assembly o the membrane attack complex. Eculizumab is approved or treatment o PNH; it signif cantly decreases hemoglobinuria and the need or erythrocyte trans usions in patients with this disorder. Eculizumab is also approved or treatment o atypical hemolytic uremic syndrome. Genetic evidence indicates that complement activation may play an etiologic role in age-dependent macular degeneration,suggesting that inhibitors o the complement cascade could be use ul local therapies or this disease.

regulation of T cell responses by inhibitory receptors

The concept that immune responses are influenced by a balance between activating and inhibitory receptors is established for all lymphocyte populations, including NK cells (see Chapter 2), B lymphocytes (see Chapter 7), and T cells. In T cells, the best-defined inhibitory receptors are CTLA-4 and PD-1. • CTLA-4. CTLA-4 is expressed transiently on activated CD4+ T cells and constitutively on regulatory T cells (described below). It functions to terminate activation of responding T cells and also mediates the suppressive function of regulatory T cells. CTLA-4 works by blocking and removing B7 molecules from the surface of APCs, thus reducing costimula tion and preventing the activation of T cells; CTLA-4 might also deliver inhibitory signals to T cells. It is intriguing that CTLA-4, which is involved in shutting off T cell responses, recognizes the same B7 costimulators that bind to CD28 and initiate T cell activation. One theory to explain how T cells choose CD28 or CTLA4, with these very different outcomes, is based on the fact that CTLA-4 has a higher affinity for B7 molecules than does CD28. Thus, when B7 levels are low (as would be expected normally when APCs are displaying self antigens), the receptor that is preferentially engaged is the high-affinity CTLA-4, but when B7 levels are high (as in infections), the lowaffinity activating receptor CD28 is engaged to a greater extent. • PD-1. PD-1 is expressed on CD4+ and CD8+ T cells after antigen stimulation. It has an immunoreceptor tyrosine-based inhibitory motif (ITIM) typical of receptors that deliver inhibitory signals. PD-1 terminates responses of T cells to self antigens and also to chronic infections, notably virus infections (see Chapter 6, Fig. 6-15). One of the most impressive therapeutic applications of our understanding of these inhibitory receptors is treatment of cancer patients with antibodies that block these receptors. Such treatment leads to enhanced antitumor immune responses and tumor regression in a significant fraction of the patients (see Chapter 10). This type of therapy has been termed checkpoint blockade, because the inhibitory receptors impose checkpoints in immune responses, and the treatment blocks these checkpoints ("removes the brakes" on immune responses). Predictably, patients treated with checkpoint blockade often develop autoimmune reactions, consistent with the idea that the inhibitory receptors are constantly functioning to keep autoreactive T cells in check. In experimental animals, if CTLA-4 or PD-1 molecules are blocked (by treatment with antibodies) or eliminated (by gene knockout), the animals develop autoimmune reactions against their own tissues. Polymorphisms in the CTLA4 gene have been associated with some autoimmune diseases in humans. Rare patients with mutations in one of their two copies of the CTLA4 gene also develop multiorgan inflammation (and a profound, as yet unexplained, defect in antibody production). Several other receptors on T cells other than CTLA-4 and PD-1 have been shown to inhibit immune responses and are currently the targets of checkpoint blockade therapy. The role of these receptors in maintaining tolerance to self antigens is not clearly established

IL-6 receptor inhibitor

The cytokine interleukin-6 (IL-6) was initially identif ed as a actor that di erentiates activated B cells into immunoglobulin-producing cells. IL-6 is a key mediator in many physiologic and pathologic processes, including the acutephase in ammatory response, angiogenesis, neutrophil migration, di erentiation o helper T cells, bone and cartilage metabolism, lipid metabolism, and cancer. Lymphocytes, monocytes, f broblasts, keratinocytes, endothelial cells, mesangial cells, and adipocytes all produce IL-6, while cells o hematopoietic origin express the IL-6 receptor Elevated serum levels o IL-6 have been implicated in the pathogenesis o many autoimmune and in ammatory conditions. Tocilizumab is a monoclonal antibody directed against the IL-6 receptor that is approved or use in polyarticular juvenile idiopathic arthritis and systemic juvenile idiopathic arthritis and in patients with rheumatoid arthritis who have had an inadequate response to anti-TNF drugs. The drug is administered every 4 weeks as an intravenous in usion.

d- xylose test

The d-xylose test serves as an indicator of mucosal absorption in the proximal small bowel and is used to determine whether defects in the epithelium of the intestine are responsible for malabsorption. d-Xylose is a 5-carbon monosaccharide that is transported across the intestinal mucosa largely by passive diffusion. In this test, the subject ingests 25 g of d-xylose, and urine is collected for the next 5 hours. Healthy subjects excrete more than 4.5 g of d-xylose in 5 hours (or ≥20% of the ingested load). Excretion of a lower amount of d-xylose suggests abnormal absorption. However, an abnormally low (false-positive) result may occur in the presence of impaired renal excretory function, gastroparesis, massive peripheral edema, or ascites. Abnormal results can also be seen in the presence of bacterial overgrowth as a result of bacterial degradation of d-xylose in the lumen, but this "pseudomalabsorption" may be corrected after treatment with antibiotics serving as a therapeutic trial.

What does Peripheral tolerance result from?

The deletion of autoreactive T cells by Fas-Fas ligand-mediated apoptosis, activation of T suppressor cells, or induction of T-cell anergy due to antigen presentation in the absence of costimulation

diagnosis and differential diagnosis

The diagnosis of IBD is based on a constellation of clinical features, laboratory tests, and endoscopic, radiographic, and histologic findings. Laboratory tests are not specific and usually reflect inflammation (leukocytosis) or anemia. Perinuclear antineutrophil cytoplasmic antibody (pANCA) is positive in up to 70% of patients with UC but is rarely positive in patients with Crohn's disease, whereas anti-Saccharomyces cerevisiae antibodies (ASCA) are common in Crohn's disease but rarely found in UC (Table 37-2). Additional markers have improved the sensitivity and specificity of serologic testing, including antibodies to OmpC (Escherichia coli outer membrane porin C) and antibodies to bacterial flagellins CBir1, FlaX, and A4-Fla2. Colonoscopy in patients with UC reveals a granular mucosa, decreased vascular markings, decreased mucosal reflection, exudate, and superficial ulcerations (Fig. 37-4). In more severe cases, the mucosa is friable, with deeper ulcerations. Patients with long-standing disease have pseudopolyps, which represent islands of normal tissue in regions of previous ulceration. In Crohn's disease (Fig. 37-5), endoscopic examination may show aphthoid erosions, deep linear or stellate ulcers, edema, erythema, exudate, and friability with intervening areas of normal mucosa (skip lesions). However, a diagnosis of indeterminate colitis is occasionally made because of an overlap of findings. For example, colonic Crohn's disease may produce superficial continuous rectal involvement similar to that seen in UC. Similarly, chronic UC can infrequently result in inflammation of the terminal ileum, called backwash ileitis. In many patients with indeterminate colitis, repeated examination is necessary, or complications may develop that help identify the disease form. Several types of radiologic studies can be used to diagnose IBD. In Crohn's disease, the most sensitive test to diagnose small bowel disease is video capsule endoscopy. Small erosions and ulcerations on the mucosa, as well as strictures, can be visualized best on video capsule endoscopy (Fig. 37-6). Patients with known or suspected strictures should be evaluated for risk of capsule retention before undergoing capsule endoscopy. On traditional small bowel radiography, segments of edematous bowel appear thickened next to uninvolved mucosa, a characteristic pattern referred to as cobblestoning. Tight, long strictures in the small bowel can be identified and are called a string sign. Crosssectional imaging with computed tomographic (CT) enterography and magnetic resonance enterography has replaced traditional small bowel radiography. Cross-sectional imaging can identify bowel wall thickening with surrounding inflammation, as well as intra-abdominal abscesses and fistulas (Figs. 37-7 and 37-8). A characteristic finding on cross-sectional imaging in Crohn's disease is infiltration of the mesentery with fat, commonly known as creeping fat. The differential diagnosis of IBD includes infectious colitis, ischemic colitis, radiation enteritis, enterocolitis induced by nonsteroidal anti-inflammatory drugs, diverticulitis, appendicitis, gastrointestinal malignancies, and irritable bowel syndrome. In patients with acute onset of bloody diarrhea, infectious causes that must be excluded with stool testing include Salmonella enteritidis, Shigella species, Campylobacter jejuni, Escherichia coli O157, and Clostridium difficile. Among the infectious causes, Yersinia enterocolitica can mimic Crohn's disease because the pathogen causes ileitis, mesenteric adenitis, fever, diarrhea, and right lower quadrant abdominal pain. Mycobacterium tuberculosis infection, strongyloidiasis, and amebiasis must be excluded in high-risk populations, because these infections can mimic IBD, and treatment with corticosteroids can lead to disseminated infection and death

Clinical presentation - History

The differential diagnosis of abdominal pain, whether acute or chronic, requires thorough history taking with regard to pain characteristics, location and radiation, timing, and the presence of any accompanying symptoms. Recognition of characteristic patterns is essential to narrowing the differential diagnosis. Pain location often indicates the organ responsible for the problem. For instance, epigastric pain is usually typical of peptic ulcer or dyspepsia, whereas right upper quadrant pain is more suggestive of cholecystitis and other biliary disorders. Early in the course of illness, pain may be perceived in one location and subsequently felt in another; this pattern of progression may be suggestive of specific pain syndromes. In acute cases, abdominal pain tends to be sharp and severe. The pain of a perforated viscus is intense, and the pain from a dissecting aneurysm may be described as tearing or crushing. Chronic pain may be less severe; pain from irritable bowel or dyspepsia is constant and dull, and the pain of chronic peptic ulcer is described as gnawing or hunger pain. The pattern of pain relief is helpful for diagnosing some conditions. The physician should also inquire about whether the pain is steady or intermittent and whether it occurs at night. For nocturnal pain, a distinction should be made between pain that awakens the patient and pain that is felt when the patient wakes up for other reasons.

special lymphocyte-signaling inhibitors- cyclosporine and tacrolimus

The discovery in 1976 that cyclosporine ( CsA ; also re erred to as cyclosporin A ) is a specif c inhibitor o T-cell-mediated immunity enabled widespread whole-organ transplantation. In act, CsA made heart transplantation a legitimate alternative in the treatment o end-stage heart ailure. CsA is a cyclic decapeptide isolated rom a soil ungus, Tolypocladium inf atum CsA inhibits the production o IL-2 by activated T cells. IL-2 is an important cytokine that acts in an autocrine and increase their biosynthesis o extracellular matrix components, resulting in interstitial f brosis Tacrolimus (also known as FK506) is a more potent immunosuppressant than CsA; although its structure di ers rom that o CsA, it acts by a similar mechanism (Fig. 46-5). Tacrolimus is a macrocyclic triene isolated rom the soil bacterium Streptomyces tsukubaensis . Tacrolimus acts by binding to FK-binding proteins ( FKBP ), and the tacrolimus- FKBP complex inhibits calcineurin. Tacrolimus inhibits IL-3, IL-4, IFN- , and TNF- production in vitro , and it appears to inhibit cell-mediated immunity without suppressing B-cell or natural killer (NK) cell unction. Tacrolimus is generally 50-100 times more potent than CsA, but, like CsA, it is nephrotoxic. Tacrolimus can also cause new-onset diabetes mellitus in post-transplant patients. In the introductory case, tacrolimus was a probable contributor to Mr. W's new-onset diabetes. This was the rationale or lowering both the tacrolimus and prednisone doses in Mr. W's immunosuppressive regimen a ter he developed diabetes mellitus Tacrolimus is approved as an immunosuppressant or transplantation. A topical ormulation is used or the treatment o atopic dermatitis and other eczematous diseases

chemical mediators of inflammation

The discussion to this point has ocused on the cells o the immune system and their roles in mounting an immune response. Equally important are the molecular mediators o immune cell activity. The ollowing discussion highlights endogenous molecules that regulate the inf ammatory process. (Note that signaling pathways or immune cells are discussed mainly in Chapter 46, although there is some overlap among the endogenous mediators o inf ammation and immunity, especially among the cytokines.) The list o mediators is long (Table 42-2), and essentially all o these signaling systems have been explored as potential pharmacologic targets. Only those mediators most crucial to inf ammation and those or which therapies already exist are discussed in detail here.

Distribution of lesions in inflammatory bowel disease

The distinction between Crohn disease and ulcerative colitis is primarily based on morphology. Transmural

What is the distinction of Ulcerative colitis and Crohn's disease based largely on?

The distribution of affected sites

enteric system

The enteric system is a division of the visceral part of the nervous system and is a local neuronal circuit in the wall of the gastrointestinal tract. It consists of motor and sensory neurons organized into two interconnected plexuses (the myenteric and submucosal plexuses) between the layers of the gastrointestinal wall, and the associated nerve fibers that pass between the plexuses and from the plexuses to the adjacent tissue The enteric system regulates and coordinates numerous gastrointestinal tract activities, including gastric secretory activity, gastrointestinal blood flow, and the contraction and relaxation cycles of smooth muscle (peristalsis). Although the enteric system is generally independent of the central nervous system, it does receive input from postganglionic sympathetic and preganglionic parasympathetic neurons that modifies its activities.

Where does the Vagus enter the abdomen associated with?

The esophagus as the esophagus passes through the diaphragm

Chronic diarrhea

The evaluation of chronic diarrhea is more variable, and the establishment of universal guidelines is more difficult, reflecting in part the many potential causes. Diarrhea may result from colonic inflammation, colonic neoplasia, small bowel inflammation, malabsorption due to small bowel mucosal disorders maldigestion due to pancreatic insufficiency, motility disorders, and functional bowel disorders. Chronic diarrhea is a common reason for referral to a gastroenterology clinic, but the true incidence is difficult to estimate because of differing definitions and populations. In one study, the estimated prevalence of chronic diarrhea in the elderly population was between 7% and 14%, but that study also included functional bowel disorders. Another estimate excluding abdominal pain placed the prevalence at 4% to 5 % Chronic diarrhea certainly can affect the quality of life, and any clinician who cares for these patients has heard of patients being housebound because of a fear of diarrhea and incontinence

tolerance to fetal antigens

The evolution of placentation in eutherian mammals allowed the fetus to mature before birth but created the problem that paternal antigens expressed in the fetus, which are foreign to the mother, have to be tolerated by the immune system of the pregnant mother. One mechanism of this tolerance is the generation of peripheral FoxP3+ regulatory T cells specific for these paternal antigens. In fact, during evolution, placentation is strongly correlated with the ability to generate stable peripheral regulatory T cells. It is unclear whether women who suffer recurrent pregnancy losses have a defect in the generation or maintenance of these regulatory T cells. Other mechanisms of fetal tolerance include exclusion of inflammatory cells from the pregnant uterus, poor antigen presentation in the placenta, and an inability to generate harmful Th1 responses in the healthy pregnant uterus. Now that we have described the principal mechanisms of immunological tolerance, we consider the consequences of the failure of selftolerance—namely, the development of autoimmunity. The mechanisms of tissue injury in autoimmune diseases and therapeutic strategies for these disorders are described in Chapter 11.

In addition to their role in cellular immunity what do T cells control?

The extent of immune responses Each T cell evolves so that it is activated by only one specific MHC:antigen complex All T cells express an MHC:antigen-specif c T-cell receptor (TCR).

pathophysiology- transplantation

The f rst transplant per ormed success ully in humans was a kidney transplant between identical twins. No immunosuppression was used, and the individuals did well. Currently, most organ transplantation occurs between unrelated individuals, termed an allograft. Donor and recipient tissues express di erent major histocompatibility complex (MHC) class I molecules, one class o alloantigens, and recipient immune cells there ore recognize the transplanted tissues as oreign. This is termed alloimmunity , and it occurs when the recipient's immune system attacks a transplanted organ. In the case o a bone marrow or stem cell transplant, graftversus-host disease (GVHD) can result when donor lymphocytes mount an assault on recipient tissues.

ileocolic artery

The final branch arising from the right side of the superior mesenteric artery is the ileocolic artery (Fig. 4.127). This passes downward and to the right toward the right iliac fossa where it divides into superior and inferior branches: ■ The superior branch passes upward along the ascending colon to anastomose with the right colic artery. ■ The inferior branch continues toward the ileocolic junction, dividing into colic, cecal, appendicular, and ileal branches The specific pattern of distribution and origin of these branches is variable: ■ The colic branch crosses to the ascending colon and passes upward to supply the first part of the ascending colon. ■ Anterior and posterior cecal branches, arising either as a common trunk or as separate branches, supply corresponding sides of the cecum ■ The appendicular branch enters the free margin of and supplies the mesoappendix and the appendix. ■ The ileal branch passes to the left and ascends to supply the final part of the ileum before anastomosing with the superior mesenteric artery.

role of costimulation in T cell activation

The full activation of T cells depends on the recognition of costimulators on APCs in addition to antigen (Fig. 5-6). We have previously referred to costimulators as second signals for T cell activation (see Chapters 2 and 3). The name costimulator derives from the fact that these molecules provide stimuli to T cells that function together with stimulation by antigen. The best-defined costimulators for T cells are two related proteins called B7-1 (CD80) and B7-2 (CD86), both of which are expressed on APCs and whose expression is increased when the APCs encounter microbes. These B7 proteins are recognized by a receptor called CD28, which is expressed on most T cells. Different members of the B7 and CD28 families serve to stimulate or inhibit immune responses (Fig. 5-7). The binding of CD28 on T cells to B7 on the APCs generates signals in the T cells that work together with signals generated by TCR recognition of antigen presented by MHC proteins on the same APCs. CD28-mediated signaling is essential for the responses of naive T cells; in the absence of CD28:B7 interactions, antigen recognition by the TCR is insufficient for T cell activation. The requirement for costimulation ensures that naive T lymphocytes are activated fully by microbial antigens and not by harmless foreign substances or by self antigens, because, as stated previously, microbes stimulate the expression of B7 costimulators on APCs. A protein called ICOS (inducible costimulator), which is related to CD28 and also expressed on T cells, plays an important role in the development and function of follicular helper T cells during germinal center responses (see Chapter 7). T cell responses are CD40 ligand (CD40L, or CD154) on activated T cells and CD40 on APCs. These molecules do not directly enhance T cell activation. Instead, CD40L expressed on an antigen-stimulated T cell binds to CD40 on APCs and activates the APCs to express more B7 costimulators and to secrete cytokines (e.g., interleukin-12 (IL-12)) that enhance T cell differentiation. Thus, the CD40L-CD40 interaction promotes T cell activation by making APCs better at stimulating T cells. The role of costimulation in T cell activation explains an observation mentioned in earlier chapters. Protein antigens, such as those used in vaccines, fail to elicit T cell-dependent immune responses unless these antigens are administered with substances that activate APCs, especially dendritic cells. Such substances are called adjuvants, and they function mainly by inducing the expression of costimulators on APCs and by stimulating the APCs to secrete cytokines that activate T cells. Most adjuvants are products of microbes (e.g., killed mycobacteria, which is often used in experimental studies) or substances that mimic microbes, and they bind to pattern recognition receptors of the innate immune system, such as Toll-like receptors and NOD-like receptors (see Chapter 2). Thus, adjuvants trick the immune system into responding to purified protein antigens in a vaccine as if these proteins were parts of infectious microbes The increasing understanding of costimulators has led to new strategies for inhibiting harmful immune responses. Agents that block B7:CD28 interactions are used in the treatment of rheumatoid arthritis, other inflammatory diseases, and graft rejection, and antibodies that block CD40:CD40L interactions are being tested in inflammatory diseases and to treat graft rejection

treatment

The goal of treatment is induction and maintenance of remission. As part of the initial management of IBD, the clinician must determine the extent and assess the severity of the disease. Patients with mild or moderate disease can be managed as outpatients. Patients with severe or fulminant disease—with abdominal pain, fever, tachycardia, anemia, and leukocytosis—require hospital admission and multidisciplinary team management. Because IBD is a chronic recurrent illness, treatment is centered on controlling the acute attack with induction of remission, followed by maintenance of remission. Treatment options for UC and Crohn's disease are summarized in Table 37-3.

What can the abundance and composition of normal commensal microbes in the gut, skin, and other sites (the microbiome) influence?

The health of the immune system and the maintenance of self-tolerance

portosystemic anastamosis

The hepatic portal system drains blood from the visceral organs of the abdomen to the liver. In normal individuals, 100% of the portal venous blood flow can be recovered from the hepatic veins, whereas in patients with elevated portal vein pressure (e.g., from cirrhosis), there is significantly less blood flow to the liver. The rest of the blood enters collateral channels, which drain into the systemic circulation at specific points (Fig. 4.133). The largest of these collaterals occur at: ■ the gastroesophageal junction around the cardia of the stomach—where the left gastric vein and its tributaries form a portosystemic anastomosis with tributaries to the azygos system of veins of the caval system; ■ the anus—the superior rectal vein of the portal system anastomoses with the middle and inferior rectal veins of the systemic venous system; and ■ the anterior abdominal wall around the umbilicus—the para-umbilical veins anastomose with veins on the anterior abdominal wall. When the pressure in the portal vein is elevated, venous enlargement (varices) tend to occur at and around the sites of portosystemic anastomoses and these enlarged veins are called: ■ varices at the anorectal junction, ■ esophageal varices at the gastroesophageal junction, and ■ caput medusae at the umbilicus Esophageal varices are susceptible to trauma and, once damaged, may bleed profusely, requiring urgent surgical intervention.

conclusion

The immune system intricately regulates the response to tissue injury and in ection. A complete review o immunology is beyond the scope o this text; instead, the discussion in this chapter presents a broad overview and highlights elements o immunity that may be addressed pharmacologically. Innate immune mechanisms respond to patterned elements shared among a class o in ectious agents, such as bacterial lipopolysaccharide or viral RNA. The innate immune system also processes these agents and presents them to lymphocytes, thereby activating the adaptive immune system The adaptive immune system develops a response speci c to an in ectious agent or inf ammatory stimulus. As part o the inf ammatory response, the adaptive immune response also includes mechanisms that mediate tolerance to distinguish sel rom nonsel ; dysregulation o these mechanisms may lead to chronic inf ammation and autoimmune disease. Many anti-inf ammatory drugs act to deplete populations o innate or adaptive immune cells; this concept is discussed in more detail in Chapter 46 The chemical mediators o the inf ammatory response—including histamine, complement, eicosanoids, and cytokines—are also major targets o current pharmacologic therapies. Macromolecular drugs are playing an increasingly important role in modulation o these chemical mediators; or example, anticytokine antibodies that inhibit tumor necrosis actor- have been developed or the treatment o rheumatoid arthritis, psoriatic arthritis, and inf ammatory bowel disease. A second approach to the modulation o inf ammatory responses is to target the intracellular signaling cascades responsible or the initiation o immune responses. Cyclosporine, discussed in Chapter 46, is an example o such a drug. As the number o agents available or treatment o immune disorders grows, it will also be important to determine whether macromolecular agents and small-molecule signaling inhibitors can be used in combination to target multiple steps in inflammation

Immunomodulators

The immunomodulators used in IBD include azathioprine (Imuran, 2 to 2.5 mg/kg/day) and its active metabolite, 6-mercaptopurine (6-MP) (Purinethol, 1 to 1.5 mg/kg/day), as well as methotrexate and cyclosporine. Azathioprine and 6-MP are effective therapies for maintaining remission in both Crohn's disease and UC and are used primarily as steroid-sparing agents (level of evidence I, A). They have a slow onset of action (weeks to months). Side effects include nausea, abnormal liver enzymes, bone marrow suppression, opportunistic infections, and an increased risk of lymphoma and nonmelanoma skin cancer Methotrexate can be used for induction (25 mg subcutaneously once weekly) and maintenance of remission (15 to 25 mg subcutaneously once weekly) in active Crohn's disease (level of evidence IIa, B for induction and I, B for maintenance); the side effect profile is similar but also includes interstitial pneumonitis. Intravenous cyclosporine (2 mg/kg/day given over 24 hours) is used as a rescue medicine and, in severe UC refractory to intravenous steroids, as a bridge treatment to one of the above immunomodulators or biologic agents. Given the potential for both short-term and long-term side effects, as well as the need for close follow-up, patients needing these medications are best managed by gastroenterologists.

What is the odds ratio or relative risk of an HLA-disease association?

The incidence of a particular autoimmune disease often is greater among individuals who inherit a particular HLA allele(s) than in the general population.

inferior mesenteric artery

The inferior mesenteric artery is the anterior branch of the abdominal aorta that supplies the hindgut. It is the smallest of the three anterior branches of the abdominal aorta and arises anterior to the body of vertebra LIII. Initially, the inferior mesenteric artery descends anteriorly to the aorta and then passes to the left as it continues inferiorly (Fig. 4.128). Its branches include the left colic artery, several sigmoid arteries, and the superior rectal artery

inferior mesenteric vein

The inferior mesenteric vein drains blood from the rectum, sigmoid colon, descending colon, and splenic flexure (Fig. 4.132). It begins as the superior rectal vein and ascends, receiving tributaries from the sigmoid veins and the left colic vein. All these veins accompany arteries of the same name. Continuing to ascend, the inferior mesenteric vein passes posterior to the body of the pancreas and usually joins the splenic vein. Occasionally, it ends at the junction of the splenic and superior mesenteric veins or joins the superior mesenteric vein

inferior pancreatoduodenal artery

The inferior pancreaticoduodenal artery is the first branch of the superior mesenteric artery. It divides immediately into anterior and posterior branches, which ascend on the corresponding sides of the head of the pancreas. Superiorly, these arteries anastomose with anterior and posterior superior pancreaticoduodenal arteries (see Figs. 4.125 and 4.126). This arterial network supplies the head and uncinate process of the pancreas and the duodenum

antigen recognition and costimulation

The initiation of T cell responses requires multiple receptors on the T cells recognizing ligands on APCs (Fig. 5-4). • The T cell receptor (TCR) recognizes MHCassociated peptide antigens. • CD4 or CD8 coreceptors on the T cells recognize MHC molecules on the APC and help the TCR complex to deliver activating signals. • Adhesion molecules strengthen the binding of T cells to APCs. • Molecules called costimulators, which are expressed on APCs after encounter with microbes, bind to costimulatory receptors on the naive T cells thus promoting responses to infectious pathogens. • Cytokines amplify the T cell response and direct it along various differentiation pathways. The roles of these molecules in T cell responses to antigens are described next. Cytokines are discussed mainly in Chapter 6.

What do Jejunal and ileal arteries supply?

The jejunum and most of the ileum These branches leave the main trunk of the artery, pass between two layers of the mesentery, and form anastomosing arches or arcades as they pass outward to supply the small intestine. The number of arterial arcades increases distally along the gut.

large intestine

The large intestine extends from the distal end of the ileum to the anus, a distance of approximately 1.5 m in adults. It absorbs fluids and salts from the gut contents, thus forming feces, and consists of the cecum, appendix, colon, rectum, and anal canal ( Beginning in the right groin as the cecum, with its associated appendix, the large intestine continues upward as the ascending colon through the right flank and into the right hypochondrium (Fig. 4.81). Just below the liver, it bends to the left, forming the right colic flexure (hepatic flexure), and crosses the abdomen as the transverse colon to the left hypochondrium. At this position, just below the spleen, the large intestine bends downward, forming the left colic flexure (splenic flexure), and continues as the descending colon through the left flank and into the left groin It enters the upper part of the pelvic cavity as the sigmoid colon, continues on the posterior wall of the pelvic cavity as the rectum, and terminates as the anal canal. The general characteristics of most of the large intestine (Fig. 4.79) are: ■ its large internal diameter compared to that of the small intestine; ■ peritoneal-covered accumulations of fat (the omental appendices) are associated with the colon; ■ the segregation of longitudinal muscle in its walls into three narrow bands (the taeniae coli), which are primarily observed in the cecum and colon and less visible in the rectum; and ■ the sacculations of the colon (the haustra of the colon).

left colic artery

The left colic artery is the first branch of the inferior mesenteric artery It ascends retroperitoneally, dividing into ascending and descending branches: ■ The ascending branch passes anteriorly to the left kidney, then enters the transverse mesocolon, and passes superiorly to supply the upper part of the descending colon and the distal part of the transverse colon; it anastomoses with branches of the middle colic artery. ■ The descending branch passes inferiorly, supplying the lower part of the descending colon, and anastomoses with the first sigmoid artery.

left gastric artery

The left gastric artery is the smallest branch of the celiac trunk. It ascends to the cardioesophageal junction and sends esophageal branches upward to the abdominal part of the esophagus (Fig. 4.123). Some of these branches continue through the esophageal hiatus of the diaphragm and anastomose with esophageal branches from the thoracic aorta. The left gastric artery itself turns to the right and descends along the lesser curvature of the stomach in the lesser omentum. It supplies both surfaces of the stomach in this area and anastomoses with the right gastric artery.

What does the left gastric artery turn into? What does it supply?

The left gastric artery itself turns to the right gastric and descends along the lesser curvature of the stomach in the lesser omentum It supplies both surfaces of the stomach in this area and anastomoses with the right gastric artery.

What do the signs and symptoms of obstruction depend on?

The level at which the obstruction has occured

What is Immunogenic vs. Tolerogenic?

The lymphocytes may be activated to proliferate and to differentiate into effector and memory cells, leading to a productive immune response; antigens that elicit such a response are said to be immunogenic The lymphocytes may be functionally inactivated or killed, resulting in tolerance; antigens that induce tolerance are said to be tolerogenic In some situations, the antigenspecific lymphocytes may not react in any way; this phenomenon has been called immunological ignorance microbes are immunogenic and self antigens are tolerogenic.

adaptive immunity

The main eatures o the adaptive immune system, speci city to oreign antigens and tolerance to sel -antigens, rely on two principles. First, there must be a mechanism to generate a speci c response to a oreign antigen. Second, adaptive immune cells must be able to distinguish native (sel ) cells and soluble actors rom oreign (nonsel ) cells and soluble actors. The rst property is provided by major histocompatibility complex (MHC) proteins and by somatic gene recombination in T cells and B cells, whereas the second property is provided by signals rom the innate immune system, by regulated immune cell development, and by costimulation.

Malabsorption- Definition and epidemiology

The main purpose of the gastrointestinal (GI) tract is to digest and absorb major nutrients (fat, carbohydrate, and protein), essential micronutrients (vitamins and trace minerals), water, and electrolytes. Impaired absorption of these nutrients is defined as malabsorption. Under normal conditions, the digestion and absorption of nutrients requires both mechanical and enzymatic breakdown of food. Mechanical processes include chewing, gastric churning, and the to-and-fro mixing in the small intestine. Enzymatic hydrolysis is initiated by intraluminal processes requiring salivary, gastric, pancreatic, and biliary secretions and is completed at the intestinal brush border. The final products of digestion are then absorbed through the intestinal epithelial cells and transported into the portal circulation. The coordinated regulation of gastric emptying, normal intestinal progression, and the presence of adequate intestinal surface area are all important factors. The human gut microbiome, which comprises the communities of microorganisms that inhabit the GI tract, has been recognized to play an important role in nutrient utilization as well. From birth, interactions between the microbiotia and the intestinal mucosa contribute to maturation of the host immune system. Disruptions to the homeostasis between the microbiota and the host immune system can lead to increased inflammation and decreased absorption. Most dietary components can be absorbed anywhere along the length of the small intestine, but there are important exceptions in which absorption is limited to specific areas (e.g., vitamin B12 and cholesterol are absorbed only in the terminal ileum). Diseases associated with diffuse mucosal involvement, such as celiac disease, can lead to impaired absorption of many nutrients, whereas diseases affecting only the terminal ileum can lead to decreased vitamin B12 absorption. Bile acids are necessary for fat absorption; they undergo an enterohepatic circulation with release into bile and reabsorption from the terminal small intestine. Diseases interfering with this mechanism deplete the bile acid pool and can lead to fat malabsorption. Water and electrolytes are absorbed primarily by the colon. In addition, there is caloric salvage of much of the carbohydrate from indigestible fiber through bacterial enzymatic activity in the colon. The following sections discuss normal assimilation of the major nutrients and the approach to evaluation of patients with suspected malabsorption.

biochemical pathways of t cell activation

The major signaling pathways linked to ζ-chain phosphorylation and ZAP-70 are the calcium-NFAT pathway, the Ras- and Rac-MAP kinase pathways, the PKCθ-NF-κB pathway, and the PI-3 kinase pathway: • Nuclear factor of activated T cells (NFAT) is a transcription factor present in an inactive phosphorylated form in the cytoplasm of resting T cells. NFAT activation and its nuclear translocation depend on the concentration of calcium (Ca2+) ions in the cytosol. This signaling pathway is initiated by ZAP-70-mediated phosphorylation and activation of an enzyme called phospholipase Cγ (PLCγ), which catalyzes the hydrolysis of a plasma membrane inositol phospholipid called phosphatidylinositol 4,5-bisphosphate (PIP2). One by-product of PLCγ-mediated PIP2 breakdown, called inositol 1,4,5-triphosphate (IP3), binds to IP3 receptors on the endoplasmic reticulum (ER) membrane and stimulates release of Ca2+ from the ER, thereby raising the cytosolic Ca2+ concentration. In response to the loss of calcium from intracellular stores, a plasma membrane calcium channel is opened, leading to the influx of extracellular Ca2+ into the cell, which further increases the intracellular Ca2+ concentration and sustains this for hours. The elevated cytoplasmic Ca2+ leads to activation of a phosphatase called calcineurin. This enzyme removes phosphates from cytoplasmic NFAT, enabling the transcription factor to migrate into the nucleus, where it binds to and activates the promoters of several genes, including the genes encoding the T cell growth factor IL-2 and components of the IL-2 receptor. A drug called cyclosporine inhibits calcineurin's phosphatase activity, and thus suppresses the NFAT-dependent production of cytokines by T cells. This agent is widely used as an immunosuppressive drug to prevent graft rejection; its introduction was one of the major factors in the success of organ transplantation (see Chapter 10). • The Ras/Rac-MAP kinase pathways include the guanosine triphosphate (GTP)- binding Ras and Rac proteins, several adaptor proteins, and a cascade of enzymes that eventually activate one of a family of mitogen-activated protein (MAP) kinases. These pathways are initiated by ZAP-70-dependent phosphorylation and accumulation of adaptor proteins at the plasma membrane, leading to the recruitment of Ras or Rac, and their activation by exchange of bound guanosine diphosphate (GDP) with GTP. Ras•GTP and Rac•GTP, the active forms of these proteins, initiate different enzyme cascades, leading to the activation of distinct MAP kinases. The terminal MAP kinases in these pathways, called extracellular signal-regulated kinase (ERK) and c-Jun amino-terminal (N-terminal) kinase (JNK), respectively, induce the expression of a protein called c-Fos and the phosphorylation of another protein called c-Jun. c-Fos and phosphorylated c-Jun combine to form the transcription factor activating protein 1 (AP-1), which enhances the transcription of several T cell genes. • Another major pathway involved in TCR signaling consists of activation of the θ isoform of the serine-threonine kinase called protein kinase C (PKCθ), which leads to activation of the transcription factor nuclear factor-κB (NF-κB).PKC is activated by diacylglycerol, which, like IP3, is generated by PLC-mediated hydrolysis of membrane inositol lipids. PKCθ acts through adaptor proteins recruited to the TCR complex to activate NF-κB. NF-κB exists in the cytoplasm of resting T cells in an inactive form, bound to an inhibitor called IκB. TCRinduced signals downstream of PKCθ activate a kinase that phosphorylates IκB and targets it for destruction. As a result, NF-κB is released and moves to the nucleus, where it promotes the transcription of several genes. • T cell receptor signal transduction also involves a lipid kinase called phosphatidylinositol-3 (PI-3) kinase, which phosphorylates the membrane phospholipid PIP2 to generate PIP3. PIP3 is required for the activation of a number of targets, including a serine-threonine kinase called Akt, or protein kinase B, which has many roles, including stimulating expression of antiapoptotic proteins and thus promoting survival of antigen-stimulated T cells. The PI-3 kinase/Akt pathway is triggered not only by the TCR but also by CD28 and IL-2 receptors. Closely linked to the Akt pathway is mTOR (mammalian target of rapamycin), a serine-threonine kinase that is involved in stimulating protein translation and promoting cell survival and growth. A drug that binds to and inactivates mTOR—rapamycin—is used to treat graft rejection. The various transcription factors that are induced or activated in T cells, including NFAT, AP-1, and NF-κB, stimulate transcription and subsequent production of cytokines, cytokine receptors, cell cycle inducers, and effector molecules such as CD40L (see Fig. 5-9). All of these signals are initiated by antigen recognition, because binding of the TCR and coreceptors to peptide-MHC complexes is necessary to initiate signaling in T cells. As stated earlier, recognition of costimulators, such as B7 molecules, by their receptor CD28 is essential for full T cell responses. The biochemical signals transduced by CD28 on binding to B7 costimulators are less well defined than are TCR-triggered signals. CD28 engagement likely amplifies some TCR signaling pathways that are triggered by antigen recognition (signal 1) and also initiates a distinct set of signals that complement TCR signals ed with a profound change in metabolic pathways. In naive (resting) T cells, low levels of glucose are taken up and used to generate energy in the form of ATP, by mitochondrial oxidative phosphorylation. Upon activation, glucose uptake increases markedly, and the cells switch to aerobic glycolysis. This process generates less ATP but facilitates the synthesis of more amino acids, lipids, and other molecules that provide building blocks for organelles and for producing new cells. As a result, it is possible for activated T cells to more efficiently manufacture the cellular constituents that are needed for their rapid increase in size and for producing daughter cells. Having described the stimuli and biochemical pathways in T cell activation, we now discuss how T cells respond to antigens and differentiate into effector cells capable of combating microbes.

What is used to estimate malabsorption of fat or bile acids ?

The measurement of radioactive carbon dioxide in the breath after ingestion of a nutrient labeled with carbon 14 (C14) also used for measurement of bacterial overgrowth (14 C- xylose)

Tolerance to Commensal Microbes in the Intestines and Skin

The microbiome of healthy humans consists of about 1014 bacteria and viruses (which is 10 times the number of human cells, prompting microbiologists to point out that we are only 10% human and 90% microbial!). These microbes reside in the intestinal and respiratory tracts and on the skin, where they serve many essential functions. For instance, in the gut, the normal bacteria aid in digestion and absorption of foods and prevent overgrowth of potentially harmful organisms. Mature lymphocytes in these tissues are capable of recognizing the organisms but do not react against them, so the microbes are not eliminated, and harmful inflammation is not triggered. In the gut, several mechanisms account for the inability of the healthy immune system to react against commensal microbes. These mechanisms include an abundance of IL-10-producing regulatory T cells, an unusual property of dendritic cells such that signaling from some Toll-like receptors leads to inhibition rather than activation, and separation of some bacteria from the intestinal immune system by the epithelium. The mechanisms that maintain tolerance to commensal bacteria in the skin are not as well defined.

middle colic artery

The middle colic artery is the first of the three branches from the right side of the main trunk of the superior mesenteric artery (Fig. 4.127). Arising as the superior mesenteric artery emerges from beneath the pancreas, the middle colic artery enters the transverse mesocolon and divides into right and left branches. The right branch anastomoses with the right colic artery while the left branch anastomoses with the left colic artery, which is a branch of the inferior mesenteric artery

conditions associated with bacterial overgrowth

The most important factors maintaining the relative sterility of the upper gut are gastric acidity, peristalsis, and intestinal immunoglobulins (IgA). Conditions that impair these functions can result in bacterial overgrowth. Impaired peristalsis may be caused by motility disorders (e.g., scleroderma, amyloidosis, diabetes mellitus) or by anatomic changes (e.g., surgically created blind loops, obstruction, jejunal diverticulosis). Achlorhydria, pancreatic insufficiency, and hypogammaglobulinemia are also associated with bacterial overgrowth but uncommonly result in clinical steatorrhea.

What is the choice between lymphocyte activation and tolerance determined largely by?

The nature of the antigen and the additional signals present when the antigen is displayed to the immune system

Congential disorders of the GI tract

The normal positions of the abdominal viscera result from a complex series of rotations that the gut tube undergoes and from the growth of the abdominal cavity to accommodate changes in the size of the developing organs (see pp. 265-268). A number of developmental anomalies can occur during gut development, many of which appear in the neonate or infant, and some of which are surgical emergencies. Occasionally, such disorders are diagnosed only in adults

What is the physical appearance of hyperacute rejection? What does this change occur as a result of?

The normal, healthy, pink appearance of the transplanted organ rapidly becomes cyanotic, mottled, and flaccid. complement activation by antibody binding to endothelial cells of the transplanted organ, resulting in thrombosis and ischemia.

What is Hematochezia?

The passage of bright-red blood or maroon stools per rectum frequently indicates a lower GI source of bleeding. 10% to 15% of patients with acute severe hematochezia have an upper GI source of brisk bleeding. This group of patients commonly displays signs of hemodynamic instability.

Irritable bowel syndrome - pathogenesis

The pathogenesis of IBS remains poorly defined, although there is clearly interplay between psychologic stressors, diet, perturbation of the gut microbiome, increased enteric sensory responses to gastrointestinal stimuli, and abnormal GI motility. For example, patients with constipation-predominant or diarrhea-predominant IBS tend to have decreased or increased colonic contractions and transit rates, respectively. Excess bile acid synthesis or bile acid malabsorption has been identified as one cause of diarrhea-predominant IBS, likely due to the effects of bile acids on intestinal motility. Other data link disturbances in enteric nervous system function to IBS, suggesting a role for defective brain-gut axis signaling. Consistent with this, deep sequencing and genome wide association studies have linked several candidate genes to IBS, including serotonin reuptake transporters, cannabinoid receptors, and TNF-related inflammatory mediators. Further, 5-HT3 receptor anatagonists are effective in many cases of diarrhea-predominant IBS. Opioids and psychoactive drugs with anti-cholinergic effects are also commonly used to treat diarrhea- predominant IBS A separate group of IBS patients, relate onset to a bout of infectious gastroenteritis, suggesting that immune activation or, alternatively, a shift in the gut microbiome may trigger some cases. While unproven, this could explain the efficacy of fecal transplantation in some IBS cases There may be some overlap in mechanisms underlying constipation-predominant and diarrhea-predominant IBS. For example, single nucleotide polymorphisms in immune mediators have been detected in both.

sympathetic innervation of the stomach

The pathway of sympathetic innervation of the stomach is as follows: ■ A preganglionic sympathetic fiber originating at the T6 level of the spinal cord enters an anterior root to leave the spinal cord. ■ At the level of the intervertebral foramen, the anterior root (which contains the preganglionic fiber) and a posterior root join to form a spinal nerve. ■ Outside the vertebral column, the preganglionic fiber leaves the anterior ramus of the spinal nerve through the white ramus communicans . ■ The white ramus communicans, containing the preganglionic fiber, connects to the sympathetic trunk. ■ Entering the sympathetic trunk, the preganglionic fiber does not synapse but passes through the trunk and enters the greater splanchnic nerve. ■ The greater splanchnic nerve passes through the crura of the diaphragm and enters the celiac ganglion. ■ In the celiac ganglion, the preganglionic fiber synapses with a postganglionic neuron. ■ The postganglionic fiber joins the plexus of nerve fibers surrounding the celiac trunk and continues along its branches. ■ The postganglionic fiber travels through the plexus of nerves accompanying the branches of the celiac trunk supplying the stomach and eventually reaches its point of distribution. ■ This input from the sympathetic system may modify the activities of the gastrointestinal tract controlled by the enteric nervous system.

What is Hematemesis?

The patient vomits bright red blood or material that resembles coffee grounds, representing partially digested blood.

Irritable Bowel Syndrome- clinical features

The peak prevalence of IBS is between 20 and 40 years of age, and there is a significant female predominance. Variability in diagnostic criteria makes it difficult to establish the incidence, but most authors report prevalence in developed countries of between 5% and 10%. IBS is presently diagnosed using clinical criteria that require the occurrence of abdominal pain or discomfort at least 3 days per month over 3 months with improvement following defecation and a change in stool frequency or form. Other causes, such as enteric infection or inflammatory bowel disease, must be excluded. IBS is not associated with serious long-term sequelae, but affected patients may undergo unnecessary abdominal surgery due to chronic pain and their ability to function socially may be compromised. The prognosis of IBS is most closely related to symptom duration, with longer duration correlating with reduced likelihood of improvement.

pelvic splanchnic nerves

The pelvic splanchnic nerves (parasympathetic root) are unique. They are the only splanchnic nerves that carry parasympathetic fibers. In other words, they do not originate from the sympathetic trunks. Rather, they originate directly from the anterior rami of S2 to S4. Preganglionic parasympathetic fibers originating in the sacral spinal cord pass from the S2 to S4 spinal nerves to the inferior hypogastric plexus (Fig. 4.136). Once in this plexus, some of these fibers pass upward, enter the abdominal prevertebral plexus, and distribute with the arteries supplying the hindgut. This provides the pathway for innervation of the distal one-third of the transverse colon, the descending colon, and the sigmoid colon by preganglionic parasympathetic fibers.

the pelvic splanchnic nerves

The pelvic splanchnic nerves, carrying preganglionic parasympathetic fibers from S2 to S4 spinal cord levels, enter the inferior hypogastric plexus in the pelvis. Some of these fibers move upward into the inferior mesenteric part of the prevertebral plexus in the abdomen (Fig. 4.138). Once there, these fibers are distributed with branches of the inferior mesenteric artery and provide parasympathetic innervation to the hindgut.

portal vein

The portal vein is the final common pathway for the transport of venous blood from the spleen, pancreas, gallbladder, and abdominal part of the gastrointestinal tract. It is formed by the union of the splenic vein and the superior mesenteric vein posterior to the neck of the pancreas at the level of vertebra L2 Ascending toward the liver, the portal vein passes posterior to the superior part of the duodenum and enters the right margin of the lesser omentum. As it passes through this part of the lesser omentum, it is anterior to the omental foramen and posterior to both the bile duct, which is slightly to its right, and the hepatic artery proper, which is slightly to its left On approaching the liver, the portal vein divides into right and left branches, which enter the liver parenchyma. Tributaries to the portal vein include: ■ right and left gastric veins draining the lesser curvature of the stomach and abdominal esophagus, ■ cystic veins from the gallbladder, and ■ the para-umbilical veins, which are associated with the obliterated umbilical vein and connect to veins on the anterior abdominal wall

What is the mechanism of action of Methotrexate?

The precise mechanism by which methotrexate exerts its anti-inflammatory effect is uncertain Although methotrexate inhibits dihydrofolate reductase, the combination of methotrexate and low-dose folate is as effective as methotrexate alone in the treatment of rheumatoid arthritis.

Pathogenesis of autoimmune diseases

The principal factors in the development of autoimmunity are the inheritance of susceptibility genes and environmental triggers, such as infections (Fig. 9-10). It is postulated that susceptibility genes interfere with pathways of self-tolerance and lead to the persistence of self-reactive T and B lymphocytes. Environmental stimuli may cause cell and tissue injury and inflammation and activate these selfreactive lymphocytes, resulting in the generation of effector T cells and autoantibodies that are responsible for the autoimmune disease Despite our growing knowledge of the immunological abnormalities that may result in autoimmunity, we still do not know the etiology of common human autoimmune diseases. This lack of understanding results from several factors: autoimmune diseases in humans usually are heterogeneous and multifactorial; the self antigens that are the inducers and targets of the autoimmune reactions often are unknown; and the diseases may manifest clinically long after the autoimmune reactions have been initiated. Recent advances, including the identification of disease-associated genes, better techniques for studying antigenspecific immune responses in humans, and the analysis of animal models that can be extrapolated to clinical situations, hold promise for providing answers to the enigma of autoimmunity.

Central T lymphocyte tolerance

The principal mechanisms of central tolerance in T cells are death of immature T cells and the generation of CD4+ regulatory T cells (Fig. 9-2). The lymphocytes that develop in the thymus consist of cells with receptors capable of recognizing many antigens, both self and foreign. If a lymphocyte that has not completed its maturation interacts strongly with a self antigen, displayed as a peptide bound to a self major histocompatibility complex (MHC) molecule, that lymphocyte receives signals that trigger apoptosis. Thus, the self-reactive cell dies before it can become functionally competent. This process, called negative selection (see Chapter 4), is a major mechanism of central tolerance. The process of negative selection affects self-reactive CD4+ T cells and CD8+ T cells, which recognize self peptides displayed by class II MHC and class I MHC molecules, respectively. It is not known why immature lymphocytes die upon receiving strong T cell receptor (TCR) signals in the thymus, whereas mature lymphocytes that get strong TCR signals in the periphery are activated. Some immature CD4+ T cells that recognize self antigens in the thymus with high affinity do not die but develop into regulatory T cells and enter peripheral tissues (see Fig. 9-2). The functions of regulatory T cells are described later in the chapter. What determines whether a thymic CD4+ T cell that recognizes a self antigen will die or become a regulatory T cell is also not known. Immature lymphocytes may interact strongly with an antigen if the antigen is present at high concentrations in the thymus and if the lymphocytes express receptors that recognize the antigen with high affinity. Antigens that induce negative selection may include proteins that are abundant throughout the body, such as plasma proteins and common cellular proteins. Surprisingly, many self proteins that are normally present only in certain peripheral tissues are also expressed in some of the epithelial cells of the thymus. A protein called AIRE (autoimmune regulator) is responsible for the thymic expression of these peripheral tissue antigens. Mutations in the AIRE gene are the cause of a rare disorder called autoimmune polyendocrine syndrome. In this disorder, several tissue antigens are not expressed in the thymus because of a lack of functional AIRE protein, so immature T cells specific for these antigens are not eliminated and do not develop into regulatory cells, remaining capable of reacting harmfully against the self antigens. These antigens are expressed normally in the appropriate peripheral tissues (because only thymic expression is under the control of AIRE). Therefore, T cells specific for these antigens emerge from the thymus, encounter the antigens in the peripheral tissues, and attack the tissues and cause disease. It is not clear why endocrine organs are the major targets of this autoimmune attack. Although this rare syndrome illustrates the importance of negative selection in the thymus for maintaining self-tolerance, it is not known if defects in negative selection contribute to common autoimmune diseases. Negative selection is imperfect, and numerous self-reactive lymphocytes are present in healthy individuals. As discussed next, peripheral mechanisms may prevent the activation of these lymphocytes.

prognosis

The prognosis in a patient with IBD is determined by the relapse rate, the rate of surgery, and the incidence of colon cancer. Approximately two thirds of patients with UC have at least one relapse in the 10 years after their diagnosis. About 20% to 30% of patients with pan-UC will require colectomy within their lifetime. Only 5% of individuals with proctitis undergo colectomy by 10 years after diagnosis. In contrast, more than 60% of Crohn's patients require surgery within the 10 years after their diagnosis. The rate of recurrence in Crohn's disease is high, with 70% of patients having an endoscopic recurrence within 1 year after surgery and 50% having a symptomatic recurrence within 4 years. Predictors of a severe course in Crohn's disease include stricturing or penetrating disease and perianal disease The risk for colon cancer is increased in patients with UC, and its magnitude is related to the extent and duration of disease. The colon cancer risk is increased 10- to 20-fold after 8 to 10 years of disease in pancolitis, and after 15 to 20 years in left-sided colitis. The cumulative incidence of colorectal cancer is 2.5% after 20 years and 7.6% after 30 years of disease. Proctitis is not associated with an increased risk of colorectal cancer. In colonic Crohn's disease, the risk of colorectal cancer is equivalent to that in patients with UC of similar extent and duration. Patients with only small bowel Crohn's disease are not thought to be at increased risk for colorectal cancer. The rates of small bowel carcinoma and lymphoma are increased in patients with Crohn's disease. Screening for dysplasia and colon cancer should be performed by colonoscopy 8 to 10 years after the onset of symptoms. Surveillance examinations are performed every 1 to 3 years. Proctitis does not require endoscopic surveillance. Patients with IBD and PSC appear to have a particularly increased risk for colon cancer, and yearly surveillance is recommended after the initial diagnosis of PSC. It is recommended that a minimum of 33 "random" mucosal biopsy samples be obtained during the colonoscopic examination, in addition to targeted samples of circumscript lesions. The use of chromoendoscopy and other enhanced imaging techniques increases the detection of dysplastic lesions in patients with UC and may replace the performance of random biopsies in the future. Colectomy is indicated in patients with flat high-grade dysplasia, multifocal flat low-grade dysplasia, or evidence of colorectal cancer. Polypoid dysplasia entirely removed by polypectomy without flat dysplasia elsewhere in the colon can be managed with continued surveillance colonoscopy As understanding of the etiologic and pathophysiologic aspects of IBD increases, major advances in diagnosis and treatment are anticipated. These will be based on better use of molecular, genetic, and serologic tests to differentiate among the subtypes of disease; earlier and more targeted use of biological agents to manage inflammation; and improvements in the detection and prevention of colorectal cancer in those at risk.

What will happen if a foreign (eg: viral) protein is being generated within a cell?

The proteolytic fragments of these viral proteins will be displayed on MHC class I proteins at the surface of the cell, and the immune system will recognize that cell as virally infected.

Bacterial overgrowth syndrome

The proximal small bowel normally contains fewer than 104 bacteria per milliliter of fluid, with no anaerobic Bacteroides organisms and few coliforms. Overgrowth of luminal bacteria can result in diarrhea and malabsorption by a number of mechanisms, including (1) deconjugation of bile salts, which leads to impaired micelle formation and impaired uptake of fat; (2) patchy injury to the enterocytes (small intestinal epithelial cells); (3) direct competition for the use of nutrients (e.g., uptake of vitamin B12 by gram-negative bacteria or the fish tapeworm Diphyllobothrium latum); and (4) stimulated secretion of water and electrolytes by products of bacterial metabolism, such as hydroxylated bile acids and short-chain (volatile) organic acids.

Functional responses of T lymphocytes to antigen and stimulation

The recognition of antigen and costimulators by T cells initiates an orchestrated set of responses that culminate in the expansion of the antigen-specific clones of lymphocytes and the differentiation of the naive T cells into effector cells and memory cells (see Fig. 5-3). Many of the responses of T cells are mediated by cytokines that are secreted by the T cells and act on the T cells themselves and on many other cells involved in immune defenses. Each component of the biologic responses of T cells is discussed next

blockade of cell adhesion

The recruitment and accumulation o in ammatory cells at sites o in ammation is an essential element o most autoimmune diseases; the only exceptions to this rule are autoimmune diseases that are purely humoral, such as myasthenia gravis. Drugs that inhibit cell migration to sites o in ammation may also inhibit antigen presentation and cytotoxicity, thus providing multiple potential mechanisms o therapeutic action

What is the immune synapse?

The region of contact between the APC and T cell, including the redistributed membrane proteins Some effector molecules and cytokines may be secreted through this region, ensuring that they do not diffuse away but are targeted to the APC Enzymes that serve to degrade or inhibit signaling molecules are also recruited to the synapse, so it may be involved in terminating lymphocyte activation

What is the right colic artery?

The second of the three branches from the right side of the main trunk of the superior mesenteric artery Nearing the colon, it divides into a descending branch, which anastomoses with the ileocolic artery, and an ascending branch, which anastomoses with the middle colic artery.

What does the clinical presentation of Crohn's disease depend on?

The section of gastrointestinal tract involved and the type of inflammation Crohn's disease can involve any portion of the gastrointestinal tract; the most common site is ileocecal (40% of patients), followed by isolated small bowel disease (30%) and isolated colonic involvement (25%). The remaining sites of Crohn's disease are rare (5%) and include the esophagus, stomach, and duodenum.

sigmoid arteries

The sigmoid arteries consist of two to four branches, which descend to the left, in the sigmoid mesocolon, to supply the lowest part of the descending colon and the sigmoid colon (Fig. 4.128). These branches anastomose superiorly with branches from the left colic artery and inferiorly with branches from the superior rectal artery

treatment

The specific treatment of malabsorption depends on identification of the underlying condition. Occasionally, therapeutic trials for treatable conditions should be instituted, such as a gluten-free diet for celiac disease, pancreatic enzyme replacement for pancreatic exocrine malfunction, metronidazole for G. lamblia infection, or broad-spectrum antibiotics for suspected bacterial overgrowth. Parenteral nutrition may have a role in maintaining adequate nutritional status. Treatment modalities are discussed in later chapters focusing on specific diseases. Two disorders, celiac disease and bacterial overgrowth, are discussed here as illustrative of the pathophysiology

What pathologic changes take place in Celiac disease?

The spectrum of pathologic changes ranges from normal villous architecture with an increase in mucosal lymphocytes and plasma cells (the infiltrative lesion) to partial blunting or total villous flattening

splanchnic nerves

The splanchnic nerves are important components in the innervation of the abdominal viscera. They pass from the sympathetic trunk or sympathetic ganglia associated with the trunk, to the prevertebral plexus and ganglia anterior to the abdominal aorta. There are two different types of splanchnic nerves, depending on the type of visceral efferent fiber they are carrying: ■ The thoracic, lumbar, and sacral splanchnic nerves carry preganglionic sympathetic fibers from the sympathetic trunk to ganglia in the prevertebral plexus, and also visceral afferent fibers. ■ The pelvic splanchnic nerves carry preganglionic parasympathetic fibers from anterior rami of S2, S3, and S4 spinal nerves to an extension of the prevertebral plexus in the pelvis (the inferior hypogastric plexus or pelvic plexus).

splenic artery

The splenic artery, the largest branch of the celiac trunk, takes a tortuous course to the left along the superior border of the pancreas (Fig. 4.123). It travels in the splenorenal ligament and divides into numerous branches, which enter the hilum of the spleen. As the splenic artery passes along the superior border of the pancreas, it gives off numerous small branches to supply the neck, body, and tail of the pancreas Approaching the spleen, the splenic artery gives off short gastric arteries, which pass through the gastrosplenic ligament to supply the fundus of the stomach. It also gives off the left gastro-omental artery, which runs to the right along the greater curvature of the stomach, and anastomoses with the right gastro-omental artery

splenic vein

The splenic vein forms from numerous smaller vessels leaving the hilum of the spleen (Fig. 4.132). It passes to the right, passing through the splenorenal ligament with the splenic artery and the tail of the pancreas. Continuing to the right, the large, straight splenic vein is in contact with the body of the pancreas as it crosses the posterior abdominal wall. Posterior to the neck of the pancreas, the splenic vein joins the superior mesenteric vein to form the portal vein. Tributaries to the splenic vein include: -short gastric veins from the fundus and left part of the greater curvature of the stomach the left gastro-omental vein from the greater curvature of the stomach, ■ pancreatic veins draining the body and tail of the pancreas, and ■ usually the inferior mesenteric vein

superior mesenteric artery

The superior mesenteric artery is the anterior branch of the abdominal aorta supplying the midgut. It arises from the abdominal aorta immediately below the celiac artery (Fig. 4.126), anterior to the lower part of vertebra LI The superior mesenteric artery is crossed anteriorly by the splenic vein and the neck of the pancreas. Posterior to the artery are the left renal vein, the uncinate process of the pancreas, and the inferior part of the duodenum. After giving off its first branch (the inferior pancreaticoduodenal artery), the superior mesenteric artery gives off jejunal and ileal arteries on its left (Fig. 4.126). Branching from the right side of the main trunk of the superior mesenteric artery are three vessels—the middle colic, right colic, and ileocolic arteries—which supply the terminal ileum, cecum, ascending colon, and two-thirds of the transverse colon.

superior mesenteric vein

The superior mesenteric vein drains blood from the small intestine, cecum, ascending colon, and transverse colon (Fig. 4.132). It begins in the right iliac fossa as veins draining the terminal ileum, cecum, and appendix join, and ascends in the mesentery to the right of the superior mesenteric artery Posterior to the neck of the pancreas, the superior mesenteric vein joins the splenic vein to form the portal vein. As a corresponding vein accompanies each branch of the superior mesenteric artery, tributaries to the superior mesenteric vein include jejunal, ileal, ileocolic, right colic, and middle colic veins. Additional tributaries include: ■ the right gastro-omental vein, draining the right part of the greater curvature of the stomach, and ■ the anterior and posterior inferior pancreaticoduodenal veins, which pass alongside the arteries of the same name; the anterior superior pancreaticoduodenal vein usually empties into the right gastro-omental vein, and the posterior superior pancreaticoduodenal vein usually empties directly into the portal vein

sympathetic trunks

The sympathetic trunks are two parallel nerve cords extending on either side of the vertebral column from the base of the skull to the coccyx (Fig. 4.135). As they pass through the neck, they lie posterior to the carotid sheath. In the upper thorax, they are anterior to the necks of the ribs, while in the lower thorax they are on the lateral aspect of the vertebral bodies. In the abdomen, they are anterolateral to the lumbar vertebral bodies and, continuing into the pelvis, they are anterior to the sacrum. The two sympathetic trunks come together anterior to the coccyx to form the ganglion impar. Throughout the extent of the sympathetic trunks, small raised areas are visible. These collections of neuronal cell bodies outside the CNS are the paravertebral sympathetic ganglia. There are usually: ■ three ganglia in the cervical region, ■ eleven or twelve ganglia in the thoracic region, ■ four ganglia in the lumbar region , ■ four or five ganglia in the sacral region, and ■ the ganglion impar anterior to the coccyx The ganglia and trunks are connected to adjacent spinal nerves by gray rami communicantes throughout the length of the sympathetic trunk and by white rami communicantes in the thoracic and upper lumbar parts of the trunk (T1 to L2). Neuronal fibers found in the sympathetic trunks include preganglionic and postganglionic sympathetic fibers and visceral afferent fibers

What are the symptoms of Irritable bowel syndrome (IBS)?

The syndrome consists of abdominal distention, flatulence, and disordered bowel function The abdominal pain of IBS tends to be in the left lower quadrant

Mechanisms of malabsorption

The term maldigestion refers to defective hydrolysis of nutrients, whereas malabsorption refers to impaired mucosal absorption. In clinical practice, however, malabsorption refers to all aspects of impaired nutrient assimilation. Malabsorption can involve multiple nutrients, or it can be more selective. Therefore, the clinical manifestations of malabsorption are highly variable. The complete process of absorption consists of a luminal phase, in which various nutrients are hydrolyzed and solubilized; a mucosal phase, in which further processing takes place at the brush border of the epithelial cell with subsequent transfer into the cell; and a transport phase, in which nutrients are moved from the epithelium to the portal venous or lymphatic circulation. Impairment in any of these phases can result in malabsorption

superior rectal artery

The terminal branch of the inferior mesenteric artery is the superior rectal artery (Fig. 4.128). This vessel descends into the pelvic cavity in the sigmoid mesocolon, crossing the left common iliac vessels. Opposite vertebra SIII, the superior rectal artery divides. The two terminal branches descend on each side of the rectum, dividing into smaller branches in the wall of the rectum. These smaller branches continue inferiorly to the level of the internal anal sphincter, anastomosing along the way with branches from the middle rectal arteries (from the internal iliac artery) and the inferior rectal arteries (from the internal pudendal artery).

overview of the immune system

The undamental role o the immune system is to distinguish sel rom nonsel . "Nonsel " can be an in ectious organism, a transplanted organ, or an endogenous tissue that is mistaken or something oreign. Because protection against in ection is the classic role o the immune system, the terms in ection and in ectious agent are generally used to denote the inciting stimulus or an immune response. The immune system can be stimulated to react against any nonsel agent Skin and other barrier tissues orm the rst line o de ense against any in ection. (In the introductory case, Mark's in ection occurred only a ter he cut his skin.) Once an o ending agent penetrates these barriers, the immune system mounts a response. The immune response consists o innate and adaptive responses. Innate responses are stereotyped reactions to a stimulus (e.g., release o histamine, phagocytosis o a bacterium). In some cases, innate responses are su cient to neutralize the o ending agent. Cells o the innate immune system, especially antigen-presenting cells, can also process the o ending agent into small ragments; this processing is necessary or activation o the adaptive immune system. Adaptive responses are neutralizing reactions that are speci c to the o ending agent (e.g., antibodies, cytotoxic T cells). In general, then, the innate immune system recognizes nonsel and activates the response to an o ending nonsel agent; the adaptive immune system generates a response that specif cally neutralizes or kills that agent Many di erent cell types are involved in the immune system, and these cell types interact in a complex web o signaling and communication to create the overall response. The cells o the immune system derive rom two types o pluripotent cells in the bone marrow: myeloid stem cells and lymphoid stem cells. The lymphoid stem cell is sometimes called the common lymphoid stem cell because it gives rise to both B cells and T cells. In general, myeloid stem cells give rise to precursor cells o the innate immune system, whereas lymphoid stem cells generate precursor cells o the adaptive immune system; there are some exceptions. Figure 42-1 depicts the myeloid and lymphoid stem cells and the mature cell types into which the precursor cells di erentiate. The derivation o these cell types is also discussed in Chapter 45. A help ul conceptual ramework is to envision the innate immune system as the immunologic memory o a species , which is invariant over the li etime o an individual and generally the same among individuals o the species. In contrast, the adaptive immune system establishes the immunologic memory o an individual over his or her li etime, depending on that individual's exposure to pathogens, vaccines, or other immunologic stimuli. Adaptive immunity, there ore, is relatively unique to each individual.

vagus nerves

The vagus nerves [X] enter the abdomen associated with the esophagus as the esophagus passes through the diaphragm (Fig. 4.138) and provide parasympathetic innervation to the foregut and midgut After entering the abdomen as the anterior and posterior vagal trunks, they send branches to the abdominal prevertebral plexus. These branches contain preganglionic parasympathetic fibers and visceral afferent fibers, which are distributed with the other components of the prevertebral plexus along the branches of the abdominal aorta

Colostomy

There are a number of instances when a colostomy may be necessary. In many circumstances it is performed to protect the distal large bowel after surgery. A further indication would include large bowel obstruction with imminent perforation wherein a colostomy allows decompression of the bowel and its contents. This is a safe and temporizing procedure performed when the patient is too unwell for extensive bowel surgery. It is relatively straightforward and carries reduced risk, preventing significant morbidity and mortality An end colostomy is necessary when the patient has undergone a surgical resection of the rectum and anus (typically for cancer).

malabsorptive procedures

There are a variety of bypass procedures that produce a malabsorption state, preventing further weight gain and also producing weight loss. There are complications, which may include anemia, osteoporosis, and diarrhea (e.g., jejunoileal bypass).

lumbar and sacral splanchnic nerves

There are usually two to four lumbar splanchnic nerves, which pass from the lumbar part of the sympathetic trunk or associated ganglia and enter the prevertebral plexus Similarly, the sacral splanchnic nerves pass from the sacral part of the sympathetic trunk or associated ganglia and enter the inferior hypogastric plexus, which is an extension of the prevertebral plexus into the pelvis

What is the pathogenesis of IBS?

There is clearly interplay between psychologic stressors, diet, perturbation of the gut microbiome, increased enteric sensory responses to gastrointestinal stimuli, and abnormal GI motility

What happens once the cells of the immune system cross the endothelial barrier?

They migrate through the interstitium to the specific site of injury or infection

What does the vagus nerve branch into after entering the abdomen as the anterior and posterior vagal trunks?

They send branches to the abdominal prevertebral plexus.

What does differential diagnosis of abdominal pain require?

Thorough history taking with regard to pain characteristics, location and radiation, timing, and the presence of any accompanying symptoms Recognition of characteristic patterns is essential to narrowing the differential diagnosis.

thoracic splanchnic nerves

Three thoracic splanchnic nerves pass from sympathetic ganglia along the sympathetic trunk in the thorax to the prevertebral plexus and ganglia associated with the abdominal aorta in the abdomen ■ The greater splanchnic nerve arises from the fifth to the ninth (or tenth) thoracic ganglia and travels to the celiac ganglion in the abdomen (a prevertebral ganglion associated with the celiac trunk). ■ The lesser splanchnic nerve arises from the ninth and tenth (or tenth and eleventh) thoracic ganglia and travels to the aorticorenal ganglion. ■ The least splanchnic nerve, when present, arises from the twelfth thoracic ganglion and travels to the renal plexus.

What is the protein called AIRE (autoimmune regulator) responsible for?

Thymic expression of peripheral tissue antigens

Where do Regulatory T cells (T-regs) develop? What do they suppress the activation of?

Thymus or peripheral tissues on recognition of self antigens Potentially harmful lymphocytes specific for these self antigens The majority of self-reactive regulatory T cells probably develop in the thymus

resolution

Tissue repair and reestablishment o homeostasis are the nal events in the acute inf ammatory response. The same mediators that activate inf ammation also initiate a cascade o tissue repair; this process is mediated by the release o growth actors and cytokines, including epidermal growth actor (EGF), platelet-derived growth actor (PDGF), basic broblast growth actor-2 (bFGF-2), trans orming growth actor- 1 (TGF- 1), IL-1, and TNF- . These actors act as mitogens or endothelial cells and broblasts and ultimately stimulate healing and scar ormation through angiogenesis ( ormation o new blood vessels) and the ormation o granulation tissue. In the introductory case, the granulation tissue and eventual scar will be the only record o Mark's acute inf ammatory event. O note, angiogenesis can be a pathologic state when it is associated with abnormal blood vessel growth or tumor growth, and pharmacologic inhibitors o angiogenesis are currently being used to treat age-related macular degeneration (where abnormal blood vessels obscure vision) and as antineoplastic agents (see Chapter 40, Pharmacology o Cancer: Signal Transduction).

What are the final events in the acute inflammatory response?

Tissue repair and reestablishment of homeostasis

Where do monocytes that exit the bloodstream take up residence?

Tissues that differentiate into macrophages As professional APCs, macrophages process and present antigenic fragments of an invading pathogen or recognition by T cells.

What is the purpose of Gastrostomy?

To feed the patient when it is impossible to take food and fluid orally (e.g., complex head and neck cancer).

What is the use of Colostomy in many circumstances?

To protect the distal large bowel after surger A further indication would include large bowel obstruction with imminent perforation wherein a colostomy allows decompression of the bowel and its contents

What environmental factor is clearly associated with Inflammatory bowel disease?

Tobacco smoking Smoking seems to be protective against UC, whereas smoking in Crohn's disease causes a more aggressive disease. No dietary triggers have been found to cause IBD, but elemental diets and diversion of the fecal stream can reduce inflammation in Crohn's disease.

What can blocking of CD40L produce?

Tolerance blockade of CD40L with anti-CD40L antibody can indeed produce tolerance and long-term graft survival in animal models of organ transplantation

What do the transcription factors induced or activated in T cells (eg: NFAT, AP-1, NF-KB) stimulate?

Transcription and subsequent production of cytokines, cytokine receptors, cell cycle inducers, and effector molecules such as CD40L All of these signals are initiated by antigen recognition, because binding of the TCR and coreceptors to peptide-MHC complexes is necessary to initiate signaling in T cells.

What is the autoantigen of Celiac disease?

Transglutaminase

What is the inflammation in Crohn's disease classified as? what can it result in?

Transmural can result in microperforations and the formation of fistulous tracts Fistulas may form between different segments of bowel (e.g., enteroenteric, enterocolonic) or between bowel and skin (enterocutaneous), bowel and bladder (enterovesicular), or rectum and vagina (rectovaginal).

solid organ rejection

Transplant rejection o solid organs can be divided into three major phases according to the time to onset. These phases, hyperacute , acute , and chronic rejection , are caused by di erent mechanisms and are there ore treated differently. The following three sections examine each o these processes, and Table 46-1 summarizes their differences

What is Tacrolimus approved as an immunosuppressant for?

Transplantation A topical formulation is used or the treatment of atopic dermatitis and other eczematous diseases

What is allograft?

Transplantation occurring between unrelated individuals

What are esophageal varices susceptible to? What can happen once they are damaged?

Trauma may bleed profusely requiring surgical intervention

treatment of acute diarrhea

Treatment of acute diarrhea begins with general supportive measures. The most important therapy is hydration, which is best accomplished by the oral route, although intravenous hydration is more frequently used in the United States. Proper oral hydration could decrease the admission rate for children in the United States by at least 100,000 patients per year. Oral hydration solutions are effective because in many small bowel diarrheal illnesses, the intestine remains able to absorb water if glucose and salt are present to allow transport of water from the lumen. The World Health Organization formula that is recommended for oral rehydration consists of the following components: This formula can easily be made and also is commercially available. Drinks made for perspiration replacement, such as Gatorade, are not the same as the hydration fluid but can be used if the individual is not volume depleted Antibiotics are not required in most cases but are a consideration in specific circumstances. The inability to obtain immediate results from cultures for enteric pathogens often necessitates a decision regarding empiric antibiotic therapy. In general, empiric antibiotics do not significantly affect the course of acute diarrhea. In one study in which all patients were culture positive, there was a 1-day benefit for those receiving antibiotic treatment compared with nontreatment. Among those who were severely ill, the results were better: 1.5 days for resolution in the treated group compared to 3.4 days in the untreated group. Antibiotics should be avoided in patients with enterohemorrhagic E. coli because no benefit has been demonstrated and there may be an increased risk of hemolytic-uremic syndrome related to increased release of Shiga toxin. These patients often have bloody diarrhea and abdominal pain but no fever. If antibiotics have been started, they should be discontinued if culture results show E. coli 0157:H7. There is also no clinical improvement with antibiotic treatment of nontyphoid Samonella gastroenteritis, and the clearance of bacteria from the stool may be prolonged.

What is dietary fat predominantly composed of?

Triglycerides (~95%) with long-chain fatty acids (16- and 18-carbon molecules) In animal fat, the constituent fatty acids are mostly saturated (e.g., palmitic acid, stearic acid), whereas those of vegetable origin are rich in unsaturated fatty acids (i.e., having one or more double bonds in the carbon chain, such as oleic and linoleic acids

T or F Adjuvants trick the immune system into responding to purified protein antigens in a vaccine as if these proteins were parts of the infectious microbes

True

T or F Certain polymorphisms in the transcription factor HNFA are associated with ulcerative colitis but not Crohn disease. These HNFA polymorphisms are also strongly associated with maturity onset diabetes of the young (MODY), which like IBD, is associated with reduced intestinal barrier function

True

T or F Pregnant women have a 20-fold increased risk of developing listeriosis from meat or unpasteurized milk, and this bacterial infection always needs to be considered in a pregnant woman with diarrhea and systemic complaints

True

T or F Self polysaccharides, lipids, and nucleic acids are T-independent antigens that are not recognized by T cells and so these antigens must induce tolerance in B lymphocytes to prevent autoantibody production

True

T or F T cell responses are initiated primarily in secondary lymphoid organs, and the effector phase occurs mainly in peripheral tissue sites of infection

True

T or F The S-shaped structure known as the Sigmoid colon is quite mobile except at its beginning, where it continues from the descending colon, and at its end, where it continues as the rectum. Between these points, it is suspended by the sigmoid mesocolon.

True

T or F The microbiome of healthy humans consists of about 10^14 bacteria and viruses

True

T or F The most frequent causes of chronic abdominal pain are functional

True

T or F Protein fragments bound to MHC class I identify infected cells, whereas fragments bound to MHC class II identify infectious agents

True However, because of the phenomenon of cross-presentation, some proteins generated in the cytosol can be presented by MHC class II to CD4 T cells, and some phagocytosed antigens can be presented by MHC class I to CD8 T cells

T or F Periods of active disease of Crohns are typically interrupted by asymptomatic periods that last for weeks to many months.

True Disease re-activation can be associated with a variety of external triggers, including physical or emotional stress, specific dietary items, and cigarette smoking

T or F In 80-90% of cases, acute GI hemorrhage resolves spontaneously without recurrence

True Nevertheless, it is prudent to localize the bleeding source, especially in those with significant bleeding or comorbidities. Proton pump inhibitors, in combination with appropriate endoscopic management, decrease the risks for ulcer rebleeding, need for urgent surgery, and death

What should all patients undergo screening of before initiating therapy with TNF inhibitors?

Tuberculosis bc of increased risk of reactivating latent tuberculosis

What is large bowel obstruction commonly caused by?

Tumor potential causes include hernias and inflammatory diverticular disease of the sigmoid colon

TNF-alpha inhibitors

Tumor necrosis factor (TNF) - is a cytokine central to many aspects o the in ammatory response. Macrophages, mast cells, and activated T H cells (especially T H 1 cells) secrete TNF- . TNF alpha stimulates macrophages to produce cytotoxic metabolites, thereby increasing phagocytic killing activity. TNF- also stimulates production o acute-phase proteins, has pyrogenic e ects, and osters local containment o the in ammatory response. Some o these e ects are indirect and are mediated by other cytokines induced by TNF- alpha TNF- has been implicated in numerous autoimmune diseases. Rheumatoid arthritis, psoriasis, and Crohn's disease are three disorders in which inhibition o TNF-alpha has demonstrated therapeutic e f cacy. Rheumatoid arthritis illustrates the central role o TNF-alpha in the pathophysiology o autoimmune diseases (Fig. 46-7). Although the initial stimulus or joint in ammation is still debated, it is thought that macrophages in a diseased joint secrete TNF- , which activates endothelial cells, other monocytes, and synovial f broblasts. Activated endothelial cells up-regulate adhesion molecule expression, resulting in recruitment o in ammatory cells to the joint. Monocyte activation has a positive eedback e ect on T-cell and synovial f broblast activation. Activated synovial f broblasts secrete interleukins, which recruit additional in ammatory cells. With time, the synovium hypertrophies and orms a pannus that leads to destruction o bone and cartilage in the joint, causing the characteristic deformity and pain o rheumatoid arthritis. Five therapies inter ering with TNF activity have been approved. Etanercept is a soluble TNF receptor dimer that links the extracellular, ligand-binding domain o human TNF receptor type II to the Fc domain o human immunoglobulin G1 (IgG1); inf iximab is a partially humanized mouse monoclonal antibody directed against human TNF- ; and adalimumab is a ully human IgG1 monoclonal antibody directed against TNF- (Fig. 46-8). Certolizumab pegol is a pegylated anti-TNF- monoclonal antibody ragment that lacks the Fc portion o the antibody; as a result, unlike in iximab and adalimumab, certolizumab does not cause antibody-dependent cell-mediated cytotoxicity or f x complement in vitro . Golimumab is a ully human IgG1 monoclonal antibody directed against TNF- that has a longer hal -li e than the other anti-TNF agents. The monoclonal antibody-based TNF- inhibitors illustrate the desirability o treatment with "humanized" or " ully human" antibodies as opposed to mouse or other nonhuman antibodies. Because mouse antibodies are oreign, treatment with them can induce the production o antibodies against the mouse-specif c regions o the therapeutic antibody. The presence o these antiantibodies can reduce drug e f cacy by sequestering the therapeutic antibody be ore it can exert its desired e ect. To address this problem, one approach is to humanize therapeutic antibodies. In this approach, the portions o the antibody not involved in binding to the antigen are changed to the corresponding human sequences. Antibodies can be partially or fully humanized , depending on the extent o these changes. Humanization limits the likelihood o production o human antibodies against the therapeutic antibody, increasing the clinical e ectiveness o the antibody and allowing its long-term use (see Chapter 54, Protein Therapeutics). A more recent approach to the preparation o therapeutic antibodies is to prepare the antibody in an experimental animal bearing a human immune system or to use an in vitro human antibody system. This strategy generates fully human antibodies that do not require urther manipulation to render them nonimmunogenic. Although all f ve o the anti-TNF agents target TNF- , etanercept is somewhat less selective than the others because it binds to both TNF- and TNF- . In iximab, adalimumab, certolizumab, and golimumab are selective or TNF- and do not bind TNF- . The Fc portions o in iximab, adalimumab, and golimumab may also have specif c activity with respect to complement f xation and binding to Fc receptors on e ector cells. The immune e ector actions o these agents may be relevant to their mechanisms o action because TNF- is expressed on the sur ace o cells, especially macrophages, and the cell sur ace orm is cleaved to yield the soluble cytokine. Anti-TNF agents with immune e ector unctions may have biological e ects di erent rom those o agents that do not bind Fc receptors or f x complement The indications or TNF inhibitors have recently expanded to include conditions spanning the f elds o dermatology, rheumatology, and gastroenterology. Etanercept is approved or use in rheumatoid arthritis, juvenile idiopathic arthritis, plaque psoriasis, psoriatic arthritis, and ankylosing spondylitis. In iximab is approved or use in rheumatoid arthritis, Crohn's disease, ulcerative colitis, plaque psoriasis, psoriatic arthritis, and ankylosing spondylitis. Adalimumab is approved or use in rheumatoid arthritis, juvenile idiopathic arthritis, Crohn's disease, ulcerative colitis, plaque psoriasis, psoriatic arthritis, ankylosing spondylitis, and hidradenitis suppurtiva. Certolizumab is approved or the treatment o rheumatoid arthritis, Crohn's disease, psoriatic arthritis, and ankylosing spondylitis. Golimumab is approved or use in adults with rheumatoid arthritis (in combination with methotrexate), ulcerative colitis, psoriatic arthritis, and ankylosing spondylitis Although high levels o TNF- are likely mediators o underlying pathophysiologic processes, treatment with an anti-TNF agent o ten improves disease symptoms without reversing the underlying pathophysiology. There ore, upon drug discontinuation, maintenance o clinical response is uncertain. Etanercept, in iximab, adalimumab, certolizumab, and golimumab are proteins and must be administered parenterally. Orally active inhibitors o TNF-alpha and inhibitors o TNF-alpha converting enzyme (TACE) are under investigation. Several important adverse e ects must be considered when administering TNF inhibitors. All patients should undergo screening or tuberculosis be ore initiating therapy because o increased risk o reactivating latent tuberculosis. Any patient who develops an in ection while taking a TNF inhibitor should undergo evaluation and aggressive antibiotic treatment. Additionally, patients diagnosed with a severe in ection are recommended to temporarily interrupt treatment with TNF inhibitors. Epidemiologic surveillance has suggested that there may be an increased risk o demyelinating disease with anti-TNF therapy, although it has not yet been determined whether the relationship is causal.

What does Microscopic colitis encompass?

Two entities: 1- Collagenous colitis 2- Lymphocytic colitis These idiopathic diseases both present with chronic, nonbloody, watery diarrhea without weight loss.

What are the sympathetic trunks?

Two parallel nerve cords extending on either side of the vertebral column from the base of the skull to the coccyx The two sympathetic trunks come together anterior to the coccyx to form the ganglion impar

Types of intracellular microbes combated by T cell-mediated immunity

Types of intracellular microbes combated by T cell-mediated immunity. A, Microbes may be ingested by phagocytes and may survive within vesicles (phagolysosomes) or escape into the cytosol, where they are not susceptible to the microbicidal mechanisms of the phagocytes. B, Viruses may infect many cell types, including nonphagocytic cells, and replicate in the nucleus and cytosol of the infected cells. Rickettsiae and some protozoa are obligate intracellular parasites that reside in non-phagocytic cells

Clinical presentations- Intestinal manifestations

UC is characterized by chronic inflammation of the mucosal surface that involves the rectum and extends proximally through the colon in a continuous manner. The extent and severity of the colonic inflammation determine prognosis and presentation (insidious versus acute onset). Most patients initially exhibit diarrhea, abdominal pain, urgency to defecate, rectal bleeding, and the passage of mucus per rectum. At presentation, approximately 40% to 50% of patients have proctitis or proctosigmoiditis, 30% to 40% have left-sided colitis (disease extending to the splenic flexure), and the remaining 20% to 25% have pancolitis. Of the patients who initially present with proctitis or proctosigmoiditis, about 15% develop more extensive disease over time The typical clinical course of UC is one of chronic intermittent exacerbations followed by periods of remission. Signs of a worsening clinical course include the development of abdominal pain, dehydration, fever, and tachycardia. Clinical features that have been used to assess severity of UC include bowel frequency, fever, increased heart rate, and hematochezia (blood in the stool), as well as the presence of anemia and an elevated erythrocyte sedimentation rate (ESR) or C-reactive protein (CRP) level. Anemia commonly occurs and is caused by chronic blood loss from the involved colonic mucosa as well as bone marrow suppression from the inflammatory condition. Perforation can occur in patients with severe or fulminant colitis, especially those taking corticosteroids, and in the setting of toxic megacolon. Toxic megacolon is characterized by gross dilation of the large bowel associated with fever, abdominal pain, dehydration, tachycardia, and bloody diarrhea. The clinical presentation of Crohn's disease depends on the section of gastrointestinal tract involved and the type of inflammation. Crohn's disease can involve any portion of the gastrointestinal tract; the most common site is ileocecal (40% of patients), followed by isolated small bowel disease (30%) and isolated colonic involvement (25%). The remaining sites of Crohn's disease are rare (5%) and include the esophagus, stomach, and duodenum. Symptoms in Crohn's disease often include right lower quadrant abdominal pain, fever, weight loss, diarrhea, and sometimes a palpable inflammatory mass. Hematochezia is less common than in UC. The disease is often present for months or years before diagnosis, and in children, growth retardation may be the sole presenting sign. In contrast to UC, the inflammation in Crohn's disease is transmural and can result in microperforations and the formation of fistulous tracts. Fistulas may form between different segments of bowel (e.g., enteroenteric, enterocolonic) or between bowel and skin (enterocutaneous), bowel and bladder (enterovesicular), or rectum and vagina (rectovaginal). Over time, as many as 30% to 40% of patients develop disabling perianal involvement with fissures, fistulas, and abscesses. Chronic inflammation can cause fibrosis and stricture formation, which in turn may result in partial or complete intestinal obstruction, with the patient complaining of abdominal pain, distention, nausea, and vomiting. Strictures can also lead to stasis with subsequent small intestinal bacterial overgrowth. Extensive ileal mucosal disease may lead to malabsorption of vitamin B12 (resulting in a megaloblastic anemia and neurologic side effects if not corrected) and malabsorption of bile salts (resulting in diarrhea induced by unabsorbed bile salts and potential fatsoluble vitamin deficiency). Depletion of the bile salt pool can lead to the formation of gallstones. Weight loss may result from generalized malabsorption caused by loss of absorptive surfaces. Chronic fat malabsorption leads to luminal binding of free fatty acids to calcium; this allows oxalate, which normally is poorly absorbed because it complexes to calcium in the gut lumen, to be absorbed. The increase in oxalate absorption increases the risk for urinary calcium oxalate stone formation. Patients with an ileostomy or chronic volume loss from diarrhea are also at increased risk for uric acid stones.

Differentiating features of Ulcerative colitis and Crohn's disease

ULCERATIVE COLITIS CROHN'S DISEASE Site of involvement Involves colon only Rectum almost always involved Any area of the gastrointestinal tract Rectum usually spared Pattern of involvement Continuous Skip lesions Diarrhea Bloody Usually nonbloody Severe abdominal pain Rare Frequent Perianal disease No In 30% of patients Fistula No Yes Endoscopic findings Erythematous and friable Superficial ulceration Aphthoid and deep ulcers Cobblestoning Radiologic findings Tubular appearance resulting from loss of haustral folds String sign of terminal ileum RLQ mass, fistulas, abscesses Histologic features Mucosa only Crypt abscesses Transmural Crypt abscesses, granulomas (about 30%) Smoking Protective Worsens course Serology pANCA more common ASCA more common

Definition and Epidemiology

Ulcerative colitis (UC) is characterized by inflammatory changes that involve the colonic mucosa in a continuous superficial fashion, typically starting in the rectum and extending proximally. Depending on the extent of the disease, UC can be divided into proctitis (rectum only), proctosigmoiditis (rectum and sigmoid), left-sided colitis (extending to the splenic flexure), and pancolitis (inflammation extends proximal to the splenic flexure). This classification is significant for both prognosis and therapy. Unlike UC, Crohn's disease can involve any segment of the gastrointestinal tract from the mouth to the anus, often in a discontinuous fashion. It is characterized by transmural inflammation, which results in complications such as abscesses, fistulas, and strictures. In the United States, about 1.4 million individuals have IBD, and the overall annual incidence is about 20 new cases per 100,000 persons years. Although the incidence of UC has remained stable for several decades, the incidence of Crohn's disease has been increasing; it now seems to have plateaued at levels approximately equivalent to those of UC. The prevalence of IBD in the United States is between 249 to 319 per 100,000 persons. A bimodal age at presentation exists, with an initial peak between the second and fourth decades of life and another peak in the sixth decade. The sexes are equally affected The incidence and prevalence of IBD reflects the interplay of complex genetic and environmental factors that contribute to these disorders. For example, both diseases are more common in northern climates and among whites, particularly populations with Northern European ancestry such as North Americans, South Africans, and Australians. Individuals of Ashkenazi Jewish descent also have a twofold to eightfold increased risk for these disorders compared with non-Jews. Although incidence rates of IBD are lowest among Hispanics and Asians, IBD can occur in any ethnic or racial group from anywhere in the world. The cause of IBD remains unknown, but it is believed that a combination of genetic, immunologic, infectious, and environmental factors plays a role. In addition, state of the art research points toward a relationship between the human microbiome and dysfunction of the immune system in patients with IBD Approximately 5% to 20% of patients with IBD have a firstdegree relative with the disease, and first-degree relatives of IBD patients have about a 10- to 15-fold increased risk for developing IBD, predominantly with the same disease as the proband. A positive family history is more frequently observed in patients with Crohn's disease compared with UC, suggesting that genetic factors contribute more significantly in the etiology of Crohn's disease. Through advances in genome-wide association studies, several susceptibility loci on multiple chromosomes have been linked to IBD, supporting a polygenic cause for these disorders. Polymorphisms in the NOD2 gene (previously known as CARD15), located on chromosome 16, were the first definitive genetic risk factors identified for Crohn's disease. Homozygous mutations of the NOD2 gene are associated with a greater than 20-fold increase in susceptibility for Crohn's disease. Defects in the NOD2 protein appear to result in abnormal intestinal immune responses to bacterial cell wall components. These gene mutations are estimated to account for 15% to 25% of the cases of Crohn's disease and are linked predominantly to fibrostenotic terminal ileal disease. In addition to NOD2, other genes associated with Crohn's disease have been identified that regulate autophagy; they include ATG16L1, IRGM, and LRRK. Genes that regulate the interleukin-17 (IL-17) and IL-23 receptor pathways have been found to increase the risk for both UC and Crohn's disease; they include IL23R, IL12B, STAT3, JAK2, and TYK2. IL27 and TNFSF15 have been implicated only in Crohn's disease. Genes regulating epithelial barrier function have also been discovered for IBD, including members of the OCTN/IBD5 susceptibility locus (SLC22A4 and SLC22A5), ECM1, CDH1, HNF4A, LAMB1, and GNA12. Currently, it is believed that IBD results from an inappropriate, overactive mucosal immune response to commensal intestinal bacteria or the microbiome in genetically susceptible individuals. Profound alterations in mucosal immunology have been demonstrated in patients with IBD. In the normal immunologic state of the intestine, recently activated lymphoid tissue is abundant within the mucosal compartment. This state has been described as controlled or physiologic inflammation, and it likely develops in response to constant encounters with antigenic substances (derived from host microbial flora or dietary and environmental sources) that have crossed the epithelial barrier from the luminal environment. Indeed, one of the main functions of the intestinal immune system is to discriminate noxious or harmful substances and organisms from nonharmful ones. As a result, a large and well-maintained network of many different mucosal immune cells exists, including cells involved in reducing immune responses (regulatory cells) and those involved in activating immune responses. In IBD, this homeostatic balance, or immune tolerance, is dysregulated, resulting in overactivation of the immune system. In Crohn's disease, there is an excessive and persistent CD4positive helper T lymphocyte subtype 1 (TH1) immune response to components of commensal bacterial flora. The TH1 cytokine profile, which includes interferon-γ, IL-2, IL-12, and tumor necrosis factor-α (TNF-α), is elevated in patients with Crohn's disease. Patients with UC demonstrate greater expression of IL-5 and IL-13, cytokines characteristically associated with a TH2 response. In addition, non-TH1/TH2 pathways have been identified as being potentially important in the pathogenesis of IBD. IL-23, for example, has been recognized as an inducer of a subset of proinflammatory T cells (TH17) that secrete high levels of IL-17 and play an important role in mediating inflammation in murine models of colitis. IL-17 expression has been shown to be upregulated in active IBD, both Crohn's disease and UC. Environmental factors also are believed to play a role in the pathogenesis of IBD, because the disease is more common in industrialized countries. Moreover, the frequency has tended to increase in countries as they become more industrialized. It has been postulated that poor sanitation, food contamination, and crowded living conditions are associated with helminthic infection, which leads to regulatory T-cell conditioning and stimulation of IL-10 and transforming growth factor-β production by mononuclear cells, thereby preventing intestinal inflammation. However, the only environmental factor clearly associated with IBD is tobacco smoking. Smoking seems to be protective against UC, whereas smoking in Crohn's disease causes a more aggressive disease. No dietary triggers have been found to cause IBD, but elemental diets and diversion of the fecal stream can reduce inflammation in Crohn's disease

Inflammatory bowel disease- epidemiology

Ulcerative colitis and Crohn disease frequently present in the teens and early 20s, with the former being slightly more common in females. IBD is most common among Caucasians and, in the United States, occurs 3 to 5 times more often among eastern European (Ashkenazi) Jews than the general population. This is at least partly due to genetic factors, as discussed later. The geographic distribution of IBD is highly variable, but it is most common in North America, northern Europe, and Australia. However, IBD incidence worldwide is on the rise, and it is becoming more common in regions such as Africa, South America, and Asia where its prevalence was historically low. The hygiene hypothesis suggests that this increasing incidence is related to improved food storage conditions, decreased food contamination, and changes in gut microbiome composition. Apparently this results in inadequate development of regulatory processes that limit mucosal immune responses. This in turn allows some mucosa-associated microbial organisms to trigger persistent and chronic inflammation in susceptible hosts. Although many details to support this hypothesis are lacking, the observation that helminth infections, which are endemic in regions where IBD incidence is low, can prevent IBD development in animal models and even reduce disease in some patients, lends support to this idea

Ulcerative colitis - clinical features

Ulcerative colitis is a relapsing disorder characterized by attacks of bloody diarrhea with stringy, mucoid material, lower abdominal pain, and cramps that are temporarily relieved by defecation. These symptoms may persist for days, weeks, or months before they subside. The initial attack may, in some cases, be severe enough to constitute a medical or surgical emergency. More than half of patients have clinically mild disease, although almost all experience at least one relapse during a 10-year period, and up to 30% require colectomy within the first 3 years after presentation because of uncontrollable symptoms. Colectomy effectively cures intestinal disease in ulcerative colitis, but extraintestinal manifestations may persist. The factors that trigger ulcerative colitis are not known, but infectious enteritis precedes disease onset in some cases. In other cases the first attack is preceded by psychologic stress, which may also be linked to relapse during remission. The initial onset of symptoms has also been reported to occur shortly after smoking cessation in some patients, and smoking may partially relieve symptoms. Unfortunately, studies of nicotine as a therapeutic agent have been disappointing.

Ulcerative colitis

Ulcerative colitis is closely related to Crohn disease. However, the disease in ulcerative colitis is limited to the colon and rectum. Common extraintestinal manifestations of ulcerative colitis overlap with those of Crohn disease and include migratory polyarthritis, sacroiliitis, ankylosing spondylitis, uveitis, and skin lesions. Approximately 2.5% to 7.5% of individuals with ulcerative colitis also have primary sclerosis cholangitis (Chapter 18). The long-term outlook for ulcerative colitis patients depends on the severity of active disease and disease duration.

What test can be useful for the diagnosis of acute Cholecystitis or appendicitis?

Ultrasonography Computed tomography (CT) scans have become more helpful with technologic improvements in scanners; early CT scans allow prompt diagnosis of sometimes unsuspected abdominal diseases.

What is immunological tolerance?

Unresponsiveness to self antigens lack of response to antigens that is induced by exposure of lymphocytes to these antigens.

Naive T cells must migrate between blood and secondary (peripheral) lymphoid organs throughout the body until what?

Until they encounter dendritic cells within the lymphoid organ that display the antigens the T cells recognize After the naive T cells are activated and differentiate into effector cells, these cells must migrate back to the sites of infection, where they function to kill microbes

phagocytosis

Upon their arrival at the site o injury or in ection, neutrophils, macrophages, and other cells o the immune system are ready to per orm their duties. However, these cells require one urther stimulus to activate their killing machinery. Foreign substances must be coated by an opsonin be ore they can be ingested (phagocytosed) by leukocytes. Opsonins are molecular adaptors that coat oreign sur aces and signal leukocytes that a particle should be attacked. The major opsonins consist o complement, immunoglobulins (antibodies), and collectins (plasma proteins that bind to certain microbial carbohydrates). The interaction o a phagocytic cell with an opsonized particle initiates engul ment and destruction o the o ending agent. This step is also a crucial point o interaction between innate and adaptive immunity. Antigenpresenting cells process engul ed particles and present their antigens to B cells and T cells, which then react to the antigens. In the introductory case, Mark's cut presumably allowed bacteria to penetrate his skin barrier, leading to in ection. The presence o these bacteria initiated an inf ammatory response that included phagocytosis o bacteria by APCs, presentation o bacterial antigens to T H cells, activation and expansion o T H cells, T H cell activation o urther APC-mediated phagocytosis, and synthesis and secretion o antibodies speci c or the bacteria

What should be the initial step in the evaluation of patients with acute GI hemorrhage who have significant blood loss?

Upper Endoscopy

What have patients with Chronic Gatrointestinal bleeding have usually undergone?

Upper and lower endoscopy at least once without identification of a bleeding source Therefore, the bleeding must be from a source that is difficult to identify or one that emanates from the small intestine

What is Dihydroorotate dehydrogenase (DHOD) a key enzyme in the synthesis of?

Uridylate (UMP) ^ which is essential for the synthesis of Pyrimidines Experimentally, leflunomide has been shown to be most effective in reducing B-cell populations, and a significant effect on T cells has also been observed

What is Uveitis? treatment?

Uveitis (or iritis) is an inflammatory lesion of the anterior chamber that produces blurred vision, photophobia, headache, and conjunctival injection Local therapy includes steroids and atropine.

What extends from the terminal arcade? What do they provide?

Vasa recta (straight arteries) provide the final direct vascular supply to the walls of the small intestine

What do inflammatory mediators lead to the contraction of?

Vascular endothelial cells leading to increased capillary permeability and the development of an exudate ie: interstitial fluid with a high protein count Pain develops due to both increased tissue pressure and the action of various inflammatory mediators.

Chemical mediators of the inflammatory response

Vasodilation Histamine C3a, C5a (complement components) Prostaglandins (PG) PGI2, PGE1, PGE2, PGD2 Nitric oxide (NO) Bradykinin Plasmin Increased vascular permeability Histamine C3a, C5a Leukotrienes (LT), especially LTC4, LTD4, LTE4 Bradykinin Platelet-activating factor Substance P Calcitonin gene-related peptide (CGRP) Chemotaxis and leukocyte activation C3a, C5a LTB4, lipoxins (LX) LXA4, LXB4 Platelet-activating factor Bacterial products Tissue damage NO Oxygen radicals Neutrophil and macrophage lysosomal products Fever PGE2, PGI2, LTB4, LXA4, LXB4 Interleukin-1 (IL-1), IL-6, tumor necrosis factor (TNF) Pain PGE2, PGI2, LTB4 Bradykinin Substance P CGR

venous drainage

Venous drainage of the spleen, pancreas, gallbladder, and abdominal part of the gastrointestinal tract, except for the inferior part of the rectum, is through the portal system of veins, which deliver blood from these structures to the liver. Once blood passes through the hepatic sinusoids, it passes through progressively larger veins until it enters the hepatic veins, which return the venous blood to the inferior vena cava just inferior to the diaphragm

How to nutrients leave cells after absorption?

Venous or lymphatic channels Consequently, malabsorption may be associated with mesenteric venous obstruction, lymphangiectasia, or lymphatic obstruction due to malignancy or infiltrative processes such as Whipple's disease

What does lymphatic drainage of the abdominal part of the GI tract, inferior part of the rectum as well as spleen, pancreas, gallbladder and liver occur through?

Vessels and nodes that eventually end in large collections called pre-aortic lymph nodes

What is the most sensitive test to diagnose small bowel disease in Crohn's disease?

Video capsule endoscopy Small erosions and ulcerations on the mucosa, as well as strictures, can be visualized best on video capsule endoscopy

What are most cases of acute diarrhea caused by?

Viral or bacterial infections most are self-limited, resolving without specific therapy It tends to occur in outbreaks involving food or waterborne contamination or recurrently in specific groups Most of the deaths associated with acute diarrhea occur in elderly persons because of the physiologic changes of aging that include abnormalities in water homeostasis and decreased thirst perception. Because of volume depletion, elderly patients are at increased risk for falls due to orthostatic hypotension, electrolyte disturbances, and delirium.

What are the steps involved in Schilling test?

Vitamin B12 is an essential micronutrient, and its absorption requires several step 1- The ingested vitamin binds to salivary R-factor protein. 2- In the stomach, gastric parietal cells secrete intrinsic factor, which mixes with the ingested meal 3- In the duodenum, pancreatic trypsin hydrolyzes the R protein, freeing the vitamin to bind with intrinsic factor. 4- The vitamin B12-intrinsic factor complex is then absorbed by specific receptors that are found only on enterocytes in the distal ileum

Schilling test

Vitamin B12 is an essential micronutrient, and its absorption requires several steps. First, the ingested vitamin binds to salivary R-factor protein. In the stomach, gastric parietal cells secrete intrinsic factor, which mixes with the ingested meal. In the duodenum, pancreatic trypsin hydrolyzes the R protein, freeing the vitamin to bind with intrinsic factor. The vitamin B12-intrinsic factor complex is then absorbed by specific receptors that are found only on enterocytes in the distal ileum. Malabsorption of vitamin B12 can occur because of lack of intrinsic factor (e.g., pernicious anemia, gastric resection), pancreatic insufficiency, bacterial overgrowth, or ileal resection or mucosal disease (e.g., Crohn's disease). The Schilling test quantifies vitamin B12 absorption using radiolabeled vitamin B12 as a marker. The test may be expanded to several stages to amplify its diagnostic spectrum. In stage 1, after the injection of 1000 µg of unlabeled vitamin B12 to saturate hepatic storage, the patient ingests 0.5 µg of radiolabeled vitamin. Urine is then collected for the measurement of radioactivity; reduced radioactivity suggests B12 malabsorption. The test is repeated (stage 2) with the addition of oral intrinsic factor to the ingested vitamin B12. If urinary excretion of the radiolabel is corrected, pernicious anemia is diagnosed. If malabsorption is still present, the patient is given a short course of oral antibiotics (stage 3), and the test is repeated; correction of radiolabeled B12 excretion establishes bacterial overgrowth. If the test result remains abnormal, oral pancreatic enzymes are given (stage 4), and the test is repeated; correction of the abnormality at this stage implies pancreatic deficiency. Finally, if all these interventions fail, ileal disease or absence of transcobalamin protein is determined by other diagnostic tests. This long outline serves is merely an example of an algorithm of clinical analysis; the usual routine in clinical settings is to administer parenteral vitamin B12 while the etiology is delineated by other modalities

What will occur as a result of obstruction?

Vomiting and absolute constipation, including the inability to pass flatus, will ensue

treatment

When appropriate, specific therapy, such as surgery for intestinal obstruction, should be provided. More commonly, patients are treated with antibiotics, most appropriately those that are effective against aerobic and anaerobic enteric organisms. Tetracycline, trimethoprim-sulfamethoxazole, or metronidazole, in combination with a cephalosporin or quinolone, are suitable agents. A single course of therapy for 7 to 10 days may be therapeutic for months. In other patients, intermittent therapy (one week of every four) or even an extended period of continuous therapy proves to be most effective.

central B lymphocyte tolerance

When immature B lymphocytes interact strongly with self antigens in the bone marrow, the B cells either change their receptor specificity (receptor editing) or are killed (deletion) (Fig. 9-8). • Receptor editing. Some immature B cells that recognize self antigens in the bone marrow may reexpress RAG genes, resume immunoglobulin (Ig) light-chain gene recombination, and express a new Ig light chain (see Chapter 4). This new light chain associates with the previously expressed Ig heavy chain to produce a new antigen receptor that may no longer be specific for the self antigen. This process of changing receptor specificity, called receptor editing, reduces the chance that potentially harmful self-reactive B cells will leave the marrow. It is estimated that 25% to 50% of mature B cells in a normal individual may have undergone receptor editing during their maturation. (There is no evidence that developing T cells can undergo receptor editing.) • Deletion. If editing fails, immature B cells that strongly recognize self antigens receive death signals and die by apoptosis. This process of deletion is similar to negative selection of immature T lymphocytes. As in the T cell compartment, negative selection of B cells eliminates lymphocytes with high-affinity receptors for abundant, and usually widely expressed, cell membrane or soluble self antigens. • Anergy. Some self antigens, such as soluble proteins, may be recognized in the bone marrow with low avidity. B cells specific for these antigens survive, but antigen receptor expression is reduced, and the cells become functionally unresponsive (anergic).

What occurs in Central B cell tolerance?

When immature B lymphocytes interact strongly with self antigens in the bone marrow, the B cells either: 1- Change their receptor specificity (receptor editing) or 2- Are killed (deletion)

When is an Ileostomy performed? purpose?

When small bowel contents need to be diverted from the distal bowel An ileostomy is often performed to protect a distal surgical anastomosis, such as in the colon to allow healing after surgery.

When do autoimmune diseases occur?

When the host immune system attacks its own tissues, mistaking self-antigen for foreign The typical result is chronic inflammation in the tissue(s) expressing the antigen.

When do patients with diverticular disease tend to develop symptoms and signs?

When the neck of the diverticulum becomes obstructed by feces and becomes infected Inflammation may spread along the wall, causing abdominal pain.

When is an end colostomy necessary?

When the patient has undergone a surgical resection of the rectum and anus (typically for cancer)

Small intestinal biopsy

Whereas the gross appearance of the mucosa during upper GI endoscopy can provide some clues regarding the presence of a disease causing malabsorption, biopsy of the small intestinal mucosal is a key diagnostic test for diseases that affect the cellular phase of absorption. In some diseases, the histologic features are diagnostic; in others, the findings may be highly suggestive (Table 33-5). Several tissue samples should be taken from the duodenal bulb and from the distal duodenum to enhance the diagnostic accuracy.

immune diversity

While MHC proteins provide a mechanism or distinguishing in ected cells and in ectious agents rom unin ected cells, somatic gene recombination and other processes or generating diversity provide a mechanism or generating a specif c response to an in ection. By recombination, immunoglobulin and T-cell receptor genes semi-randomly create millions o modular three-dimensional protein structures, re erred to as variable regions . Recombined variable regions may undergo somatic hypermutation to create additional diversity that, in the aggregate, can recognize almost any structure. This is the primary mechanism by which the immune system generates an astounding diversity o immune responses.

Initial evaluation

While resuscitation is underway, the following information should be obtained by history and physical examination to determine the source of bleeding: 1. The nature of the bleeding: melena, hematemesis, hematochezia, or occult blood. A digital rectal examination is essential for determination of stool color and identification of anal fissures or rectal neoplasms. 2. The duration of GI bleeding, which helps dictate the appropriate pace of the evaluation to determine the bleeding source 3. The presence or absence of abdominal pain; for example, hematochezia caused by diverticula or angiodysplasia typically is painless, but hematochezia due to intestinal ischemia it is often accompanied by abdominal pain. 4. Other associated symptoms, including fever, urgency or tenesmus, recent change in bowel habits, and weight loss 5. Current or recent medication use, particularly nonsteroidal anti-inflammatory drugs (NSAIDs), including aspirin, which may predispose to ulceration or gastritis (see Chapter 37), anticoagulants, and alcohol. Many over-the-counter products may contain aspirin or NSAIDs. 6. Relevant past medical and surgical history, including a history of prior GI bleeding, abdominal surgery (prior abdominal aorta repair should raise suspicion for an aortoenteric fistula), radiation therapy (radiation proctitis), major organ diseases (including cardiopulmonary, hepatic, or renal disease), inflammatory bowel diseases, and recent polypectomy (postpolypectomy bleeding). The physical examination must include an assessment of vital signs, cardiac and pulmonary examinations, and abdominal and digital rectal examinations. The initial laboratory examination should include a complete blood cell count, blood typing and cross-matching, and measurements of serum electrolytes, blood urea nitrogen, creatinine, and coagulation factors. The first hematocrit measurement may not reflect the degree of blood loss, but it will decrease gradually to a stable level over 24 to 48 hours The initial disposition of the patient must also be considered. Patients older than 60 years of age, those with severe blood loss or continued bleeding (as reflected by a significant decrease in hematocrit or postural changes in blood pressure or pulse rate), and those with significant comorbid illness are at the greatest risk for complications of GI hemorrhage and are best managed in an intensive care setting until stabilized.

What is cirrhosis characterized by?

Widespread hepatic fibrosis interspersed with areas of nodular regeneration and abnormal reconstruction of preexisting lobular architecture The presence of cirrhosis implies previous or continuing liver cell damage

dilation of vessels

Within hours o being cut, Mark's thumb begins to exhibit the ve classic signs o inf ammation presented in the introduction. Initially, these signs and symptoms result rom alterations in vascular hemodynamics at the site o injury. Injury to a tissue causes the release o inf ammatory mediators (discussed earlier) that dilate arterioles and postcapillary venules; in turn, vasodilation leads to increased blood f ow to the site o injury, causing the clinical signs o redness and warmth. Inf ammatory mediators also cause contraction o vascular endothelial cells, leading to increased capillary permeability and the development o an exudate (i.e., interstitial f uid with a high protein content); in turn, the exudate causes the clinical mani estation o swelling. Pain develops due to both increased tissue pressure and the action o various inf ammatory mediators

What infectious agent can mimic Crohn's disease?

Yersinia enterocolitica the pathogen can mimic bc it also causes ileitis, mesenteric adenitis, fever, diarrhea, and right lower quadrant abdominal pain

Are Bile acids necessary for fat absorption?

Yes they undergo an enterohepatic circulation with release into bile and reabsorption from the terminal small intestine. Diseases interfering with this mechanism deplete the bile acid pool and can lead to fat malabsorption

What is Inflammatory Bowel disease (IBD)?

a chronic condition resulting from inappropriate mucosal immune activation

What is Hepatic cirrhosis? How is the diagnosis confirmed? What happens after a diagnosis is suspected?

a complex disorder of the liver diagnosis is confirmed histologically a liver biopsy

What is Natalizumab? What is it used in the treatment of?

a humanized anti-α4-integrin antibody, blocks inflammatory cell migration and adhesion, approved for the treatment of moderate to severe Crohn's disease in patients who have had an inadequate response to, or are unable to tolerate, conventional Crohn's disease therapies including inhibitors of TNF-α (level of evidence IIa, B for induction and IIa, A for maintenance).

What is Vedolizumab? What is it used in the treatment of?

a humanized monoclonal antibody to α4β7 integrin approved for the treatment and maintenance of both Crohn's and ulcerative colitis

What is Central tolerance?

a mechanism of tolerance only to self antigens that are present in the generative lymphoid organs—namely, the bone marrow and thymus Tolerance to self antigens that are not present in these organs must be induced and maintained by peripheral mechanisms.

What is Infliximab?

a partially humanized mouse monoclonal antibody directed against human TNF-alpha

What is Etanercept ?

a soluble TNF receptor dimer that links the extracellular, ligand-binding domain of human TNF receptor type II to the Fc domain o human immunoglobulin G1 (IgG1)

What is the Complement?

a system of Serine proteases that is one of the first innate mechanisms to be activated in response to injury.

What are included as Abdominal viscera receiving extrinsic innervation?

abdominal part of the gastrointestinal tract, the spleen, the pancreas, the gallbladder, and the liver These viscera send sensory information back to the central nervous system through visceral afferent fibers and receive motor impulses from the central nervous system through visceral efferent fibers

What does elevated Ca2+ in the NFAT pathway lead to the activation of?

activation of a phosphatase called calcineurin This enzyme removes phosphates from cytoplasmic NFAT, enabling the transcription factor to migrate into the nucleus, where it binds to and activates the promoters of several genes, including the genes encoding the T cell growth factor IL-2 and components of the IL-2 receptor

What is a normal inflammatory response?

an acute process that resolves after removal of the inciting stimulus

What are the complications associated with malabsorptive procedures?

anemia, osteoporosis, and diarrhea (e.g., jejunoileal bypass).

What is the Superior mesenteric artery and what does it supply? What does it arise from?

anterior branch of the abdominal aorta supplies the midgut arises from the abdominal aorta immediately below the celiac artery, anterior to the lower part of vertebra LI.

What are other agents that can be used for the treatment of IBD?

antibiotics, probiotics, antidiarrheals, bile salt resin binders, and nutritional support

During the response what is the survival and proliferation of T cells maintained by?

antigen, costimulatory signals from CD28, and cytokines such as IL-2.

What area does Crohn's disease involve? What is it characterized by?

any segment of the gastrointestinal tract from the mouth to the anus, often in a discontinuous fashion. Transmural inflammation-which results in complications such as abscesses, fistulas, and strictures

What does endoscopic examination show in Crohn's disease?

aphthoid erosions, deep linear or stellate ulcers, edema, erythema, exudate, and friability with intervening areas of normal mucosa

What are Chemokines?

are a subset of cytokines that promote immune cell trafficking, transmigration, and localization to sites of inflammation eg: Macrophage chemoattractant protein-1 (MCP-1) promotes monocyte transmigration and activation Other notable cytokines include the Hematopoietic growth factors granulocyte-monocyte colony-stimulating factor (GMCSF) and granulocyte colony-stimulating actor (G-CSF)

What are MHC proteins?

are transmembrane proteins that bind and display on their surface proteolytically degraded protein fragments and, in some cases, glycolipid antigens

What are Toll-like receptors (TORs)?

are transmembrane proteins that bind to shared microbial components such as LPS expressed by Gram-negative bacteria, mannans expressed by fungi, and double-stranded RNA expressed by viral pathogens Ten TLRs are expressed in humans, and each has a characteristic immune cell distribution and set of pathogen-associated ligands eg: TLR4 is expressed by antigen-presenting cells and it binds to LPS

Where is the surface projection of the base of the appendix ?

at the junction of the lateral and middle one-third of a line from the anterior superior iliac spine to the umbilicus (McBurney's point).

A patient was recently diagnosed with mild to moderate Crohn's disease. He was prescribed a medication where side effects include reversible oligospermia. Which of the following medications was the patient prescribed? a) Prednisone b) 5-ASA c) Azathioprine d) Infliximab e) Natalizumab

b) 5-ASA

A 65-year-old male presents to the ER with abdominal pain and says it looks like there are "small snakes" running across his abdomen. The physician explains that he has caput medusae due to his history of elevated portal vein pressure from his cirrhosis of the liver. Which of the following caval vessels will provide collateral blood supply? a) Left gastric veins b) Epigastric veins c) Superior rectal veins d) Omental veins e) Patent ductus venosus

b) Epigastric veins

surgery for obesity

bariatric surgery. This type of surgery has become increasingly popular over the last few years for patients who are unable to achieve significant weight loss through appropriate diet modification and exercise programs. It is often regarded as a last resort. Importantly, we have to recognize the increasing medical impact that overweight patients pose. With obesity the patient is more likely to develop diabetes and cardiovascular problems and may suffer from increased general health disorders. All of these have a significant impact on health care budgeting and are regarded as serious conditions for the "health of a nation There are a number of surgical options to treat obesity. Surgery for patients who are morbidly obese can be categorized into two main groups: malabsorptive procedures and restrictive procedures

How can IBS be diagnosed?

based on a typical history and clinical findings and does not usually require an extensive evaluation

Why is Histamine referred to as a Vasoactive amine?

because its inflammatory effects occur mainly on the vasculature: histamine release stimulates dilation of arterioles and postcapillary venules, constriction of veins, and contraction of endothelial cells

Why do effector T cells migrate to sites of infection?

because they express adhesion molecules and chemokine receptors that bind to ligands expressed or displayed on vascular endothelium in innate immune responses to microbes

Where does the hindgut begin ? end at?

begins just before the left colic flexure (the junction between the proximal two-thirds and distal one-third of the transverse colon) ends midway through the anal canal.

What does the foregut begin with ? end with?

begins with the abdominal esophagus ends just inferior to the major duodenal papilla, midway along the descending part of the duodenum

What does an autoimmune disease involving IL-2 receptor alpha chain (CD25) lead to?

believed to influence the balance of effector and regulatory T cells

What is required for diagnosis of Microscopic colitis? What is it caused by?

biopsies are necessary to document its presence Although the underlying cause of the colitis is not known, it is associated with autoimmune disease in up to 50% of cases.

A 35-year-old female is scheduled for a kidney transplant due to end-stage kidney disease. The donated kidney is an allograft and the recipient and donor are matched to try and minimized rejection. The surgery is performed successfully. Which of the following medication would most likely be prescribed to prevent organ rejection in this patient? a) Methotrexate b) Azathioprine c) Mycophenolate mofetil d) Prednisone e) Etanercept

c) Mycophenolate mofetil

A 15-year-old male presents to the Emergency Department with nausea and sharp abdominal pain. The patient's mother states her son complained of central abdominal pain hours earlier but the pain has increased and moved to his lower right quadrant. The patient is taken back for an appendectomy and the surgeon ligates the appendicular artery, which is a branch off of which main artery? a) Inferior mesenteric artery b) Celiac artery c) Superior mesenteric artery d) Internal iliac artery e) Inferior epigastric artery

c) Superior mesenteric artery

What are the 3 major divisions of the abdominal prevertebral plexus and associated ganglia?

celiac, aortic, and superior hypogastric plexuses

What is Infliximab? What is it useful in the treatment of?

chimeric monoclonal antibody to TNF-alpha effective in the treatment of both moderate to severe Crohn's disease, including fistulizing disease, and UC Because infliximab is a chimeric antibody, its toxicities include infusion reactions, delayed-type hypersensitivity reactions, and formation of autoantibodies (which can reduce its efficacy).

What are the main symptoms associated with diarrhea-predominant Inflammatory bowel syndrome (IBS)?

chronic abdominal pain and altered bowel movements These patients complain of small-volume, frequent diarrhea, often with interspersed normal or constipated stools may report marked urgency as well as a feeling of incomplete evacuation Approximately one half of these patients have mucus in the stools.

What are Biologic agents?

class of medications that target specific aspects of the immune system.

What does the inferior branch of the Ileocolic artery divide into?

colic, cecal, appendicular, and ileal branches 1- The colic branch crosses to the ascending colon and passes upward to supply the first part of the ascending colon 2- Anterior and posterior cecal branches, arising either as a common trunk or as separate branches, supply corresponding sides of the cecum 3- The appendicular branch enters the free margin of and supplies the mesoappendix and the appendix. 4- The ileal branch passes to the left and ascends to supply the final part of the ileum before anastomosing with the superior mesenteric artery

What can Chronic diarrhea result from?

colonic inflammation, colonic neoplasia, small bowel inflammation, malabsorption due to small bowel mucosal disorders, maldigestion due to pancreatic insufficiency, motility disorders, and functional bowel disorders

What is inflammation composed of?

complex web of responses to tissue injury and infection, characterized by the classic signs of: 1- Rubor (redness) 2- Calor (heat) 3- Tumor (swelling) 4- Dolor (pain) 5- Functio laesa (loss of function)

What is the main role of antidiarrheal mediactions?

controlling diarrhea in patients who have undergone previous resections

What test should be performed when fat malabsorption is demonstrated?

d-xylose absorption-excretion test Fat malabsorption is demonstrated when (>6 g/24 hours, or increased qualitative stool fat and decreased serum carotene) A normal d-xylose test result makes diffuse mucosal disease unlikely and suggests maldigestion, principally pancreatic enzyme or bile salt deficiency

The anal canal is the continuation of the large intestine inferior to the rectum. The arterial supply to the rectum and anal canal includes the superior rectal, middle rectal, and inferior rectal arteries. The superior rectal artery is specifically a branch from which artery? a. The superior mesenteric artery. b. The internal pudendal artery. c. The internal iliac artery. d. The inferior mesenteric artery. e. None of the above.

d. The inferior mesenteric artery.

The ultimate fate of mature B-cells that respond to self-antigen in peripheral lymphoid tissue is: a.) Stimulation to become a plasma cell b.) Returning to bone marrow to receptor edit into a B-cell non reactive to self-antigen c.) Costimulation by helper T-cells d.) Anergy due to block in signalling from antigen receptor e.) Escapes detection and triggers an autoinflammatory response

d.) Anergy due to block in signalling from antigen receptor

What do patients with constipation-predominant or diarrhea-predominant IBS tend to have?

decreased or increased colonic contractions and transit rates, respectively Excess bile acid synthesis or bile acid malabsorption has been identified as one cause of diarrhea-predominant IBS, likely due to the effects of bile acids on intestinal motility.

In addition to cell-mediated immunity what do T lymphocytes also play an important role in?

defense against microbes that replicate outside cells, including several types of bacteria, fungi, and helminthic parasites

What does osmaotic gap in fecal water suggest?

dietary (rather than a secretory) cause of the diarrhea related to luminal short-chain fatty acids or carbohydrates.

What occurs in the breath test involving disaccharidase deficiency?

disaccharidase deficiency, a specific disaccharide (e.g., lactose) that is orally ingested but not properly absorbed in the small intestine is delivered to the colon, where bacterial fermentation liberates metabolites; hydrogen gas is the marker assayed in the breath.

When does Chronic rejection usually occur?

does not occur until months or years after transplantation chronic rejection is now the most common life-threatening pathology associated with organ transplantation

Where does the Celiac trunk drain to?

drains to pre-aortic nodes near the origin of the celiac trunk these celiac nodes also receive lymph from the superior mesenteric and inferior mesenteric groups of pre-aortic nodes lymph from the celiac nodes enters the cisterna chyli

Where does the Inferior mesenteric artery drain into?

drains to pre-aortic nodes near the origin of the inferior mesenteric artery lymph from the inferior mesenteric nodes drains to the superior mesenteric nodes.

Where does the Superior mesenteric artery drain into?

drains to pre-aortic nodes near the origin of the superior mesenteric artery these superior mesenteric nodes also receive lymph from the inferior mesenteric groups of pre-aortic nodes, lymph from the superior mesenteric nodes drains to the celiac nodes.

A patient comes into the clinic complaining of localized tenderness in the right groin. The patient states that initially the pain began in the central region and would come and go. After 7 hours the pain began to become constant. The patient also complained of fever, nausea, and vomiting. What is the cause of the patient's pain? a) The rectum. b) The sigmoid colon. c) The small intestines. d) The cecum. e) The appendix.

e) The appendix.

What is Celiac disease induced by?

exposure to storage proteins found in grain plants such as wheat (which contains gliadin), barley, and rye and their products. Oats are implicated, not because of gliadin, but because of contamination with wheat during packaging and transportation.

What is an Ileal conduit?

extraanatomical procedure and is performed after resection of the bladder for tumor

What is the Left colic artery? What are its branches?

first branch of the inferior mesenteric artery, It ascends retroperitoneally, dividing into ascending and descending branches: 1- The ascending branch passes anteriorly to the left kidney, then enters the transverse mesocolon, and passes superiorly to supply the upper part of the descending colon and the distal part of the transverse colon; it anastomoses with branches of the middle colic artery 2- The descending branch passes inferiorly, supplying the lower part of the descending colon, and anastomoses with the first sigmoid artery

What does a colonoscopy reveal in patients with ulcerative colitis?

granular mucosa, decreased vascular markings, decreased mucosal reflection, exudate, and superficial ulceration In more severe cases, the mucosa is friable, with deeper ulcerations.

Describe the bowel movements in people suffering from Malabsorption

greasy or oily, foul smelling, and large volume, although not usually watery

What is IL-23 involved in?

has been recognized as an inducer of a subset of proinflammatory T cells (TH17) that secrete high levels of IL-17 and play an important role in mediating inflammation in murine models of colitis.

What does secretion of IL-2 by CD4 T cells increase expression of?

high-affinity IL-2 receptor thus rapidly enhancing the ability of the T cells to bind and respond to IL-2 IL-2 produced by antigen-stimulated T cells preferentially binds to and acts on the same T cells, an example of autocrine cytokine action.

How is Pericholangitis identified?

identified only by abnormalities in alkaline phosphatase and γ-glutamyl transpeptidase on laboratory tests and histologically by portal tract inflammation and bile ductule degeneration Small-duct sclerosing cholangitis may progress to cirrhosis

What does ICOS (inductible costimulator) play a role in?

important role in the development and function of follicular helper T cells during germinal center responses

acute abdomen

in medical practice. The acute abdomen is caused by sudden inflammation, perforation, obstruction, or infarction of an intraabdominal organ. The urgent question to be answered is whether immediate surgery is needed; a quick but complete evaluation is necessary to avoid undue delay in intervention for patients who require surgery. The physician must assess for abdominal tenderness, rebound, and guarding. Early surgical consultation should be obtained, even in doubtful cases, rather than awaiting confirmation of the diagnosis via laboratory or radiologic studies. However, many extra-abdominal conditions such as pneumonia, myocardial infarction, nephrolithiasis, and metabolic disorders can cause acute abdominal pain. In some instances of the acute abdomen in its early stages, there are few findings. The examiner should be aware that patients with benign chronic conditions may have severe pain at presentation that is out of proportion to any physical findings. The context provided by the medical history, particularly previous abdominal surgery, is very valuable. Indeed, a patient with sudden crampy pain and abdominal distention may have an intestinal obstruction caused by adhesions or an incarcerated hernia. Therefore, examination of the entire patient, looking for jaundice, skin lesions, evidence of prior surgery, or evidence of chronic liver disease, is important In evaluating a patient with acute abdominal symptoms, a complete blood cell count with differential, a urinalysis, and measurements of serum amylase, lipase, bilirubin, and electrolytes are necessary components of the laboratory examination. Additional studies may be done but usually do not aid in the rapid decision making required. An elevated white blood cell count may indicate inflammatory disease, and extremely high values are typical of acute intestinal ischemia. An elevated serum amylase concentration usually indicates acute pancreatitis, although a perforated ulcer or mesenteric thrombosis can also cause hyperamylasemia Radiographic examination with an abdominal film is important to reveal the intra-abdominal gas pattern, and an upright film that includes the diaphragm or a left lateral decubitus film may identify intra-abdominal air suggesting perforation of a hollow viscus. Ultrasonography can be helpful in the diagnosis of acute cholecystitis or appendicitis. Computed tomography (CT) scans have become more helpful with technologic improvements in scanners; early CT scans allow prompt diagnosis of sometimes unsuspected abdominal diseases. Examination with a radiopaque medium should be used judiciously, especially if surgery is anticipated. E-Figures 33-1 through 33-4 are CT images of appendicitis, diverticulitis, pancreatitis, and ulcerative colitis, respectively.

What are clinical features that have been used to assess severity of Ulcerative colitis?

include bowel frequency, fever, increased heart rate, and hematochezia (blood in the stool), as well as the presence of anemia and an elevated erythrocyte sedimentation rate (ESR) or C-reactive protein (CRP) level.

What does the etiology of cirrhosis include?

includes toxins (alcohol), viral inflammation, biliary obstruction, vascular outlet obstruction, nutritional (malnutrition) causes, and inherited anatomical and metabolic disorders.

What is diarrhea?

increased frequency of stools with decreased consistency and increased volume

What does increased Oxalate lead to?

increases the risk for urinary calcium oxalate stone formation

What is the function of adjuvants?

inducing the expression of costimulators on APCs and by stimulating the APCs to secrete cytokines that activate T cells Most adjuvants are products of microbes (e.g., killed mycobacteria, which is often used in experimental studies) or substances that mimic microbes, and they bind to pattern recognition receptors of the innate immune system, such as Toll-like receptors and NOD-like receptors

What is included in the differential diagnosis of IBD?

infectious colitis, ischemic colitis, radiation enteritis, enterocolitis induced by nonsteroidal anti-inflammatory drugs, diverticulitis, appendicitis, gastrointestinal malignancies, and irritable bowel syndrome

What is Ulcerative colitis characterized by?

inflammatory changes that involve the colonic mucosa in a continuous superficial fashion typically starting in the rectum and extending proximally.

What do Ras-GTP and Rac-GTP initiate and lead to the activation of?

initiate different enzyme cascades, leading to the activation of distinct MAP kinases

What is the NFAT signaling pathway initiated by?

initiated by ZAP-70-mediated phosphorylation and activation of an enzyme called phospholipase Cγ (PLCγ), which catalyzes the hydrolysis of a plasma membrane inositol phospholipid called phosphatidylinositol 4,5-bisphosphate (PIP2).

What can high grade dysplasia be associated with? How can it be treated?

invasive carcinoma at the same site or elsewhere in the colon therefore, often prompts colectomy

What are some nooninfectious causes of acute diarrhea?

irritable bowel syndrome (IBS), inflammatory bowel disease (IBD), ischemic bowel disease (either ischemic colitis or mesenteric vascular insufficiency), partial bowel obstruction, fecal impaction with overflow diarrhea, and bacterial overgrowth.

What is IL-17 mainly produced by? What role does it play?

is a cytokine that is mainly produced by IL-23-induced Th17 cells. plays a central role in inflammation by stimulating the production of key inflammatory mediators, including IL-6, TNF-alpha , and IL-1 IL-17 also stimulates the proliferation and survival of neutrophils, T cells, and B cells IL-17 exerts its pro-inflammatory effsect by signaling through two receptor subtypes, IL-17 receptor A (IL-17RA) and IL-17 receptor C (IL-17RC).

What is the MOA of Leflunomide?

is an inhibitor of pyrimidine synthesis specifically blocking the synthesis of Uridylate (UMP) by inhibiting dihydroorotate dehydrogenase (DHOD)

What is CTLA-4 expressed on?

is expressed transiently on activated CD4+ T cells and constitutively on regulatory T cells

What is Diverticular disease?

is the development of multiple colonic diverticula predominantly throughout the sigmoid colon, though the whole colon may be affected The sigmoid colon has the smallest diameter of any portion of the colon and is therefore the site where intraluminal pressure is potentially the highest.

What does secondary diarrhea manifest with?

is uncommon and typically manifests with large-volume (>1 L/day), watery diarrhea that occurs both day and night and continues in spite of fasting A secretory diarrhea will have an osmolar gap of less than 50 mOsm/kg, whereas the gap in an osmotic diarrhea will be greater than 125 mOsm/kg

When patients have either an ileostomy, colostomy, or ileal conduit what is necessary?

it is necessary for them to fix a collecting bag onto the anterior abdominal wall. Contrary to one's initial thoughts these bags are tolerated extremely well by most patients and allow patients to live a nearly normal and healthy life.

What does the Gastroduodenal artery divide into as it reaches the lower border of the superior part of the duodenum?

its terminal branches: 1- The right gastro-omental artery 2- The anterior superior pancreaticoduodenal artery

What does Anergy refer to in T cells?

long-lived functional unresponsiveness that is induced when these cells recognize self antigens Self antigens are normally displayed with low levels of costimulators Antigen recognition without adequate costimulation is thought to be the basis of anergy induction,

CTLA-4 blockade

lpilimumab is a f rst-generation immune checkpoint inhibitor. This recombinant, humanized IgG1 monoclonal antibody binds to CTLA-4 and blocks the interaction o CTLA-4 with its ligands B7-1 and B7-2 (see Chapter 42). In clinical studies, ipilimumab showed an overall survival benef t in patients with unresectable or metastatic melanoma who had been previously treated with one or more anticancer therapies. Ipilimumab was approved or use in 2011 in advanced melanoma that is unresectable or metastatic. Patients taking ipilimumab should be monitored or possible immunerelated hepatotoxicity and endocrinopathie

Where does Crohns's disease occur?

may involve any area of the GI tract is typically transmural

What may a polymorphism in the gene encoding the Tyrosine phosphatase PTPN22 (protein tyrosine phosphatase N22) lead to?

may lead to uncontrolled activation of both B and T cells are associated with numerous autoimmune diseases, including rheumatoid arthritis, systemic lupus erythematosus, and type 1 diabetes mellitus

Where can Arteriosclerosis occur?

may occur throughout the abdominal aorta and at the openings of the celiac trunk and the superior mesenteric and inferior mesenteric arteries frequently it is the Inferior mesenteric artery that becomes occluded

What is the Schilling test?

medical procedure used to determine whether you're absorbing vitamin B-12 properly quantifies vitamin B12 absorption using radiolabeled vitamin B12 as a marker

What are derivatives oral 5-ASA compounds?

mesalamine (Pentasa, 4 g/day in divided doses; Delzicol, 2.4 g/day in divided doses; Asacol HD, 2.4 to 4.8 g/day in divided doses; Lialda, 2.4 to 4.8 g once daily; Apriso, 1.5 g once a day), olsalazine (Dipentum, 1 to 2 g/day in divided doses), and balsalazide (Colazal, 6.75 g/day in divided doses; Giazo 3.3 g/day in divided doses). Topical forms of mesalamine (Canasa suppositories, 1000 mg once daily; Rowasa enemas, 4 g once nightly) are commonly used because of a more favorable side-effect profile

What are the branches of the SMA from the right side of the main trunk? What do they supply?

middle colic, right colic, and ileocolic arteries supply the terminal ileum, cecum, ascending colon, and two-thirds of the transverse colon.

Who is Microscopic colitis most commonly seen in? What does it manifest with?

middle-aged women but can be found at all ages and in men as well chronic watery diarrhea There can be mild cramping and weight loss, but dehydration and malnutrition are rare. Disease is present throughout the colon but is often more severe on the right side.

What extraintestinal manifestations overlap between UC and Crohns?

migratory polyarthritis, sacroiliitis, ankylosing spondylitis, uveitis, and skin lesions Approximately 2.5% to 7.5% of individuals with ulcerative colitis also have primary sclerosis cholangiti

What does the Enteric nervous system consists of?

motor and sensory neurons organized into two interconnected plexuses (the myenteric and submucosal plexuses) between the layers of the gastrointestinal wall, and the associated nerve fibers that pass between the plexuses and from the plexuses to the adjacent tissue

What happens when the region of the splenic flexure of the colon becomes ischemic?

mucosa sloughs off, rendering the patient susceptible to infection and perforation of the large bowel which then requires urgent surgical attention

Acute diarrhea part 2

nclude information regarding travel, work exposure, and pets. Fever usually indicates an invasive organism, such as Salmonella, Shigella, Campylobacter, certain viruses, Entamoeba histolytica, or C. difficile. Risk factors include food consumption or preparation involving raw or undercooked meats and dairy products or contaminated fruits and vegetables. Pregnant women have a 20-fold increased risk of developing listeriosis from meat or unpasteurized milk, and this bacterial infection always needs to be considered in a pregnant woman with diarrhea and systemic complaints. Ingestion of preformed bacterial toxins from Staphylococcus aureus, Bacillus cereus, or Clostridium perfringens typically causes diarrhea within 6 hours. Acute traveler's diarrhea is most commonly caused by enterotoxigenic E. coli Infectious agents sometimes can cause mucosal inflammation ranging from mild to severe. These include noroviruses, rotaviruses, and the human immunodeficiency virus (HIV). Bacterial mucosal invasion can be present with Salmonella, enteroinvasive E. coli, Campylobacter jejuni, and Yersina enterocolitica; E. histolytica, C. difficile, Shigella spp., E. coli O157:H7, Vibrio, and Aeromonas secrete toxins and invade the mucosa. Patients often relate a history of initially watery diarrhea later progressing to bloody diarrhea. Noninfectious causes of acute diarrhea are less common. They can include irritable bowel syndrome (IBS), inflammatory bowel disease (IBD), ischemic bowel disease (either ischemic colitis or mesenteric vascular insufficiency), partial bowel obstruction, fecal impaction with overflow diarrhea, and bacterial overgrowth. Most often, these disorders manifest as persistent or chronic diarrhea, but they do have a specific onset. Medications and over-the-counter supplements can be a cause of acute or chronic diarrhea, and the diagnosis may be suggested by introduction of a new medication or an increase in dose. Frequently, oral magnesium replacement, donepezil hydrochloride (Aricept), tube feedings, liquid medications, and chewing gum made with nonabsorbable sugars (e.g., sorbitol) are associated with diarrhea. An accurate medication history (prescription and over-the-counter), including supplements, is necessary.

What are some common viruses that cause acute diarrhea?

noroviruses, rotaviruses, and adenoviruses The symptoms typically last approximately 48 hours and clear spontaneously.

What infectious agents can cause mucosal inflammation?

noroviruses, rotaviruses, and the human immunodeficiency virus (HIV). Bacterial mucosal invasion can be present with Salmonella, enteroinvasive E. coli, Campylobacter jejuni, and Yersina enterocolitica; E. histolytica, C. difficile, Shigella spp., E. coli O157:H7, Vibrio, and Aeromonas secrete toxins and invade the mucos

What does the Superior hypogastric plexus contain?

numerous small ganglia and is the final part of the abdominal prevertebral plexus before the prevertebral plexus continues into the pelvic cavity.

Upon recognition of self antigens what may T cells preferentially engage?

one of the inhibitory receptors of the CD28 family, cytotoxic T lymphocyte-associated antigen 4 (CTLA-4, or CD152) or programmed death protein 1 (PD-1) Anergic T cells may express higher levels of these inhibitory receptors, which will inhibit responses to subsequent antigen recognition

What is Histamine? Where is it synthesized and stored?

one of the initiators of the inflammatory response synthesized and stored in the granules of mast cells and basophils

Digestion and absorption of proteins

only source for the essential amino acids. Digestion starts in the stomach with pepsins secreted by the gastric mucosa, but most of the hydrolysis is accomplished by pancreatic enzymes in the proximal small bowel. The pancreas secretes the proteases trypsin, elastase, chymotrypsin, and carboxypeptidase as inactive proenzymes. Enterokinase (more properly, enteropeptidase) is secreted by the intestinal brush border; it splits trypsinogen to its active form, trypsin, which in turn converts the other proenzymes to their active forms. The products of luminal brush border peptidase digestion consist of amino acids and oligopeptides, which are transported across the epithelial cell. The transfer of most amino acids is sodium dependent and takes place in the proximal small bowel. Dietary requirements for amino acid nitrogen are met with about 15% of calories from protein.

What other gene associations are implicated in IBD besides NOD2?

other genes associated with Crohn's disease have been identified that regulate autophagy: they include ATG16L1, IRGM, and LRRK. Genes that regulate the interleukin-17 (IL-17) and IL-23 receptor pathways have been found to increase the risk for both UC and Crohn's disease; they include IL23R, IL12B, STAT3, JAK2, and TYK2. IL27 and TNFSF15 have been implicated only in Crohn's disease. Genes regulating epithelial barrier function have also been discovered for IBD, including members of the OCTN/IBD5 susceptibility locus (SLC22A4 and SLC22A5), ECM1, CDH1, HNF4A, LAMB1, and GNA12.

What antibody is seen in Ulcerative colitis but not in Crohn's disease?

pANCA Perinuclear antineutrophil cytoplasmic antibody (pANCA) is positive in up to 70% of patients with UC but is rarely positive in patients with Crohn's disease

Where do both A and C fibers originate from?

parietal peritoneum and abdominal wall

What is the treatment for Bacterial overgrowth?

patients are treated with antibiotics, most appropriately those that are effective against aerobic and anaerobic enteric organisms. eg: Tetracycline, trimethoprim-sulfamethoxazole, or metronidazole, in combination with a cephalosporin or quinolone

When is surgery indicated for patients with UC or Crohn's?

patients with severe complications such as obstruction, perforation, massive gastrointestinal hemorrhage, or toxic megacolon not responsive to medical treatment. The other main indication for surgical treatment is the presence of dysplasia or cancer

What is the function of Lck?

phosphorylates tyrosine residues contained within the ITAMs of the CD3 and ζ proteins. The phosphorylated ITAMs of the ζ chain become docking sites for a tyrosine kinase called ZAP-70 (zeta-associated protein of 70 kD), which also is phosphorylated by Lck and thereby made enzymatically active The active ZAP-70 then phosphorylates various adaptor proteins and enzymes, which assemble near the TCR complex and mediate additional signaling events.

What extra-abdominal conditions can cause Acute abdominal pain?

pneumonia, myocardial infarction, nephrolithiasis, and metabolic disorders can cause acute abdominal pain.

What structural components serve as conduits for the afferent and efferent fibers?

posterior and anterior roots of the spinal cord, respectively, spinal nerves, anterior rami, white and gray rami communicantes, the sympathetic trunks, splanchnic nerves carrying sympathetic fibers (thoracic, lumbar, and sacral), parasympathetic fibers (pelvic), the prevertebral plexus and related ganglia, and the vagus nerves [X].

Neuronal fibers found in the sympathetic trunk include?

preganglionic and postganglionic sympathetic fibers and visceral afferent fibers.

IL-4 produced by? effect?

produced by: CD4+ T cells (Th2), (mast cells?) effect: Anti-Inflammatory: Th2 differentiation and proliferation, B cells, isotype switching to IgE

IL-17 produced by? effect?

produced by: CD4+ Tcells (Th17) effect: Pro-Inflammatory: increased chemokine and cytokine production from epithelial cells and macrophages

IL-13 produced by? effect?

produced by: CD4+ Tcells (Th2) effect: Anti-Inflammatory: B cells, isotype switching to IgE, increased mucus production from epithelial cells, activation of macrophages, inhibits Th1

IL-23 produced by? effect?

produced by: Macrophages and Dendritic cells (DC cell) effect: Differentiation and proliferation of Th17 cells

TNF produced by? effect?

produced by: Macrophages, NK cells, T cells effect: Pro-Inflammatory: activation of endothelial cells (inflammation and coagulation), activation of neutrophils, hypothalamus: fever; muscle & fat: cachexia

IFN-gamma produced by? effect?

produced by: T cells (Th1 and CD8+), NK cells effect: Pro-Inflammatory: activation of macrophages, B cells isotype switching to IgG, Th1 differentiation, increased expression of MHC class I and class II molecules, increased antigen processing and presentation to T cells

IL-2 produced by? effect?

produced by: T cells (Th1), NK cells effect: Pro-Inflammatory- T cells, proliferation and differentiation

IL-5 produced by? effect?

produced by: TCD4+ Tcells (Th2) effect: Help antibody production by B cells; activation of eosinophils

IL-12 produced by? effect?

produced by: macrophages, dendritic cells effect: Pro-Inflammatory: Th1 differentiation, interferon gamma synthesis from NK and T cells, inhibits Th2

What are the indications warranting a complete evaluation involving clinical signs of severe illness in diarrhea?

profuse watery diarrhea and hypovolemia; frequent passage of small-volume, bloody stools containing mucus; bloody diarrhea; fever greater than 101° F; illness lasting longer than 48 hours; severe abdominal pain; hospitalization; recent use of antibiotics; age greater than 70 years; immunocompromise; and systemic illness and diarrhea in pregnancy (listeriosis).

What protein antigens are displayed by MHC I molecules?

protein antigens present in the cytosol are processed by proteasomes into peptides displayed by class I MHC molecules

What are cytokines?

proteins that act in a Paracrine manner to regulate leukocyte activity

What does Adapative innate immunity provide vs adaptive immunity?

provides a broad gating function, attempting to counteract harmful effects of foreign invaders in a rapid manner and to determine whether an infectious agent should be further attacked by adaptive immunity while adaptive immunity provides a specialized response that is specific to the particular invading infectious agent. Adaptive immune cells mount a faster and more intense response upon reexposure to the infectious agent

What does mutation in AIRE gene cause?

rare disorder called Autoimmune polyendocrine syndrome (APS) In this disorder, several tissue antigens are not expressed in the thymus because of a lack of functional AIRE protein, so immature T cells specific for these antigens are not eliminated and do not develop into regulatory cells, remaining of reacting harmfully against the self antigens

What is the prognosis of patient with IBD determined by?

relapse rate, the rate of surgery, and the incidence of colon cancer.

What does the Superior mesenteric artery supply?

rest of the duodenum, the jejunum, the ileum, the ascending colon, and the proximal two-thirds of the transverse colon

What does the Inferior mesenteric artery supply?

rest of the transverse colon, the descending colon, the sigmoid colon, and most of the rectum

What symptoms are associated with Crohn's disease?

right lower quadrant abdominal pain, fever, weight loss, diarrhea, and sometimes a palpable inflammatory mass Hematochezia is less common than in UC.

What does significant GI bleeding (hemorrhage) manifest with?

some combination of weakness, dizziness, lightheadedness, shortness of breath, postural changes in blood pressure or pulse, cramping abdominal pain, and diarrhea

Receptor editing reduces the chance that?

that potentially harmful self-reactive B cells will leave the marrow It is estimated that 25% to 50% of mature B cells in a normal individual may have undergone receptor editing during their maturation.

What does Costimulation ensure? What do the 2 signals provide respectively?

that stimulation of a single immune receptor does not activate a damaging immune reaction Signal 1: provides specificity Signal 2: is permissive, ensuring that an inflammatory response is appropriate

What does initiation of T cell response require?

that the cells recognize antigens displayed by dendritic cells, which capture antigens and concentrate them in lymphoid organs Thus, T lymphocytes work by communicating with other cells.

Where does the Superior mesenteric vein drain blood from?

the Small intestine, cecum, ascending colon, and transverse colon It begins in the right iliac fossa as veins draining the terminal ileum, cecum, and appendix join, and ascends in the mesentery to the right of the superior mesenteric artery.

What processes is IL-6 a key mediator in?

the acute phase inflammatory response, angiogenesis, neutrophil migration, differentiation of helper T cells, bone and cartilage metabolism, lipid metabolism, and cancer

What must occur to induce a response?

the binding of T cells to APCs must be stabilized for a sufficiently long period to achieve the necessary signaling threshold This stabilization function is performed by adhesion molecules on the T cells that bind to ligands expressed on APCs

What does NFAT activation and its nuclear translocation depend on?

the concentration of calcium (Ca2+) ions in the cytosol.

What is one of the most feared and long-term complications of Ulcerative coltiis and colonic Crohns disease?

the development of Neoplasia

What does an autoimmune disease involving IL-23 promote?

the development of proinflammatory Th17 cells

What is the Portal vein?

the final common pathway for the transport of venous blood from the spleen, pancreas, gallbladder, and abdominal part of the gastrointestinal tract.

What are innate vs adaptive immune responses?

the innate immune system recognizes nonself and activates the response to an offending nonself agent the adaptive immune system generates a response that specifically neutralizes or kills that agent

What happens as Cirrhosis progresses?

the intrahepatic vasculature is distorted, which in turn leads to increased pressure in the portal vein and its draining tributaries - portal hypertension Portal hypertension produces increased pressure in the splenic venules, leading to splenic enlargement. At the sites of portosystemic anastomosis , large dilated veins (varices) develop. These veins are susceptible to bleeding and may produce marked blood loss, which in some instances can be fatal.

What occurs in a Jejunostomy? Purpose?

the jejunum is brought to the anterior abdominal wall and fixed used as a site where a feeding tube is placed through the anterior abdominal wall into the proximal efferent small bowel.

What does the celiac trunk supply?

the lower esophagus, stomach, superior part of the duodenum, and proximal half of the descending part of the duodenum

What is Costimulation?

the paradigm that cells of the immune system typically require two signals for activation If a first signal is provided in the absence of a second signal, the target immune cell may become anergic rather than activated.

What happens once the immune response is over?

the system must return to its steady state, homeostasis, so that it is prepared to respond to the next infectious pathogen

What does Colonic diverticula result from?

the unique structure of the colonic muscularis propria and elevated intraluminal pressure in the sigmoid colon focal discontinuities in the muscle are created In other parts of the intestine these gaps are reinforced by the external longitudinal layer of the muscularis propria, but, in the colon, this muscle layer is gathered into the three bands termed taeniae coli

What do Adhesion molecules on T cells recognize?

their ligands on APCs stabilize the binding of the T cells to the APCs Most TCRs bind the peptide-MHC complexes for which they are specific with low affinity.

What occurs in checkpoint blockade?

therapeutic applications of inhibitory receptors in the treatment of cancer patients with antibodies that block the CTLA-4 and PD-1 receptors the inhibitory receptors impose checkpoints in immune responses, and the treatment blocks these checkpoints ("removes the brakes" on immune responses).

What is an important function of APC in addition to displaying nonself antigens to T cells?

they provide the costimulatory signals that are necessary for T-cell activation

What is the fundamental role of the immune system?

to distinguish self from nonself Nonself can be an infectious organism, a transplanted organ, or an endogenous tissue that is mistaken or something foreign protection against infection is the classic role of the immune system

What can occur as a failure of normal liver metabolism? What can patients develop?

toxic metabolic by-products do not convert to nontoxic metabolites. This buildup of noxious compounds is made worse by the numerous portosystemic shunts, which allow the toxic metabolites to bypass the liver Severe neurological features, called hepatic encephalopathy, that can manifest as acute confusion, epileptic fits, or psychotic state

What does the activation of Theta isoform of the Protein Kinase C lead to teh activation of?

transcription factor Nuclear factor-κB (NF-κB) PKC is activated by diacylglycerol, which, like IP3, is generated by PLC-mediated hydrolysis of membrane inositol lipids. PKCθ acts through adaptor proteins recruited to the TCR complex to activate NF-κB As a result, NF-κB is released and moves to the nucleus, where it promotes the transcription of several genes

What is NFAT (nuclear factor of activated T cells)?

transcription factor present in an inactive phosphorylated form in the cytoplasm of resting T cells

How is Ulcerative colitis different from Crohns?

ulcerative colitis is limited to the colon and rectum

What can occur as a result of poorly functioning liver cells?

unable to break down blood and blood products leading to an increase in the serum bilirubin level, which manifests as jaundice.

What are some extraintestinal manifestations of Crohns disease?

uveitis, migratory polyarthritis, sacroiliitis, ankylosing spondylitis, erythema nodosum, and clubbing of the fingertips Pericholangitis and primary sclerosing cholangitis occur in Crohn disease with a higher frequency than in those without Crohn disease,

What are probiotics? What are they used in the treatment of?

viable nonpathogenic organisms that, after ingestion, may prevent or treat intestinal diseases and have been explored in the treatment of IBD

When is Peripheral tolerance induced? What does it lead to?

when mature T cells recognize self antigens in peripheral tissues leads to functional inactivation (anergy) or death, or when the self-reactive lymphocytes are suppressed by regulatory T cells

What is autoimmunity?

when the immune system attacks the individual's own cells and tissues the diseases they cause are called autoimmune diseases

When is Gastrostomy performed?

when the stomach is attached to the anterior abdominal wall and a tube is placed through the skin into the stomach. The procedure can be performed either surgically or through a direct needlestick puncture under sedation in the anterior abdominal wall.

IBD- pathogenesis part 2

• Epithelial defects. A variety of epithelial defects have been described in both Crohn disease and ulcerative colitis. Some examples follow: • Defects in intestinal epithelial tight junction barrier function are present in Crohn disease patients and a subset of their healthy first-degree relatives. In patients with Crohn disease and their relatives, this barrier dysfunction is associated with specific disease-associated NOD2 polymorphisms; experimental models demonstrate that barrier dysfunction can activate innate and adaptive mucosal immunity and sensitize subjects to disease. • Some polymorphisms, such as those involving ECM1 (extracellular matrix protein 1), which inhibits matrix metalloproteinase 9, are linked to ulcerative colitis but not Crohn disease. In this context it is notable that inhibition of matrix metalloproteinase 9 reduces the severity of colitis in experimental models. • Certain polymorphisms in the transcription factor HNFA are associated with ulcerative colitis but not Crohn disease. These HNFA polymorphisms are also strongly associated with maturity onset diabetes of the young (MODY), which like IBD, is associated with reduced intestinal barrier function Together these data suggest that, derangements in epithelial function is an important component are critical to IBD pathogenesis. Microbiota. The abundance of microbiota in the GI lumen is overwhelming, amounting to as much as 1012 organisms per milliliter in the colon and 50% of fecal mass. In total, these organisms greatly outnumber human cells in our bodies, a sober reminder that at a cellular level, we may be only about 10% human. A sampling of data that supports the notion that microbiota play a role in the evolution of IBD follows: • As mentioned earlier, linkage to NOD2, points to the involvement of microbes in the causation of Crohn disease. • The presence of antibodies against the bacterial protein flagellin are most common in Crohn disease patients who have disease associated NOD2 variants, stricture formation, perforation, and small-bowel involvement. In contrast, anti-flagellin antibodies are uncommon in ulcerative colitis patients . • Microbial transfer studies are able to induce or reduce disease in animal models of IBD, and clinical trials suggest that probiotic (or beneficial) bacteria or even fecal microbial transplants from healthy individuals may benefit IBD patients. One model that unifies the roles of intestinal microbiota, epithelial function, and mucosal immunity suggests a cycle by which transepithelial flux of luminal bacterial components activates innate and adaptive immune responses. In a genetically susceptible host, the subsequent release of TNF and other immune-mediated signals direct epithelia to increase tight junction permeability, which causes further increases in the in flux of luminal material. These events may establish a self-amplifying cycle that gives rise to maladaptative and injurious immune responses.

Key concepts

■ Irritable bowel syndrome (IBS) is characterized by chronic, relapsing abdominal pain, bloating, and changes in bowel habits without obvious gross or histologic pathology. The pathogenesis of IBS is not defined, but includes contributions by psychologic stressors, diet, the gut microbiome, abnormal GI motility, and increased enteric sensory responses to gastrointestinal stimuli. ■ Inflammatory bowel disease (IBD) is an umbrella term for ulcerative colitis and Crohn disease. Indeterminate colitis is used for cases of IBD without definitive features of either ulcerative colitis or Crohn disease. ■ Ulcerative colitis is limited to the colon, is continuous from the rectum, and ranges from only rectal disease to pancolitis; neither skip lesions nor granulomas are present. ■ Crohn disease most commonly affects the terminal ileum and cecum, but any site within the gastrointestinal tract can be involved; skip lesions are common and noncaseating granulomas also occur. ■ Both forms of IBD typically present in the teens and early 20s and are associated with extraintestinal manifestations. ■ IBD is thought to arise from a combination of alterations in host interactions with intestinal microbiota, intestinal epithelial dysfunction, and aberrant mucosal immune responses. Molecular analyses have identified more than 160 IBD-associated genes, of which the function of only a few is understood. ■ The risk of colonic epithelial dysplasia and adenocarcinoma is increased in IBD patients who have had colonic disease for more than 8 to 10 years. ■ The two forms of microscopic colitis, collagenous colitis and lymphocytic colitis, both cause chronic watery diarrhea. The intestines are grossly normal, and the diseases are identified by their characteristic histologic features. ■ Diverticular disease of the sigmoid colon is common in western populations older than age 60. The causes include low fiber diets, colonic spasm, and the unique anatomy of the colon. Inflammation of diverticula, diverticulitis, affects a minority of those with diverticulosis, but can cause perforation in its most severe form.

What are the mechanisms in which overgrowth of luminal bacteria can result in diarrhea and malabsorption?

(1) Deconjugation of bile salts, which leads to impaired micelle formation and impaired uptake of fat (2) Patchy injury to the enterocytes (small intestinal epithelial cells) (3) Direct competition for the use of nutrients (e.g., uptake of vitamin B12 by gram-negative bacteria or the fish tapeworm Diphyllobothrium latum) (4) Stimulated secretion of water and electrolytes by products of bacterial metabolism, such as hydroxylated bile acids and short-chain (volatile) organic acids.

clinical features of colonic diverticula

. Most individuals with diverticular disease remain asymptomatic throughout their lives. However, about 20% of individuals with diverticuli develop manifestations of diverticular disease, such as intermittent cramping, continuous lower abdominal discomfort, constipation, distention, or a sensation of never being able to completely empty the rectum. Patients sometimes experience alternating constipation and diarrhea that can mimic IBS. Occasionally there may be minimal chronic or intermittent blood loss, and, rarely, massive hemorrhage. When present, bleed ing is macroscopically visible in the stools. Whether a high-fiber diet prevents such progression or protects against diverticulitis is unclear, but diets supplemented with fiber may provide symptomatic improvement. Even when diverticulitis occurs, it most often resolves spontaneously and relatively few patients require surgical intervention.

Epidemiology of IBD

1- 1.4 million individuals have IBD, and the overall annual incidence is about 20 new cases per 100,000 persons years 2- C. The prevalence of Inflammatory bowel disease(IBD) in the United States is between 249 to 319 per 100,000 persons. 3-A bimodal age at presentation exists, with an initial peak between the second and fourth decades of life and another peak in the sixth decade. 4- The sexes are equally affected.

What lab tests are required for a patient presenting with Acute abdominal symptoms?

1- A complete blood cell count with differential 2- Urinalysis 3- Measurements of serum amylase, lipase, bilirubin, and electrolytes

What are the 2 types of neurons that carry pain fibers?

1- A fibers- rapid conduction 2- C fibers- slow conduction Most visceral neurons are of the C type, and the pain resulting from their stimulation tends to be variable with regard to sensation and localization

What is the pathway of sympathetic innervation of the stomach?

1- A preganglionic sympathetic fiber originating at the T6 level of the spinal cord enters an anterior root to leave the spinal cord. 2- At the level of the intervertebral foramen, the anterior root (which contains the preganglionic fiber) and a posterior root join to form a spinal nerve 3- Outside the vertebral column, the preganglionic fiber leaves the anterior ramus of the spinal nerve through the white ramus communicans. 4- The white ramus communicans, containing the preganglionic fiber, connects to the sympathetic trunk. 5- Entering the sympathetic trunk, the preganglionic fiber does not synapse but passes through the trunk and enters the greater splanchnic nerve. 6- The greater splanchnic nerve passes through the crura of the diaphragm and enters the celiac ganglion. 7- In the celiac ganglion, the preganglionic fiber synapses with a postganglionic neuron. 8- The postganglionic fiber joins the plexus of nerve fibers surrounding the celiac trunk and continues along its branches 9- The postganglionic fiber travels through the plexus of nerves accompanying the branches of the celiac trunk supplying the stomach and eventually reaches its point of distribution. 10-This input from the sympathetic system may modify the activities of the gastrointestinal tract controlled by the enteric nervous system.

How is an Ileal conduit carried out?

1- A short segment of small bowel is identified 2- The bowel is divided twice to produce a 20-cm segment of small bowel on its own mesentery. 3- This isolated segment of bowel is used as a conduit 4- The remaining bowel is joined together 5- The proximal end is anastomosed to the ureters, and the distal end is anastomosed to the anterior abdominal wall 6- Hence, urine passes from the kidneys into the ureters and through the short segment of small bowel to the anterior abdominal wall

What are 2 Crohn related genes of interest in addition to NOD2?

1- ATG16L1 (autophagyrelated 16-like) 2- IRGM (immunity-related GTPase M) Both are part of the autophagy pathways that are critical for cellular responses to intracellular bacteria ATG16L1 may also regulate epithelial homeostasis

What are the 2 best defined mechanisms responsible for induction of anergy ?

1- Abnormal signaling by the TCR complex 2- The delivery of inhibitory signals from receptors other than the TCR complex.

What are the mechanisms in the gut that account for the inability of the immune system to react against commensal microbes?

1- Abundance of IL-10-producing regulatory T cells 2- Dendritic cells such that signaling from some Toll-like receptors leads to inhibition rather than activation 3- Separation of some bacteria from the intestinal immune system by the epithelium

What are the steps involved in the migration of activated T cells into peripheral tissues?

1- Activated T cells express high levels of the glycoprotein ligands for E- and P-selectins and the integrins LFA-1 and VLA-4 (very late antigen 4). Innate immune cytokines produced at the site of infection, such as TNF and IL-1, act on the endothelial cells to increase expression of E- and P-selectins as well as ligands for integrins, especially ICAM-1 and vascular cell adhesion molecule 1 (VCAM-1), the ligand for the VLA-4 integrin. 2- Effector T cells that are passing through the blood vessels at the infection site bind first to the endothelial selectins, leading to rolling interactions. 3- Effector T cells also express receptors for chemokines that are produced by macrophages and endothelial cells at these inflammatory sites and are displayed on the surface of the endothelium. The rolling T cells recognize these chemokines, leading to increased binding affinity of the integrins for their ligands and firm adhesion of the T cells to the endothelium. 4-After the effector T lymphocytes are arrested on the endothelium, they engage other adhesion molecules at the junctions between endothelial cells, crawling through these junctions into the tissue. Chemokines that were produced by macrophages and other cells in the tissues stimulate the motility of the transmigrating T cells.

What are the steps involved in the inhibition of IL-2?

1- Activated T cells increase their production o IL-2 via a pathway that begins with dephosphorylation of a cytoplasmic transcription factor, NFAT (nuclear factor of activated T cells) 2- NFAT is dephosphorylated by the cytoplasmic phosphatase calcineurin . 3- Upon dephosphorylation, NFAT translocates to the nucleus and enhances transcription of the IL-2 gene 4- Cyclosporine(CsA) acts by binding to cyclophilin , and the CsA-cyclophilin complex binds to calcineurin and inhibits its phosphatase activity 5- By inhibiting calcineurin-mediated NFAT dephosphorylation, Cyclosporine prevents translocation of NFAT to the nucleus and thereby suppresses IL-2 production

What may Abdominal pain be classified as?

1- Acute - occurs suddenly and more often suggests serious physiologic alterations 2- Chronic - may be present for several months; although it does not mandate immediate attention, chronic pain may lead to prolonged evaluation

What are the 3 types of diarrhea?

1- Acute diarrhea - is limited to 2 weeks or less and is usually infectious in origin 2- Persistent diarrhea - longer than 14 days 3- Chronic diarrhea -is considered to last longer than 4 weeks

What are Anti-TNF agents that are administered subcutaneously? What are they used to treat?

1- Adalimumab (Humira) 2- Golimumab (Simponi) which are fully human monoclonal antibodies 3-Certolizumab pegol (Cimzia), which is a humanized anti-TNF antibody Fab fragment. Adalimumab and certulizumab are efficacious in patients with moderate to severe Crohn's disease whereas adalimumab and golimumab are approved to treat moderate to severe Ulcerative Colitis.

What do Naive T cells express which mediate the selective migration of naive cells into lymph nodes through specialized vessels called high endothelial venules (HEVs)?

1- Adhesion molecule L-selectin -CD62L 2- Chemokine receptor- CCR7

What are the ganglia and sympathetic trunk connected to?

1- Adjacent spinal nerves by gray rami communicantes throughout the length of the sympathetic trunk 2- White rami communicantes in the thoracic and upper lumbar parts of the trunk (T1 to L2)

What is Mechanical obstruction caused by?

1- An intraluminal, mural, or extrinsic mass which can be secondary to a foreign body, 2- Obstructing tumor in the wall, 3- Extrinsic compression from an adhesion, or embryological band

What are the 2 mechanisms of death of mature T lymphocytes induced by self antigens?

1- Antigen recognition induces the production of pro-apoptotic proteins in T cells that induce cell death by causing mitochondrial proteins to leak out and activate caspases, cytosolic enzymes that induce apoptosis. In normal immune responses, the activity of these pro-apoptotic proteins is counteracted by anti-apoptotic proteins that are induced by costimulation and by growth factors produced during the responses. However, self antigens, which are recognized without strong costimulation, do not stimulate production of anti-apoptotic proteins, and the relative deficiency of survival signals induces death of the cells that recognize these antigens. 2- • Recognition of self antigens may lead to the coexpression of death receptors and their ligands. This ligand-receptor interaction generates signals through the death receptor that culminate in the activation of caspases and apoptosis. The best-defined death receptor- ligand pair involved in self-tolerance is a protein called Fas (CD95), which is expressed on many cell types, and Fas ligand (FasL), which is expressed mainly on activated T cells. The Fas pathway may also be involved in death of some B cells in germinal centers

What 2 classes of cytotoxic agents are commonly used as Immunosuppressants?

1- Antimetabolites 2-Alkylating agents

What may B cells that recognize self antigens in the periphery undergo?

1- Apoptosis 2- Inhibitory receptors on the B cells may be engaged thus preventing activation

What is the pathway of the Hepatic artery proper? What does it divide into?

1- Ascends toward the liver in the free edge of the lesser omentum 2- It runs to the left of the bile duct and anterior to the portal vein, and divides into: 1- Right hepatic artery 2- Left hepatic artery

What are the immunomodulators used in IBD?

1- Azathioprine and its active metabolite, 6-mercaptopurine (6-MP) as well 2- Methotrexate 3- Cyclosporine.

What does the interaction of Helper T cell (Th) with B cell (CD40) promote?

1- B cell activation 2- Isotype switching 3- Clonal expansion

Describe the pathway of the large intestine

1- Begins in the right groin as the cecum with its associated appendix 2- It continues upward as the ascending colon through the right flank and into the hypochondrium 3- Just below the liver, it bends to the left, forming the right colic flexure (hepatic flexure), 4- It then crosses the abdomen as the transverse colon to the left hypochondrium 5-At this position, just below the spleen, the large intestine bends downward, forming the left colic flexure (splenic flexure), and continues as the descending colon through the left flank and into the left groin.

What are common features of Ulcerative colitis?

1- Broad based ulcers 2- Isolated islands of regenerating mucosa often bulge into the lumen to create pseudopolyps, and the tips of these polyps may fuse to create mucosal bridges 3- Chronic disease may lead to mucosal atrophy with a flat and smooth mucosal surface that lacks normal folds. 4- Unlike Crohn disease, mural thickening is not present, the serosal surface is normal, and strictures do not occur. 5-Inflammation and inflammatory mediators can damage the muscularis propria and disturb neuromuscular function leading to colonic dilation and toxic megacolon, which carries a significant risk of perforation. 6- The inflammatory process is diffuse and generally limited to the mucosa and superficial submucosa 7- Granulomas are NOT present in ulcerative colitis.

How does CTLA-4 work?

1- By blocking and removing B7 molecules from the surface of APCs, thus reducing costimula tion 2- Preventing the activation of T cells 3- CTLA-4 might also deliver inhibitory signals to T cells

What is Rituximab approved for the use in?

1- CD20 Non-Hodgkin's lymphoma 2- Rheumatoid arthritis refractory to TNF inhibitors 3- Granulomatosis with polyangitis

What are the 2 best-defined costimulators for T cells? What are they recognized by on the T cell?

1- CD80 (B7-1) 2- CD86 (B7-2) both of which are expressed on APCs and whose expression is increased when the APCs encounter microbes These are recognized by a receptor called CD28, which is expressed on most T cells

What are 2 important inhibitory receptors?

1- CTLA-4 2- PD-1 CTLA-4, like CD28, recognizes B7-1 and B7-2 on APCs, and PD-1 recognizes different but structurally related ligands on many cell types

What are the major signaling pathways linked to the zeta-chain phosphorylation and ZAP-70 ?

1- Calcium-NFAT pathway 2- Ras- and Rac-MAP kinase pathways 3- PKCθ-NF-κB pathway 4- PI-3 kinase pathway

What diseases does Lymphocytic colitis show strong association with?

1- Celiac disease 2- Autoimmune diseases including: Graves disease, rheumatoid arthritis, and autoimmune or lymphocytic gastritis.

What are the 3 branches of the abdominal aorta that supply the GI viscera?

1- Celiac trunk 2- Superior mesenteric artery 3- Inferior mesenteric artery

What are the abdominal parts of the GI system mainly supplied by?

1- Celiac trunk 2- Superior mesenteric artery (SMA) 3- Inferior mesenteric artery (IMA)

What are the pathways in which complement system can be activated?

1- Classical pathway 2- Alternative pathway 3- Lectin pathway In each pathway, a series of proteolytic reactions converts a complement precursor protein into its active form eg: C3 is cleaved into its active forms C3a and C3b

What are the 2 primary mechanisms for avoiding autoimmunity?

1- Clonal deletion -T cells die during development when they express high affinity receptors that recognize self-antigen 2- Tolerance or anergy -cells of the immune system undergo a carefully regulated series of steps during development to ensure that mature immune cells do not recognize native proteins.

Epidemiology of Diverticula

1- Colonic diverticula are rare in persons younger than age 30, but the prevalence approaches 50% in Western adult populations older than age 60 2- This disease is much less common in Japan as well as developing countries, probably because of dietary differences. 3- Moreover, most diverticula in Asia and Africa occur in the right colon, while right-sided diverticula are uncommon in Western countries. Diverticula are generally multiple and the condition is referred to as diverticulosis.

What do the major opsonins consist of?

1- Complement 2- Immunoglobulins (antibodies), 3- Collectins (plasma proteins that bind to certain microbial carbohydrates).

What are examples of inhibitory checkpoint molecules?

1- Cytotoxic T lymphocyte antigen 4 (CTLA-4) 2- Programmed cell death protein 1 (PD-1) ligation of these molecules on T cells inhibits the immune response

What is Collagenous colitis characterized by?

1- Dense subepithelial collagen layer 2- increased numbers of intraepithelial lymphocytes 3- A mixed inflammatory infiltrate within the lamina propria occurs primarily in middle-aged and older women

What long term effects are associated with the use of Glucocorticoids?

1- Diabetes 2- Reduced resistance to infection, 3- Osteoporosis 4- Cataracts 5- Increased appetite leading to weight gain 6- Hypertension and its sequelae 7- Masking of inflammation

What occurs in the Luminal phase of absorption?

1- Digestion is accomplished for the most part by pancreatic enzymes, particularly lipase, colipase, and trypsin; the gastric digestive enzymes do not play a major role as a result chronic pancreatitis can result in malabsorption, particularly for fat and protein

What factors are related to the risk of dyplasia?

1- Duration of the disease -Risk increases sharply 8 to 10 years after disease onset 2- Extent of the disease -Patients with pancolitis are at greater risk than those with only left-sided disease 3- Nature of the inflammatory response -Greater frequency and severity of active inflammation (characterized by the presence of neutrophils) confers increased risk.

What are common features of Crohns disease?

1- Edema and loss of the normal mucosal texture are common. 2-Crohn disease, results in a coarsely textured, cobblestone appearance in which diseased tissue is depressed below the level of normal mucosa 3- Fissures frequently develop between mucosal folds and may extend deeply to become fistula tracts or sites of perforation 4-In cases with extensive transmural disease, mesenteric fat frequently extends around the serosal surface (creeping fat) 5- The microscopic features of active Crohn disease include abundant neutrophils that infiltrate and damage crypt epithelium. Clusters of neutrophils within a crypt are referred to as crypt abscesses and are often associated with crypt destruction. 6- Ulceration is common in Crohn disease, and there may be an abrupt transition between ulcerated and adjacent normal mucosa. 7- Repeated cycles of crypt destruction and regeneration lead to distortion of mucosal architecture; the normally straight and parallel crypts take on bizarre branching shapes and unusual orientations to one another 8- Epithelial metaplasia, another consequence of chronic relapsing injury, often takes the form of gastric antral-appearing glands, and is called pseudopyloric metaplasia. 9- Paneth cell metaplasia may also occur in the left colon, where Paneth cells are normally absent. These architectural and metaplastic changes may persist even when active inflammation has resolved 10- Noncaseating granulomas (Fig. 17-35B), a hallmark of Crohn disease, are found in approximately 35% of cases and may occur in areas of active disease or uninvolved regions in any layer of the intestinal wall 11- Cutaneous granulomas form nodules that are referred to as metastatic Crohn disease (a misnomer since there is no cancer). The absence of granulomas does not preclude a diagnosis of Crohn disease.

What are the adverse effects associated with Abatacept use?

1- Exacerbations of preexisting chronic obstructive lung disease 2- Increased susceptibility to infection Abatacept should not be administered concurrently with TNF inhibitors because the combination carries an unacceptably high risk of infection.

After activation complement triggers inflammatory responses by what 2 mechanisms?

1- First, several cleavage products of the complement cascade are potent stimulators of inflammation eg: C3b is an important opsonin, and C3a and C5a mediate leukocyte chemotaxis 2- Second, the final step in complement activation is the assembly of the membrane attack complex -This complex of complement proteins produces large pores in the outer membrane of Gram negative bacteria, leading to lysis of the bacteria

What are the 4 phases of the inflammatory response?

1- First, the vasculature around a site of injury reacts to recruit cells of the immune system 2- Second, circulating immune cells migrate from these vessels into the injured tissues, and the mechanisms of innate and adaptive immunity serve to neutralize and remove the inciting stimulus 3- Next, the process of repair and tissue healing ensues and the acute inflammatory process is terminated 4- If the process of acute inflammation is not halted but continues to smolder, chronic inflammation can occur.

What do the main features of adaptive immunity rely on?

1- First, there must be a mechanism to generate a specific response to a foreign antigen - is provided by major histocompatibility complex (MHC) proteins and by somatic gene recombination in T cells and B cells 2- Second, adaptive immune cells must be able to distinguish native (self) cells and soluble factors from foreign (nonself) cells and soluble factors - is provided by signals from the innate immune system, by regulated immune cell development, and by costimulation.

What 3 important tasks do innate immune cells perform?

1- First, these cells defend against bacterial and parasitic infections, either by neutralizing the infectious agent with secreted cytotoxic proteins or by phagocytosis (engulfing) of the bacterium or parasite 2- Second, phagocytosis of the of ending agent initiates proteolytic digestion of microbial macromolecules to fragments (antigens) that are then displayed, together with major histocompatibility complex (MHC) class II proteins, on the surface of antigen presenting cells. In turn, these antigen-presenting cells, which include macrophages and dendritic cells, activate cells of the adaptive immune system 3- Third, innate immune cells secrete numerous cytokines (see below) that further amplify the immune response.

What can the primitive gut tube be divided into?

1- Foregut 2- Midgut 3- Hindgut the boundaries of these regions are directly related to the areas of distribution of the 3 anterior branches of the abdominal aorta

What do Biliary tract diseases induce increased incidence of?

1- Gallstones 2- Primary sclerosing cholangitis (PSC).

What are the most important factors maintaining the relative sterility of the upper gut?

1- Gastric acidity 2- Peristalsis 3-Intestinal immunoglobulins (IgA). Conditions that impair these functions can result in bacterial overgrowth

What is a mechanism that allows tolerance to fetal antigens?

1- Generation of peripheral FoxP3+ regulatory T cells 2- exclusion of inflammatory cells from the pregnant uterus 3- Poor antigen presentation in the placenta 4- An inability to generate harmful Th1 responses in the healthy pregnant uterus.

What are the major cell types of the innate immune system?

1- Granulocytes (neutrophils, eosinophils, and basophils) 2- Mast cells 3- Antigen-presenting cells (macrophages and dendritic cells).

What does the Ras/Rac-MAP kinase pathway include?

1- Guanosine triphosphate (GTP)- binding Ras and Rac proteins, 2- Several adaptor proteins 3- Cascade of enzymes that eventually activate one of a family of mitogen-activated protein (MAP) kinases

What does the Superior pancreatoduodenal artery supply?

1- Head of the pancreas 2- Duodenum These vessels eventually anastomose with the anterior and posterior branches of the inferior pancreaticoduodenal artery

What are T cells divided into?

1- Helper T cells (Th)- CD4 T cells (recognize MHC II) 2- Cytotoxic T cells (Tc)- CD8 T cells (recognize MHC I)

What can IBD-associated dysplasia be classified histologically as?

1- High grade dysplasia 2- Low grade dysplasia 3- Multifocal

What is Adaptive immunity generally divided into?

1- Humoral immunity- B cells 2- Cellular immunity- T cells

What are the 3 major phases that transplant rejection of solid organs can be divided into?

1- Hyperacute rejection 2- Acute rejection 3- Chronic rejection are all caused by different mechanisms and are therefore treated differently

What cytokines does Th17 produce?

1- IL-17 2- IL-21 3- IL-22

What cytokines does Th2 produce? function?

1- IL-4 2- IL-5 3- IL-10 these cytokines enhance antibody production by B cells The Th2 cell subtype is more often associated with autoimmunity

Describe the pattern of autoimmunity in Monozygotic vs Dizygotic twins

1- If an autoimmune disease develops in one of two twins, the same disease is more likely to develop in the other twin than in an unrelated member of the general population 2- The incidence is greater among monozygotic (identical) twins than among dizygotic twins

Describe the level of glucose activation/uptake in T cells

1- In naive (resting) T cells, low levels of glucose are taken up and used to generate energy in the form of ATP, by mitochondrial oxidative phosphorylation 2- Upon activation, glucose uptake increases markedly, and the cells switch to aerobic glycolysis 3- This process generates less ATP but facilitates the synthesis of more amino acids, lipids, and other molecules that provide building blocks for organelles and for producing new cells 4- As a result, it is possible for activated T cells to more efficiently manufacture the cellular constituents that are needed for their rapid increase in size and for producing daughter cells

The responses of naive T lymphocytes to cell-associated microbial antigens consist of a series of sequential steps that result in what?

1- Increase in the number of antigen-specific T cells 2- The conversion of naive T cells to effector and memory cells

What can an infection in a tissue induce and what can it lead to?

1- Induce a local innate immune response 2- May lead to increased production of costimulators and cytokines by tissue APCs 3- These activated tissue APCs may be able to stimulate self-reactive T cells that encounter self antigens in the tissue In other words, infection may break T cell tolerance and promote the activation of selfreactive lymphocytes

What are the branches of the SMA?

1- Inferior pancreaticoduodenal artery 2- Jejunal and ileal arteries on its left

What are AZA and 6-MP also used as immunosupressive agents in the treatment of?

1- Inflammatory bowel disease (IBD) 2- Acute lymphoblastic leukemia 3- Autoimmune skin disorders

What is the principal factors in the development of autoimmunity?

1- Inheritance of susceptibility genes 2- Environmental triggers, such as infections It is postulated that susceptibility genes interfere with pathways of self-tolerance and lead to the persistence of self-reactive T and B lymphocytes. Environmental stimuli may cause cell and tissue injury and inflammation and activate these self-reactive lymphocytes, resulting in the generation of effector T cells and autoantibodies that are responsible for the autoimmune disease

What are the 8 mechanistic approaches that pharmacologic suppression of the immune system utilizes?

1- Inhibition of gene expression to modulate inflammatory responses 2- Depletion of expanding lymphocyte populations with cytotoxic agents 3. Inhibition of lymphocyte signaling to block activation of lymphocytes and expansion of lymphocyte populations 4. Neutralization of cytokines and cytokine receptors essential or mediating the immune response 5. Depletion of specific immune cells, usually via cellspecific antibodies 6. Blockade of costimulation to induce anergy 7. Blockade of cell adhesion to prevent migration and homing of inflammatory cells 8. Inhibition of innate immunity, including complement activation

Describe the progression of pain in appendicitis

1- Initially, the pain begins as a central, periumbilical, colicky type of pain, which tends to come and go 2- After 6 to 10 hours, the pain tends to localize in the right iliac fossa and becomes constant Patients may develop a fever, nausea, and vomiting.

What liver complications are associated with IBD?

1- Intraheaptic diseases 2- Biliary tract diseases

What is the general characteristics of the large intestine?

1- Its large internal diameter compared to that of the small intestine; 2- Peritoneal-covered accumulations of fat (the omental appendices) are associated with the colon; 3- The segregation of longitudinal muscle in its walls into three narrow bands (the taeniae coli), which are primarily observed in the cecum and colon and less visible in the rectum; and 4-The sacculations of the colon (the haustra of the colon).

What is Abatacept approved for the treatment of?

1- Juvenile idiopathic arthritis 2- Rheumatoid arthritis that is refractory to Methotrexate or TNF inhibitors Clinically, abatacept significantly improves symptoms of rheumatoid arthritis in patients who ail to respond to methotrexate or TNF inhibitors

What is posterior to the superior mesenteric artery?

1- Lefft renal vein 2- Uncinate process of the pancreas 3- Inferior part of the duodenum

What are the branches of the Inferior mesenteric artery (IMA)?

1- Left colic artery 2- Several sigmoid arteries 3- Superior rectal artery

What does the Celiac trunk immediately divide into?

1- Left gastric artery 2- Splenic artery 3- Common hepatic arteries

What phases are involved in the complete process of absorption?

1- Luminal phase - various nutrients are hydrolyzed and solubilized 2- Mucosal phase -further processing takes place at the brush border of the epithelial cell with subsequent transfer into the cell 3- Transport phase - nutrients are moved from the epithelium to the portal venous or lymphatic circulation. impairment of any of these phases can lead to malabsorption

What are the 2 types of Microscopic colitis?

1- Lymphocytic 2- Collagenous

What are the 2 classes of MHC proteins?

1- MHC I 2- MHC II MHC class I proteins primarily display fragments of cytosolic proteins

What can extensive ileal mucosal disease lead to?

1- Malabsorption of Vitamin B12 resulting in megaloblastic anemia and neurologic side effects if not corrected 2- Malabsorption of Bile salts resulting in diarrhea induced by unabsorbed bile salts and potential fatsoluble vitamin deficiency Depletion of the bile salt pool can lead to the formation of gallstones

What 2 main procedures/surgeries can patients who are morbidly obese undergo?

1- Malabsorptive procedures 2- Restrictive procedures

What characteristics do memory T cells have?

1- Memory cells survive even after the infection is eradicated and antigen is no longer present. Certain cytokines, including IL-7 and IL-15, which are produced by stromal cells in tissues, may serve to keep memory cells alive and cycling slowly. 2- Memory T cells may be rapidly induced to produce cytokines or kill infected cells on encountering the antigen that they recognize 3-Memory T cells can be found in lymphoid organs, in various peripheral tissues, especially mucosa and skin, and in the circulation

What can cause impaired peristalsis be caused by?

1- Motility disorders eg: scleroderma, amyloidosis, diabetes mellitus 2- Anatomic changes eg: Surgically created blind loops, obstruction, jejunal diverticulosis Achlorhydria, pancreatic insufficiency, and hypogammaglobulinemia are also associated with bacterial overgrowth but uncommonly result in clinical steatorrhea.

What are examples of newer antimetabolites that have fewer adverse effects?

1- Mycophenolate mofetil 2- Leflunomide

The cells of the immune system derive from what 2 types of pluripotent cells in the bone marrow?

1- Myeloid stem cells 2- Lymphoid stem cells Myeloid stem cells give rise to precursor cells of the innate immune system Lymphoid stem cells generate precursor cells of the adaptive immune system

What is the formula recommended by the world health organization

1- NaCl 2.6 g (0.092 ounce) 2- Trisodium citrate dihydrate 2.9 g (0.10 ounce) 3- KCl 1.5 g (0.053 ounce) 4- Anhydrous glucose 13.5 g (0.48 ounce) or sucrose 27 g (0.96 ounce) 5- 1 L water

What are the steps involved in the migration of leukocytes through blood vessels?

1- Naive T cells in the blood engage in L-selectin- mediated rolling interactions with the HEV, allowing chemokines to bind to CCR7 on the T cells. 2- CCR7 transduces intracellular signals that activate the integrin leukocyte function-associated antigen 1 (LFA-1) on the naive T cell, increasing the binding affinity of the integrin. 3- The increased affinity of the integrin for its ligand, intercellular adhesion molecule 1 (ICAM-1) on the HEV, results in firm adhesion and arrest of the rolling T cells. 4- The T cells then exit the vessel through the endothelial junctions and are retained in the T cell zone of the lymph node because of the chemokines produced there

What mechanisms have evolved to overcome the challenges that T cells face in the generation of a useful cell-mediated immune response?

1- Naive T cells need to find the antigen. This problem is solved by APCs that capture the antigen and concentrate it in specialized lymphoid organs in the regions through which naive T cells recirculate. 2- The correct type of T lymphocytes (i.e., CD4+ helper T cells or CD8+ CTLs) must respond to antigens from the endosomal and cytosolic compartments. This selectivity is determined by the specificity of the CD4 and CD8 coreceptors for class II and class I MHC molecules, and by the segregation of extracellular (vesicular) and intracellular (cytosolic) protein antigens for display by class II and class I MHC molecules, respectively 3- T cells should respond to microbial antigens but not to harmless proteins. This preference for microbes is maintained because T cell activation requires costimulators that are induced on APCs by microbes. 4- Antigen recognition by a small number of T cells must lead to a response that is large enough to be effective. This is accomplished by robust clonal expansion after stimulation and by several amplification mechanisms induced by microbes and activated T cells themselves that enhance the response. 5- The response must be optimized to combat different types of microbes. This is accomplished largely by the development of specialized subsets of effector T cells

What are the steps involved the process of elimination of infectious agents starting with naive T lymphocytes?

1- Naive T lymphocytes constantly recirculate through peripheral lymphoid organs searching for foreign protein antigens. 2- The antigens of microbes are transported from the portals of entry of the microbes to the same regions of peripheral lymphoid organs through which naive T cells recirculate 3- In these organs, the antigens are processed and displayed by MHC molecules on dendritic cells, the antigenpresenting cells (APCs) that are the most efficient stimulators of naive T cell 4- When a T cell recognizes antigen, it is transiently arrested on the dendritic cell and it initiates an activation program 5- Following activation and differentiation, the cells may leave the lymphoid organ and migrate preferentially to the inflamed tissue, the original source of the antigen

What are the side effects associated with use of Anakinra?

1- Neutropenia 2- Increased susceptibility to infection

What is Cyclosporine approved for the use in?

1- Organ transplantation 2- Psoriasis 3- Rheumatoid arthritis An ophthalmic preparation of Cyclosporine is approved for the treatment of chronic dry eyes.

Describe the different types of pains

1- Pain from Perforated viscus- intense 2- Pain from dissecting aneurysm- tearing or crushing 3- Chronic pain- maybe less severe 4- Pain from Irritable bowel syndrome- constant and dull 5- Pain of Chronic peptic ulcer- gnawing or hunger pain

WHat is Eculizumab approved for the treatment of?

1- Paroxysmal nocturnal hemoglobinuria (PNH) - it significantly decreases hemoglobinuria and the need or erythrocyte transfusions in patients with this disorder 2- Atypical hemolytic uremic syndrome

What does Restrictive procedures involve?

1- Placing a band or stapling in or around the stomach to decrease the size of the organ 2- This reduction produces an earlier feeling of satiety and prevents the patient from overeating.

What 2 factors are linked to diverticular disease?

1- Poor dietary fiber intake 2- Obesity

What may be the various locations of the appendix?

1- Posterior to the cecum or the lower ascending colon, or both, in a retrocecal or retrocolic position; 2- Suspended over the pelvic brim in a pelvic or descending position 3-Below the cecum in a subcecal location 4-Anterior to the terminal ileum, possibly contacting the body wall, in a pre-ileal position or posterior to the terminal ileum in a postileal position.

What does the abdominal prevertebral plexus recieve?

1- Preganglionic parasympathetic and visceral afferent fibers from the vagus nerves [X] 2- Preganglionic sympathetic and visceral afferent fibers from the thoracic and lumbar splanchnic nerves 3- Preganglionic parasympathetic fibers from the pelvic splanchnic nerves.

Depending on the extent of the disease what can Ulcerative colitis be divided into?

1- Proctitis (rectum only) 2- Proctosigmoiditis (rectum and sigmoid) 3- Left-sided colitis (extending to the splenic fexure) 4- Pancolitis (inflammation extends proximal to the splenic flexure)

What are 3 general forms that the specific expression of autoimmunity can take?

1- Production of autoantibodies against a specific antigen - causes antibody-dependent opsonization of cells in the target organ with subsequent cyotoxicity eg: Goodpasture's syndrome- results from autoantibodies against type IV collagen in the renal glomerular basement membrane 2- In some autoimmune vasculitis syndromes, circulating antibody-antigen complexes deposit in blood vessels, causing inflammation and injury to the vessels -eg: mixed essential cryoglobulinemia and systemic lupus erythematosus 3- T cell mediated disease are caused by cytotoxic T cells that react with a specific self-antigen, resulting in destruction of the tissue(s) expressing that antigen eg: Type I diabetes mellitus, in which the cytotoxic T cells react against self-antigens in the pancreatic beta cells

What are the 2 classic dermatologic manifestations of IBD?

1- Pyoderma gangrenosum 2- Erythema nodosum

What is the function of the Enteric system?

1- Regulates and coordinates numerous gastrointestinal tract activities, including gastric secretory activity, gastrointestinal blood flow, 2- Regulates the contraction and relaxation cycles of smooth muscle (peristalsis).

What conditions is Brentuximab vedotin used to treat?

1- Relapsing Hodgkin's lymphoma after failure of multi-agent chemotherapy or failure of autologous stem cell transplant 2- Systemic anaplastic large cell lymphoma after failure of at least one multi-agent chemotherapy regimen

What is Infliximab approved for use in treating?

1- Rheumatoid arthritis 2- Crohn's disease 3- Ulcerative colitis 4- Plaque psoriasis 5- Psoriatic arthritis 6- Ankylosing spondylitis

What are 3 disorders where inhibition of TNF-alpha has demonstrated therapeutic effect?

1- Rheumatoid arthritis 2- Psoriasis 3- Crohn's disease

What are the tributaries to the portal vein?

1- Right and left gastric veins draining the lesser curvature of the stomach and abdominal esophagus 2- Cystic veins from the gallbladder, 3- The para-umbilical veins, which are associated with the obliterated umbilical vein and connect to veins on the anterior abdominal wall

Describe the role of genetics in IBD

1- Risk of disease is increased when there is an affected family member and, in Crohn disease, the concordance rate for monozygotic twins approaches 50% 2- Twins affected by Crohn disease tend to present within a few years of each other and develop disease in similar regions of the GI tract 3- The concordance of monozygotic twins for ulcerative colitis is only about 15%, suggesting that genetic factors are less dominant than in Crohn disease. 4- Concordance for dizygotic twins is less than 10% for both forms of IBD.

What 3 families of proteins control the migration of naive and effector T cells?

1- Selectins 2- Integrins 3- Chemokines

What are the ways in which self antigens differ from foreign microbial antigens?

1- Self antigens are present in the thymus, where they induce deletion and generate regulatory T cells; by contrast, most microbial antigens tend to be excluded from the thymus, because they are typically captured from their sites of entry and transported into peripheral lymphoid organs (see Chapter 3). 2- Self antigens are displayed by resting APCs in the absence of innate immunity and second signals, thus favoring the induction of T cell anergy or death, or suppression by regulatory T cells. By contrast, microbes elicit innate immune reactions, leading to the expression of costimulators and cytokines that promote T cell proliferation and differentiation into effector cells. 3- Self antigens are present throughout life and may therefore cause prolonged or repeated TCR engagement, again promoting anergy, apoptosis, and the development of regulatory T cells

What does the Splenic artery give off as it approaches the spleen?

1- Short gastric arteries, which pass through the gastrosplenic ligament to supply the fundus of the stomach 2- Left gastro-omental artery, which runs to the right along the greater curvature of the stomach, and anastomoses with the right gastro-omental artery

What 2 signals do T lymphocytes need to induce their proliferation and differentiation?

1- Signal 1 is always antigen 2- Signal 2 is provided by costimulators that are expressed on antigen-presenting cells (APCs) in Self reactive T lymphocytes with receptors for the self antigens are able to recognize the antigens and thus receive signals from their antigen receptors (signal 1), but the T cells do not receive strong costimulation because there is no accompanying innate immune respons

What are the 2 costimulatory signals for T cells?

1- Signal 1 is mediated by MHC-antigen- TCR interaction 2- Signal 2 is mediated predominantly by the interaction of CD28 on T cells with B7-1 (CD80) or B7-2 (CD86) on activated antigen presenting cells ie: CD28-B7-1(CD80) or CD28-B7-2 The lack of expression of B7 molecules in the absence of an innate immune response may help to limit inappropriate adaptive immune responses

What 2 types of pain can abdominal pain be classified as?

1- Somatic pain - pain originates from the abdominal wall and parietal peritoneum 2- Visceral pain - whereas visceral pain originates in internal organs and from the visceral peritoneum

How does Receptor editing occur in B cells?

1- Some immature B cells that recognize self antigens in the bone marrow may reexpress RAG genes, resume immunoglobulin (Ig) light-chain gene recombination, and express a new Ig light chain 2- This new light chain associates with the previously expressed Ig heavy chain to produce a new antigen receptor that may no longer be specific for the self antigen.

What are the mechanisms by which Regulatory T cells can supress immune responses?

1- Some regulatory cells produce cytokines (e.g., IL-10, TGF-β) that inhibit the activation of lymphocytes, dendritic cells, and macrophages 2- Regulatory cells express CTLA-4, which, may block or remove B7 molecules made by APCs and make these APCs incapable of providing costimulation via CD28 and activating T cells. 3- Regulatory T cells, by virtue of the high level of expression of the IL-2 receptor, may bind and consume this essential T cell growth factor (IL-2), thus reducing its availability for responding T cells.

What is the SMA crossed anteriorly by?

1- Splenic vein 2- The neck of the pancreas

What does Visceral pain result from?

1- Stretching of the walls of hollow organs or of the capsule of solid organ 2-Inflammation or ischemia.

What makes up the TCR complex?

1- TCR 2- CD3 3- Zeta chain

What is the function of CTLA-4?

1- Terminate activation of responding T cells 2-Mediates the suppressive function of regulatory T cells

What are the subsets of Helper T cells?

1- Th1 2- Th2 3- Th17 based on the cytokines produced by the cells

What occurs when the T cell receives both signal 1 and signal 2?

1- The T cell is activated 2- IL-2 is expressed 3- Clonal expansion of Th cells specific for that foreign epitope occurs

How should one perform palpatation?

1- The abdomen should be palpated gently, starting in an area away from the pain 2- The examiner searches for areas of localized tenderness and rebound as well as for masses and enlarged organs. 3- Percussion is performed to identify the size of organs or to determine the presence of ascites

What is the arterial supply to the cecum and appendix?

1- The anterior cecal artery from the ileocolic artery (from the superior mesenteric artery) 2- The posterior cecal artery from the ileocolic artery (from the superior mesenteric artery) 3- The appendicular artery from the ileocolic artery (from the superior mesenteric artery

What is included in the arterial supply to the ascending colon?

1- The colic branch from the ileocolic artery (from the superior mesenteric artery), 2- The anterior cecal artery from the ileocolic artery (from the superior mesenteric artery) 3- The posterior cecal artery from the ileocolic artery (from the superior mesenteric artery) 4- The right colic artery from the superior mesenteric artery.

What do the motor and sensory neurons of the enteric nervous system control?

1- The coordinated contraction and relaxation of intestinal smooth muscle 2- Regulation gastric secretion and blood flow.

What complications can occur in diverticular disease?

1- The diverticula can perforate to form an abscess in the pelvis. 2- The inflammation may produce an inflammatory mass, obstructing the left ureter. 3- Inflammation may also spread to the bladder, producing a fistula between the sigmoid colon and the bladder 4- . In these circumstances patients may develop a urinary tract infection and rarely have fecal material and gas passing per urethra

What are the 2 pharmacologic strategies that are used to target the pathology of immune disease?

1- The first involves modification of the signaling mediators of the inflammatory process or suppression of components of the immune system 2- The second pharmacologic approach involves modifcation of the underlying pathophysiologic stimulus, thus removing the impetus for inflammation

What are the 2 types of IBD related Arthritis?

1- The first is a peripheral, large-joint, asymmetrical, seronegative, oligoarticular, nondeforming arthritis that may involve the knees, hips, wrists, elbows, and ankles. This peripheral arthropathy usually parallels the course of the large bowel disease and typically lasts for only a few weeks 2- The second type of IBDrelated arthritis is axial in location, consisting of sacroiliitis or ankylosing spondylitis, and does not parallel the activity level of the bowel disease Ankylosing spondylitis occurs in 5% to 10% of IBD patients and manifests with low back pain and stiffness that is usually worse during the night, in the morning, or after inactivity. Sacroiliitis alone (without ankylosing spondylitis) is common in IBD (up to 20% of patients), but in many cases is asymptomatic.

In normal individuals, 100% of the portal venous blood flow can be recovered from the hepatic veins, whereas in patients with elevated portal vein pressure (e.g., from cirrhosis), there is significantly less blood flow to the liver. The rest of thesystemic circulation at specific points. The largest of these collaterals occur at?

1- The gastroesophageal junction around the cardia of the stomach—where the left gastric vein and its tributaries form a portosystemic anastomosis with tributaries to the azygos system of veins of the caval system 2- The anus—the superior rectal vein of the portal system anastomoses with the middle and inferior rectal veins of the systemic venous system 3- The anterior abdominal wall around the umbilicus—the para-umbilical veins anastomose with veins on the anterior abdominal wall.

What 3 thoracic splanchnic nerves pass from sympathetic ganglia along the sympathetic trunk in the thorax to the prevertebral plexus ?

1- The greater splanchnic nerve arises from the fifth to the ninth (or tenth) thoracic ganglia and travels to the celiac ganglion in the abdomen (a prevertebral ganglion associated with the celiac trunk). 2- The lesser splanchnic nerve arises from the ninth and tenth (or tenth and eleventh) thoracic ganglia and travels to the aorticorenal ganglion 3- The least splanchnic nerve, when present, arises from the twelfth thoracic ganglion and travels to the renal plexus.

Describe the features of IBS

1- The peak prevalence of IBS is between 20 and 40 years of age 2- There is a significant female predominance. 3- Prevalence in developed countries of between 5% and 10%. 4- IBS is not associated with serious long-term sequelae, but affected patients may undergo unnecessary abdominal surgery due to chronic pain and their ability to function socially may be compromised 5- The prognosis of IBS is most closely related to symptom duration, with longer duration correlating with reduced likelihood of improvement.

What helps differentiate Crohns from UC?

1- The presence of multiple, separate, sharply delineated areas of disease, resulting in skip lesions 2- Strictures are common in Crohn disease, but do not generally develop in ulcerative colitis

What is the arterial supply to the transverse colon?

1- The right colic artery from the superior mesenteric artery 2- The middle colic artery from the superior mesenteric artery 3- The left colic artery from the inferior mesenteric artery

What are tributaries to the superior mesenteric vein?

1- The right gastro-omental vein, draining the right part of the greater curvature of the stomach 2- The anterior and posterior inferior pancreaticoduodenal veins, which pass alongside the arteries of the same name; the anterior superior pancreaticoduodenal vein usually empties into the right gastro-omental vein, and the posterior superior pancreaticoduodenal vein usually empties directly into the portal vein.

What is the arterial blood supply to the rectum and anal canal?

1- The superior rectal artery from the inferior mesenteric artery, 2- The middle rectal artery from the internal iliac artery 3- The inferior rectal artery from the internal pudendal artery (from the internal iliac artery).

What are the 2 different types of Splanchnic nerves depending on the type of visceral efferent fiber they are carrying?

1- The thoracic, lumbar, and sacral splanchnic nerves carry preganglionic sympathetic fibers from the sympathetic trunk to ganglia in the prevertebral plexus, and also visceral afferent fibers 2- The pelvic splanchnic nerves carry preganglionic parasympathetic fibers from anterior rami of S2, S3, and S4 spinal nerves to an extension of the prevertebral plexus in the pelvis (the inferior hypogastric plexus or pelvic plexus

These collections of neuronal cell bodies outside the CNS are the paravertebral sympathetic ganglia, there are usually?

1- Three ganglia in the cervical region 2- Eleven or twelve ganglia in the thoracic region 3- Four ganglia in the lumbar region, 4- Four or five ganglia in the sacral region 5- The ganglion impar anterior to the coccyx

Glucocorticoids downregulate the expression of what inflammatory mediators?

1- Tumor necrosis factor (TNF) 2- Interleukin-1 (IL-1) 3- IL-4

What are the 2 disorders that comprise IBD?

1- Ulcerative Colitis 2- Crohn's disease

What 2 disorders does inflammatory bowel disease (IBD) comprise of?

1- Ulcerative colitis 2- Crohn's disease diagnosis of IBD is based on review of clinical, endoscopic, radiologic, and histologic data

Epidemiology of UC and Crohns

1- Ulcerative colitis and Crohn disease frequently present in the teens and early 20s, with the former being slightly more common in females 2- IBD is most common among Caucasians and, in the United States, occurs 3 to 5 times more often among eastern European (Ashkenazi) Jews than the general population 3- The geographic distribution of IBD is highly variable, but it is most common in North America, northern Europe, and Australia. However, IBD incidence worldwide is on the rise, and it is becoming more common in regions such as Africa, South America, and Asia where its prevalence was historically low 4- The hygiene hypothesis suggests that this increasing incidence is related to improved food storage conditions, decreased food contamination, and changes in gut microbiome composition 5-

When does acute appendicitis occur?

1- Usually occurs when the appendix is obstructed by either a fecalith or enlargement of the lymphoid nodules 2- Within the obstructed appendix, bacteria proliferate and invade the appendix wall, which becomes damaged by pressure necrosis. 3- In some instances, this may resolve spontaneously; in other cases, inflammatory change continues and perforation ensues, which may lead to localized or generalized peritonitis

What are the 2 ocular manifestations of IBD?

1- Uveitis - 2- Episcleritis

What are the 2 sources of parasympathetic innervation of the abdominal part of the GI tract and of the pancreas, spleen, gallbladder and liver?

1- Vagus Nerve (CN X) 2- Pelvic splanchnic nerves

When the pressure in the portal vein is elevated, venous enlargement (varices) tend to occur at and around the sites of portosystemic anastomoses and these enlarged veins are called?

1- Varices at the anorectal junction, 2- Esophageal varices at the gastroesophageal junction 3- Caput medusae at the umbilicus

How does Abatacept down regulate T cells?

1- When the antigen-presenting cell interacts with a T cell, MHC:antigen- TCR interaction signal 1 occurs 2- But the complex of B7 with abatacept prevents delivery of a costimulatory signal signal 2, and the T cell develops anergy or undergoes apoptosis 3- By this mechanism, abatacept therapy appears to be effective in down-regulating specific T-cell populations

What is the Ras/Rac-MAP kinase pathways initiated by? What does it lead to?

1- ZAP-70-dependent phosphorylation 2- Accumulation of adaptor proteins at the plasma membrane recruitment of Ras or Rac, and their activation by exchange of bound guanosine diphosphate (GDP) with GTP

What are some rare causes of secondary diarrhea?

1- Zollinger-Ellison syndrome (gastrinoma) 2- Vasoactive intestinal peptide-producing tumor (VIPoma) 3- Carcinoid syndrome.

What does Crohns disease presentation begin with in most patients?

1- intermittent attacks of relatively mild diarrhea, fever, and abdominal pain 2- Approximately 20% of patients present acutely with right lower quadrant pain, fever, and bloody diarrhea that may mimic acute appendicitis or bowel perforation.

What are the different causes that can lead to malabsorption of Vitamin B12?

1- lack of intrinsic factor (e.g., pernicious anemia, gastric resection) 2- Pancreatic insufficiency 3- Bacterial overgrowth 4- Ileal resection or mucosal disease (e.g., Crohn's disease).

How long does it take for T lymphocytes activated by antigen and costimulation to begin proliferating? What does it result in?

1-2 days expansion of antigen-specific clones This expansion quickly provides a large pool of antigen-specific lymphocytes from which effector cells can be generated to combat infection. The expansion of CD4+ T cells appears to be 100-fold to 1000-fold less than that of CD8+ cells.

What do mucosal biopsies reveal in Inflammatory bowel disease?

1-Acute and chronic inflammation with infiltration by plasma cells, neutrophils, lymphocytes, and eosinophils 2- Focal ulcerations 3- Crypt architectural distortion 4- Crypt abscesses

Describe the morphology of Colonic diverticula

1-Anatomically, colonic diverticula are small, flask-like outpouchings, usually 0.5 to 1 cm in diameter, that occur in a regular distribution alongside the taeniae coli 2-These are most common in the sigmoid colon, but more extensive areas may be affected in severe cases. 3- Colonic diverticula have a thin wall composed of a flattened or atrophic mucosa, compressed submucosa, and attenuated or, most often, totally absent muscularis propria 4- Obstruction of diverticula leads to inflammatory changes, producing diverticulitis and peridiverticulitis 5- With or without perforation, diverticulitis may cause segmental diverticular disease-associated colitis, fibrotic thickening in and around the colonic wall, or stricture formation 6- Perforation is uncommon but it can result in pericolonic abscesses, sinus tracts, and, occasionally, peritonitis.

What can disruptions to the homeostasis between the microbiota and the host immune system can lead to?

1-Increased inflammation 2-Decreased absorption.

What are tributaries of the Splenic vein?

1-Short gastric veins from the fundus and left part of the greater curvature of the stomach 2- The left gastro-omental vein from the greater curvature of the stomach 3- Pancreatic veins draining the body and tail of the pancreas 4- Usually the inferior mesenteric vein

What are some examples of clinical manifestations that occur in response to specific micronutrient deficiences?

1-iron deficiency anemia may be the only manifestation of celiac disease in some patients 2- Muscle wasting and edema result from protein malabsorption. 3- Nutritional anemia, caused by deficiencies of iron, folate, and vitamin B12, contributes to fatigue 4- Bleeding tendency (e.g., ecchymosis) may be attributed to prolonged prothrombin time resulting from vitamin K deficiency related to fat malabsorption

What is Etanercept approved for use in treating?

1-rheumatoid arthritis 2- Juvenile idiopathic arthritis 3-Plaque psoriasis 4- Psoriatic arthritis 5- Ankylosing spondylitis

What are important parts in the history a physician should take when trying to evaluate causes of Chronic diarrhea?

1. Character of the onset of diarrhea—sudden or gradual 2. Continuous versus intermittent symptoms 3. The presence of nocturnal diarrhea 4. Duration of diarrhea 5. Epidemiology—travel, exposure to contaminated food or water, family members with similar illness 6. Stool characteristics—watery, bloody, greasy 7. Fecal incontinence versus diarrhea or both 8. Abdominal pain—IBD, IBS, mesenteric vascular insufficiency 9. Weight loss—often significant in malabsorption, IBD, ischemia, and neoplasm 10. Aggravating factors—stress, specific foods (e.g., milk) 11. Prior evaluations to avoid repeating tests 12. Mitigating factors—what the patient has tried to control the diarrhea 13. Previous operations, radiation therapy, medications, supplements 14. Factitious diarrhea—always a consideration in eating disorders, malingering, or secondary gain 15. Review of systems—hyperthyroidism, scleroderma, tumor syndromes, diabetes mellitus 16. Risk factors for HIV and other immunosuppressed state

While the resuscitation is underway what information should be obtained to determine the the source of bleeding?

1. The nature of the bleeding: melena, hematemesis, hematochezia, or occult blood. A digital rectal examination is essential for determination of stool color and identification of anal fissures or rectal neoplasms. 2. The duration of GI bleeding, which helps dictate the appropriate pace of the evaluation to determine the bleeding source 3. The presence or absence of abdominal pain; for example, hematochezia caused by diverticula or angiodysplasia typically is painless, but hematochezia due to intestinal ischemia it is often accompanied by abdominal pain 4. Other associated symptoms, including fever, urgency or tenesmus, recent change in bowel habits, and weight loss 5. Current or recent medication use, particularly nonsteroidal anti-inflammatory drugs (NSAIDs), including aspirin, which may predispose to ulceration or gastritis (see Chapter 37), anticoagulants, and alcohol. Many over-the-counter products may contain aspirin or NSAIDs. 6. Relevant past medical and surgical history, including a history of prior GI bleeding, abdominal surgery (prior abdominal aorta repair should raise suspicion for an aortoenteric fistula), radiation therapy (radiation proctitis), major organ diseases (including cardiopulmonary, hepatic, or renal disease), inflammatory bowel diseases, and recent polypectomy (postpolypectomy bleeding).

What are the terminal branches of the abdominal aorta?

2 common iliac arteries

How many Lumbar splanchnic nerves usually pass from the lumbar part of the sympathetic trunk and enter the prevertebral plexus?

2-4

Steps in the activation of lymphocytes

3 Steps in the activation of T lymphocytes. Naive T cells recognize major histocompatibility complex (MHC)-associated peptide antigens displayed on antigen-presenting cells and other signals (not shown). The T cells respond by producing cytokines, such as interleukin-2 (IL-2), and expressing receptors for these cytokines, leading to an autocrine pathway of cell proliferation. The result is expansion of the clone of T cells that are specific for the antigen. Some of the progeny differentiate into effector cells, which serve various functions in cellmediated immunity, and memory cells, which survive for long periods. Other changes associated with activation, such as the expression of various surface molecules, are not shown. APC, Antigen-presenting cell; CTL, cytotoxic T lymphocyte; IL-2R, interleukin-2 receptor.

What does the drug 5-Aminosalicyclic acid consist of?

5-ASA linked to a sulfapyridine moiety; the 5-ASA is released after bacterial lysis of an azo bond in the colon.

What can be used to treat diarrhea-predominant IBS?

5-HT3 (Serotonin) receptor anatagonists Opioids and psychoactive drugs with anti-cholinergic effects are also commonly used to treat diarrhea- predominant IBS.

What occurs in the d-xylose test?

5-carbon monosaccharide that is transported across the intestinal mucosa largely by passive diffusion. In this test, the subject ingests 25 g of d-xylose, and urine is collected for the next 5 hours 1-Healthy subjects excrete more than 4.5 g of d-xylose in 5 hours (or ≥20% of the ingested load) 2- Excretion of a lower amount of d-xylose suggests abnormal absorption 3- However, an abnormally low (false-positive) result may occur in the presence of impaired renal excretory function, gastroparesis, massive peripheral edema, or ascites Abnormal results can also be seen in the presence of bacterial overgrowth as a result of bacterial degradation of d-xylose in the lumen, but this "pseudomalabsorption" may be corrected after treatment with antibiotics serving as a therapeutic trial.

What is Azathioprine a prodrug of?

6-mercaptopurine (6-MP) is the active form which is slowly released as AZA reacts nonenzymatically with sulhydryl compounds such as glutathione

What is the most accurate test for fat malabsorption?

72 hour fecal fat analysis however, the test is difficult to carry out in clinical practice Surrogate screening for steatorrhea is done with the qualitative stool fat examination (Sudan stain) and measurement of serum carotene

What is Natalizumab?

A Monoclonal antibody directed against alpha-4 integrin that inhibits immune-cell interactions with cells expressing VCAM-1 or MAdCAM-1.

Bowel obstruction

A bowel obstruction can be either functional or due to a true obstruction. Mechanical obstruction is caused by an intraluminal, mural, or extrinsic mass which can be secondary to a foreign body, obstructing tumor in the wall, or extrinsic compression from an adhesion, or embryological band A functional obstruction is usually due to an inability of the bowel to peristalse, which again has a number of causes, and most frequently is a postsurgical state due to excessive intraoperative bowel handling. Other causes may well include abnormality of electrolytes (e.g., sodium and potassium) rendering the bowel paralyzed until correction has occurred. The signs and symptoms of obstruction depend on the level at which the obstruction has occurred. The primary symptom is central abdominal, intermittent, colicky pain as the peristaltic waves try to overcome the obstruction. Abdominal distention will occur if it is a low obstruction (distal), allowing more proximal loops of bowel to fill with fluid. A high obstruction (in the proximal small bowel) may not produce abdominal distention. Vomiting and absolute constipation, including the inability to pass flatus, will ensue Early diagnosis is important because considerable fluid and electrolytes enter the bowel lumen and fail to be reabsorbed, which produces dehydration and electrolyte abnormalities. Furthermore, the bowel continues to distend, compromising the blood supply within the bowel wall, which may lead to ischemia and perforation. The symptoms and signs are variable and depend on the level of obstruction. Small bowel obstruction is typically caused by adhesions following previous surgery, and history should always be sought for any operations or abdominal interventions (e.g., previous appendectomy). Other causes include bowel passing into hernias (e.g., inguinal) and bowel twisting on its own mesentery (volvulus). Examination of hernial orifices is mandatory in patients with bowel obstruction Large bowel obstruction is commonly caused by a tumor. Other potential causes include hernias and inflammatory diverticular disease of the sigmoid colon The treatment is intravenous replacement of fluid and electrolytes, analgesia, and relief of obstruction. The passage of a nasogastric tube allows aspiration of fluid from the stomach. In many instances, small bowel obstruction, typically secondary to adhesions, will settle with nonoperative management. Large bowel obstruction may require an urgent operation to remove the obstructing lesion, or a temporary bypass procedure (e.g., defunctioning colostomy

What is the abdominal prevertebral plexus?

A collection of nerve fibers that surrounds the abdominal aorta and is continuous onto its major branches.

What is Functional abdominal pain syndrome?

A condition in which the pain has been present for months or years The complaints of pain often are not related to eating, defecation, or menses, unlike other causes of chronic pain

What is the enteric nervous system? What does it consist of?

A division of the visceral part of the nervous system and is a local neuronal circuit in the wall of the gastrointestinal tract.

Development of Memory T lymphocytes

A fraction of antigen-activated T lymphocytes differentiates into long-lived memory cells. These cells are a pool of lymphocytes that are induced by microbes and are waiting for the infection to return. We do not know what factors determine whether the progeny of antigenstimulated lymphocytes will differentiate into effector cells or memory cells. Memory cells have several important characteristics. • Memory cells survive even after the infection is eradicated and antigen is no longer present. Certain cytokines, including IL-7 and IL-15, which are produced by stromal cells in tissues, may serve to keep memory cells alive and cycling slowly. • Memory T cells may be rapidly induced to produce cytokines or kill infected cells on encountering the antigen that they recognize. These cells do not perform any effector functions until they encounter antigen, but once activated, they respond much more vigorously and rapidly than do naive lymphocytes. • Memory T cells can be found in lymphoid organs, in various peripheral tissues, especially mucosa and skin, and in the circulation. They can be distinguished from naive and effector cells by several criteria (see Chapter 1). A subset of memory T cells, called central memory cells, populate lymphoid organs and are responsible for rapid clonal expansion after reexposure to antigen. Another subset, called effector memory cells, localize in mucosal and other peripheral tissues and mediate rapid effector functions on reintroduction of antigen to these sites

What is IL-12? What is its function?

A heterodimer composed of p40 and p35 subunits directs the differentiation of naïve T cells into Th1 cells, which secrete IL-2, IFN- gamma and TNF-alpha

What is IL-23? What is its function?

A heterodimer that has the same p40 subunit covalently linked to a p19 subunit directs the differentiation of naïve T cells into Th17 cells, which secrete IL-17 and IL-22.

What is Belimumab?

A human monoclonal antibody directed against the B-lymphocyte stimulator (BLyS) cytokine

What is Eculizumab?

A humanized monoclonal antibody directed against C5 a complement protein that mediates late steps in complement activation and triggers assembly of the membrane attack complex

What is CTLA-4?

A key inhibitory receptor in T cells

What is Sweet's syndrome (acute febrile neutrophilic dermatosis)? What is it characterized by?

A less common dermatologic manifestation of IBD characterized by the sudden onset of fever, leukocytosis, and tender, erythematous, well-demarcated papules and plaques that show dense neutrophilic infiltrates on histologic examination.

What is Graft versus Host disease?

A major complication of allogeneic bone marrow or stem cell transplantation GVHD is an alloimmune inflammatory reaction that occurs when transplanted immune cells attack the cells of the recipient

etiology

A major early goal is to distinguish between upper and lower GI sources because the management strategies are different. In addition to the symptoms already described, certain aspects of the history and physical examination, the age of the patient, and results of laboratory studies may be suggestive. However, in many patients, the site of bleeding frequently remains uncertain after the initial evaluation.

What is the common hepatic artery? What are its 2 branches?

A medium-sized branch of the celiac trunk 1- Hepatic artery proper 2- Gastroduodenal artery

What is Linaclotide?

A new agent for chronic constipation and IBS with constipation (IBSC) This medication causes increased secretion of chloride and bicarbonate into the intestinal lumen via a cyclic guanosine monophosphate (cGMP) pathway. This pathway also may be responsible for relief of visceral pain in patients with IBS-C.

What are some diseases associated with Celiac disease?

A number of diseases, including dermatitis herpetiformis, type 1 diabetes mellitus, autoimmune thyroid disease, and selective immunoglobulin A (IgA) deficiency, are found in significant association with celiac disease.

What is Chronic inflammation?

A pathologic state characterized by the continued and inappropriate response of the immune system to an inflammatory stimulus Chronic inflammation accounts for the symptoms of many autoimmune diseases and may be an important cause of organ transplant rejection

What is Antithymocyte globulin (ATG)?

A preparation of antibodies induced by injecting rabbits or horses with human thymocytes The rabbit or horse antibodies are polyclonal and target many antigens on human T cells

What do CD4 and CD8 coreceptors facilitate signaling through?

A protein kinase called Lck is noncovalently attached to the cytoplasmic tails of these coreceptors Lck, is brought near the TCR complex by the CD4 or CD8 molecules

approach to the patient with acute GI bleeding- assessment of vital signs and resuscitation

A simple mnemonic for the approach to gastrointestinal bleeding is SET: Stabilization, Evaluation (endoscopy), and Treatment. The first goal in management is to stabilize the patient and determine the severity of blood loss (Fig. 33-2). Vital signs with postural changes should be recorded immediately. If the systolic blood pressure drops more than 10 mm Hg or the pulse increases more than 10 beats per minute as the patient changes position from supine to standing, it is likely the patient has lost at least 800 mL (15%) of circulating blood volume. Hypotension, tachycardia, tachypnea, and mental status changes in the setting of acute GI hemorrhage suggest the loss of at least 1500 mL (30%) of circulating blood volume The goals of resuscitation are to restore the normal circulatory volume and to prevent complications from red blood cell loss, such as cardiac, pulmonary, renal, or neurologic consequences. Initially, at least two large-bore intravenous catheters are used to administer isotonic solutions (e.g., lactated Ringer's solution, 0.9% NaCl), and blood products if indicated. If the patient is in shock, central venous access should be established. Although the amount of blood to be infused must be individually determined in each case, recent randomized trials and a retrospective review suggest that use of a lower hemoglobin threshold of 7 g/dL, rather than a more liberal level of 9 g/dL, results in improved mortality rates, lower total transfusion requirements, and lower rates of rebleeding in both peptic ulcer bleeding and variceal bleeding in patients in whom early endoscopy (<5 hours) is available. In view of the costs and potential risks of blood transfusion, it is not appropriate to simply transfuse until an arbitrary target hematocrit is achieved. If coagulation studies are abnormal, as is commonly observed in cirrhotic patients, freshfrozen plasma, platelets, or both may be required to control ongoing hemorrhage.

What is the net effect of Mucosal disease?

A smaller effective mucosal surface and a relative loss of mucosal absorption Selective defects in an otherwise normal intestine may result in specific entities such as lactase deficiency or abetalipoproteinemia.

What is Cyclosporine?

A specific inhibitor of T-cell-mediated immunity enabled widespread whole-organ transplantation CsA is a cyclic decapeptide isolated rom a soil fungus, Tolypocladium inlfatum

What does mutation in FOXP3 cause?

A systemic, multiorgan autoimmune disease, demonstrating the importance of FoxP3+ regulatory T cells for the maintenance of self-tolerance The human disease is known by the acronym IPEX, for immune dysregulation, polyendocrinopathy, enteropathy, X-linked syndrome.

Who is the patient more likely to be in Functional abdominal pain syndrome?

A woman who has undergone numerous examinations and diagnostic studies with negative findings and, in many cases, surgical operations without any relief. Depression may be the result rather than the cause of the pain.

Which of the following cell types presents the antigen to T cells in the lymph node and serves as an APC? A) Dendritic Cell B) Neutrophil C) Eosinophil D) Mast Cell E) Macrophages

A) Dendritic Cell

Amaya visits her PCP because a few days ago she noticed bright red blood in her stool. Over the past few months, she has been experiencing abdominal pain and diarrhea with urgency to defecate but claimed that it would "come and go". Based on an endoscopy and laboratory and histologic findings, Amaya's PCP diagnoses her with Ulcerative Colitis. Which of the following would most likely be involved? A) IL-5 B)IL-12 C) IL-2 D) INF-gamma E) TNF-alpha

A) IL-5

A 36-year-old female presents to her family physician with relapsing abdominal pain, bloating, and recurrent constipation. The patient says that she has been experiencing these symptoms for at least 3 days per month over the last 3 months and her discomfort improves after defecation. Her physical examination is normal and a fecal occult blood test came out negative. To rule out other diseases an endoscopy was ordered which revealed no abnormalities and there was no obvious histological pathology. Which disorder is most likely to be diagnosed? A) Irritable bowel syndrome B) Ulcerative colitis C) Crohn's disease D) Collagenous colitis E) Lymphocytic colitis

A) Irritable bowel syndrome

A 45 year-old man presents to the clinic for a routine examination. He is found to be overweight and his labs reveal abnormally high cholesterol levels. He is referred to a gastroenterologist for an abdominal angiography procedure. Results reveal arteriosclerosis of the inferior mesenteric artery which has become completely occluded. The patient elects to have surgery to repair the occlusion but is bewildered that he is still alive. The doctor explains that anastomoses within the abdominal cavity keep the blood circulating in these situations. Which of the following arteries is most likely to provide collateral circulation to supply the distal large bowel if the inferior mesenteric is occluded? A) Marginal B) Inferior Epigastric C) Inferior Pancreaticoduodenal D) Jejunal E) Superior Pancreaticoduodenal

A) Marginal

You are presented with a patient undergoing counseling for her recent diagnosis of Crohn's Disease. The patient was wondering if any other members of her family, or any future children, would also be at risk for developing the disease. You explain to her that while Crohn's has multifactorial inheritance there are some definitive genetic risk factors for developing disease. The most common defective protein in Crohn's results in abnormal intestinal immune responses to bacterial cell wall components. What gene is associated with this effect? A) NOD2 B)LRRK C) STAT3 D) BRCA1 E) VHL

A) NOD2

A patient with severe Crohn's Disease presents to her gastroenterologist in hopes to be placed on a new prescription because she doesn't feel like her current medication is working. The physician decides to place the patient on a biologic agent that is a humanized anti-a4-integrin antibody and blocks inflammatory cell migration and adhesion. She explains to the patient that this drug may develop a rare demyelinating disorder that is caused by the human JC. What drug is the physician planning on prescribing to the patient? A) Natalizumab B) Infliximab C) Cyclosporine D) Methotrexate E) Golimumab

A) Natalizumab

A 35 year old male presents to the gastroenterologist with right lower quadrant pain, progressive weight loss, and diarrhea. The physician suspects ileocecal Crohn's Disease and would like to run the most sensitive diagnostic test available. What test should the physician choose? A) Video capsule endoscopy B) Colonoscopy C) Perinuclear Anti-Neutrophil Cytoplasmic Antibody (pANCA) D) Antibodies to OmpC E) Traditional small bowel radiography

A) Video capsule endoscopy

Microscopic pathology of ulcerative colitis

A, Crypt abscess. B, Pseudopyloric metaplasia (bottom). C, Disease is limited to the mucosa. Compare

Uncommon cases of Colitis

A, Diversion colitis. Note the large lymphoid aggregates with germinal centers. B, Collagenous colitis with intraepithelial lymphocytes and a dense subepithelial collagen band. C, Lymphocytic colitis

Colitis-associated dysplasia

A, Dysplasia with extensive nuclear stratification and marked nuclear hyperchromasia. B, Cribriform glandular arrangement in high-grade dysplasia. C, Colectomy specimen with high-grade dysplasia on the surface and underlying invasive adenocarcinoma. A large cystic, neutrophil-filled space lined by invasive adenocarcinoma is apparent (arrow) beneath the muscularis mucosae. Also seen are small invasive glands (arrowhead).

Mechanisms by which microbes may promote autoimmunity

A, Normally, an encounter of mature T cells with self antigens presented by resting tissue antigen-presenting cells (APCs) results in peripheral tolerance. B, Microbes may activate the APCs to express costimulators, and when these APCs present self antigens, the specific T cells are activated, rather than being rendered tolerant. C, Some microbial antigens may cross-react with self antigens (mimicry). Therefore, immune responses initiated by the microbes may become directed at self cells and tissues. This figure illustrates concepts as they apply to T cells; molecular mimicry also may apply to self-reactive B lymphocytes

Roles of non-MHC genes in autoimmunity

A, Select examples of variants (polymorphisms) of genes that confer susceptibility to autoimmune diseases but individually have small or no effects. B, Examples of genes whose mutations result in autoimmunity. These are rare examples of autoimmune diseases with mendelian inheritance. The pattern of inheritance varies in the different diseases. APS-1 is autosomal recessive, and in most patients, both alleles of the gene (AIRE) have to be abnormal to cause the disease. IPEX is X-linked, so mutation in one allele of the gene (FOXP3) is sufficient to cause a defect in boys. ALPS is autosomal dominant with highly variable penetrance, because FAS and FASL are trimeric proteins and mutations in one of the alleles of either gene result in reduced expression of intact trimers. The disease caused by CTLA4 mutations is also autosomal dominant, perhaps because mutation in one allele reduces the expression of the protein enough to impair its function. AS, Ankylosing spondylitis; IBD, inflammatory bowel disease; MS, multiple sclerosis; PS, psoriasis; RA, rheumatoid arthritis; SLE, systemic lupus erythematosus; T1D, type 1 diabetes.

A patient presents with generalized, systemic autoimmunity to peripheral tissue antigens in multiple organ systems. The physician suspects Autoimmune Polyendocrine Syndrome in which several peripheral tissue antigens are not properly expressed in the thymus for negative selection, leading to defective elimination of self-reactive T cells. What gene must be nonfunctional for the prevalence of these self-reactive immature T cells released in peripheral tissue? A. AIRE B. FOXP3 C. FCGRIIB D. NOD2 E. RAG

A. AIRE

A 21-year-old female is out for her 21st birthday when she starts to get a pain centrally around her belly button. In the morning she wakes up with a fever, nausea and she's vomiting. She decides to wake up her parents because she is having a sharp, constant pain in her right hip region. They tell her she is hung over and to go back to bed. An hour later she calls herself an ambulance because she can no longer bear the pain. At the hospital an ultrasound is taken around the right lilac fossa region. She is immediately taken into surgery. What did she most likely present to the hospital with? A. Appendicitis B. Diverticular disease C. Bowel obstruction D. Esophageal cancer E. Carcinoma of the stomach

A. Appendicitis

A patient presents to the emergency room and is diagnosed with acute lymphoblastic leukemia after several rounds of testing. The physician would like the prescribe an antimetabolite, prodrug that releases 6-MP slowly as it reacts with sulfhydryl compounds, such as glutathione. Which of the following is the drug the physician would like to prescribe? A. Azathioprine B. Methotrexate C. Glucocorticoids D. Cyclophosphamide TNF-alpha inhibitor

A. Azathioprine

A 12-year-old male is brought in by their parents with a chief complaint of fever and joint pain. The patient has no relevant family history and recent medical history of a bacterial infection that subsided 3 weeks prior. The patient also complains of fatigue and sore throat. Physical exam confirms pain in multiple joints and reveals heart palpitations but no abnormal lung sounds. Believing this presentation to be an autoimmune disease, what auto-antigen is the cause of this condition? A. Cross reaction of cardiac muscle with bacterial agents B. Platelet GPIIb-IIIa C. Epidermal cadherin D. Rh blood group antigens E. Renal glomerular type 4 collagen

A. Cross reaction of cardiac muscle with bacterial agents

Calcineurin is an enzyme that plays an important role in activating the nuclear factor of activated T cells (NFAT). If a patient were prescribed a medicine that inhibited calcineurin, what would the drug prevent? A. Dephosphorylation of NFAT B. Phosphorylation of NFAT C. Phosphorylation of c-Jun D. Dephosphorylation of c-Jun E. None of the above

A. Dephosphorylation of NFAT

A 54-year-old female presents to her PCP with the complaint of dry, red, inflamed eyes. She states these symptoms have been occurring for the past few months with no relief. The physician diagnoses her with Keratoconjunctivitis sicca and prescribes her a topical immunosuppressant drug that inhibits the production of IL-2 by binding to cyclophilin. Which of the following is a unique side effect of this drug? A. Gingival hyperplasia B. Hepatotoxicity C. Alopecia D. Heart failure E. Diabetes mellitus

A. Gingival hyperplasia

A CD8+ T cell in the thymus reacts to self antigen. Assuming the body's defense mechanisms are working and autoimmunity does not occur, what will happen to this cell in this tissue? A. It will undergo Apoptosis B. It will undergo receptor editing C. It will develop into a regulatory T cell D. It will undergo anergy E. More than one of these is possible

A. It will undergo Apoptosis

A scientist is doing research regarding MHC proteins. She reads that this MHC protein displays protein fragments derived from endocytic vesicles. Which of the following correctly pairs this MHC protein with the cell surface protein it presents antigens to? A. MHC II, CD4 B. MHC II, CD8 C. MHC I, CD4 D. MHC I, CD8 E. MHC I, CD80

A. MHC II, CD4

A 22-year-old female decided to spend her days in quarantine constructing a new project for her apartment. While she walked over to pick up a box she stepped on a nail that pierced through her left foot. As a second year medical student she couldn't help but think of her immune system and what was occurring at the puncture site. Which of the following cells is NOT accurately paired with its function? A. Macrophage; controls adaptive immune responses B. Dendritic cell; transports and presents antigen to T cells in lymph nodes C. Neutrophil; phagocytoses and kills invading pathogens, especially bacteria D. Eosinophil; defends against parasites E. Basophil/mast cell; release histamine, leukotrienes and other mediators after exposure to antigen

A. Macrophage; controls adaptive immune responses

A patient arrives with a primary complaint or aching joints. After testing, the patient is diagnosed with Rheumatoid arthritis, the physician prescribes infliximab. What long term side effect should the patient be aware of before starting treatment? A. Myelosuppression and potential increased risk of demyelination diseases B. Rash and reactivation of HBV C. Hypertension and Nasopharyngitis D. Pericarditis and Endocrine disorder E. Prolonged QT interval and Diabetes Mellitus

A. Myelosuppression and potential increased risk of demyelination diseases

A 6-y.o boy was immediately brought to his pediatrician by his mother after he fell while running outside and got a cut on his hand. The mother is concerned about bacterial infection because the area around the cut is already red. The doctor explains that ________ are the most abundant cell type of the innate immune system and the "first responders" in inflammation, are phagocytic cells primarily responsible for defense against bacterial infection. A. Neutrophils B. Eosinophils C. Basophils D. Mast Cells E. NK Cells

A. Neutrophils

A 1-year-old boy is admitted to the hospital due to bloody stool. An endoscopic examination reveals ulceration of the intestinal wall. The physician suspects an anomaly of the small intestine that occurs in 2% of the population. Which of the following best describes the congenital disorder responsible for this condition? A. Persistence of the vitelline duct B. Incomplete rotation of the midgut C. Incomplete fixation of the small bowel mesentery D. Volvulus of the small bowel E. Peritoneal folds compressing the duodenum

A. Persistence of the vitelline duct

Which of the following inflammatory Bowel Disease is characterized by a discrete ulcer with a necrotic base, present usually on the leg? The ulcer may spread and become large and deep. A. Pyoderma gangrenous B. Erythema nodosum C. Sweet's Syndrome D. Uveitis E. Episcleritis

A. Pyoderma gangrenous

A 66-year-old patient has been referred to an endocrinologist after displaying elevated glucose levels on a fasting blood glucose test. His PCP is concerned that he may have developed new-onset diabetes after his transplant procedure. The patient was given a very potent immunosuppressant for his transplantation that causes the suppression of IL-2. Which of the following statements is true about the immunosuppressant? A. binds to FK-binding proteins (FKBP), inhibiting calcineurin-mediated NFAT dephosphorylation B. an ophthalmic preparation is approved for treatment of chronic dry eyes C. blocks the synthesis of uridylate (UMP) by inhibiting dihydroorotate dehydrogenase (DHOD) D. binds to cyclophilin, inhibiting calcineurin-mediated NFAT dephosphorylation E. is a chimeric, partially humanized monoclonal antibody

A. binds to FK-binding proteins (FKBP), inhibiting calcineurin-mediated NFAT dephosphorylation

A 40-year-old female who underwent a bone marrow transplant post aggressive chemotherapy treatment is experiencing diarrhea and a rash. What is occurring in this patient? A. transplanted immune cells are attacking the cells of the patient B. the patient's cells are undergoing an adjustment period to the transplanted bone marrow C. the patient is experiencing delayed side effects from the chemotherapy D. the patient's IgE antibodies are reacting to a foreign substance E. transplanted bone marrow is attacking the intestinal brush border

A. transplanted immune cells are attacking the cells of the patient

A 46 year old male presents to his primary care physician with the chief complaint of persistent greasy foul smelling stool. Suspecting intestinal malabsorption of fat, which diagnostic test would the physician most likely order? A.) Sudan staining B.) Schilling test C.) 14C-xylose test D.) Serum amylase E.) Scintigraphic erythrocyte scan

A.) Sudan staining

What antibody is seen in Crohn's disease but not in ulcerative colitis?

ASCA anti-Saccharomyces cerevisiae antibodies (ASCA) are common in Crohn's disease but rarely found in UC

abatacept

Abatacept consists o CTLA-4 used to an IgG1 constant region. Abatacept complexes with costimulatory B7 molecules on the sur ace o antigen-presenting cells. When the antigen-presenting cell interacts with a T cell, MHC:antigen- TCR interaction ("signal 1") occurs, but the complex o B7 with abatacept prevents delivery o a costimulatory signal ("signal 2"), and the T cell develops anergy or undergoes apoptosis. By this mechanism, abatacept therapy appears to be e ective in down-regulating specif c T-cell populations Abatacept is approved or the treatment o juvenile idiopathic arthritis and rheumatoid arthritis that is re ractory to methotrexate or TNF inhibitors. Clinically, abatacept signif cantly improves symptoms o rheumatoid arthritis in patients who ail to respond to methotrexate or TNF inhibitors The major adverse e ects o abatacept are exacerbations of preexisting chronic obstructive lung disease and increased susceptibility to infection . Abatacept should not be administered concurrently with TNF inhibitors because the combination carries an unacceptably high risk o infection

What will occur if a low obstruction (distal) occurs? What does it allow?

Abdominal distention allowing more proximal loops of bowel to fill with fluid A high obstruction (in the proximal small bowel) may not produce abdominal distention

What does the foregut include? What is it supplied by?

Abdominal esophagus, stomach, duodenum (superior to the major papilla), liver, pancreas and gallbladder, the spleen also develops in relation to the foregut region Celiac trunk

What can occur when the sigmoid colon becomes inflamed (diverticulitis)?

Abdominal pain and fever ensue

Definition and epid.

Abdominal pain is a frequent manifestation of intra-abdominal disease. However, abdominal pain is difficult to localize or grade because the sensation of pain often is colored by emotional and physical factors. Abdominal pain may be classified as acute or chronic. Acute pain occurs suddenly and more often suggests serious physiologic alterations. Chronic pain may be present for several months; although it does not mandate immediate attention, chronic pain may lead to prolonged evaluation. According to a recent report, abdominal pain is the most common symptom in patients presenting to gastroenterology clinics in the U.S., with almost 16 million estimated visits in 2009 alone. Appropriate evaluation of abdominal pain requires knowledge of pain mechanisms, close attention to history and physical examination findings, and recognition of important accompanying symptoms as well as awareness of the strengths and weaknesses of the tests that might be used.

physiology

Abdominal pain results from stimulation of receptors specific for thermal, mechanical, or chemical stimuli. Once these receptors are excited, pain impulses travel through sympathetic fibers. Abdominal pain can be characterized as somatic or visceral. Somatic pain originates from the abdominal wall and parietal peritoneum, whereas visceral pain originates in internal organs and from the visceral peritoneum. Two types of neurons carry pain: A fibers, which have rapid conduction, and C fibers, which have slow conduction. Most visceral neurons are of the C type, and the pain resulting from their stimulation tends to be variable with regard to sensation and localization. In contrast, both A and C fibers originate from the parietal peritoneum and abdominal wall, and somatic pain tends to be sharp and distinctly localized. Because of this pattern of innervation, abdominal viscera are not sensitive to cutting, tearing, burning, or crushing. However, visceral pain results from stretching of the walls of hollow organs or of the capsule of solid organs, as well as from inflammation or ischemia.

What should a physician assess in Acute abdomen?

Abdominal tenderness, rebound and guarding

innervation

Abdominal viscera are innervated by both extrinsic and intrinsic components of the nervous system: ■ Extrinsic innervation involves receiving motor impulses from, and sending sensory information to, the central nervous system. ■ Intrinsic innervation involves the regulation of digestive tract activities by a generally self-sufficient network of sensory and motor neurons (the enteric nervous system). Abdominal viscera receiving extrinsic innervation include the abdominal part of the gastrointestinal tract, the spleen, the pancreas, the gallbladder, and the liver. These viscera send sensory information back to the central nervous system through visceral afferent fibers and receive motor impulses from the central nervous system through visceral efferent fibers The visceral efferent fibers are part of the sympathetic and parasympathetic parts of the autonomic division of the peripheral nervous system Structural components serving as conduits for these afferent and efferent fibers include posterior and anterior roots of the spinal cord, respectively, spinal nerves, anterior rami, white and gray rami communicantes, the sympathetic trunks, splanchnic nerves carrying sympathetic fibers (thoracic, lumbar, and sacral), parasympathetic fibers (pelvic), the prevertebral plexus and related ganglia, and the vagus nerves [X]. The enteric nervous system consists of motor and sensory neurons in two interconnected plexuses in the walls of the gastrointestinal tract. These neurons control the coordinated contraction and relaxation of intestinal smooth muscle and regulate gastric secretion and blood flow

Angiogenesis can be a pathologic state when associated with what?

Abnormal blood vessel growth or tumor growth

How is specificity of the immune response achieved ?

According to the pattern of adhesion molecules expressed by the activated endothelium and by the various types of leukocytes eg: Neutrophils dominate the early inflammatory response, while monocytes predominate after 24 hours.

What does Sigmoid Diverticular disease generally refer to?

Acquired pseudodiverticular outpouchings of the colonic mucosa and submucosa Unlike true diverticula, such as Meckel diverticulum, they are not invested by all three layers of the colonic wall.

What does Methotrexate cause apoptosis of?

Activated CD4 and CD8 T cells

What is CD30 expressed on?

Activated CD4 and CD8 T cells is also highly expressed on Reed-Sternberg cells in Hodgkin's lymphoma and on anaplastic large cell lymphoma cell

Where are both CTLA-4 and PD-1 induced? what is their function?

Activated T cells terminate responses of these cells

Leflunomide

Activated lymphocytes both proli erate and synthesize large quantities o cytokines and other e ector molecules, and these processes require increased DNA and RNA synthesis. There ore, agents that reduce intracellular nucleotide pools have suppressive e ects on these activated cells. Lef unomide is an inhibitor o pyrimidine synthesis, specif cally blocking the synthesis o uridylate (UMP) by inhibiting dihydroorotate dehydrogenase (DHOD). DHOD is a key enzyme in the synthesis o UMP (Fig. 46-4), which is essential or the synthesis o all pyrimidines. (See Chapter 39 or a review of pyrimidine synthesis.) Experimentally, le unomide has been shown to be most e ective in reducing B-cell populations, and a signif cant e ect on T cells has also been observed. Le unomide is currently approved or use in rheumatoid arthritis. The drug has also shown signif cant e f cacy in the treatment o other immune diseases, including systemic lupus erythematosus and myasthenia gravis. Le unomide has antiviral activity against cytomegalovirus (CMV) and has been used to treat this in ection in cases o drug-resistant CMV and in transplant patients. Le unomide prolongs transplant gra t survival and limits GVHD in animal models. The most signif cant adverse e ects o le unomide are diarrhea and reversible alopecia. Le unomide undergoes signif cant enterohepatic circulation, resulting in a prolonged pharmacologic e ect. I le unomide must be removed quickly rom a patient's system, cholestyramine may be administered. By binding to bile acids, cholestyramine interrupts the enterohepatic circulation and causes a rapid "washout" o leflunomide.

What is most IL-1 generated by?

Activated mononuclear cells

What occurs in Cellular response?

Activation and clonal expansion of T cells that recognize a specific antigen Some T cells recognize infected cells and then lyse those cells using cytotoxic proteins called perforins and granzymes Cellular immune responses are there ore effective against many intracellular infectious agents such as viruses

What does Cytokine release syndrome result from?

Activation of T cells and release of T-cell cytokines before the antibody-coated T cells can be cleared by macrophages The cytokine release syndrome typically occurs after the first few doses of ATG therapy, and the syndrome dissipates as T cells are eliminated

What is PIP3 required for the activation of?

Activation of a number of targets, including a serine-threonine kinase called Akt, or protein kinase B, which has many roles, including stimulating expression of antiapoptotic proteins and thus promoting survival of antigen-stimulated T cells

What is the MOA of Tacrolimus?

Acts by binding to FK-binding proteins ( FKBP ), and the tacrolimus- FKBP complex inhibits calcineurin. Tacrolimus inhibits IL-3, IL-4, IFN-gamma, and TNF-alpha production in vitro it appears to inhibit cell-mediated immunity without suppressing B-cell or natural killer (NK) cell unction

Appendicits

Acute appendicitis is an abdominal emergency. It usually occurs when the appendix is obstructed by either a fecalith or enlargement of the lymphoid nodules. Within the obstructed appendix, bacteria proliferate and invade the appendix wall, which becomes damaged by pressure necrosis. In some instances, this may resolve spontaneously; in other cases, inflammatory change (Figs. 4.86 and 4.87) continues and perforation ensues, which may lead to localized or generalized peritonitis Most patients with acute appendicitis have localized tenderness in the right groin. Initially, the pain begins as a central, periumbilical, colicky type of pain, which tends to come and go. After 6 to 10 hours, the pain tends to localize in the right iliac fossa and becomes constant. Patients may develop a fever, nausea, and vomiting. The etiology of the pain for appendicitis is described in Case 1 of Chapter 1 on p. 48. The treatment for appendicitis is appendectomy

GI hemorrhage- acute Gastrointestinal hemorrhage

Acute gastrointestinal (GI) bleeding remains a common major medical problem despite recent advances in diagnosis and treatment. Bleeding occurs as a complication of many diverse disease processes, and adequate treatment depends on careful assessment and management that focuses on ensuring hemodynamic stability, determining blood loss, and identifying sources of bleeding. Although advances in medical and surgical intensive care, pharmacologic therapy, and the prompt deployment of endoscopic therapies have significantly decreased the rate of rebleeding, the overall mortality rate from acute bleeding episodes has remained essentially unchanged during the past half-century, at about 5% to 10%, owing to an aging population and an increased prevalence of serious concomitant illnesses.

What does an elevated serum amylase concentration (hyperamylasemia) indicate?

Acute pancreatitis although a perforated ulcer or mesenteric thrombosis can also cause hyperamylasemia

acute rejection

Acute rejection has cellular and humoral components. Acute cellular rejection is mediated by cytotoxic T cells and causes interstitial as well as vascular damage. This cellular response is most commonly seen in the initial months a ter transplantation. Immunosuppression o T cells is highly e ective at preventing or limiting activation o the recipient immune system by the transplanted organ, thereby preventing acute cellular rejection. In acute humoral rejection , recipient B cells become sensitized to donor antigens in the transplanted organ and produce antibodies against these alloantigens a ter a period o 7-10 days. The antibody response is typically directed against endothelial cells and is thus also known as acute vascular rejection . Like acute cellular rejection, acute humoral rejection can usually be prevented by immunosuppression o the recipient a ter transplantation. Even with immunosuppression, however, episodes o acute rejection can occur months or even years a ter transplantation. Transplant recipients experiencing acute rejection are usually asymptomatic, and symptoms o ever or malaise are usually nonspecif c.

humoral and cellular immunity

Adaptive immunity is generally divided into humoral immunity and cellular immunity . In the basic (simplif ed) model o the immune system, the primary cells mediating humoral immunity are B cells, and those mediating cellular immunity are T cells (Table 42-1). The humoral response involves the production o antibodies specif c or an antigen. Mature B cells are characterized by CD19 and CD20 expression, and they can di erentiate into plasma cells. Upon antigen stimulation, plasma cells secrete antibodies against extracellular in ectious agents such as bacteria. In contrast, the cellular response involves activation and clonal expansion o T cells that recognize a specif c antigen. Some T cells recognize in ected cells and then lyse those cells using cytotoxic proteins called perforins and granzymes . Cellular immune responses are there ore e ective against many intracellular in ectious agents such as viruses. In addition to their role in cellular immunity, T cells control the extent o immune responses. Each T cell evolves so that it is activated by only one specif c MHC:antigen complex. All T cells express an MHC:antigen-specif c T-cell receptor (TCR). T cells are divided into cytotoxic T cells (T C ) and helper T cells (T H ) based on the type o coreceptor expressed and the unction imparted by that coreceptor (Fig. 42-3). T C cells are the mediators o cellular adaptive immunity. These cells express the CD8 coreceptor, which recognizes a constant (i.e., antigen-independent) domain on MHC class I proteins. This coreceptor unction allows the antigen-speci c TCR on T C cells to bind a speci c class I MHC:antigen complex with su ciently high a nity that the T C cell is activated by the cell expressing the class I MHC:antigen complex. Speci c activation o the T C cell initiates a chain o events, including the secretion o membrane-penetrating per orins and apoptosis-inducing granzymes, which result in the death o the cell displaying the oreign antigen T H cells are primarily the regulators o adaptive immunity. T H cells are identi ed by their expression o the CD4 coreceptor, which recognizes an antigen-independent domain on MHC class II proteins. This coreceptor unction allows the antigen-speci c TCR on T H cells to bind a specific class II MHC:antigen complex with su ciently high a nity that the T H cell is activated by the antigen-presenting cell. In addition to initiating and strengthening the immune response, T H cells control the type o immune response by producing one or another set o cytokines. T H cells can be generally divided into T H 1, T H 2, and T H 17 subtypes based on the cytokines produced by the cells. T H 1 cells characteristically produce IFN- and IL-2, and these cytokines inf uence the development o cell-mediated immune responses o both CD8 T C cells and other CD4 T H cells. IL-2 interacts with CD25, a receptor expressed on activated T cells, to mediate the early steps in T cell activation. T H 2 cells characteristically produce IL-4, IL-5, and IL-10, and these cytokines enhance antibody production by B cells. The T H 2 cell subtype is more o ten associated with autoimmunity (see Chapter 46). T H 17 cells characteristically produce IL-17, IL-21, and IL-22. IL-17 iso orms recruit neutrophils and ampli y the immune response. T H 17 cells are generated rom CD4 cells when these cells are stimulated by IL-23. Drugs that block the maturation or growth o T H 17 cells are available or clinical use in the treatment o certain autoimmune diseases.

Methotrexate can act as an anti inflammatory agent by increasing the levels of what?

Adenosine Adenosine is a potent endogenous anti inflammatory mediator that inhibits neutrophil adhesion, phagocytosis, and superoxide generation

What strengthens the binding of T cells to APCs?

Adhesion molecules

What does the homing of effector T cells to the site of infection mainly depend on?

Adhesion molecules and chemokines. Therefore, any effector T cell present in the blood, regardless of antigen specificity, can enter the site of any infection.

role of adhesion molecules in T cell responses

Adhesion molecules on T cells recognize their ligands on APCs and stabilize the binding of the T cells to the APCs. Most TCRs bind the peptide-MHC complexes for which they are specific with low affinity. To induce a response, the binding of T cells to APCs must be stabilized for a sufficiently long period to achieve the necessary signaling threshold. This stabilization function is performed by adhesion molecules on the T cells that bind to ligands expressed on APCs. The most important of these adhesion molecules belong to the family of heterodimeric (two-chain) proteins called integrins. The major T cell integrin involved in binding to APCs is leukocyte function-associated antigen 1 (LFA1), whose ligand on APCs is called intercellular adhesion molecule 1 (ICAM-1). On resting naive T cells, which are cells that have not previously recognized and been activated by antigen, the LFA-1 integrin is in a lowaffinity state. Antigen recognition by a T cell increases the affinity of that cell's LFA-1. Therefore, once a T cell sees antigen, it increases the strength of its binding to the APC presenting that antigen, providing a positive feedback loop. Thus, integrin-mediated adhesion is critical for the ability of T cells to bind to APCs displaying microbial antigens. Integrins also play an important role in directing the migration of effector T cells and other leukocytes from the circulation to sites of infection. This process is described in Chapter 2 and later in this chapter.

At the onset of an inflammatory response the endothelial cells at the site of injury are activated to express what?

Adhesion molecules that bind specifi receptors expressed by leukocytes. eg: Intercellular adhesion molecules ( ICAMs ) and vascular cell adhesion molecules ( VCAMs ) expressed by the vascular endothelium bind integrins expressed on the cell surface of leukocytes This interaction causes the leukocytes, which normally roll along the surface of the endothelium by means of loose, transient binding interactions, to adhere tightly to the activated endothelium at the site of injury

What is Small bowel symptoms typically caused by?

Adhesions following previous surgery history should always be sought for any operations or abdominal interventions . Other causes include bowel passing into hernias (e.g., inguinal) and bowel twisting on its own mesentery (volvulus). Examination of hernial orifices is mandatory in patients with bowel obstruction (

What can abrupt cessation of glucocorticoid therapy result in?

Adreanl insufficiency because the hypothalamus and pituitary gland require weeks to months to reestablish adequate ACTH production During this time, the underlying disease can worsen because of disinhibition of the immune system.

What was Ipilimumab approved for use in?

Advanced melanoma that is unresectable or metastatic Patients taking ipilimumab should be monitored or possible immunerelated hepatotoxicity and endocrinopathies.

transport phase

After absorption, nutrients leave the cells through venous or lymphatic channels. Consequently, malabsorption may be associated with mesenteric venous obstruction, lymphangiectasia, or lymphatic obstruction due to malignancy or infiltrative processes such as Whipple's disease. The absorptive process can be impaired at many stages. For example, patients with subtotal gastrectomy or bariatric surgery often experience malabsorption. There are resultant defects at all phases: impaired gastric churning, premature emptying, and impaired mixing (in the jejunum) of food with bile and pancreatic enzymes. The impaired mixing is a consequence of anatomic changes (gastrojejunostomy bypassing the duodenum) and reduced production of pancreatic enzymes (because cholecystokinin and secretin release is blunted when gastric contents bypass the duodenum). Moreover, stasis may lead to bacterial overgrowth in the afferent loop with changes in the bile acids needed for fat absorption. Another example of manifold mechanisms is diabetes mellitus, which may lead to delayed gastric emptying, abnormal intestinal motility, bacterial overgrowth, and pancreatic exocrine insufficiency.

What is the likely source of Hematemesis

After exclusion of swallowed blood from the nasopharynx or the respiratory tract (hemoptysis), the source of bleeding is likely to be proximal to the ligament of Treitz.

Development of cells in the immune system

All hematopoietic cells develop rom the pluripotent hematopoietic stem cell. This cell gives rise to the lymphoid stem cell and the trilineage myeloid stem cell. The lymphoid stem cell and its progenitor cells (not shown) give rise to mature lymphocytes (B cells and T cells), the cells that mediate adaptive immune responses. When exposed to speci c antigens, B cells di erentiate into antibody-producing plasma cells, and T cells adopt an activated phenotype. The myeloid stem cell and its progenitor cells, including megakaryocytes, erythroblasts, and myeloid precursors (not shown), proli erate and di erentiate into mature neutrophils, eosinophils, basophils, mast cells, monocytes, platelets, and erythrocytes. In the tissues, monocytes di erentiate into macrophages or dendritic cells, and mast cell precursors di erentiate into mast cells. (See Fig. 45-1 or more details about the di erentiation o cell lineages in the bone marrow.)

What do the older antimetabolites eg: Azathioprine and Methotrexate affect?

All rapidly dividing cells can have toxic effects on the gastrointestinal mucosa and bone marrow

What do T cell receptors expressed on all CD4 and CD8 T cells consist of?

Alpha chain and Beta chain both participate in antigen recognition

Natalizumab

Alpha-4 integrins are critical to immune-cell adhesion and homing. The 4 1 integrin mediates immune-cell interactions with cells expressing vascular cell adhesion molecule 1 (VCAM-1), while the 4 7 integrin mediates immune-cell binding to cells expressing mucosal addressin cell adhesion molecule 1 (MAdCAM-1). Natalizumab is a monoclonal antibody directed against 4 integrin that inhibits immune-cell interactions with cells expressing VCAM-1 or MAdCAM-1. Natalizumab was approved or the treatment o relapsing multiple sclerosis. During postmarketing surveillance o the drug, however, several patients treated with natalizumab developed progressive multi ocal leukoencephalopathy (PML), a rare demyelinating disorder caused by in ection with JC virus. This f nding resulted in voluntary withdrawal o the drug. A ter urther FDA investigation, it was decided to resume testing o natalizumab and to add a warning to the product label regarding the possible association. Natalizumab was subsequently reapproved or use in the treatment o multiple sclerosis and Crohn's disease

Extraintestinal manifestations

Although both UC and Crohn's disease primarily involve the bowel, they are also associated with inflammatory manifestations in other organ systems. This reflects the systemic nature of these disorders (Table 37-1). Extraintestinal manifestations can occur in parallel or independently of colonic activity, and they can become more difficult to treat than the bowel disease itself. The most common extraintestinal manifestation is arthritis, which is seen in about 9% to 50% of patients and is divided into two major types. The first is a peripheral, large-joint, asymmetrical, seronegative, oligoarticular, nondeforming arthritis that may involve the knees, hips, wrists, elbows, and ankles. This peripheral arthropathy usually parallels the course of the large bowel disease and typically lasts for only a few weeks. The second type of IBDrelated arthritis is axial in location, consisting of sacroiliitis or ankylosing spondylitis, and does not parallel the activity level of the bowel disease. Ankylosing spondylitis occurs in 5% to 10% of IBD patients and manifests with low back pain and stiffness that is usually worse during the night, in the morn ing, or after inactivity. Sacroiliitis alone (without ankylosing spondylitis) is common in IBD (up to 20% of patients), but in many cases is asymptomatic. Liver complications of IBD include both intrahepatic and biliary tract diseases. Intrahepatic diseases include fatty liver, pericholangitis, and chronic active hepatitis. Pericholangitis, also known as small-duct sclerosing cholangitis, is the most common of these diseases. It usually is asymptomatic, identified only by abnormalities in alkaline phosphatase and γ-glutamyl transpeptidase on laboratory tests and histologically by portal tract inflammation and bile ductule degeneration. Small-duct sclerosing cholangitis may progress to cirrhosis Biliary tract disease includes an increased incidence of gallstones and of primary sclerosing cholangitis (PSC). PSC is a chronic cholestatic liver disease marked by fibrosis of the intrahepatic and extrahepatic bile ducts; it occurs1% to 4% of patients with UC and less often in those with Crohn's disease. Overall, about 70% of patients with PSC have UC. Fibrosis leads to strictures of the bile ducts, which in turn may lead to recurrent cholangitis (with fever, right upper quadrant pain, and jaundice) and progression to cirrhosis. In addition, about 10% of patients develop cholangiocarcinoma. Medical or surgical therapy for the IBD does not modify the course of PSC, and most patients progress to cirrhosis and may require liver transplantation. The two classic dermatologic manifestations of IBD are pyoderma gangrenosum and erythema nodosum. Pyoderma gangrenosum occurs in about 5% of patients and is characterized by a discrete ulcer with a necrotic base, usually on the legs. The ulcer may spread and become large and deep, destroying soft tissues. Pyoderma parallels the bowel activity in 50% of cases. Treatment is usually with systemic or intralesional steroids, or both. Other treatment options include dapsone, cyclosporine, and the anti-TNF agents. Erythema nodosum occurs in 10% of IBD patients, usually with peripheral arthropathy, and produces raised, tender nodules, usually over the anterior surface of the tibia. Erythema nodosum responds to treatment for the underlying bowel disease. A less common dermatologic manifestation of IBD is Sweet's syndrome, or acute febrile neutrophilic dermatosis. This condition is characterized by the sudden onset of fever, leukocytosis, and tender, erythematous, well-demarcated papules and plaques that show dense neutrophilic infiltrates on histologic examination. Ocular manifestations of IBD, including uveitis and episcleritis, occur in 1% to 5% of patients. Uveitis (or iritis) is an inflammatory lesion of the anterior chamber that produces blurred vision, photophobia, headache, and conjunctival injection. Local therapy includes steroids and atropine. Episcleritis produces burning eyes and scleral injection without vision deficits and is treated with topical steroids.

IBD- pathogenesis

Although precise causes are not yet defined most investigators believe that IBD results from the combined effects of alterations in host interactions with intestinal microbiota, intestinal epithelial dysfunction, aberrant mucosal immune responses, and altered composition of the gut microbiome. This view is supported by epidemiologic, genetic, and clinical studies as well as data from laboratory models of IBD Genetics . There is compelling evidence that genetic factors contribute to IBD. Risk of disease is increased when there is an affected family member and, in Crohn disease, the concordance rate for monozygotic twins approaches 50%. Genetic factors may also contribute to phenotypic expression of the disease, because twins affected by Crohn disease tend to present within a few years of each other and develop disease in similar regions of the GI tract. The concordance of monozygotic twins for ulcerative colitis is only about 15%, suggesting that genetic factors are less dominant than in Crohn disease. Concordance for dizygotic twins is less than 10% for both forms of IBD. Population based genome wide association studies have identified over 160 IBD-associated genes. Most of these are shared between Crohn disease and ulcerative colitis, as well as other complex immune-mediated diseases. Interestingly, several IBD associated genes overlap with genes involved in responses to mycobacteria, including Mycobacterium tuberculosis and Mycobacterium leprae. This supports the idea that host-microbial interactions are critical to the pathogenesis of IBD and may explain some overlap in the histopathology of Crohn disease and mycobacterial infection. One of genes most strongly associated with Crohn disease is NOD2 (nucleotide oligomerization binding domain 2), which encodes an intracellular protein that binds to bacterial peptidoglycans and activates signaling events, including the NF-κB pathway. Despite the increase in risk attributable to NOD2 polymorphisms, it should be remembered that fewer than 10% of individuals carrying risk associated NOD2 variants develop disease. Thus, as is the case with all IBD-associated genes, any one gene confers only a small increase in the risk of developing these diseases. In addition to NOD2, two Crohn disease-related genes of particular interest are ATG16L1 (autophagyrelated 16-like), and IRGM (immunity-related GTPase M). Both are part of the autophagy pathways that are critical for cellular responses to intracellular bacteria; ATG16L1 may also regulate epithelial homeostasis NOD2, ATG16L1, and IRGM are expressed in multiple cell types, and their precise roles in the pathogenesis of Crohn disease have yet to be defined. However, all cellular pathogens, supporting the hypothesis that inappropriate immune reactions to luminal bacteria are an important component of IBD pathogenesis. • Mucosal immune responses. Several observations support a role for mucosal immune responses in the pathogenesis of IBD. Some of these are: • T helper cells are activated in Crohn disease and the response is polarized to the TH1 type (see Chapter 6) • TH17 T cells most likely contribute to disease pathogenesis. Consistent with this, certain polymorphisms of the IL-23 receptor, which is involved in the development and maintenance of TH17 cells, confer marked reductions in the risk of both Crohn disease and ulcerative colitis. • Many other pro-inflammatory cytokines, including TNF, interferon-γ and IL-13, as well as immunoregulatory molecules such as IL-10 and TGF-β, appear to be play a role the pathogenesis of IBD. The role of IL-10 is supported by the observations that autosomal recessive mutations of the IL-10 and IL-10 receptor genes are linked to severe, early onset IBD. Overall, while details remain to be defined, it is clear that deranged mucosal immune activation and defective immunoregulation contribute to the development of ulcerative colitis and Crohn disease. Immunosuppresive agents remain the mainstay of treatment for these conditions.

Ileal conduit

An ileal conduit is an extraanatomical procedure and is performed after resection of the bladder for tumor. In this situation a short segment of small bowel is identified. The bowel is divided twice to produce a 20-cm segment of small bowel on its own mesentery. This isolated segment of bowel is used as a conduit. The remaining bowel is joined together. The proximal end is anastomosed to the ureters, and the distal end is anastomosed to the anterior abdominal wall. Hence, urine passes from the kidneys into the ureters and through the short segment of small bowel to the anterior abdominal wall. When patients have either an ileostomy, colostomy, or ileal conduit it is necessary for them to fix a collecting bag onto the anterior abdominal wall. Contrary to one's initial thoughts these bags are tolerated extremely well by most patients and allow patients to live a nearly normal and healthy life.

Ileostomy

An ileostomy is performed when small bowel contents need to be diverted from the distal bowel. An ileostomy is often performed to protect a distal surgical anastomosis, such as in the colon to allow healing after surgery.

What is Autoimmunity?

An immune response against self (autologous) antigens Different autoimmune diseases may be organ-specific, affecting only one or a few organs, or systemic, with widespread tissue injury and clinical manifestations

What is the current belief of what IBD results from?

An inappropriate, overactive mucosal immune response to commensal intestinal bacteria or the microbiome in genetically susceptible individuals

What does binding of TLR4 to their ligands acitvate?

An intracellular signaling cascade that converges on the expression of proinflammatory cytokines leading to further immune cell recruitment and activation of the inflammatory response

What does the recognition of antigen and costimulators by T cells initiate?

An orchestrated set of responses that culminate in the expansion of the antigen-specific clones of lymphocytes and the differentiation of the naive T cells into effector cells and memory

Colonic diverticula morphology

Anatomically, colonic diverticula are small, flask-like outpouchings, usually 0.5 to 1 cm in diameter, that occur in a regular distribution alongside the taeniae coli (Fig. 17-40A). These are most common in the sigmoid colon, but more extensive areas may be affected in severe cases. Because diverticula are compressible, easily emptied of fecal contents, and often surrounded by the fat-containing epiploic appendices on the surface of the colon, they may be missed on casual inspection. Colonic diverticula have a thin wall composed of a flattened or atrophic mucosa, compressed submucosa, and attenuated or, most often, totally absent muscularis propria (Fig. 17-40B, C). Hypertrophy of the circular layer of the muscularis propria in the affected bowel segment is common. Obstruction of diverticula leads to inflammatory changes, producing diverticulitis and peridiverticulitis. Because the wall of the diverticulum is supported only by the muscularis mucosae and a thin layer of subserosal adipose tissue, inflammation and increased pressure within an obstructed diverticulum can lead to perforation. With or without perforation, diverticulitis may cause segmental diverticular disease-associated colitis, fibrotic thickening in and around the colonic wall, or stricture formation. Perforation is uncommon but it can result in pericolonic abscesses, sinus tracts, and, occasionally, peritonitis.

What may occur if an antigen is presented without a coincident costimulatory signal?

Anergy whereby a cell becomes unreactive and will not respond to further antigenic stimuli

anergy

Anergy in T cells refers to long-lived functional unresponsiveness that is induced when these cells recognize self antigens (Fig. 9-4). Self antigens are normally displayed with low levels of costimulators, as discussed earlier. Antigen recognition without adequate costimulation is thought to be the basis of anergy induction, by mechanisms that are described below. Anergic cells survive but are incapable of responding to the antigen. The two best-defined mechanisms responsible for the induction of anergy are abnormal signaling by the TCR complex and the delivery of inhibitory signals from receptors other than the TCR complex. • When T cells recognize antigens without costimulation, the TCR complex may lose its ability to transmit activating signals. In some cases, this is related to the activation of enzymes (ubiquitin ligases) that modify signaling proteins and target them for intracellular destruction by proteases. • On recognition of self antigens, T cells also may preferentially engage one of the inhibitory receptors of the CD28 family, cytotoxic T lymphocyte-associated antigen 4 (CTLA-4, or CD152) or programmed death protein 1 (PD-1), which were introduced in Chapter 5. Anergic T cells may express higher levels of these inhibitory receptors, which will inhibit responses to subsequent antigen recognition. The functions and mechanisms of action of these receptors are described in more detail below. Although several experimental animal models support the importance of T cell anergy in the maintenance of self-tolerance, it is still not clear if anergic T cells specific for self antigens are present in most healthy people and if their loss is associated with the development of autoimmunity. Forced expression of high levels of B7 costimulators in a tissue in a mouse, using transgenic technology, results in autoimmune reactions against antigens in that tissue. Thus, artificially providing second signals may break anergy and activate autoreactive T cells.

What is the Celiac trunk? What does it arise from?

Anterior branch of the abdominal aorta supplying the foregut arises from the Abdominal aorta immediately below the aortic hiatus of the diaphragm, anterior to the upper part of vertebra LI

What is the Inferior mesenteric artery (IMA) what does it supply?

Anterior branch of the abdominal aorta that supplies the hindgut

Where do Pelvic splanchnic nerves originate from? What does it innervate?

Anterior rami of S2 to S4. Preganglionic parasympathetic fibers originating in the sacral spinal cord pass from the S2 to S4 spinal nerves to the inferior hypogastric plexus Once in this plexus, some of these fibers pass upward, enter the abdominal prevertebral plexus, and distribute with the arteries supplying the hindgut. . This provides the pathway for innervation of the distal one-third of the transverse colon, the descending colon, and the sigmoid colon by preganglionic parasympathetic fibers.

What is the cause of tissue injury in autoimmune diseases?

Antibodies against self antigens or by T cells reactive with self antigens

What does the Humoral immune response involve the production of?

Antibodies specific for an antigen Mature B cells can differentiate into plasma cells Upon antigen stimulation, plasma cells secrete antibodies against extracellular infectious agents such as bacteria

What is hyperacute rejection most commonly mediated by?

Antibodies that react with blood group antigens in donor organs eg: Type AB donor in a Type O recipient

What can be used as adjuncts for management of diarrhea in patients with IBD?

Antidiarrheal agents and bile salt resin binders antidiarrheal agents should be used cautiously during exacerbations of colitis because they can precipitate toxic megacolon

Signal transduction pathways in T lymphocytes

Antigen recognition by T cells induces early signaling events, which include tyrosine phosphorylation of molecules of the T cell receptor (TCR) complex and the recruitment of adaptor proteins to the site of T cell antigen recognition. These early events lead to the activation of several biochemical intermediates, which in turn activate transcription factors that stimulate transcription of genes whose products mediate the responses of the T cells. The possible effects of costimulation on these signaling pathways are not shown. These signaling pathways are illustrated as independent of one another, for simplicity, but may be interconnected in more complex networks. AP-1, Activating protein 1; APC, antigen-presenting cell; GTP/GDP, guanosine triphosphate/diphosphate; ITAM, immunoreceptor tyrosine-based activation motif; mTOR, mammalian target of rapamycin; NFAT, nuclear factor of activated T cells; PKC, protein kinase C; PLCγ1, γ1 isoform of phosphatidylinositol-specific phospholipase C; PI-3, phosphatidylinositol-3; ZAP-70, zetaassociated protein of 70 kD.

What occurs in anergy?

Antigen recognition without adequate costimulation results in T cell anergy or death or makes T cells sensitive to suppression by regulatory T cells

What do cytokines stimulate the proliferation of? What does it result in?

Antigen-activated T cells increase in the number of antigen-specific lymphocytes, a process called clonal expansion

antigen presenting cells

Antigen-presenting cells (APCs) process the macromolecules (especially proteins) o an invading agent to display the processed ragments on the sur ace o the APC. In this orm, the ragments serve as molecular ngerprints used by cells o the adaptive immune system to recognize the invading agent. APCs are important initiators o immune responses because, in addition to displaying nonsel antigens to T cells (see below), they provide the costimulatory signals that are necessary or T-cell activation. The concept o costimulation , in which two separate signals are required to initiate an immune response to a stimulus, is discussed below. Monocytes that exit the bloodstream and take up residence in the tissues can di erentiate into macrophages . As "pro essional APCs," macrophages process and present antigenic ragments o an invading pathogen or recognition by T cells. The ability o macrophages to envelop and destroy pathogens is enhanced by other components o the immune system, including antibodies and complement (which mediate opsonization) and cytokines (which enhance killing ability). In addition, macrophages produce cytokines such as TNF- that modi y immune responses. Dendritic cells are the most important APCs or the initiation o adaptive immune responses. In the nonlymphoid tissues, dendritic cells engul and process oreign antigens. Dendritic cells then migrate to lymphoid tissues, where they present these cognate antigens to T cells via speci c molecular interactions

Antibody:Antineutrophil cytoplasmic antibodies (ANCA) Antigen/Tissue: Indication:

Antigen/Tissue: Indication: Autoimmune vasculitis,

Antibody:Anti-dsDNA antibodies Antigen/Tissue: Indication:

Antigen/Tissue: Antinuclear antibodies (ANA), specific Indication: Systemic Lupus Erythematosus

Antibody: anti-La/SSB antibodies Antigen/Tissue: Indication:

Antigen/Tissue: Antinuclear antibodies (ANA), specific Indication: Systemic Lupus Erythmatosus, Sjogren's Syndrome, Subacute cutaneous lupus erythmatosus and neonatal lupus erythematosus

Antibody:anti- Ro/SSA antibodies Antigen/Tissue: Indication:

Antigen/Tissue: Antinuclear antibodies (ANA), specific Indication: Systemic Lupus Erythmatosus, Sjogren's Syndrome, Subacute cutaneous lupus erythmatosus and neonatal lupus erythematosus

Antibody:Anti-CCCP Anti-cyclic citrullinated peptide Antigen/Tissue: Indication:

Antigen/Tissue: Citrulline is produced as part of arginine metabolism. In joints with RA this may occur at a higher rate; citrulline can change protein structure and trigger an immune response, producing antibodies against joint proteins Indication: Rheumatoid arthritis

Antibody:RF rheumatoid factor Antigen/Tissue: Indication:

Antigen/Tissue: RF is antibody against the Fc portion of IgG and different RFs recognize different parts of the IgG-Fc. precipitate as immune complexes Indication: Rheumatoid arthritis

Antibody: TSHR antibodies Antigen/Tissue: Indication:

Antigen/Tissue: TSH receptor on Thyroid, Hyperthyroid Indication: Graves Disease, autoimmune thyroid disease

Antibody:AChR Antigen/Tissue: Indication:

Antigen/Tissue: acetylcholine receptor AChR, decrease in acetylcholine, NMJ Indication: Myasthenia gravis (MG)

Antibody:Antinuclear antibodies (ANA) Antigen/Tissue: Indication:

Antigen/Tissue: bind to contents of the cell Nucleus (below are types of antinuclear antibodies) Indication: Not specific (lupus ?)

Antibody: P/Q VGCC Antigen/Tissue: Indication:

Antigen/Tissue: calcium channels, decrease in acetylcholine, NMJ Indication: Lambert Eaton Myasthenic Syndrome (LEMs)

Antibody: Anti-thyroid peroxidase antibodies Antigen/Tissue: Indication:

Antigen/Tissue: enzyme found in the thyroid gland important with production of thyroid hormone, Thyroid, Hyperthyroid Indication: Hashimoto's disease, Graves' Disease

Antibody:Antineutrophil cytoplasmic antibodies (ANCA)- cANCA: cytoplasmic Antigen/Tissue: Indication:

Antigen/Tissue: neutrophils, specifically binds proteinase 3 (PR3) or myeloperoxidase (MPO) in the neutrophil cytoplasm Indication: Granulomatosis with polyangitis (GPA),

Antibody:Antineutrophil cytoplasmic antibodies (ANCA)- pANCA: perinuclear Antigen/Tissue: Indication:

Antigen/Tissue: neutrophils, specifically binds proteinase 3 (PR3) or myeloperoxidase (MPO) in the neutrophil cytoplasm Indication: Microscopic polyangitis (MPA), Eosinophilic granulomatosis with polyangitis (EGPA), Ulcerative colitis

Antibody: Anti-transglutaminase antibodies Antigen/Tissue: Indication:

Antigen/Tissue: transglutaminase enzyme is a cellular adhesion molecule found in both intracellular and extracellular spaces; in heart, liver, and small intestine Indication: Celiac disease, juvenile diabetes, inflammatory bowel disease and various forms of arthritis

Infections can injury tissues and release what?

Antigens that normally are sequestered from the immune system eg: Some sequestered antigens (e.g., in testis and eye) normally are not seen by the immune system and are ignored. Release of these antigens (e.g., by trauma or infection) may initiate an autoimmune reaction against the tissue.

antimetabolites

Antimetabolites have been a mainstay o immunosuppressive treatment or many years. Their power ul suppressive e ect on immune cells is accompanied by many adverse e ects related to their lack o selectivity. The older antimetabolites, such as azathioprine and methotrexate, a ect all rapidly dividing cells and can have toxic e ects on the gastrointestinal mucosa and bone marrow. Newer antimetabolites, such as mycophenolate mo etil and lef unomide , cause ewer adverse e ects. Mycophenolate mo etil may also be relatively selective or immune cells, urther reducing its toxicity. Antimetabolites typically a ect both cell-mediated and humoral immunity, rendering patients more susceptible to in ection than would occur i only one o these immune systems were a ected Antimetabolites are widely used in the treatment o cancer, and their mechanisms o action are described in detail in Chapter 39, Pharmacology o Cancer: Genome Synthesis, Stability, and Maintenance. Here, we ocus on the antimetabolites used or immunosuppression and brie y discuss the anti-in ammatory aspects o their mechanisms o action

polyclonal antibodies- antithymocyte globulin

Antithymocyte globulin (ATG) is a preparation o antibodies induced by injecting rabbits or horses with human thymocytes. The rabbit or horse antibodies are polyclonal and target many antigens on human T cells. Because ATG targets essentially all T cells and leads to pro ound lymphocyte depletion, ATG treatment results in broad immunosuppression that can predispose to in ection. ATG is approved or use in prevention or treatment o renal transplant rejection, and the equine-derived material is also approved or treatment o aplastic anemia. ATG is administered intravenously once daily or up to 28 days ATG therapy is o ten complicated by ever and headache as prominent mani estations o the cytokine release syndrome . This syndrome, common to many antibody drugs that target lymphocytes, results rom activation o T cells and release o T-cell cytokines be ore the antibody-coated T cells can be cleared by macrophages. The cytokine release syndrome typically occurs a ter the f rst ew doses o ATG therapy, and the syndrome dissipates as T cells are eliminated. However, administration o successive ATG doses can also be complicated by the development o antibodies against rabbit- or horsespecif c epitopes on the administered immunoglobulins. ATG is generally co-administered with glucocorticoids, an antihistamine, and an antipyretic to mitigate in usion reactions. Note that Mr. W received medications in these three classes when starting induction ATG therapy a ter kidney transplant

What is the diagnosis of Mesenteric angina determined?

Aortic angiography the stenoses of the celiac trunk and superior mesenteric artery are best appreciated in the lateral view

What is the earliest lesion in Crohns disease? What can it progress to?

Aphthous ulcer may progress, and multiple lesions often coalesce into elongated, serpentine ulcers oriented along the axis of the bowel

What is the treatment for appendicitis?

Appendectomy

Key Abdominal pain syndromes

Appendicitis Crampy, steady Periumbilical, RLQ (right lower quadrant) Back Cholecystitis Intermittent, steady Epigastric, RUQ Right scapula Pancreatitis Steady Epigastric, periumbilical Back Perforation Sudden, severe Epigastric Entire abdomen Obstruction Crampy Periumbilical Back Infarction Severe, diffuse Periumbilical Entire abdomen CHRONIC ABDOMINAL PAIN Esophagitis Burning Retrosternal Left arm, back Peptic ulcer Gnawing Epigastric Back Dyspepsia Bloating, dull Epigastric None IBS Crampy LLQ, RLQ Non

What does the mesoappendix contain?

Appendicular vessels

Approach to the patient with suspected malabsorption

Approach to the patient with suspected malabsorption. CT, Computed tomography; ERCP, endoscopic retrograde cholangiopancreatography; GI, gastrointestinal. (Modified from Riley SA, Marsh MN: Maldigestion and malabsorption. In Feldman M, Scharschmidt BF, Sleisenger MH, editors: Sleisenger and Fordtran's gastrointestinal and liver disease: pathophysiology/diagnosis/management, ed 6, Philadelphia, 1998, WB Saunders, pp 1501-1522.)

depletion of specific immune cells

Appropriately targeted antibodies deplete the immune system o reactive cells and thereby provide e ective therapy or autoimmune diseases and transplant rejection. When the adaptive immune system reacts to an antigen, the resulting immunologic response includes the clonal expansion o cells specif cally reactive against that antigen. Treatment with exogenous antibodies directed against cell sur ace molecules that are expressed selectively on reactive immune cells can pre erentially deplete the immune system o these reactive cells. Antibodies that target cell sur ace receptors expressed selectively on malignant cells o immune origin are discussed in Chapter 40, Pharmacology o Cancer: Signal Transduction.

Physiology

Approximately 8 to 9 L of fluid enters the small bowel in a 24-hour period. This includes 1 to 2 L from dietary consumption, with the remainder produced by salivary, gastric, biliary, pancreatic, and intestinal secretions. All but 1 to 2 L is absorbed in the small intestine and then enters the colon. Almost all of this fluid is absorbed as it travels through the colon, leaving less than 200 g/day of stool. Disruption of the absorption of ions, solutes, and water or increased secretion of electrolytes results in water accumulation in the lumen and therefore diarrhea

What are Eosinophils primarily involved in defense against?

Are circulating granulocytes primarily involved in defense against parasitic infections Because parasites are often too large to engulf ,eosinophils attach to a parasite's exterior and secrete cytotoxic substances directly on the parasite

What are the symptoms faced by transplant patients facing acute rejection?

Are usually asymptomatic, and symptoms of fever or malaise are usually nonspecific

What are the clinical manifestations of Malabsorption?

Are usually nonspecific, particularly in the early stages. A change in bowel movements, usually with diarrhea, and weight loss despite adequate food intake may occur in more severe cases Usually, however, patients have relatively mild symptoms such as bloating and flatulence.

What is a Rectal examination important for identifying ?

Arectal tumor in the case of colon obstruction or tenderness high in the rectum in acute appendicitis A pelvic examination should be performed in women to rule out pelvic inflammatory disease

What is the most common extraintestinal manifestation of Inflammatory bowel disease (both UC and Crohn's)?

Arthritis seen in about 9% to 50% of patients

inhibition of immune checkpoints

As discussed above in the "Inhibition o Costimulation" section, the immune system uses costimulatory signals to activate antigen-specif c immune responses (see also Chapter 42). The immune system also uses immune checkpoint signals to inhibit such responses. Inhibitory checkpoint molecules include cytotoxic T lymphocyte antigen 4 (CTLA-4), programmed cell death protein 1 (PD-1), and several others. In general, ligation o these molecules on T cells inhibits the immune response; the signaling pathways that mediate these inhibitory responses are subjects o active investigation. Some tumors have been ound to up-regulate PD-1 ligands and thereby inhibit T-cell immune surveillance o tumors. Studies have shown that inhibition o CTLA-4 and/or PD-1 augments T-cell activation, proli eration, and cytokine production, and can be used to enhance the anti-tumor immune response

other agents

As noted in Table 42-2, other signaling molecules also coordinate the inf ammatory response. These include kinins, platelet-activating actor, nitric oxide, oxygen radicals, and other leukocyte and bacterial products released during phagocytosis. Although pharmacologic agents have been developed to modulate each o these pathways, there are, as yet, no approved anti-inf ammatory drugs that speci cally interrupt the action o these mediators.

migration of lymphocytes in cell-mediated immune reactions

As we discussed at the beginning of this chapter, T cell responses are initiated primarily in secondary lymphoid organs, and the effector phase occurs mainly in peripheral tissue sites of infection (see Fig. 5-2). Thus, T cells at different stages of their lives have to migrate in different ways: • Naive T cells must migrate between blood and secondary (peripheral) lymphoid organs throughout the body, until they encounter dendritic cells within the lymphoid organ that display the antigens the T cells recognize • After the naive T cells are activated and differentiate into effector cells, these cells must migrate back to the sites of infection, where they function to kill microbes. The migration of naive and effector T cells is controlled by three families of proteins—selectins, integrins, and chemokines—that regulate the migration of all leukocytes, as described in Chapter 2 (see Fig. 2-16, Chapter 2). The routes of migration of naive and effector T cells differ significantly because of selective expression of different adhesion molecules and chemokine receptors on naive T cells versus effector T cells, together with the selective expression of endothelial adhesion molecules and chemokines in lymphoid tissues and sites of inflammation Naive T cells express the adhesion molecule L-selectin (CD62L) and the chemokine receptor CCR7, which mediate the selective migration of the naive cells into lymph nodes through specialized vessels called high endothelial venules (HEVs) (see Fig. 5-15). HEVs are located in the T cell zones of lymphoid tissues and are lined by specialized endothelial cells, which express carbohydrate ligands that bind to L-selectin. HEVs also display chemokines that are made only in lymphoid tissues and are specifically recognized by CCR7. The migration of naive T cells proceeds in a multistep sequence like that of migration of all leukocytes through blood vessels (see Chapter 2): • Naive T cells in the blood engage in L-selectin- mediated rolling interactions with the HEV, allowing chemokines to bind to CCR7 on the T cells. • CCR7 transduces intracellular signals that activate the integrin leukocyte function-associated antigen 1 (LFA-1) on the naive T cell, increasing the binding affinity of the integrin. • The increased affinity of the integrin for its ligand, intercellular adhesion molecule 1 (ICAM-1) on the HEV, results in firm adhesion and arrest of the rolling T cells. • The T cells then exit the vessel through the endothelial junctions and are retained in the T cell zone of the lymph node because of the chemokines produced there. Thus, many naive T cells that are carried by the blood into an HEV migrate to the T cell zone of the lymph node stroma. This happens constantly in all lymph nodes and mucosal lymphoid tissues in the body. Effector T cells do not express CCR7 or L-selectin, and thus they are not drawn into lymph nodes. The phospholipid sphingosine 1-phosphate (S1P) plays a key role in the egress of T cells from lymph nodes. The levels of S1P are higher in the blood and lymph than inside lymph nodes. S1P binds to and thereby reduces expression of its receptor, which keeps the expression of the receptor on circulating naive T cells low. When a naive T cell enters the node, it is exposed to lower concentrations of S1P, and expression of the receptor begins to increase. If the T cell does not recognize any antigen, the cell leaves the node through efferent lymphatic vessels, following the gradient of S1P into the lymph. If the T cell does encounter specific antigen and is activated, the surface expression of the S1P receptor is suppressed for several days. As a result, recently activated T cells stay in the lymph node long enough to undergo clonal expansion and differentiation. When this process is completed, S1P receptor is reexpressed on the cell surface; at the same time, the cells lose expression of L-selectin and CCR7, which previously attracted the naive T cells to the lymph nodes. Therefore, activated T cells are drawn out of the nodes into the draining lymph, which then transports the cells to the circulation. The net result of these changes is that differentiated effector T cells leave the lymph nodes and enter the circulation. The importance of the S1P pathway has been highlighted by the development of a drug (fingolimod) that binds to the S1P receptor and blocks the exit of T cells from lymph nodes. This drug is approved for the treatment of the inflammatory disease multiple sclerosis Effector T cells migrate to sites of infection because they express adhesion molecules and chemokine receptors that bind to ligands expressed or displayed on vascular endothelium in innate immune responses to microbes. The process of differentiation of naive T lymphocytes into effector cells is accompanied by changes in the types of adhesion molecules and chemokine receptors expressed on these cells (see Fig. 5-15). The migration of activated T cells into peripheral tissues is controlled by the same kinds of interactions involved in the migration of other leukocytes into tissues (see Chapter 2): • Activated T cells express high levels of the glycoprotein ligands for E- and P-selectins and the integrins LFA-1 and VLA-4 (very late antigen 4). Innate immune cytokines produced at the site of infection, such as TNF and IL-1, act on the endothelial cells to increase expression of E- and P-selectins as well as ligands for integrins, especially ICAM-1 and vascular cell adhesion molecule 1 (VCAM-1), the ligand for the VLA-4 integrin. • Effector T cells that are passing through the blood vessels at the infection site bind first to the endothelial selectins, leading to rolling interactions. • Effector T cells also express receptors for chemokines that are produced by macrophages and endothelial cells at these inflammatory sites and are displayed on the surface of the endothelium. The rolling T cells recognize these chemokines, leading to increased binding affinity of the integrins for their ligands and firm adhesion of the T cells to the endothelium. • After the effector T lymphocytes are arrested on the endothelium, they engage other adhesion molecules at the junctions between endothelial cells, crawling through these junctions into the tissue. Chemokines that were produced by macrophages and other cells in the tissues stimulate the motility of the transmigrating T cells. The net result of these molecular interactions between the T cells and endothelial cells is that the T cells migrate out of the blood vessels to the site of infection. Naive T cells do not express ligands for E- or P-selectin and do not express receptors for chemokines produced at inflammatory sites. Therefore, naive T cells do not migrate into sites of infection or tissue injury. The homing of effector T cells to a site of infection is independent of antigen recognition, but lymphocytes that recognize antigens are preferentially retained and activated at the site. The homing of effector T cells to sites of infection mainly depends on adhesion molecules and chemokines. Therefore, any effector T cell present in the blood, regardless of antigen specificity, can enter the site of any infection. This nonselective migration presumably maximizes the chances of effector lymphocytes entering tissues where they may encounter the microbes they recognize. The effector T cells that leave the circulation, and that specifically recognize microbial antigen presented by local tissue APCs, become reactivated and contribute to the killing of the microbe in the APC. One consequence of this reactivation is an increase in the expression of VLA integrins on the T cells. Some of these integrins specifically bind to molecules present in the extracellular matrix, such as hyaluronic acid and fibronectin. Therefore, the antigen-stimulated lymphocytes adhere firmly to the tissue matrix proteins near the antigen, which may serve to keep the cells at the inflammatory sites. This selective retention contributes to accumulation of more and more T cells specific for microbial antigens at the site of infection. As a result of this sequence of T cell migration events, the effector phase of T cell-mediated immune responses may occur at any site of infection. In contrast with the activation of naive T cells, which requires antigen presentation and costimulation by dendritic cells, differentiated effector cells are less dependent on costimulation. Therefore, the proliferation and differentiation of naive T cells are confined to lymphoid organs, where dendritic cells (which express abundant costimulators) display antigens, but the functions of effector T cells may be reactivated by any host cell displaying microbial peptides bound to MHC molecules, not just dendritic cells. Elucidation of the molecular interactions involved in leukocyte migration has spurred many attempts to develop agents to block the process of cell migration into tissues. Antibodies against integrins are effective in the inflammatory diseases multiple sclerosis and inflammatory bowel disease, but the clinical utility of these drugs is limited because reducing leukocyte entry into tissues, especially the central nervous system, allows the reactivation of latent viruses in occasional treated patients. A small molecule inhibitor of the S1P pathway is used for treating multiple sclerosis, as mentioned above. Small molecules that bind to and block chemokine receptors have also been developed, and some have shown efficacy in inflammatory bowel disease.

What may be the best index of pancreatic exocrine function?

Aspiration of duodenal contents for evaluation of bicarbonate and enzyme output after stimulation of the pancreas However, the test is invasive, is time-consuming, and is performed only in a few specialized centers

Tests of Pancreatic exocrine function

Aspiration of duodenal contents for evaluation of bicarbonate and enzyme output after stimulation of the pancreas may be the best index of pancreatic exocrine function. However, the test is invasive, is time-consuming, and is performed only in a few specialized centers. The measurement of pancreatic enzymes (i.e., fecal elastase 1) in the stool is simple and provides helpful laboratory evidence for the diagnosis of moderate to severe pancreatitic insufficiency. Pancreatic calcifications seen on abdominal films or computed tomography (CT) scans indicate the presence of chronic pancreatitis. Magnetic resonance cholangiopancreatography (MRCP) and endoscopic retrograde cholangiopancreatography (ERCP) can help outline abnormal duct anatomy and may supplement CT scanning for diagnostic purposes to evaluate the sequelae of chronic pancreatitis. However, normal findings on pancreatography do not exclude the presence of pancreatic exocrine insufficiency.

Where does the abdominal aorta begin?

At the aortic hiatus of the diaphragm anterior to the lower border of vertebra T12

Where is the right colic flexure located?

At the junction of the ascending and transverse colon is just inferior to the right lobe of the liver

Where does the left colic flexure occur?

At the junction of the transverse and descending colon This bend is just inferior to the spleen, is higher and more posterior than the right colic flexure, and is attached to the diaphragm by the phrenicocolic ligament.

What is MMF also effective in the treatment of?

Autoimmune diseases In rheumatoid arthritis, levels of rheumatoid factor, immunoglobulin, and T cells are all reduced by treatment with MMF

autoimmunity

Autoimmune diseases occur when the host immune system attacks its own tissues, mistaking sel -antigen or oreign. The typical result is chronic in ammation in the tissue(s) expressing the antigen Autoimmune diseases are most commonly due to a breakdown o sel -tolerance, both central and peripheral. Central tolerance re ers to the specif c clonal deletion o autoreactive T and B cells during their development rom precursor cells in the thymus (T cells) and bone marrow (B cells). Central tolerance ensures that the majority o immature autoreactive T and B cells do not develop into sel -reactive clones. The thymus and bone marrow do not express every antigen in the body, however; a number o proteins are expressed only in specif c tissues. For this reason, peripheral tolerance is also important. Peripheral tolerance results rom deletion o autoreactive T cells by Fas-Fas ligand-mediated apoptosis, activation o T suppressor cells, or induction o T-cell anergy due to antigen presentation in the absence of costimulation Although breakdown in tolerance lies at the center o virtually all autoimmune diseases, the inciting stimulus leading to loss o tolerance is o ten unknown. Genetic actors may play a role, in that the presence o certain MHC subtypes may predispose T cells to the loss o sel -tolerance. For example, human leukocyte antigen (HLA)-B27 is causally related to many orms o autoimmune spondylitis. Several other autoimmune diseases are linked to specif c HLA loci, supporting an association, i not a causal role, or genetic predisposition to autoimmunity. Molecular mimicry , in which epitopes rom in ectious agents are similar to sel -antigens, can also lead to a breakdown o tolerance and may be the mechanism underlying poststreptococcal glomerulonephritis. Several other processes, including ailure o T-cell apoptosis, polyclonal lymphocyte activation, and exposure o cryptic sel -antigens, have also been hypothesized to lead to autoimmunity. The details o these mechanisms are beyond the scope o this book; however, the result o each is a loss of tolerance Once sel -tolerance has been compromised, the specif c expression o autoimmunity can take one o three general orms (Table 46-2). In some diseases, production o autoantibodies against a specif c antigen causes antibody-dependent opsonization o cells in the target organ, with subsequent cytotoxicity. One example is Goodpasture's syndrome, which results rom autoantibodies against collagen type IV in the renal glomerular basement membrane. In some autoimmune vasculitis syndromes, circulating antibody-antigen complexes deposit in blood vessels, causing in ammation and injury to the vessels. Two examples o immune complex disease are mixed essential cryoglobulinemia and systemic lupus erythematosus. Finally, T-cell-mediated diseases are caused by cytotoxic T cells that react with a specif c sel -antigen, resulting in destruction o the tissue(s) expressing that antigen. One example is type 1 diabetes mellitus, in which the cytotoxic T cells react against sel -antigens in pancreatic beta cells The pharmacologic therapy or autoimmune diseases does not yet match the exquisite specif city o the o ending biological process. Most currently available pharmacologic agents cause generalized immunosuppression and do not target the specif c pathophysiology. Better understanding o the molecular pathways leading to autoimmune diseases should reveal new pharmacologic targets that can be used to suppress the specif c autoimmune response be ore disease arises

What can patients treated with checkpoint blockade often develop?

Autoimmune reactions

What is the HLA-B27 related to forms of?

Autoimmune spondylitis

autoimmunity

Autoimmunity is defined as an immune response against self (autologous) antigens. It is an important cause of disease, estimated to affect 2% to 5% of the population in developed countries, and the prevalence of several autoimmune diseases is increasing. Different autoimmune diseases may be organ-specific, affecting only one or a few organs, or systemic, with widespread tissue injury and clinical manifestations. Tissue injury in autoimmune diseases may be caused by antibodies against self antigens or by T cells reactive with self antigens (see Chapter 11). A cautionary note is that in many cases, diseases associated with uncontrolled immune responses are called autoimmune without formal evidence that the responses are directed against self antigens.

What was the first drug to be used for suppression of the immune system after organ transplantation?

Azathioprine (AZA)

Azathioprine

Azathioprine (AZA) was the f rst drug to be used or suppression o the immune system a ter organ transplantation, and it remains a mainstay or this indication. AZA is a prodrug o the purine analogue 6-mercaptopurine (6-MP), which is slowly released as AZA reacts nonenzymatically with sul hydryl compounds such as glutathione (Fig. 46-2). The slow release of 6-MP from AZA favors immunosuppression. Although AZA prolongs organ gra t survival, this drug is less e f cacious than mycophenolate mo etil in improving the long-term survival o kidney allogra ts. AZA and 6-MP are also used as immunosuppressants in the treatment o in ammatory bowel disease, acute lymphoblastic leukemia, and autoimmune skin disorders.

What drugs are effective therapies for maintaining remission in both Crohn's disease and Ulcerative colitis and are used primarily as steroid-sparing agents? side effects?

Azathioprine and 6-MP They have a slow onset of action (weeks to months). Side effects include nausea, abnormal liver enzymes, bone marrow suppression, opportunistic infections, and an increased risk of lymphoma and nonmelanoma skin cancer

What happens if B cells recognize an antigen but does not receive T cell help?

B cells become anergic because of a block in signaling from the antigen receptor Anergic B cells may leave lymphoid follicles and are subsequently excluded from the follicles. These excluded B cells may die because they do not receive necessary survival stimuli.

What is a member of the TNF family that promotes B cell survival and differentiation?

B lymphocyte stimulator (BLyS)

A 6 year old patient is diagnosed with a rare disorder called autoimmune polyendocrine syndrome (APS). The attending physician wants to explain her condition to her parents. He tells them that their child has a mutation in a gene encoding a protein that is responsible for presenting peripheral tissue antigen in thymus. What protein is responsible for causing this disorder? A) MHC class II B) AIRE C) FAS D) MHC class I E) CTLA-4

B) AIRE

A 55-year-old male comes into the emergency department with abdominal pain. He explains that this has been occurring for the past few weeks and almost always after a meal. He also explains he has not been eating as often anymore resulting in weight loss. After physical examination and an aortic angiography, the physician explains that he has been experiencing mesenteric angina from narrowing of the celiac trunk and SMA. What is the best explanation for the pain he is experiencing after meals? A) Due to the stenosed vessels, not enough blood is supplied to the colon causing the mucosa to slough off, resulting in pain after meals. B) After a meal oxygen demand increases, and the stenosed vessels are not supplying enough blood causing abdominal pain. C) The stenosed vessels themselves are causing the pain D) The stenosed vessels results in inflammation of the abdominal wall, causing somatic pain only after meals.

B) After a meal oxygen demand increases, and the stenosed vessels are not supplying enough blood causing abdominal pain.

A group of Immunologists are dressing up for their research lab halloween contest as different costimulatory signals. Each of their spouses is dressing up as the complementary molecule.The group ends up losing the contest because one couple was not wearing the correct complementary costumes. Which of the following costimulatory signals was the reason they lost the contest? A) CD40 & CD40L B) CD86 & B7-2 C) MHC:antigen & TCR D) CD28 & CD80 E) CD28 & B7-1

B) CD86 & B7-2

A 25-year old patient with a history of type 1 diabetes is brought to the clinic with complaints of abdominal discomfort, bloating, and loose stools. When asked about her diet, the patient states that she "likes to eat everything." Suspecting Celiac Disease, the physician sends the patient for further testing. What is the most likely diagnostic tool the physician used in order to confirm this disease? A) Fecal fat analysis B) Intestinal biopsy C) Schilling test D) Digital rectal exam E) Hydrogen breath test

B) Intestinal biopsy

A 20-year-old male presents to the emergency room with complaints of bloody diarrhea, right lower quadrant pain, and fever. An initial stool sample was collected and showed abundant neutrophils. After ruling out acute appendicitis and bowel perforation, the physician ordered a tissue biopsy that showed neutrophils infiltrating and damaging crypt epithelium. What is a characteristic feature of this disease that differentiates it from other forms of inflammatory bowel disease? A) Broad-based ulcers B) Skip lesions C) Small, flask-like outpouchings D) Pseudopolyps E) Fissures

B) Skip lesions

Antigens of which of the following pathogens would most likely be displayed by class I MHC molecules? A) Protozoa B) Viruses C) Fungi D) Intracellular bacteria E) Extracellular bacteria

B) Viruses

A 35 year old woman came into the clinic with complaints of a swollen red leg. The doctor knew that a local inflammatory response was happening. A blood test was run and there was a presence of antibodies. The doctor knew this meant Helper T cells were activated. What interaction is occurring that promotes Helper T cell activation? A) CTLA-4 from Helper T cells is bound to B7-1 and B7-2 from APCs B)CD40L from Helper T cells bind to CD40 from APCs C) Phagocytosis of extracellular bacteria with antigen presentation via MHC class I D) Presentation of viral antigens from an infected cell via MHC class II E) Formation of the membrane attack complex

B)CD40L from Helper T cells bind to CD40 from APCs

A 45 year old female presents to her doctor's office because she has been experiencing abdominal pain, diarrhea, weight loss, and fatigue for the past month. Looking at the patient's chart, she has a family history of inflammatory bowel disease (IBD). The doctor performs an endoscopy as well as genetic testing and diagnoses the patient with Crohn's disease. Interestingly, the genetic testing indicates that the patient has increased risk for Crohn's disease due to a mutation in a gene that regulates the interleukin-17 (IL-17) and interleukin-23 (IL-23) receptor pathways. A mutation in which of the following genes would lead to increased risk of Crohn's disease and differentiate from a diagnosis of Ulcerative colitis (UC)? A) TYK2 B)IL27 C) JAK2 D) IL23R E) STAT3

B)IL27

A 34 year old female presents to the office for management of severe rheumatoid arthritis. The patient states that he current treatment with methotrexate hasn't provided her with any relief for her RA symptoms. The physician decides to prescribe her a drug that works by inhibiting costimulation of lymphocytes but before doing so must screen her for TB because this drug has been known to reactivate latent TB infection. What drug did the physician most likely prescribe and what is its mechanism of action? A. Abatacept; forms a complex with costimulatory B7 molecules on the surface of APCS, preventing the delivery of a costimulatory signal to B cells, leading to B cell anergy and apoptosis. B. Abatacept; forms a complex with costimulatory B7 molecules on the surface of APCs, preventing the delivery of a costimulatory signal to T cells, leading to T cell anergy and apoptosis. C. Anakinra; forms a complex with costimulatory B7 molecules on the surface of APCs, preventing the delivery of a costimulatory signal to T cells, leading to T cell anergy and apoptosis. D. Abatacept; blocks the binding of the T cell receptor to the MHC-antigen complex, inhibiting T cell activation, leading to T cell anergy and apoptosis. E. Alefacept; forms a complex with costimulatory B7 molecules on surface of APCs, preventing the delivery of a costimulatory signal to B cells, leading to B cell anergy and apoptosis.

B. Abatacept; forms a complex with costimulatory B7 molecules on the surface of APCs, preventing the delivery of a costimulatory signal to T cells, leading to T cell anergy and apoptosis.

A 63 year old man is admitted to the emergency department with severe pain that radiates from his epigastric region, towards his left flank, and down to his left groin. An arteriogram reveals a 95% blockage at the origin of his inferior mesenteric artery. Which of the following could explain his symptoms? A. Blockage of his right colic artery causes ischemia of the transverse colon B. Blockage of his left colic artery causes ischemia of the transverse colon C. Blockage of his middle colic artery causes ischemia of the descending colon D. Blockage of his right colic artery causes ischemia of the descending colon E. Blockage of his left colic artery causes ischemia of the sigmoid colon

B. Blockage of his left colic artery causes ischemia of the transverse colon

In the midst of the Covid-19 pandemic, a biotech company decided to enter the race to Covid-19 immunity and started developing an adjuvanted vaccine. This vaccine would target which set of molecules on the T lymphocyte and antigen-presenting cell respectively? A. CD4 and MHC Class II B. CD40L and CD40 C. CTLA-4 and B7-1 D. PD-1 and PD-L1 E. LFA-1 and ICAM-1

B. CD40L and CD40

A professor explains to his students about the importance of CD8 T cells in killing virus infected cells or cancer cells. He goes on to say that patients with HIV, which kills CD4 and not CD8 T cells, suffer from increased susceptibility to viral infections and cancer. What is the best explanation as to why those with HIV have increased susceptibility to viral infections and cancer? A. HIV also kills CD8 T cells B. CD8 T cell activation requires CD4 Helper T cells C. CD4 T cells are the main factor in killing viral infected and tumor cells. D. CD4 T cells inhibit the growth and differentiation of CD8 T cells E. CD8 T cells are not involved in killing viral infected and tumor cells.

B. CD8 T cell activation requires CD4 Helper T cells

A student researching lymphocytes observes that there is a remarkable increase in a certain T lymphocyte about 1-2 days after antigen-specific activation and co-stimulation. She notes that there is a 10,000-fold increase in this T lymphocyte, most of which will exhibit cytotoxic activity. Which of the following classifies this lymphocyte? A. T-helper cell B. CD8+ T cell C. Memory T cell D. Regulatory T cell E. NK cell

B. CD8+ T cell

Dave is a 33-year-old male who presents to the emergency department with complaints of intermittent epigastric pain that radiates to his right shoulder. Based on his current symptoms, what is the most likely differential diagnosis at this time? A. Appendicitis B. Cholecystitis C. Pancreatitis D. Small bowel obstruction E. Peptic ulcer

B. Cholecystitis

A 50-year-old female presents with nonbloody, watery diarrhea for the past six months. Colonoscopy is normal but histological exam reveals thickened subepithelial connective tissue layer and increased intraepithelial lymphocytes. Which of the following is the most likely diagnosis? A. Ulcerative colitis B. Collagenous colitis C. Lymphocytic colitis D. Diversion colitis E. Irritable Bowel Syndrome

B. Collagenous colitis

After an effector T Cell migrates from the lymph node into circulation, it travels to sites of infection where it then binds to the vascular endothelium. What is the first receptor:ligand interaction that begins the process of migrating out of the blood and into the infected tissue? A. b2-integrin with ICAM-1 B. E and P selectin ligand with E and P selectin C. CXCR3 with CXCL10 D. L-Selectin with L-selectin ligand E. S1P with S1PR1

B. E and P selectin ligand with E and P selectin

A 45-year-old female presents to the clinic with complaints of fatigue, dyspnea, dark urine occurring at night, back pain and easy bruising. After several workups, Flow cytometry confirmed the diagnosis of Paroxysmal Nocturnal Hemoglobinuria. What is the appropriate treatment? A. Natalizumab B. Eculizumab C. Ipilimumab D. Belimumab E. Rituximab

B. Eculizumab

Mature MHC-restricted T cells express either CD4 or CD8, both of which bind to the appropriate peptide-MHC complex, MHC class II and MHC class I respectively. When activated CD4+ cells recognize antigens ingested from _______, while CD8+ T cells recognize peptides derived from _______. A. Cytosolic or nuclear antigens; extracellular microbes B. Extracellular microbes; cytosolic or nuclear antigens C. Both CD4+ T cells and CD8+ T cells recognize extracellular antigens D. Both CD4+ T cells and CD8+ T cells recognize cytosolic or nuclear antigens E. None of the above

B. Extracellular microbes; cytosolic or nuclear antigens

In a clinic, a patient presents with features of immune dysregulation polyendocrinopathy enteropathy X-Linked syndrome or IPEX. In explaining to the patient the basis of their disease, the physician would describe the issue as a mutation in what gene affecting what major immune cell type? A. FOXP3, B Cells B. FOXP3, T Cells C. AIRE, B Cells D. AIRE, T Cells E. FAS, B and T Cells

B. FOXP3, T Cells

A 28-year-old female schedules an appointment with her primary care physician with complaints of abdominal pain in her right lower quadrant, weight loss, and non-bloody diarrhea. Serological testing confirms the presence of anti-Saccharomyces cerevisiae antibodies (ASCA). Mucosal biopsy reveals transmural inflammation and the formation of fistulas. Which of the following is also likely to be found in this patient? A. Continuous pattern of inflammation B. Granulomas C. Superficial ulcerations D. Perinuclear Anti-Neutrophil Cytoplasmic Antibodies (pANCA) E. Erythematous and friable ulcers

B. Granulomas

A person who is suffering from chronic respiratory infections undergoes a multitude of tests to diagnose the root cause. Their physician informs them they have a deficiency in T cells. What deficit in a specific cytokine would best explain this scenario? A. IFN-γ B. IL-2 C. IL-17 D. TGF-β E. IL-5

B. IL-2

A 65-year-old male is referred to an oncologist by his primary care physician and subsequently diagnosed and treated for metastatic melanoma. After six months of therapy with a monoclonal antibody that specifically targets CTLA-4, the patient develops checkpoint inhibitor-induced insulin dependent diabetes mellitus. Which one of the following agents did the oncologist most likely prescribe? A. Natalizumab B. Ipilimumab C. Eculizumab D. Infliximab E. Rituximab

B. Ipilimumab

T lymphocytes have a variety of different surface molecules that all serve a different important function. T-cell receptors have low affinity for peptide-MHC complexes on antigen presenting cells. Which of the following surface molecules function for adhesion/stabilization of the T cell binding to antigen presenting cells? A. CD3 B. LFA-1 C. CD28 D. CD4 E. PD-1

B. LFA-1

A 26 year old patient comes to his PCP complaining of diarrhea and alopecia. Medical history includes treatment for rheumatoid arthritis. The PCP believes that the patient's current symptoms are a result of treatment with a drug that is an inhibitor of pyrimidine synthesis. What drug was the patient most likely given to cause these symptoms? A. Azathioprine B. Leflunomide C. Mycophenolate mofetil D. methotrexate E. cyclosporine

B. Leflunomide

A patient has arrived with complaints of vomiting blood. Patient history shows excessive consumption of alcohol for many years. Upon physical examination, the patient has ascites of the abdomen and yellowing of the skin and eyes. The physician suspects varices of the veins in the esophagus as the reason for vomiting blood. Which of the following are the portosystemic anastomoses that the esophageal varices occur? A. Superior rectal vein of portal system to middle and inferior rectal vein of caval system B. Left gastric vein of portal system with azygos veins of caval system C. Left gastric vein of caval system with azygos veins of portal system D. Para-umbilical veins of the portal system with the veins of the anterior abdominal wall E. Portal vein with the inferior vena cava

B. Left gastric vein of portal system with azygos veins of caval system

A 54 year old male presents to the clinic with a history of occlusion of the inferior mesenteric artery but has not experienced any complications due to anastomoses among the right, middle, and left colic arteries made possible by which of the follow arteries: A. Left anterior descending artery B. Marginal artery of Drummond C. Gastric artery D. Pancreaticoduodenal artery E. Superior mesenteric artery

B. Marginal artery of Drummond

A 30-year-old woman has an appointment with her gastroenterologist. She recently had a colonoscopy to look at her colon which was inconclusive, even though she had prior hemorrhage and blockage. She has been referred for an angiogram of her ileum and the results are shown below (need to see image through the answer key), what is the woman's diagnosis? A. Constipation B. Meckel's Diverticulum C. Ulcerative colitis D. Diverticular disease E. Hemorrhoids

B. Meckel's Diverticulum

A 45-year-old woman presents to a new primary care physician with complaints of diarrhea and hair loss in large patches on her head. Upon reviewing her chart and drug list, the physician notices that she has recently been prescribed an antiviral for drug-resistant cytomegalovirus which has caused her hair to fall out. What is the drug the patient is taking currently approved for? A. Psoriasis B. Rheumatoid Arthritis C. Keratoconjunctivitis sicca D. Crohn's Disease E. Multiple Sclerosis

B. Rheumatoid Arthritis

Maria recently visited her primary care clinic and was diagnosed with Type 1 Diabetes, an autoimmune disease which occurs due to antibodies that bind to the body's own insulin-producing beta cells and destroys them, resulting in a lack of insulin. The physician describes a relatively new drug which could possibly alleviate some of her symptoms by inhibiting the immune response. What is the mechanism by which this drug most likely acts? A. The drug binds to and inhibits CTLA-4 on T-cells B. The drug blocks the interaction of B7 and CD28 between APCs and T-cells C. The drug binds to and blocks PD-1 on T-cells D. The drug blocks ICAM-1 E. The drug binds to and blocks ICOS

B. The drug blocks the interaction of B7 and CD28 between APCs and T-cells

A 65 year old man with a history of poorly controlled high cholesterol and arteriosclerosis goes to his doctor for his annual physical check-up. The physician notes that the patient has lost a significant amount of weight since his last check-up. The patient says he has been experiencing severe pain and discomfort after eating a heavy meal and has avoided eating to not feel the pain. The patient hasn't traveled anywhere recently and denies having any diarrhea, blood in stool, or fever. When asked about feeling pain throughout the day, the patient insisted on only feeling pain after eating a meal. After an aortic angiography, the doctor was able to confirm a diagnosis. What is the patient's diagnosis? A. Atherosclerotic plaque at the inferior mesenteric artery causing reduced blood flow to the hindgut which led to the patient's symptoms. B. The patient is experiencing mesenteric angina which is caused by stenosis of celiac trunk and superior mesenteric artery, impeding blood flow to the gut. C. Tumor in the small bowel wall causing a mechanical obstruction. D. Patient has multiple colon diverticula in the sigmoid colon.

B. The patient is experiencing mesenteric angina which is caused by stenosis of celiac trunk and superior mesenteric artery, impeding blood flow to the gut.

A surgeon is performing a liver transplant when he notices that organ becoming cyanotic, mottled, and flaccid within minutes of performing the transplant. He suspects that a mistake was made causing hyperacute rejection. Which of the following is most likely the mistake that was made leading to this type of organ rejection? A. The patient was not started on immunosuppressive therapy prior to transplant B. The patient's blood group antigens were not matched to the donor's C. T cells were not removed from the organ prior to transplant D. The patient's T cells were not deactivated prior to transplant E. There was no way this could have been prevented

B. The patient's blood group antigens were not matched to the donor's

A 23 year old female presents to the emergency department with complaints of abdominal pain. Upon further examination it is concluded that her discomfort is due to peptic ulcers in her stomach. Which of the following regions is affected in this patient and what is its blood supply? A. midgut, superior mesenteric artery B. foregut, celiac trunk C. rectum, inferior mesenteric artery D. hindgut, inferior mesenteric artery E. midgut, celiac trunk

B. foregut, celiac trunk

A 6-year-old patient arrives in the clinic with complaints of abdominal pain and dyspepsia. After further testing, it is confirmed that the patient has a bacterial infection originating in the mucosal layer of the gastric epithelium. What is the most likely complication of this infection? A: Microscopic polyangiitis B: Gastroduodenal ulcers C: Angiodysplasia D: Esophageal cancer E: Mallory-Weiss tear

B: Gastroduodenal ulcers

What do breath tests rely on?

Bacterial degradation of luminal compounds, which releases metabolic byproduct gases (e.g., hydrogen, methane, carbon dioxide) that can be measured in the exhaled breath

anti CD25 monoclonal antibody

Basiliximab is a monoclonal antibody directed against CD25, the high-a f nity IL-2 receptor. IL-2 mediates early steps in T-cell activation. Because CD25 is expressed only on activated T cells, anti-CD25 antibody therapy selectively targets T cells that have been activated by an MHC-antigen stimulus Basiliximab is administered prophylactically in renal transplantation to inhibit acute organ rejection. It is also used as a component o general immunosuppressive regimens a ter organ transplantation. Basiliximab is typically administered in a two-dose regimen, with the f rst administration 2 hours be ore transplantation surgery and the second dose 4 days a ter transplantation. This type o dosing regimen, in which drug is administered or a limited period immediately a ter transplantation, is re erred to as induction therapy . Daclizumab is another antibody with the same mechanism o action that was voluntarily withdrawn rom the market by the manu acturer in 2009.

Why is Etanercept somewhat less selective than other anti-TNF agents?

Because it binds to both TNF-alpha and TNF-beta

How is a relatively blood-free mobilization of the ascending and descending colon possible?

Because major vessels and lymphatics are on the medial or posteromedial sides of the ascending and descending colon, a relatively blood-free mobilization of the ascending and descending colon is possible by cutting the peritoneum along these lateral paracolic gutters

Indeterminante colitis

Because of the extensive pathologic and clinical overlap between ulcerative colitis and Crohn disease definitive diagnosis is not possible in approximately 10% of IBD patients. These cases, termed indeterminate colitis, do not involve the small bowel and have colonic disease in a continuous pattern that would typically indicate ulcerative colitis. However, patchy histologic disease, fissures, a family history of Crohn disease, perianal lesions, onset after initiating use of cigarettes, or other features that are not typical of ulcerative colitis may prompt more detailed endoscopic, radiographic, and histologic examination. Serologic studies can be useful in these cases because perinuclear anti-neutrophil cytoplasmic antibodies are found in 75% of individuals with ulcerative colitis but only 11% with Crohn disease. In contrast, ulcerative colitis patients tend to lack antibodies to Saccharomyces cerevisiae, which are often present in those with Crohn disease. However, even the serologic results can be ambiguous in cases that are indeterminate on clinical grounds. Despite diagnostic uncertainty, extensive overlap in medical management of ulcerative colitis and Crohn disease allows patients carrying a diagnosis of indeterminate colitis to be treated effectively.

evaluation of chronic diarrhea

Because of the myriad causes and differing severities, optimal guidelines are not established. Most recommendations are based on expert opinion and may include bias as a result of the types of referrals or regional differences. A thorough history and physical examination are essential. The history should try to establish the likelihood of organic versus functional disorders, to differentiate malabsorptive from inflammatory etiologies, and to determine the cause of the diarrhea. Consequently, important parts of the history include the following: 1. Character of the onset of diarrhea—sudden or gradual 2. Continuous versus intermittent symptoms 3. The presence of nocturnal diarrhea 4. Duration of diarrhea 5. Epidemiology—travel, exposure to contaminated food or water, family members with similar illness 6. Stool characteristics—watery, bloody, greasy 7. Fecal incontinence versus diarrhea or both 8. Abdominal pain—IBD, IBS, mesenteric vascular insufficiency 9. Weight loss—often significant in malabsorption, IBD, ischemia, and neoplasm 10. Aggravating factors—stress, specific foods (e.g., milk) 11. Prior evaluations to avoid repeating tests 12. Mitigating factors—what the patient has tried to control the diarrhea 13. Previous operations, radiation therapy, medications, supplements 14. Factitious diarrhea—always a consideration in eating disorders, malingering, or secondary gain 15. Review of systems—hyperthyroidism, scleroderma, tumor syndromes, diabetes mellitus 16. Risk factors for HIV and other immunosuppressed states Physical examination rarely provides a specific diagnosis, but it does allow an assessment of fluid status and nutritional status. Some helpful findings include mouth ulcers or perianal disease that suggest the possibility of IBD, rashes or flushing, abdominal mass, and findings of hyperthyroidism. Physical examination rarely provides a specific diagnosis, but it does allow an assessment of fluid status and nutritional status. Some helpful findings include mouth ulcers or perianal disease that suggest the possibility of IBD, rashes or flushing, abdominal mass, and findings of hyperthyroidism. Some causes are related to socioeconomic status. In contrast to acute diarrhea, infectious causes of chronic diarrhea are unusual in the United States, although they are frequently encountered in developing countries. Infectious causes are a concern in newly arrived immigrants or travelers. Occasionally, persisting infections such as Giardia, Entamoeba, C. difficile, Aeromonas, Plesiomonas, Cryptosporidium, Tropheryma whipplei (Whipple's disease), Blastocystis hominis, and Cyclospora can cause chronic diarrhea. In addition, up to 30% of patients with these infections develop postinfectious IBS as a cause of their chronic diarrhea. Patients with diarrhea-predominant IBS can have a wide variety of symptoms, but the main complaints are usually chronic abdominal pain and altered bowel movements. These patients complain of small-volume, frequent diarrhea, often with interspersed normal or constipated stools. They may report marked urgency as well as a feeling of incomplete evacuation. Approximately one half of these patients have mucus in the stools. Largevolume diarrhea, bloody diarrhea, nocturnal diarrhea, and greasy stools are not compatible with IBS and raise the probability of organic disease. The Rome III criteria, a consensus statement on functional bowel disorders established for research purposes, may help establish the diagnosis of IBS as well as functional diarrhea. The criteria for IBS are recurrent abdominal pain or discomfort occurring on at least 3 days per month in the last 3 months and associated with two or more of following: (1) improvement of pain with bowel movement; (2) onset associated with a change in frequency of stool; and (3) onset associated with a change in form or consistency of stool.

decline of the immune response

Because of the remarkable expansion of antigenspecific lymphocytes at the peak of an immune response, it is predictable that once the response is over, the system must return to its steady state, called homeostasis, so that it is prepared to respond to the next infectious pathogen (see Fig. 5-12). During the response, the survival and proliferation of T cells are maintained by antigen, costimulatory signals from CD28, and cytokines such as IL-2. Once an infection is cleared and the stimuli for lymphocyte activation disappear, many of the cells that had proliferated in response to antigen are deprived of these survival signals. As a result, these cells die by apoptosis (programmed cell death). The response subsides within 1 or 2 weeks after the infection is eradicated, and the only sign that a T cell-mediated immune response had occurred is the pool of surviving memory lymphocytes. Numerous mechanisms have evolved to overcome the challenges that T cells face in the generation of a useful cell-mediated immune response: • Naive T cells need to find the antigen. This problem is solved by APCs that capture the antigen and concentrate it in specialized lymphoid organs in the regions through which naive T cells recirculate. • The correct type of T lymphocytes (i.e., CD4+ helper T cells or CD8+ CTLs) must respond to antigens from the endosomal and cytosolic compartments. This selectivity is determined by the specificity of the CD4 and CD8 coreceptors for class II and class I MHC molecules, and by the segregation of extracellular (vesicular) and intracellular (cytosolic) protein antigens for display by class II and class I MHC molecules, respectively. • T cells should respond to microbial antigens but not to harmless proteins. This preference for microbes is maintained because T cell activation requires costimulators that are induced on APCs by microbes. • Antigen recognition by a small number of T cells must lead to a response that is large enough to be effective. This is accomplished by robust clonal expansion after stimulation and by several amplification mechanisms induced by microbes and activated T cells themselves that enhance the response. • The response must be optimized to combat different types of microbes. This is accomplished largely by the development of specialized subsets of effector T cells.

Tolerance to commensal microbes and fetal antigens

Before concluding our discussion of the mechanisms of immunological tolerance, it is useful to consider two other types of antigens that are not self but are produced by cells or tissues that have to be tolerated by the immune system. These are products of commensal microbes that live in symbiosis with humans and paternally derived antigens in the fetus. Coexistence with these antigens is dependent on many of the same mechanisms that are used to maintain peripheral tolerance to self antigens. Foremost among these mechanisms are regulatory T cells.

Where does the midgut begin ? end at?

Begins just inferior to the major duodenal papilla, in the descending part of the duodenum ends at the junction between the proximal two-thirds and distal one-third of the transverse colon

Belatacept

Belatacept is a close structural congener o abatacept that has increased a f nity or B7-1 and B7-2. In a large clinical trial, belatacept was as e ective as cyclosporine at inhibiting acute rejection in renal transplant recipients. Belatacept is approved as an immunosuppressant or renal transplantation in patients who are seropositive or Epstein-Barr virus (EBV).

B-lyphocyte stimulator

Belimumab is a human monoclonal antibody directed against the B-lymphocyte stimulator (BLyS) cytokine. BLyS binding to normal B cells activates signaling cascades that stimulate cell survival and cell di erentiation into antibody- and autoantibody-producing cells. Belimumab blocks the normal function of BLyS, resulting in B-cell apoptosis. As mentioned previously, autoantibodies against sel -antigens are one mechanism o tissue injury and in ammation in systemic lupus erythematosus. Reducing the number o circulating B cells leads to a subsequent decrease in autoantibody production and reduced disease activity

Where do Fistulae develop in Crohns disease?

Between loops of bowel may also involve the urinary bladder, vagina, and abdominal or perianal skin Perforations and peritoneal abscesses are common.

Biologic agents

Biologics are a class of medications that target specific aspects of the immune system. The first such agent to be used in IBD was infliximab (Remicade), a chimeric monoclonal antibody to TNF-α, which has been shown to be effective in the treatment of both moderate to severe Crohn's disease, including fistulizing disease, and UC (level of evidence I, A). Because infliximab is a chimeric antibody, its toxicities include infusion reactions, delayed-type hypersensitivity reactions, and formation of autoantibodies (which can reduce its efficacy). Anti-TNF agents that are administered subcutaneously include adalimumab (Humira) and golimumab (Simponi), which are fully human monoclonal antibodies, and certolizumab pegol (Cimzia), which is a humanized anti-TNF antibody Fab fragment. Adalimumab and certulizumab are efficacious in patients with moderate to severe Crohn's disease, whereas adalimumab and golimumab are approved to treat moderate to severe UC Natalizumab (Tysabri), a humanized anti-α4-integrin antibody, blocks inflammatory cell migration and adhesion, and has been approved for the treatment of moderate to severe Crohn's disease in patients who have had an inadequate response to, or are unable to tolerate, conventional Crohn's disease therapies including inhibitors of TNF-α (level of evidence IIa, B for induction and IIa, A for maintenance). Vedolizumab (Entyvio), a humanized monoclonal antibody to α4β7 integrin, has been recently approved for the treatment and maintenance of both Crohn's and ulcerative colitis (level of evidence I, A). Because of the potent effects these biologic agents have on the immune system, careful patient selection and monitoring for complications are necessary. Reactivation of latent tuberculosis and other serious infections have been reported with the antiTNF agents. Other rare but serious complications include nonHodgkin's lymphoma, exacerbation of congestive heart failure, abnormal complete blood count (CBC) and liver function test results, and demyelinating disease. Natalizumab has been linked to rare cases of progressive multifocal leukoencephalopathy caused by the human JC virus Future biologic agents with alternate mechanisms of action are being discovered and developed. Ustekinumab (Stellara), an IL-12/IL-23 inhibitor already approved for psoriasis, has shown promise and is in clinical trials for Crohn's disease. Tofacitinib, a JAK inhibitor, already approved for rheumatoid arthritis, is now being investigated for Crohn's disease.

What is Melena?

Black, tarry, usually foul-smelling stools are most often a manifestation of upper GI bleeding A small bowel or proximal colonic source of bleeding may on occasion lead to melenic stools. Volumes as little as 50 to 100 mL of blood in the stomach can result in melena.

What does Acute Gastrointestinal Hemorrhage occur as a result of?

Bleeding occurs as a complication of many diverse disease processes, and adequate treatment depends on careful assessment and management that focuses on ensuring hemodynamic stability, determining blood loss, and identifying sources of bleeding.

What happens if the openings of the celiac trunk and superior mesenteric artery become narrowed?

Blood supply to the gut is diminished After a heavy meal, the oxygen demand of the bowel therefore outstrips the limited supply of blood through the stenosed vessels, resulting in severe pain and discomfort -mesenteric angina Patients with this condition tend not to eat because of the pain and rapidly lose weight- pain occurs after eating

What can Leukemia (primary immunodeficiency) and other conditions be treated with?

Bone marrow or peripheral stem cell transplantation In this procedure, hematopoietic and immune function is restored after the patient's bone marrow has been eradicated by aggressive chemotherapy and/or radiation therapy

What types of immunity do Antimetabolites affect?

Both Cell-mediated and humoral immunity rendering patients more susceptible to infection than would occur if only one of these immune systems were a ected.

If the urinary d-xylose excretion is abnormal what test may be used to diagnose bacterial overgrowth using glucose for the carbohydrate load?

Breath hydrogen test

Breath tests

Breath tests rely on bacterial degradation of luminal compounds, which releases metabolic byproduct gases (e.g., hydrogen, methane, carbon dioxide) that can be measured in the exhaled breath. In the case of disaccharidase deficiency, a specific disaccharide (e.g., lactose) that is orally ingested but not properly absorbed in the small intestine is delivered to the colon, where bacterial fermentation liberates metabolites; hydrogen gas is the marker assayed in the breath. In the presence of bacterial overgrowth of the small intestine, orally ingested glucose ferments in the proximal small bowel (instead of being absorbed), resulting in increased hydrogen in the breath; here, the timing of exhaled hydrogen aids in the diagnosis. The measurement of radioactive carbon dioxide in the breath after ingestion of a nutrient labeled with carbon 14 (14C) has been used to estimate the malabsorption of fat or bile acids and for measurement of bacterial overgrowth (14C-xylose). The radioactive tests are cumbersome, and their usefulness in clinical practice is limited. The overlap of symptoms and the large number of diagnostic tests available for evaluation of malabsorption necessitate the use of a systematic approach and a rational algorithm (see Fig. 33-3). The most accurate test for fat malabsorption remains the 72-hour fecal fat analysis; however, the test is difficult to carry out in clinical practice. Surrogate screening for steatorrhea is done with the qualitative stool fat examination (Sudan stain) and measurement of serum carotene. If the stool fat content is normal, the patient may still have selective impairment of absorption of a specific carbohydrate. This latter condition should be suspected if the primary symptoms are cramps, flatulence, and diarrhea. The most common example of carbohydrate malabsorption is lactose intolerance; specific tests include the oral lactose tolerance test, but measurement of breath hydrogen is more sensitive and more specific. More generally, an osmotic gap in fecal water suggests a dietary (rather than a secretory) cause of the diarrhea related to luminal short-chain fatty acids or carbohydrates. The osmotic gap is calculated by the following formula: The osmotic gap is not calculated by directly measuring stool osmolality because it increases with time in the specimen container. In addition, luminal osmolality is equal to serum osmolality because the colon cannot establish a gradient against the serum concentration of solutes. When fat malabsorption is demonstrated (>6 g/24 hours, or increased qualitative stool fat and decreased serum carotene), a d-xylose absorption-excretion test should be performed next. A normal d-xylose test result makes diffuse mucosal disease unlikely and suggests maldigestion, principally pancreatic enzyme or bile salt deficiency. Clues to chronic pancreatitis include a history of alcohol abuse or previous episodes of pancreatitis. Unusual causes of pancreatic malabsorption, such as cystic fibrosis, microlithiasis, or drug toxicity, require specific testing and a detailed history. Serum enzyme tests and abdominal imaging (plain filmsor, with much greater sensitivity, abdominal CT scans) can be obtained next to identify pancreatic disease. If the urinary d-xylose excretion is abnormal, the breath hydrogen test may be used to diagnose bacterial overgrowth using glucose for the carbohydrate load. If no bacterial overgrowth is present, a mucosal biopsy should be performed (see Table 33-5). Imaging studies of the small bowel may be helpful on occasion.

antibody drug conjugates

Brentuximab vedotin is a chimeric human monoclonal antibody-drug conjugate directed against CD30. CD30 is a membrane glycoprotein in the TNF receptor amily that signals through multiple mechanisms, including the NF B pathway, to promote cell proli eration and survival. CD30 is expressed on activated CD4 and CD8 T cells and on B cells. It is also highly expressed on Reed-Sternberg cells in Hodgkin's lymphoma and on anaplastic large cell lymphoma cells. The specif c role o CD30 in the pathogenesis o lymphoma is under investigation. In brentuximab vedotin, the anti-CD30 antibody is linked to the antimitotic drug monomethyl auristatin E (MMAE) through a valine-citrulline dipeptide. This linker dipeptide is enzymatically cleaved a ter endocytosis at the target site to release MMAE into the cytoplasm. MMAE prevents microtubule polymerization, causing cell cycle arrest in the G2 to Brentuximab vedotin is approved or the treatment o relapsing Hodgkin's lymphoma a ter ailure o multi-agent chemotherapy or ailure o autologous stem cell transplant. It is also approved or systemic anaplastic large cell lymphoma a ter ailure o at least one multi-agent chemotherapy regimen. Additional anti-CD30 monoclonal antibodies, bispecif c antibodies, and antibody-drug conjugates are under active investigation or the treatment o Hodgkin's lymphoma

What effects do Glucocorticoids have?

Broad anti-inflammatory effects

What does Antithymocyte globulin (ATG) result in?

Broad immunosuppression that can predispose to infection.

What types of ulcers are involved in UC?

Broad-based ulcers Grossly, involved colonic mucosa may be slightly red and granular

Where does the final hydrolysis to glucose monomers occur?

Brush border includes hydrolysis by sucrase and lactase Glucose and galactose are actively transported in conjunction with sodium, whereas fructose absorption occurs by facilitated diffusion About one half of dietary energy is derived from carbohydrate, with a nutritional goal of 55% and an increased component of insoluble fiber (i.e., that which is indigestible by mammalian enzymes but variably broken down by colonic bacteria.)

What is a hallmark of Steatorrhea?

Bulky oily stools resulting from fat malabsorption

Sarah goes to her PCP with a primary complaint of bloody diarrhea with stringy, mucoid material. When asked if she has any other complaints, she explains that she has skin lesions on her arms. History reveals that she has recently quit smoking and has been very stressed since. What will be shown upon histological examination to confirm diagnosis of Ulcerative Colitis? A) Broad based ulcers and skip lesions B) Mucosal atrophy and aphthous ulcer C) Broad based ulcers and pseudopolyps D) Skip lesions and mucosal atrophy E) Pseudopolyps and granulomas

C) Broad based ulcers and pseudopolyps

A 21-year-old female presents to the Emergency Room with abdominal pain, nausea, diarrhea, fever and right lower abdominal pain. She has a recent history of some eye irritation. Upon physical examination, she appears to have clubbing of her fingertips, as well as skin tender lumps on the front of her legs, below the knees. Patient had a positive fecal occult blood test, indicating the presence of blood in her stool. Which of the following is a feature associated with the patient's disease? A) Thin intestinal wall appearance B) Superficial, broad-based ulcers C) Fat and vitamin malabsorption D) Inflammation limited to the mucosa E) Toxic megacolon

C) Fat and vitamin malabsorption

A 15-year-old female presents with fever, abdominal pain, and bloody diarrhea. She is iron deficient and upon colonoscopy, fibrosing strictures are observed. A NOD2 polymorphism is noted. Which of the following conditions is she diagnosed with? A) Ulcerative colitis B) Indeterminate colitis C) Microscopic colitis D) Crohn's disease E) Sigmoid diverticular disease

C) Microscopic colitis

A 21 year old female presents to the gastroenterologist upon referral from her primary care physician. She notes reduced appetite and weight loss, fatigue, severe abdominal cramping, bloody stool, and vomiting about an hour after eating. After ruling out other possible medical conditions, she was diagnosed with Chron's Disease. Which of the following genes may be associated with her condition? A) PTPN22 B) CTLA4 C) NOD2 D) CD25 E) IL23R

C) NOD2

A 25-year-old male presents to his primary care physician with complaints of frequent diarrhea, bloating, and left lower quadrant pain that has been occurring for the past 3 months. Based on the patient's history and clinical findings, the physician suspects a diagnosis of irritable bowel syndrome (IBS). According to the Rome III criteria, what must be true in order to conclude that the patient suffers from IBS? A) Diarrhea is greasy or oily, foul smelling B) Diarrhea is of large volume C) Pain improves with bowel movement D) Pain is more severe on the right side E) Weight loss and anemia are seen

C) Pain improves with bowel movement

A 34 year old patient comes in complaining about mild cramping, episodes of bloody mucus and intermittent diarrhea with fewer than 4 stools/ day. The physician decides to order a endoscopic exam and an Antibody test. During the endoscopic procedure, he notices superficial ulcerations and pseudopolyp in the intestines and has a suspicion it is a mild ulcerative colitis. He confirms his diagnosis through the antibody test, which came out positive for pANCA. What treatment should be given to this patient with mild ulcerative colitis? A) Hydrocortisone w/ Ca2+ and Vitamin D supplement B) Antibiotics C) Sulfasalazine w/ Folic acid supplement D) Cyclosporine E) Adalimumab

C) Sulfasalazine w/ Folic acid supplement

A 42 year old male presents to their PCP with symptoms resembling diabetes mellitus. The patient described that these symptoms began a few weeks after an organ transplant. He was prescribed an immunosuppressant drug that inhibits IL-3, IL-4, IFN-y and TNF-a production in vitro, as well as inhibiting cell-mediated immunity without suppressing B-cell or his natural killer cell functions. Which of the following drugs was prescribed? A) Etanercept B) Leflunomide C) Tacrolimus D) Anakinra E) Azathioprine

C) Tacrolimus

A 24-year-old male is fully recovered from a recent coronavirus infection. Which of the following is true about his memory T cells? A. The memory T cells require IL-23 stimulation to say alive B. A subset of the memory T cells will proliferate in the bone marrow C. A subset of the memory T cells will localize in mucosal and peripheral tissues D. Memory T cells will only be present during an infection E. Memory T cells will be induced to proliferate within 3-4 days of encountering the antigen

C. A subset of the memory T cells will localize in mucosal and peripheral tissues

Amanda is a 32 year old who presents to her primary care physician with small, painful lumps under the skin in her armpits. She is referred to a dermatologist who diagnoses her with hidradenitis suppurtiva. To treat her condition, her physician prescribes a fully human IgG1 monoclonal antibody directed against TNF-a. Which drug was prescribed to Amanda? A. Etanercept B. Infliximab C. Adalimumab D. Basiliximab E. Certolizumab

C. Adalimumab

A 72 patient presents to the clinic with abdominal pain. The physician decides to run an angiography and finds that the splenic artery has a great deal of atherosclerosis so there is limited blood flow through it. However, the greater curvature of the stomach is still receiving adequate blood flow, why is this? A. Anastomoses from the left gastric artery also supplies the greater curvature of the stomach B. Anastomoses from the IMA also supplies the greater curvature of the stomach C. Anastomoses from the common hepatic artery also supply the greater curvature of the stomach D. The splenic artery has nothing to do with the greater curvature of the stomach E. Anastomoses from the subclavian artery will supply the greater curvature of the stomach

C. Anastomoses from the common hepatic artery also supply the greater curvature of the stomach

In T cells there is a long-lived functional inactivation that occurs when cells recognize antigens without adequate levels of costimulators. What is this process of functional inactivation called and what antigen stimulating complex do T cells use? A. Anergy, BCR complex B. Proliferation, TCR complex C. Anergy, TCR complex D. Suppression, BCR complex E. Proliferation, T cell complex

C. Anergy, TCR complex

A graduate student researching Naive T cell differentiation observes an unknown microbe has caused a spike in T cells. Within 3 to 4 days of exposure, the student observes an increase in effector cells, specifically Th1, Th2, and Th17. Based on these observations, the graduate student concludes the infection spread to peripheral tissue. Which of the following correctly classifies the effector cells that played a role in this cell-mediated immunity? A. Genes encoding cytokines B. CD8+ T cell C. CD4+ T cell D. Genes encoding cytotoxic proteins E. Both A and C

C. CD4+ T cell

A 26-year-old female presents to the emergency department with acute upper abdominal pain. The patient states the pain was intermittent when it onset a couple hours ago but is now steady. She also states that the pain is radiating to her right upper back. On physical exam, the pain is noted to be epigastric and localized in the right upper quadrant, specifically radiating to her right scapula. Prior to ordering imaging and labs, what is the most likely differential diagnosis? A. Appendicitis B. Pancreatitis C. Cholecystitis D. Perforation E. Obstruction

C. Cholecystitis

Michael, a 39-year-old male visits his local doctor for the 2nd time in 2 weeks because of his ongoing stomach problems. On his first visit he was diagnosed with an Inflammatory Bowel Disease (IBD) and prescribed 5-Aminosalicylic Acid, which he says seems to not be working. Given that the 5-Aminosalicylic Acid does not treat his symptoms what would be the next type of drug he would be given in order to treat his IBD? A. Azathioprine B. 5-Aminosalicylic Acid C. Corticosteroids D. Infliximab E. 6-mercaptopurine (6-MP)

C. Corticosteroids

A 32-year-old male presented to his primary care provider with a several month history of reduced flexibility in his spine that has been associated with pain. Imaging confirms the diagnosis of ankylosing spondylitis. Which MHC allele locus has been associated with increased risk of developing this autoimmune disease? A. HLA-DRB1*01 B. HLA-DR4 C. HLA-B27 D. HLA-DRB1*0301 E. HLA-DR3

C. HLA-B27

A patient presents to the emergency room stating that they have been vomiting a mixture of "coffee grounds" and bright red blood. The attending physician examines the nasal passages along with the patient's nasopharynx and doesn't find evidence of a bleed. The patient informs the physician that they have not had a cough and hemoptysis is ruled out. Which of the following is not a likely cause of the bleed in this patient? A. Duodenal ulcer B. Esophageal varices C. Ileal ulcer D. Atrio-esophageal fistula E. Gastric ulcer

C. Ileal ulcer

A 28-year-old male patient who has been suffering from a GI ulcer is told by his specialist that his ulcer has a Forest 1 rating. He is told this puts him at a high risk for continued or recurrent bleeding, but there are a few therapies that may help. After researching his options, he elects to have thermal therapy by electrocautery performed. Which of the following is NOT a reason why these beneficial endoscopic therapies are performed? A. Decreased rates of rebleeding B. Decreased mortality rates C. Increased need for transfusions D. Decreased need for surgery E. Decreased length of hospital stay

C. Increased need for transfusions

A 55-year-old man comes into the clinic complaining of joint pain, swelling, and redness. After further testing, it is shown that he has elevated rheumatoid factor. He is prescribed a partially humanized mouse monoclonal antibody directed against a cytokine that is central to many aspects of inflammatory diseases and implicated in numerous autoimmune diseases. What drug was he most likely prescribed and against which cytokine does it exert its action? A. Leflunomide and TNF-β B. Etanercept and TNF-α C. Infliximab and TNF-α D. Anakinra and IL-1 E. Abatacept and B7

C. Infliximab and TNF-α

A 53 year-old female presents to the emergency department visibly jaundiced. History shows a kidney transplant two years ago. It is revealed that the procedure was uneventful, she responded well to immunosuppressant therapy, and has had no prior major complication with the graft tissue. She is diagnosed with renal failure caused by tissue rejection of the grafted kidney, and is scheduled for dialysis on the same day. Visibly frustrated, she initially refuses, stating that the main reason she underwent surgery was to eliminate the need for dialysis. In counseling her on this matter, which of the following likely describes part of the process that lead to her kidney failure? A. Her B-lymphocytes became synthesized by a grafted tissue antigen, eventually producing enough antibodies to cause tissue rejection. B. Pre-formed antibodies in her circulation immediately began attacking with the graft tissue. C. Macrophages played a role in inducing chronic inflammation of the tissue, leading to eventual scarring. D. This dialysis is necessary to stabilize the patient before she is treated for the chronic tissue rejection she is experiencing. E. The patient's cytotoxic T-cells attacked the grafted organ's endothelial tissue, leading to vascular damage.

C. Macrophages played a role in inducing chronic inflammation of the tissue, leading to eventual scarring.

A 21 Male patient presents at clinic in order to establish care. Upon going over medical history it is noted the patient has Crohn's disease. Inherited risk for most autoimmune diseases is attributable to multiple gene loci, of which the largest contribution is made by MHC genes. Family and Linkage studies have shown a greater likelihood of developing certain autoimmune diseases in persons who inherit particular MHC alleles. Polymorphisms in non-HLA genes are also associated with various autoimmune diseases. Lastly, single gene defects can also cause autoimmunity these are referred to as mendelian diseases. Which of the following is associated with this autoimmune disease? A. CD25 B. AIRE C. NOD2 D. HLA-B27 E. PTPN22

C. NOD2

Frank, 46 years old, presents to his primary care physician complaining of abdominal pain and diarrhea for the past two months. He describes the pain as a cramping and aching, more localized on the right side near his belly button. He also reports seeing blood in his stool about 4-5 times and has unexpectedly lost 12 pounds since the symptoms began. Frank has never experienced this before and has not tried any medication to alleviate the symptoms. He denies any illicit drug use and abstains from any alcoholic beverages. Based on his history, his physician believes that Frank may be experiencing a condition known as Crohn's Disease. However, the physician would like to order further tests before prescribing him medication. Which of the following would deem the physician's diagnosis incorrect? A. Frank stopped smoking cigarettes because it worsened his symptoms B. Serology tests are positive for ASCA C. Radiologic findings of the lower GI tract show a tubular appearance resulting from loss of haustral folds D. MRI illustrates an anal fistula E. Endoscopic findings reveal cobblestoning

C. Radiologic findings of the lower GI tract show a tubular appearance resulting from loss of haustral folds

Which of the following mechanisms that have evolved to overcome the challenges that T cells face in the generation of a useful cell-mediated immune response is incorrect? A. Naive T cells need to find the antigen B. The correct type of T lymphocytes must respond to antigens from the endosomal and cytosolic compartments C. T cells should respond to microbial antigens and to harmless proteins D. Antigen recognition by a small number of T cells must lead to a response that is large enough to be effective E. The response must be optimized to combat different types of microbes

C. T cells should respond to microbial antigens and to harmless proteins

A patient is prescribed a prodrug inhibitor of IMPDH for a kidney transplant. Which of the following is false regarding this drug? A. This drug affects lymphocytes by inhibiting de novo purine synthesis and not salvage purine synthesis B. This drug's active metabolite inhibits only type II IMPDH C. This drug reduces guanosine which regulates endothelial NOS D. The patient may feel GI discomfort E. This drug is used for chronic rejection by inhibiting lymphocytes as well as smooth muscle proliferation

C. This drug reduces guanosine which regulates endothelial NOS

A 21-year-old male presents to the ER with complaints of diarrhea, bloating and pain, weight loss, dizziness, and fatigue. When asked about his diet, the patient reports that he usually eats burgers, fries and pasta at his school's dining halls. The patient confirms that he does drink beer but stopped recently because his symptoms seemed to get worse after drinking. The patient is put on a saline IV to treat his dehydration while the attending physician orders a genetic test, which ultimately finds the gene HLA-DQ8. The physician determines that the patient may be suffering from a malabsorptive disease that has led to mucosal damage and the current symptoms. What is the autoantigen of this particular disease? A. Alpha-3 subunits of Type IV collagen B. Thyroid-stimulating hormone receptor C. Tissue transglutaminase D. Pancreatic beta-cell antigen E. Myelin basic proteins, proteolipid proteins

C. Tissue transglutaminase

A 50-year-old male presents to the clinic with RLQ abdominal pain and fever. After taking a detailed medical history and performing a physical exam, findings show that the patient has lost 20 lbs. within the last 3 months and has palpable abdominal tenderness. The physician orders a colonoscopy and histological findings include transmural inflammation and numerous abscesses. Which of the following symptoms would be least associated with their illness? A. Bloody Diarrhea B. Deep Ulcers C. Tubular histological appearance D. Exacerbation of symptoms with consumption of tobacco E. ASCA

C. Tubular histological appearance

What are mature B cells characterized by the expression of?

CD19 or CD20

What do Memory T cells express that naive T cells dont?

CD25

What receptor does IL-2 interact with on activated T cells? What does it mediate?

CD25 mediates the early steps in T cell activation

What T cell recognizes MHC II?

CD4 T cells

Are the majority of regulatory T cells CD4 or CD8 T cells? What do they express high levels of?

CD4 T cells CD25, the α chain of the interleukin-2 (IL-2) receptor. They also express a transcription factor called FoxP3, which is required for the development and function of the cells

Which MHC do CD4 and CD8 T cells recognize respectively?

CD4 T cells - MHC II CD8 T cells - MHC I

What are Th17 cells generated from?

CD4 T cells when these cells are stimulated by IL-23 Drugs that block the maturation or growth of Th17 cells are available or clinical use in the treatment of certain autoimmune diseases.

What may the differentiation of naive CD8+ T cells into fully active cytotoxic T lymphocytes (CTLs), and into memory cells require?

CD4+ Helper T cells The CD4+ T cells may produce cytokines or membrane molecules that help to activate the CD8+ T cells

What is PD-1 expressed on?

CD4+ and CD8+ T cells after antigen stimulation

What is the most important cell surface protein involved in the effector function of CD4 T cells?

CD40 ligand The CD40L gene is transcribed in CD4+ T cells in response to antigen recognition and costimulation, and so CD40L is expressed on activated helper T cells The interaction of CD40L on T cells with CD40 on dendritic cells increases the expression of costimulators on these APCs and the production of T cell-stimulating cytokines, thus providing a positive feedback (amplification) mechanism for APC-induced T cell activation

What role do CD40l and CD40 play?

CD40 ligand (CD40L, or CD154) on activated T cells and CD40 on APCs. These molecules do not directly enhance T cell activation. Instead, CD40L expressed on an antigen-stimulated T cell binds to CD40 on APCs and activates the APCs to express more B7 costimulators and to secrete cytokines (e.g., interleukin-12 (IL-12) that enhance T cell differentiation Thus, the CD40L-CD40 interaction promotes T cell activation by making APCs better at stimulating T cells.

What other mediator do activated T cells express?

CD40 ligand (CD40l) CD40 is expressed on antigen-presenting cells, including B cells and macrophages

Antigens presented by MHC I are recognized by what T cell?

CD8 T cell

Cells of the immune system

CELL TYPE FUNCTION Innate Immunity Macrophage Tissue-resident cell, derived from monocyte Phagocytoses cellular and foreign debris Involved in chronic inflammation Antigen-presenting cell Dendritic cell Transports and presents antigen to T cells in lymph nodes Antigen-presenting cell Neutrophil Phagocytoses and kills invading pathogens, especially bacteria Eosinophil Defends against parasites Basophil/mast cell Release histamine, leukotrienes, and other mediators after exposure to antigen Adaptive Immunity Cytotoxic T cell (TC) Effector of cellular adaptive immunity Helper T cell (TH) Controls adaptive immune responses B cell Synthesizes and secretes antibody Antigen-presenting cell

CI and ADRs: infliximab

CI: Hypersensitivity to infliximabmoderate to severe heart failure (infliximab > 5mg/kg) ADRs: Similar to etanercept; GI upset *Partially humanized mouse antibody against human TNF-alpha*

CI and ADRs: natalizumab

CI: Hypersensitivity to natalizumab; history of progressive multifocal leukoencephalopathy or existing PML ADRs: progressive multifocal leukoencephalopathy; bowel obstruction; hepatotoxicity; herpes enphalitis and meningitis; depression, suicidal ideation; pneumonia; rash; GI upset; arthralgia; headache; UTI; fatigue

CI and ADRs: etanercept

CI: Sepsis ADRs: Heart failure, myelosuppression, necrotizing fasciitis, Stevens-Johnson syndrome, toxic epidermal necrolysis, hepatotoxicity, optic neuritis, reactivation of tuberculosis, increased risk of infection or malignancy, demyelinating disease of CNS

CI and ADRs: brentuximab vedotin

CI: concominant use of bleomycin due to pulmonary toxicity ADRs: supraventricular arrhythmias, Steven-Johnson syndrome, anemia, thrombocytopenia, neutropenia, neuropathy, leukoencephalopathy, pyelonephritis, pneumothorax, pulmonary embolism, pneumonitis, tumor lysis syndrome CD30 is expressed on CD4 and CD8 T cells, B cells, and lymphoma cells the anti-CD30 is linked to another drug (MMAE) which disrupts microtubule formation and leads to cell cycle arrest. MMAE is also a CYP3A4/5 inhibitor

CI and ADRs: mycophenolate mofetil (MMF)

CI: hypersensitivity to MMF, hypersensitivity to polysorbate 80 (IV formulation) ADR: GI hemorrhage, leukopenia, myelosuppression, increased risk of infection or lymphoma, leukoencephalopathy, pleural effusion, pulmonary fibrosis, hypertension, hypercholesterolemia *don't use with iron*; *this is a prodrug*

CI and ADRs: anakinra

CI: hypersensitivity to anakinra or E.coli derived proteins ADRs: cardiorespiratory arrest, neutropenia, increased risk of infection or malignancy, injection site reaction reduces bony erosions, possibly by decreasing metalloproteinase release from synovial cells avoid administration with live vaccines

CI and ADRs: cyclosporine

CI: hypersensitivity to cyclosporine, abnormal renal function, uncontrolled hypertension, malignancies (rheumatoid arthritis and psoriasis patients), concominant PUVA or UVB therapy, methotrexate, immunosuppressive agents, colar tar, radiation therapy (psoriasis patients), active ocular infection (topical cyclosporine) ADRs: hyperkalemia, hypomagnesemia, nephrotoxicity, neurotoxicity, hepatotoxicity, hypertension, gingival hyperplasia, hirsutism *rifampin and St. Johns wort decrease plasma levels*

CI and ADRs: prednisone

CI: hypersensitivity to drug, systemic fungal infection ADR: Cushing's syndrome, pulmonary toxicity, increased risk of infection, impaired wound healing, psychotic disorder, cardiotoxicity, seizure, muscle atrophy, decreased body growth, electrolyte imbalance, thromboembolic disorder *short DOA*

CI and ADRs: eculizumab

CI: hypersensitivity to eculizumab; Neisseria meningitidis infection; no vaccination against N. meningitidis ADRs: infections, leukopenia, anemia, hypertension, GI upset, headache, insomnia, nasopharyngitis, fever

CI and ADRs: leflunomide

CI: hypersensitivity to leflunomide, pregnancy ADR: Stevens-Johnson syndrome, toxic epidermal necrolysis, panctyopenia, hepatotoxicity, interstitial lung disease, alopecia, rash, mouth ulcer, dizziness *enterohepatic circulation - prolonged effect*

CI and ADRs: methotrexate

CI: hypersensitivity to methotrexate, pregnancy, breastfeeding, patients with psoriasis or rheumatoid arthritis who have: alcoholism, alcoholic liver disease, chronic liver disease, preexisting blood dyscrasia, or laboratory evidence of immunodeficiency syndrome ADRs: thromboembolic disorder, Steven-Johnson syndrome, myelosuppresion, hepatotoxicity, kidney disease, interstitial pulmonary disease, leukoencephalopathy, seizure, malignant lymphoma, opportunistic infection *v. toxic to fetus, avoid with alcohol, avoid w. naproxen, avoid w polio vaccine*

CI and ADRs: tacrolimus

CI: hypersensitivity to tacrolimus, hypersensitivity to hydrogenated castor oil (IV formulation) ADRs: prolonged QT interval, atrial fibrillation, heart failure, diabetes mellitus, hyperkalemia, hypomagnesemia, GI perforation, hepatotoxicity, neurotoxicity, acute respiratory distress syndrome, increased risk of lymphoma or infection *rifampin and St. Johns wort decrease plasma levels*

CI and ADRs: rituximab

CI: none ADRs: anemia, significant immunosuppression, leukoencephalopathy, tumor lysis syndrome, bowel obstruction, hepatitis, infusion reaction, cardiac arrhythmia, heart failure, Stevens-Johnson syndrome, toxic epidermal necrolysis, keratitis, nephrotoxicity, pulmonary fibrosis, angioedema avoid administration of live vaccinesanti-CD20 monoclonal antibody (CD20 is expressed on surface of all mature B cells); causes profound depletion of circulating B cells

CI and ADRs: abatacept

CI: none ADRs: cellulitis, sepsis, pyelonephritis, pneumonia, acute exacerbation of COPD, susceptibility to malignancy, nausea, headache, UTI, nasopharyngitis, URI

CI and ADRs: Ipilimunab

CI: none ADRs: pericarditis, endocrine disorder, enterocolitis, eosinophilia, anemia, hepatotoxicity, myositis, encephalitis, neuropathy, renal failure, iritis, uvetitis, pneumonitis, rash diarrhea, fatigue

CI and ADRs: azathioprine

CI: pregnancy, hypersensitivity to azathioprine, rheumatoid arthritis with prior treatment with alkylating agents ADR: pancreatitis, myelosuppression, hepatotoxicity, increased risk of infection or malignancy, pericarditis, leukoencephalopathy *toxicity increased by allopurinol*

CI and ADRs: belimumab

CI: previous anaphylaxis to belimumab ADRs: susceptibility to infection or malignancy, infusion reaction, leukoencephalopathy, depression fully human monoclonal antibody that inhibits B lymphocyte stimulator which is required for the survival for B cells BLyS function - binds to normal B cells and activates signaling cascades that stimulate cell survival and cell differentiation into autoantibody-producing cells

What inhibitory receptor plays an important role in the suppressive function of regulatory T cells (T-reg)?

CTLA-4 Because these inhibitory receptors evolved to prevent immune responses against self antigens, genetic deletion or blockade of these molecules in mice and humans results in systemic autoimmune disease

What does Abatacept consist of? What does it complex with?

CTLA-4 fused to an IgG1 constant region complexes with costimulatory B7 molecules on the surface of antigen-presenting cells

Anti- CD52 mAB

Campath-1 (CD52) is an antigen expressed on most mature lymphocytes and on some lymphocyte precursors. An antibody against this antigen was originally tested in rheumatoid arthritis and ound to cause prolonged and sustained depletion o all T cells, o ten lasting or years. The reason or the sustained lymphocyte depletion is unknown. Anti-CD52 mAb therapy did lead to some improvement in the symptoms o arthritis; however, the sustained depletion o lymphocytes and concern about in ections precluded urther study o this antibody in autoimmune conditions. Under the generic name alemtuzumab , anti-CD52 monoclonal antibody had been approved as an adjunctive therapy in the treatment o B-cell chronic lymphocytic leukemia—a condition in which sustained suppression o the leukemic cells is desirable. However, alemtuzumab has been voluntarily withdrawn rom the market in the United States and Europe

Why should Antibiotics be avoided in patients infected with Enterohemorrhagic E Coli (EHEC)?

Can lead to an increased risk of hemolytic-uremic syndrome related to increased release of Shiga toxin These patients often have bloody diarrhea and abdominal pain but no fever

What are antimetabolites widely used in the treatment of?

Cancer

What do Bloating (abdominal distention) and soft diarrheal movements occur as a result of?

Carbohydrate malabsorption

malabsorptive therapy

Cardiovascular disease and other consequences of obesity have reached epidemic proportions in the United Sates, and one approach to this problem has been the deliberate induction of malabsorption (primarily of fats) to reduce a patient's lipid levels and body mass index. Medications used for this purpose includes bile acid-binding resins, such as cholestyramine and colestipol, and the lipase inhibitors, orlistsat (Xenical) and ezetimibe (Zetia). Surgical treatment (bariatric operations) usually consists of gastric partition combined with some degree of small intestinal bypass, which induces significant weight loss by several proposed mechanisms, including malabsorption, improved nutrient deposition, and enhanced satiety.

What cases are considered as Intdeterminante colitis?

Cases where a definitive diagnosis is not possible between Ulcerative colitis and Crohns disease Because of the extensive pathologic and clinical overlap between ulcerative colitis and Crohn disease definitive diagnosis is not possible in approximately 10% of IBD patients

What is the first part of the large intestine?

Cecum It is inferior to the ileocecal opening and in the right iliac fossa

What is continuous with the ascending colon at the entrance of the ileum?

Cecum and is usually in contact with anterior body wall It may cross the pelvic brim to lie in the true pelvis.

What does the large intestine consist of?

Cecum, appendix, colon, rectum, and anal canal

What disease is the leading consideration in someone with malabsorption syndrome?

Celiac disease

celiac disease

Celiac disease (also called celiac sprue, nontropical sprue, or gluten-sensitive enteropathy) is characterized by intestinal mucosal injury resulting from gluten-related immunologic damage in persons genetically predisposed to this condition. The prevalence of the disease among relatives of patients with celiac disease is approximately 10%. There is a strong association of celiac disease with human leukocyte antigen (HLA) class II molecules, particularly HLA-DQ2 and HLA-DQ8. The disease is induced by exposure to storage proteins found in grain plants such as wheat (which contains gliadin), barley, and rye and their products. Oats are implicated, not because of gliadin, but because of contamination with wheat during packaging and transportation. The exposure initiates a cellular immune response that results in mucosal damage, particularly in the proximal intestine. Results of investigations suggest that an enzyme, tissue transglutaminase, may be the autoantigen of celiac disease.

What is the Clinical presentation of Celiac disease?

Celiac disease can manifest with the classic constellation of symptoms and signs of a malabsorption syndrome Manifestation is atypical, with nonspecific GI symptoms such as bloating, chronic diarrhea (with or without steatorrhea), flatulence, lactose intolerance, or deficiencies of a single micronutrient (e.g., iron deficiency anemia Extraintestinal complaints such as depression, weakness, fatigue, arthralgias, osteoporosis, or osteomalacia may predominate.

clinical presentation

Celiac disease can manifest with the classic constellation of symptoms and signs of a malabsorption syndrome. Not uncommonly, however, the manifestation is atypical, with nonspecific GI symptoms such as bloating, chronic diarrhea (with or without steatorrhea), flatulence, lactose intolerance, or deficiencies of a single micronutrient (e.g., iron deficiency anemia). Extraintestinal complaints such as depression, weakness, fatigue, arthralgias, osteoporosis, or osteomalacia may predominate. A number of diseases, including dermatitis herpetiformis, type 1 diabetes mellitus, autoimmune thyroid disease, and selective immunoglobulin A (IgA) deficiency, are found in significant association with celiac disease.

Diagnosis

Celiac disease is a leading consideration in every patient with the malabsorption syndrome, and it should be included as well in the differential of atypical manifestations. Fiberoptic or capsule endoscopy may show the typical features of broad and flattened villi; with the former instrument, tissue can be sampled for histologic analysis. Intestinal biopsy is the most valuable test in establishing the diagnosis. The spectrum of pathologic changes ranges from normal villous architecture with an increase in mucosal lymphocytes and plasma cells (the infiltrative lesion) to partial blunting or total villous flattening. Although abnormal biopsy findings are not specific, they are highly suggestive, particularly because most other conditions that can mimic celiac disease (e.g., Crohn's disease, gastrinoma, lymphoma, tropical sprue, graft-versus-host disease, immune deficiency) may be distinguished clinically. A clinical response to a gluten-free diet establishes the diagnosis and precludes the need, in adults, to document healing by repeated biopsies. Serologic blood tests (antigliadin, antiendomysial, and reticulin antibodies) are helpful in screening of patients with atypical symptoms and asymptomatic relatives of patients with celiac disease

What is a major role of T lymphocytes?

Cell mediated immunity provides defense against infections by intracellular microbes. In several types of infections, microbes may find a haven inside cells, from where they must be eliminated by cell-mediated immune responses

What does increased vascular permeability allow?

Cells in the blood to enter the interstitium

innate immunity

Cells o the innate immune system are the rst responders to an o ending agent that has penetrated the skin or another barrier (Table 42-1). Innate immune cells per orm three important tasks. First, these cells de end against bacterial and parasitic in ections, either by neutralizing the in ectious agent with secreted cytotoxic proteins or by phagocytosis (engul ng) o the bacterium or parasite. Second, phagocytosis o the o ending agent initiates proteolytic digestion o microbial macromolecules to ragments (antigens) that are then displayed, together with major histocompatibility complex (MHC) class II proteins, on the sur ace o antigenpresenting cells. In turn, these antigen-presenting cells, which include macrophages and dendritic cells, activate cells o the adaptive immune system. Third, innate immune cells secrete numerous cytokines (see below) that urther ampli y the immune response. The major cell types o the innate immune system include granulocytes (neutrophils, eosinophils, and basophils), mast cells , and antigen-presenting cells (macrophages and dendritic cells). Some immunologists consider natural killer (NK) cells, NK T cells, and T cells to have innate immune roles; the biology o these cell types is beyond the scope of this text. Granulocyte is a descriptive term based on the appearance o the cytoplasmic granules within these cells. Neutrophils , the most abundant cell type o the innate immune system and the " rst responders" in inf ammation, are phagocytic cells primarily responsible or de ense against bacterial in ection. These cells envelop invading bacteria in phagocytic vesicles and destroy the bacteria within these vesicles using enzymes such as myeloperoxidase. Eosinophils are circulating granulocytes primarily involved in de ense against parasitic in ections. Because parasites are o ten too large to engul , eosinophils attach to a parasite's exterior and secrete cytotoxic substances directly on the parasite. Both basophils (circulating) and mast cells (tissue-resident) bind IgE antibody, display this IgE on the cell sur ace, and maintain histamine-containing granules that are released when exogenous antigen binds to and cross-links the IgE. Basophils and mast cells are important in allergic responses. Eosinophils and basophils are so named because they exhibit eosinophilic and basophilic patterns, respectively, when stained with Wright-Giemsa stain.

Acute rejection has both?

Cellular and Humoral components

Chronic rejection is possibly both?

Cellular and humoral in nature

What is the primary symptoms of obstruction?

Central abdominal, intermittent, colicky pain as the peristaltic waves try to overcome the obstruction

Central tolerance in immature B lymphocytes

Central tolerance in immature B lymphocytes. An immature B cell that recognizes self antigen in the bone marrow changes its antigen receptor (receptor editing), dies by apoptosis (negative selection, or deletion), or reduces antigen receptor expression and becomes functionally unresponsive

What is Pyoderma gangrenosum characterized by? treatment?

Characterized by a discrete ulcer with a necrotic base, usually on the legs The ulcer may spread and become large and deep, destroying soft tissues. Pyoderma parallels the bowel activity in 50% of cases. Treatment is usually with systemic or intralesional steroids, or both. Other treatment options include dapsone, cyclosporine, and the anti-TNF agents.

What is immune cell targeting accomplished by?

Chemotaxis - chemical signaling Inflammatory mediators released at the site ofinjury, such as N- formyl peptides derived from bacterial proteins or endogenous mediators such as C5a and leukotriene B 4 (LTB 4 ), create a chemical gradient to which the leukocytes respond, allowing them to crawl preferentially toward the site of the inflammatory reaction.

What is Brentuximab vedotin?

Chimeric human monoclonal antibody-drug conjugate directed against CD30 CD30 is a membrane glycoprotein in the TNF receptor family that signals through multiple mechanisms, including the NFkB pathway, to promote cell proliferation and survival

What is Rituximab?

Chimeric, partially humanized anti-CD20 monoclonal antibody

How can Leflunomide be removed quickly from the system?

Cholestyramine may be administered. By binding to bile acids, cholestyramine interrupts the enterohepatic circulation and causes a rapid washout of leflunomide.

Chronic GI hemorrhage

Chronic GI bleeding is a diagnostic challenge. It can manifest as self-limited, recurrent episodes of melena or hematochezia, but usually without the degree of hemodynamic compromise discussed earlier. Some patients have no overt evidence of blood loss but rather have persistent anemia and persistent occult blood loss. The evaluation of this condition differs from that of acute GI hemorrhage; the pace of the evaluation is less urgent, and the likely causes of bleeding differ from those associated with acute GI bleeding these imaging techniques have limited diagnostic utility. Flat mucosal lesions such as vascular ectasias, a common cause of obscure bleeding, may easily be missed. If radiographic studies are unrevealing, endoscopic evaluation of the small bowel may be attempted by capsule endoscopy or with push or balloon enteroscopy (see Chapter 34). For the patient with persistent blood loss, no endoscopically identified source of bleeding in the upper GI tract or colon, and negative findings on radiologic studies, the entire small intestine may be examined at laparotomy with endoscopy in the operative suite. In addition, angiographic evaluation of the whole GI tract may reveal the source of chronic blood loss. Patients with this condition usually have undergone upper and lower endoscopy at least once without identification of a bleeding source. Therefore, the bleeding must be from a source that is difficult to identify or one that emanates from the small intestine. The small intestine is a difficult area to examine because of its length and configuration. In general, the small intestine is initially evaluated radiographically. The patient may ingest barium, which is followed through the length of the small intestine. To distend the small bowel and provide greater mucosal detail, an enteroclysis tube may be placed with its distal tip near the ligament of Treitz, allowing more forceful administration of barium and air. However, computed tomography and magnetic resonance enterography are rapidly replacing fluoroscopic imaging.

What is Primary Sclerosing Cholangitis (PSC)?

Chronic cholestatic liver disease marked by fibrosis of the intrahepatic and extrahepatic bile ducts occurs in 1% to 4% of patients with UC and less often in those with Crohn's disease Overall, about 70% of patients with PSC have UC

What are inflammatory bowel diseases like ulcerative colitis and Crohns disease classic examples of?

Chronic diarrhea

chronic inflammation

Chronic inf ammation is a pathologic state characterized by the continued and inappropriate response o the immune system to an inf ammatory stimulus. Chronic inf ammation accounts or the symptoms o many autoimmune diseases and may be an important cause o organ transplant rejection. In contrast to the acute inf ammatory response, which is dominated by neutrophils, one o the hallmarks o chronic inf ammation is the predominance o macrophages. Activated macrophages secrete collagenases and growth actors in addition to inf ammatory mediators such as proteases and eicosanoids. These secreted products initiate and maintain a cycle o tissue injury and repair, leading to tissue remodeling. Over time, chronic inf ammation can cause relentless tissue destruction. Promising treatments or chronic inf ammation could include cytokine inhibitors that neutralize mediators o the signaling cascades that perpetuate chronic inf ammation. These agents are discussed in Chapter 46

What does Chronic rejection occur as a result of?

Chronic inflammation caused by the response of activated recipient T cells to donor antigen Activated T cells release cytokines that recruit macrophages into the graft The macrophages induce chronic inflammation that leads to intimal proliferation of the vasculature and scarring of the graft tissue

What is Ulcerative colitis (UC) characterized by?

Chronic inflammation of the mucosal surface that involves the rectum and extends proximally through the colon in a continuous manner

What is Dyspepsia characterized by?

Chronic intermittent epigastric discomfort, sometimes accompanied by nausea or bloating These symptoms are not always relieved by acid suppression and may be the result of an underlying motor disorder

What is the typical clinical course of Ulcerative colitis? What are signs of worsening clinical course?

Chronic intermittent exacerbations followed by periods of remission development of abdominal pain, dehydration, fever, and tachycardia

What does the presence of postprandial nausea and vomiting suggest?

Chronic peptic ulcer, disorders of gastric emptying, or outlet obstruction If anorexia accompanies weight loss, particularly in elderly patients, cancer must be excluded If no cancer can be found and all objective tests are normal, the possibility of chronic depression must be entertained

chronic rejection

Chronic rejection is believed to be both cellular and humoral in nature and does not occur until months or years a ter transplantation. Because hyperacute and acute rejection are generally well controlled by donor/recipient matching and immunosuppressive therapy, chronic rejection is now the most common li e-threatening pathology associated with organ transplantation. Chronic rejection is thought to result rom chronic in ammation caused by the response o activated recipient T cells to donor antigen. Activated T cells release cytokines that recruit macrophages into the gra t. The macrophages induce chronic in ammation that leads to intimal proli eration o the vasculature and scarring o the gra t tissue. The chronic changes eventually lead to irreversible organ ailure. Other contributing nonimmune actors can include ischemia-reper usion injury and in ection. No e ective treatment regimens are currently available to eliminate chronic rejection. It is believed, however, that several experimental therapies have a reasonable chance o reducing chronic rejection. Especially promising is the possibility o developing tolerance through elimination o costimulation (see below).

What is Irritable bowel syndrome (IBS) characterized by? What does the diagnosis depend on?

Chronic, relapsing abdominal pain, bloating, and changes in bowel habits depends on clinical symptoms and functional testing

Hepatic cirrhosis

Cirrhosis is a complex disorder of the liver, the diagnosis of which is confirmed histologically. When a diagnosis is suspected, a liver biopsy is necessary Cirrhosis is characterized by widespread hepatic fibrosis interspersed with areas of nodular regeneration and abnormal reconstruction of preexisting lobular architecture. The presence of cirrhosis implies previous or continuing liver cell damage The etiology of cirrhosis is complex and includes toxins (alcohol), viral inflammation, biliary obstruction, vascular outlet obstruction, nutritional (malnutrition) causes, and inherited anatomical and metabolic disorders As the cirrhosis progresses, the intrahepatic vasculature is distorted, which in turn leads to increased pressure in the portal vein and its draining tributaries (portal hypertension). Portal hypertension produces increased pressure in the splenic venules, leading to splenic enlargement. At the sites of portosystemic anastomosis (see below), large dilated veins (varices) develop. These veins are susceptible to bleeding and may produce marked blood loss, which in some instances can be fatal. The liver is responsible for the production of numerous proteins, including those of the clotting cascade. Any disorder of the liver (including infection and cirrhosis) may decrease the production of these proteins and so prevent adequate blood clotting. Patients with severe cirrhosis of the liver have a significant risk of serious bleeding, even from small cuts; in addition, when varices rupture, there is a danger of rapid exsanguination. As the liver progressively fails, the patient develops salt and water retention, which produces skin and subcutaneous edema. Fluid (ascites) is also retained in the peritoneal cavity, which can hold many liters The poorly functioning liver cells (hepatocytes) are unable to break down blood and blood products, leading to an increase in the serum bilirubin level, which manifests as jaundice With the failure of normal liver metabolism, toxic metabolic by-products do not convert to nontoxic metabolites. This buildup of noxious compounds is made worse by the numerous portosystemic shunts, which allow the toxic metabolites to bypass the liver. Patients may develop severe neurological features, called hepatic encephalopathy, that can manifest as acute confusion, epileptic fits, or psychotic state Hepatic encephalopathy is one of the urgent criteria for liver transplantation; if the condition is not reversed, it leads to irreversible neurological damage and death.

What is the diagnosis of Diverticular disease based on?

Clinical examination and often CT scanning

How can low grade dysplasia be treated?

Colectomy or followed closely, depending on a variety of factors including patient age and the number of dysplastic foci present Colonic adenomas (discussed later) also occur in IBD patients, and in some cases these may be difficult to differentiate from a polypoid focus of IBD-associated dysplasia.

What do activated macrophages secrete?

Collagenases and growth factors in addition to inflammatory mediators such as proteases and eicosanoids These secreted products initiate and maintain a cycle of tissue injury and repair, leading to tissue remodeling.

Where are Water and electrolytes primarily absorbed?

Colon

Where areas is Ulcerative Colitis limited to? Where does it extend to?

Colon and rectum extends only into the mucosa and submucosa.

Colonic biopsy of Crohn's disease

Colonic biopsy specimen demonstrates a chronic inflammatory infiltrate with a granuloma in a patient with Crohn's colitis (hematoxylin and eosin stain

sigmoid diverticular disease pathogenesis

Colonic diverticula result from the unique structure of the colonic muscularis propria and elevated intraluminal pressure in the sigmoid colon. Where nerves, arterial vasa recta, and their connective tissue sheaths penetrate the inner circular muscle coat, focal discontinuities in the muscle wall are created. In other parts of the intestine these gaps are reinforced by the external longitudinal layer of the muscularis propria, but, in the colon, this muscle layer is gathered into the three bands termed taeniae coli. Increased intraluminal pressure is probably due to exaggerated peristaltic contractions, with spasmodic sequestration of bowel segments, and may be enhanced by diets low in fiber, which reduce stool bulk, particularly in the sigmoid colon.

complement

Complement is a system o serine proteases that is one o the rst innate mechanisms to be activated in response to injury. The complement system can be activated by antigen-antibody interactions (the classical pathway), by direct interactions with oreign sur aces (the alternative pathway), or by interactions with certain complex carbohydrates (the lectin pathway). In each pathway, a series o proteolytic reactions converts a complement precursor protein, re erred to by the letter "C" ollowed by a number ( or example, C3), into its active orm(s), indicated by the letter "a" or "b" ( or example, C3a and C3b; in this case, both orms are active). The general scheme o this pathway is analogous to that o the coagulation cascade (see Chapter 23, Pharmacology o Hemostasis and Thrombosis), in which precursor proteins are proteolytically cleaved to active products that contribute to the actions o the cascade. A ter activation, complement triggers inf ammatory responses by two mechanisms. First, several cleavage products o the complement cascade are potent stimulators o inf ammation. For example, C3b is an important opsonin, and C3a and C5a mediate leukocyte chemotaxis. Second, the nal step in complement activation is the assembly o the membrane attack complex . This complex o complement proteins produces large pores in the outer membrane o Gramnegative bacteria, leading to lysis o the bacteria. A large number o complement regulatory proteins, both soluble and on the cell sur ace, care ully govern and localize complement activation to the site o inf ammation. Pharmacologic inhibitors o complement activation are used to lessen the tissue injury associated with inappropriate inf ammatory responses ( or example, in paroxysmal nocturnal hemoglobinuria and atypical hemolytic uremic syndrome

What is the incidence and prevalence of IBD reflect an interplay of?

Complex genetic and environmental factors eg: both diseases are more common in northern climates and among whites, particularly populations with Northern European ancestry such as North Americans, South Africans, and Australians Individuals of Ashkenazi Jewish descent also have a twofold to eightfold increased risk for these disorders compared with non-Jews Although incidence rates of IBD are lowest among Hispanics and Asians, IBD can occur in any ethnic or racial group from anywhere in the world

What is the diagnosis of IBD based on?

Constellation of clinical features, laboratory tests, and endoscopic, radiographic, and histologic findings. Laboratory tests are not specific and usually reflect inflammation (leukocytosis) or anemia

What is the anal canal?

Continuation of the large intestine inferior to the rectum

right colic artery

Continuing distally along the main trunk of the superior mesenteric artery, the right colic artery is the second of the three branches from the right side of the main trunk of the superior mesenteric artery (Fig. 4.126). It is an inconsistent branch, and passes to the right in a retroperitoneal position to supply the ascending colon. Nearing the colon, it divides into a descending branch, which anastomoses with the ileocolic artery, and an ascending branch, which anastomoses with the middle colic artery

Corticosteroids

Corticosteroids may be used topically, orally, or intravenously. They are effective for controlling active disease but not for maintaining remission (level of evidence I,A for induction and III, A for maintenance). They are indicated for moderate or severe disease in UC patients for whom treatment with 5-ASA has failed. The most commonly used agent is oral prednisone, started in doses between 40 and 60 mg/day. Patients typically improve rapidly, and the medication is usually tapered down slowly (i.e., by 5 to 10 mg/week until discontinuation. Patients who do not improve after 1 week of oral treatment and those with more severe disease are best treated in the hospital with intravenous corticosteroids, such as intravenous hydrocortisone (300 mg/day), or methylprednisolone (which can be given either by continuous infusion or in three divided doses). Corticosteroids have numerous side effects with long-term use. Controlled trials have shown that budesonide EC (Entocort EC) is more effective than placebo or oral 5-ASA and has similar efficacy to prednisolone for the induction of remission in Crohn's disease (level of evidence I, A). Entocort EC (9 mg given once daily) undergoes extensive first-pass hepatic metabolism, is available for inducing and maintaining remission of ileal and ileocolonic Crohn's disease (level of evidence III A) and may offer long term benefits with decreased corticosteroid side effects. Budesonide MMX (Uceris 9 mg given once daily) has an extended release that targets the colon and is available for the treatment of mild to moderate UC (level of evidence I, A for induction and III, A for maintenance)

inhibition of Costimulation

Costimulation re ers to the paradigm that cells o the immune system typically require two signals or activation (see Chapter 42, Principles o In ammation and the Immune System). I a f rst signal is provided in the absence o a second signal, the target immune cell may become anergic rather than activated. Because induction o anergy could lead to long-term acceptance o an organ gra t or limit the extent o an autoimmune disease, inhibition o costimulation represents a viable strategy or immunosuppression. Several therapeutic agents inhibit costimulation by blocking the second signal required or cell activation, and more such agents are under development.

What does the full stimulation of T cell depend on in addition to antigens?

Costimulators

What do costimulators, which are expressed on APCs, bind to on the naive T cell after encountering microbes and what does it promote?

Costimulatory receptors promotes responses to infectious pathogens.

What may Surgical treatment of ulcerative colitis, Hirschsprung disease and other intestinal disorders sometimes require ?

Creation of a temporary or permanent ostomy and a blind distal segment of colon, from which the normal fecal flow is diverted

What are Cytokines?

Critical signaling mediators in immune function

Crohn disease morphology

Crohn disease may occur in any area of the GI tract, but the most common sites involved at presentation are the terminal ileum, ileocecal valve, and cecum. Disease is limited to the small intestine alone in about 40% of cases; the small intestine and colon are both involved in 30% of patients; the remainder have only colonic involvement. The presence of multiple, separate, sharply delineated areas of disease, resulting in skip lesions, is characteristic of Crohn disease and may help in the differentiation from ulcerative colitis. Strictures are common in Crohn disease, but do not generally develop in ulcerative colitis The earliest lesion, the aphthous ulcer, may progress, and multiple lesions often coalesce into elongated, serpentine ulcers oriented along the axis of the bowel (Fig. 17-34B). Edema and loss of the normal mucosal texture are common. Sparing of interspersed mucosa, a result of the patchy distribution of Crohn disease, results in a coarsely textured, cobblestone appearance in which diseased tissue is depressed below the level of normal mucosa (Fig. 17-34B). Fissures frequently develop between mucosal folds and may extend deeply to become fistula tracts or sites of perforation (Fig. 17-34C). The intestinal wall is thickened and rubbery as a consequence of transmural edema, inflammation, submucosal fibrosis, and hypertrophy of the muscularis propria, all of which contribute to stricture formation (Fig. 17-34A). In cases with extensive transmural disease, mesenteric fat frequently extends around the serosal surface (creeping fat The microscopic features of active Crohn disease include abundant neutrophils that infiltrate and damage crypt epithelium. Clusters of neutrophils within a crypt are referred to as crypt abscesses and are often associated with crypt destruction. Ulceration is common in Crohn disease, and there may be an abrupt transition between ulcerated and adjacent normal mucosa. Even in areas where gross examination suggests diffuse disease, microscopic pathology can appear patchy. Repeated cycles of crypt destruction and regeneration lead to distortion of mucosal architecture; the normally straight and parallel crypts take on bizarre branching shapes and unusual orientations to one another (Fig. 17-35A). Epithelial metaplasia, another consequence of chronic relapsing injury, often takes the form of gastric antral-appearing glands, and is called pseudopyloric metaplasia. Paneth cell metaplasia may also occur in the left colon, where Paneth cells are normally absent. These architectural and metaplastic changes may persist even when active inflammation has resolved. Mucosal atrophy, with loss of crypts, may occur after years of disease. Noncaseating granulomas (Fig. 17-35B), a hallmark of Crohn disease, are found in approximately 35% of cases and may occur in areas of active disease or uninvolved regions in any layer of the intestinal wall (Fig. 17-35C). Granulomas may also be present in mesenteric lymph nodes. Cutaneous granulomas form nodules that are referred to as metastatic Crohn disease (a misnomer since there is no cancer). The absence of granulomas does not preclude a diagnosis of Crohn disease.

Defects in intestinal epithelial tight junction barrier function are present in what patients?

Crohn disease patients and their first degree relatives this barrier dysfunction is associated with specific disease-associated NOD2 polymorphisms; experimental models demonstrate that barrier dysfunction can activate innate and adaptive mucosal immunity and sensitize subjects to disease.

crohn disease

Crohn disease, an eponym based on the 1932 description by Crohn, Ginzburg, and Oppenheimer, has existed for centuries. Louis XIII of France (1601-1643) suffered relapsing bloody diarrhea, fever, rectal abscess, small intestinal and colonic ulcers, and fistulae beginning at age 20 years, most likely due to Crohn disease

What are variants that cause reduced resistance of NOD-2 to intestinal microbes associated with?

Crohn's disease an inflammatory bowel disease, in some ethnic populations

Where does Crohn's disease present?

Crohn's disease can involve any portion of the GI tract, from the mouth to the anus Patients with Crohn's disease typically have abdominal pain, diarrhea, weight loss, and fever; gross bleeding is not common with Crohn's disease Anemia, leukocytosis, and elevated inflammatory markers are common laboratory findings

What is the presence of antibodies against the bacterial protein flagellin most common in?

Crohns disease patients who have disease associated NOD2 variants, stricture formation, perforation, and small-bowel involvement.

What syndromes are mediated in part by increased levels of IL-1? examples? treatment?

Cryopyrin-associated periodic syndromes (CAPS) Muckle-wells syndrome, Hibernian fever Anakinra

What is Anakinra approved for use in?

Cryopyrin-associated periodic syndromes (CAPS) syndrome neonatal-onset multisystem inflammatory disease

alkylating agents- cyclophosphamide

Cyclophosphamide (Cy) is a highly toxic drug that alkylates DNA. The mechanism o action and uses o Cy are discussed extensively in Chapter 39; there ore, the discussion here is limited to Cy's utility in treating diseases o the immune system. Because Cy has a major suppressive e ect on B-cell proli eration but can enhance T-cell responses, the use o Cy in immune diseases is limited to disorders o humoral immunity, particularly systemic lupus erythematosus. Another use under consideration or Cy is the suppression o antibody ormation against xenotransplant gra ts. Adverse e ects o Cy are severe and widespread, including leukopenia, cardiotoxicity, pulmonary toxicity, and increased risk o cancer because o mutagenicity. The risk o bladder cancer is especially notable because Cy produces a carcinogenic metabolite, acrolein , which is concentrated in the urine. When high-dose Cy is administered by intravenous in usion, acrolein can be detoxif ed by co-administration o mesna (a sul hydryl-containing compound that neutralizes the reactive moiety o acrolein).

What does the right hepatic artery give off as it nears the liver?

Cystic artery to the gallbladder

cyotkine and cytokine receptor inhibiton

Cytokines are critical signaling mediators in immune unction. Cytokines are also pleiotropic; that is, they exert di erent e ects depending on the target cell and overall cytokine milieu. For this reason, pharmacologic uses o cytokines or cytokine inhibitors may have unpredictable e ects. Anticytokine therapy has been in clinical use or immunologically mediated diseases or more than a decade. The f rst anticytokine agent approved or use was etanercept , an anti-TNF drug developed or rheumatoid arthritis. During the initial clinical studies, some patients with severe, drugre ractory rheumatoid arthritis literally got up rom their wheelchairs and walked a ter receiving etanercept. This dramatic e f cacy ushered in a new era o biological therapies or autoimmune disease, and the number o new drugs that inhibit proin ammatory cytokines continues to grow rapidly. An alternative approach to block the action o in ammatory cytokines is to target the cytokine receptor

cytokines

Cytokines are proteins that act in a paracrine manner to regulate leukocyte activity. Interleukins and tumor necrosis actor (TNF) amily members are cytokines secreted primarily by cells o the hematopoietic lineage. Interleukin-1 (IL-1) and tumor necrosis actor- (TNF- ) are among the key cytokines elaborated in the acute inf ammatory response; these cytokines were two o the mediators responsible or Mark's ever. Another member o the TNF amily is the B lymphocyte stimulator (BLyS), which promotes B cell survival and di erentiation. Located on T cells and B cells, TNF receptors contain membrane glycoprotein CD30, which is important in cellular proli eration and survival and serves as a pharmacologic target (see Chapter 46). Chemokines are a subset o cytokines that promote immune cell tra cking, transmigration, and localization to sites o inf ammation. For example, macrophage chemoattractant protein-1 (MCP-1) promotes monocyte transmigration and activation. Other notable cytokines include the hematopoietic growth actors granulocyte-monocyte colony-stimulating actor (GMCSF) and granulocyte colony-stimulating actor (G-CSF) (see Chapter 45). Because cytokines a ect the proli eration and unction o cells that mediate innate and adaptive immune responses, selective inhibition or stimulation o cytokine action has the potential to modulate immune and inf ammatory responses. Pharmacologic uses or cytokine and anticytokine therapies are discussed in Chapter 45 and Chapter 46, respectively.

What does Leflunomide have antiviral activity agaisnt?

Cytomegalovirus (CMV) has been used to treat this infection in cases of drug-resistant CMV and in transplant patients Leflunomide prolongs transplant graft survival and limits GVHD in animal models

What does activation of CD8 T cells often require?

Cytosolic antigen from one cell (eg: virus infected or tumor cells) to be cross-presented by dendritic cells

What effect does MPA have on lymphocytes? What can it lead to the induction of on T cells?

Cytostatic effect on lymphocytes Apoptosis of activated T cells, to the elimination of reactive clones of proliferating cells

What is Acute rejection mediated by? What does it cause?

Cytotoxic T cells Interstitial as well as vascular damage

cytotoxic agents

Cytotoxic agents are used both or immunosuppression and or antineoplastic chemotherapy. The therapeutic goal in both cases is the elimination o pathogenic cells. Two classes o cytotoxic agents, antimetabolites and alkylating agents , are commonly used as immunosuppressants. Antimetabolites are structural analogues o natural metabolites that inhibit essential pathways involving these metabolites. Alkylating agents inter ere with DNA replication and gene expression by alkylation of DNA

What does TNF-alpha stimulate macrophages to produce?

Cytotoxic metabolites thereby increasing phagocytic killing TNF-alpha also stimulates production of acute-phase proteins, has pyrogenic effects, and fosters local containment of the inflammatory response

An 18-year-old male presents to the emergency department with severe localized tenderness in his right groin. During the history, he explained to the physician that the pain began very generalized around his belly button and would come and go. The patient informed the physician that he is currently feeling nauseous and vomited right before arriving at the emergency department. Which of the following correctly describes the anatomical feature most likely affected? A) A depression formed between the lateral margins of the ascending and descending colon B) A bend of bowel found at the junction of the ascending and transverse colon, just inferior to the right lobe of the liver C) A bend of bowel found at the junction of the transverse colon and descending bowel, just inferior to the spleen D) A narrow, hollow, blind-ended tube with a base that is extended from the cecum E) A blind-ended tubular outgrowth of bowel that lies on the antimesenteric border of the ileum

D) A narrow, hollow, blind-ended tube with a base that is extended from the cecum

A 38-year-old male presents to the Emergency Department for abdominal pain. While conducting a physical exam the physician notes that the pain is located in the right lower quadrant and periumbilical region of the patient's abdomen. The patient describes the pain as crampy and consistent. He also claims that he feels pain in his back. When asked about his medical history the patient states he has never had any health issues in his entire life, not even a broken bone. The physician suspects an acute abdominal pain syndrome. Which of the following is the most likely answer choice? A) IBS B) Cholecystitis C) Perforation D) Appendicitis E) Peptic Ulcer

D) Appendicitis

Anergy in T-cells means long lived functional inactivation, what are the two mechanisms of anergy that can occur when cells do not receive adequate co-stimulation? A) Negative selection and receptor editing B) Presence of antigen and co-stimulation C) TCR complex gains ability to transmit signals and CTLA-4 has lower affinity for B7 D) Block in signaling by the TCR complex and delivery of inhibitory signals from receptors other than the TCR complex

D) Block in signaling by the TCR complex and delivery of inhibitory signals from receptors other than the TCR complex

Which of the following answers correctly describes CTLA-4 A) CTLA-4's major site of action is in secondary lymphoid organs and it's inhibited during the effector phase B)CTLA-4 is most active during situations of chronic or repeated antigen C) CTLA-4 inhibits CD8+ cells more than CD4 D) CTLA-4 is a competitive inhibitor of the B7-CD28 interaction

D) CTLA-4 is a competitive inhibitor of the B7-CD28 interaction

Following severe progression of complex neck cancer, Veronica's oncologist decided to halt all oral food and water consumption. Which of the following surgical interventions would be most appropriate to ensure adequate consumption of nutrients? A) Colostomy B) Ileostomy C) Jejunostomy D) Gastrostomy E) Ileal conduit

D) Gastrostomy

An abdominal CT scan was performed on a male patient. A physician identifies an incidental finding, an atherosclerotic plaque occluding the inferior mesenteric artery. Which of the following best explains why this patient is not suffering any complications due to the occluded vessel? A. Anastomotic connection of the internal iliac artery B. Anastomotic connection of the external iliac artery C. Anastomotic connection of the internal pudendal artery D. Anastomotic connection of the marginal artery E. Anastomotic connection of the anterior cecal artery

D. Anastomotic connection of the marginal artery

A 23 year old male presents to the emergency department with complaints of severe right lower quadrant pain that has been constant for the past 3 hours. He tells the physician he experienced 3 episodes of non-bloody emesis prior to arrival. He states he has never had pain like this in the past and admits to a fairly-healthy diet and active lifestyle. It is noted the patient has positive Mcburney's sign during physical exam, his temperature is 39 C, and he has been able to pass gas since arriving at the emergency room. What is the most likely diagnosis? A. Carcinoma of the stomach B. Meckel's Diverticulum C. Diverticulitis D. Appendicitis E. Small bowel obstruction

D. Appendicitis

A patient with acute diarrhea is prescribed Lomotil (diphenoxylate-atropine) in an effort to decrease the frequency of their watery stools. The patient asks her physician if there are any dietary changes that she can implement to improve her situation. She explains that her current diet is pretty normal. Which of the following dietary changes is most likely recommended by the physician in this scenario? A. Consume alcohol in moderation B. Decrease intake of clear liquids C. Remove soft and low fiber foods from diet D. Avoid the consumption of milk E. Increase intake of high fiber foods

D. Avoid the consumption of milk

A 25-year-old female presents to the physician's office complaining of diarrhea, abdominal pain, urgency to defecate and occasional rectal bleeding. Upon testing, the patient is diagnosed with Ulcerative Colitis. The physician prescribes the immunomodulator Azathioprine for treatment. What potential side effect should be monitored for this patient? A. Leukoencephalopathy B. Reversible oligospermia C. Skin reactions D. Bone marrow suppression E. Demyelinating disease

D. Bone marrow suppression

Which of the following molecules is responsible for a chemotaxis response for leukocytes in an inflammatory response? A. TNF-α B. IL-1 C. NO D. C5a E. Substance P

D. C5a

The migration of naive and effector T cells is controlled by three families of proteins - selectins, integrins, and chemokines - that regulate the migration of all leukocytes. Naive T cells express certain adhesion molecules and chemokine receptors which mediate the selective migration of the naive cells into lymph nodes through high endothelial venules (HEVs). Which of the following is the main chemokine receptor responsible for the activation of integrins and the chemotaxis of naive T cells to the T cell zone of lymph nodes? A. LFA-1 B. CXCR3 C. CCL19 D. CCR7 E. CCL21

D. CCR7

A 10-year-old boy was recently diagnosed with Type 1 Diabetes. The boy's parents wanted to know what caused the disease in their son. The doctor explained that the disease is influenced by many genetic and environmental factors. He orders genetic testing to look for possible mutations in different genes that could lead to Type 1 Diabetes. Polymorphisms in which of the following genes could contribute to this patient developing Type 1 Diabetes? A. NOD2 B. Complement proteins C2 and C4 C. FOXP3 D. CD25 (IL-2Rα) E. AIRE

D. CD25 (IL-2Rα)

A 40-year-old male patient has a genetic deficiency leading to the expression of nonfunctional CD40 ligand (CD40L). In addition to defects in humoral immunity, the patient showed an impaired cell-mediated immunity as evident by increased susceptibility to infections caused by intracellular pathogens. Which statement correctly describes CD40L-dependent mechanisms involved in cell-mediated immunity? A. CD40L-dependent isotype switching is required to produce antibody isotypes that activate T cells. B. CD40 ligand is required for killing of CD40-expressing infected cells by CTLs. C. CD40 ligand is required for maturation of CD4+ T cells in the thymus. D. CD40 ligand on T cells binds to CD40 on macrophages and enhances their phagocytic and Ag-presenting properties. E. CD40 ligand on T cells binds to B7-1 and B7-2 on APCs, and this enhances the function of the APCs.

D. CD40 ligand on T cells binds to CD40 on macrophages and enhances their phagocytic and Ag-presenting properties.

After trading away Deandre Hopkins, Bill O'Brien began to feel lower quadrant pain, weight loss, and diarrhea. After some examination you deduce that his ileocecal area is inflamed and his biopsy reveals that there are granulomas in the inflamed area. What is Bill O'Brien suffering from? A. Ulcerative Colitis B. Allergic reaction to something he ate C. He is currently experiencing a lot of stress D. Crohn's Disease

D. Crohn's Disease

A 32-year-old patient is admitted to the ER with severe abdominal pain, fever, and tachycardia. Lab testing has revealed anemia, leukocytosis, and a colonoscopy reveals deep ulcerations with pseudo polyps. A positive pANCA test confirmed their suspicion and the patient was started on which of the following treatment? A. Methotrexate B. Oral 5-ASA C. Oral Steroids D. Cyclosporine E. Children's Motrin

D. Cyclosporine

Jack is a 28-year old male who was just recently diagnosed with leukemia. After discussing various treatment options, he elects to receive a bone marrow transplant. As part of this procedure, Jack's immune system will be eradicated by aggressive chemotherapy and/or radiation therapy. Assuming a mild-to-moderate graft-versus-host disease, which of the following would most likely lead to the best outcome? A. Do not completely remove the immune cells from the recipient bone marrow before transplantation. B. Remove the helper T cells from the donor bone marrow before transplantation. C. Remove the cytotoxic T cells from the donor bone marrow before transplantation. D. Do not remove donor T cells from the bone marrow graft. E. A stem cell transplant will provide better outcomes than a bone marrow transplant

D. Do not remove donor T cells from the bone marrow graft.

A student is researching the effects of various cytokines in the immune response. The student wants to figure out which cytokine promotes the increase in the number of antigen specific T lymphocytes, a process known as "clonal expansion". They learn that this cytokine is produced by antigen-activated T cells. Which of the following cytokines fits this role? A. IL-5 B. IFN-gamma C. IL- 22 D. IL- 2 E. IL - 17

D. IL- 2

A 35-year-old female presents to the clinic with intermittent hematochezia and an overall increase in bowel frequency. Physical exam findings show that the patient is slightly febrile and has diffuse, nonlocalized, abdominal tenderness. Suspecting that the patient has ulcerative colitis, the physician wants to order a new test to screen for increased levels of cytokines in the bowel. Which of the following cytokines in an increased concentration would confirm his diagnosis? A. IL-2 B. IL-12 C. IL-17 D. IL-5 E. IL-10

D. IL-5

Which of the following is not a mechanism of tolerance to fetal antigens? A. Generation of peripheral FoxP3+ regulatory T cells specific for these paternal antigens B. Exclusion of inflammatory cells from the pregnant uterus C. Poor antigen presentation in the placenta D. Mature T cells recognize fetal antigens leading to functional anergy E. Inability to generate harmful Th1 responses in the healthy pregnant uterus

D. Mature T cells recognize fetal antigens leading to functional anergy

A 23 year old male presents to the emergency room with intermittent central abdominal pain. After a few hours the pain begins to localize to the right lower quadrant in the right iliac fossa. He also presents with a fever and nausea. After being referred to an abdominal surgeon, he was informed he had appendicitis and his inflamed appendix was removed. Which of the following is NOT an artery that supplies the appendix? A. Appendicular artery B. Ileocolic artery C. Superior mesenteric artery D. Middle colic artery E. Anterior cecal artery

D. Middle colic artery

A 3-year-old female was brought into the Westside pavilion clinic by her parents. They said that the child was experiencing consistent non-bloody diarrhea with excessive flatulence. During the physical exam, the physician noted that the child was below average for her height and was not able to walk for more than a few steps. The physician suspects that she may have a gluten intolerance and diagnoses her with Celiac disease. As a follow-up, an intestinal biopsy was ordered. Which of the following was the most likely finding in the biopsy results? A. An osmotic gap in the fecal specimen B. Polyps present within the colon C. Severe inflammation within the sigmoid colon D. Normal villous structure with increased mucosal lymphocytes and plasma cells E. A microscopic Giardia parasite attached to the lining of the small intestines

D. Normal villous structure with increased mucosal lymphocytes and plasma cells

A 42-year-old female patient, Samantha, presents to the clinic with a chief complaint of persistent weakness and fatigue, loss of appetite, and blinding headaches. She also mentions that when she does eat, she experiences indigestion, and that her hands and feet have been feeling colder than usual. A CBC reveals low hematocrit and elevated MCV. Her primary care physician suspects a Type II hypersensitivity disorder that is neutralizing a secretion from her gastric parietal cells and fears the possibility of long-term neurologic damage. Which of the following would be the appropriate diagnostic to confirm the physician's suspected diagnosis? A. 14C-xylose test B. Endoscopy C. Serologic blood test D. Schilling Test E. Erythrocyte Sedimentation Rate

D. Schilling Test

A middle-aged man comes into the ER complaining of severe pain and discomfort after eating meals. The physician, after performing an aortic angiography, explains that his gut is not getting a proper supply of oxygen due to narrowing of a certain artery. Which artery below is most likely stenosed? A. Inferior Mesenteric Artery B. Superior Rectal Artery C. Short Gastric Arteries D. Superior Mesenteric Artery E. Cystic Artery

D. Superior Mesenteric Artery

A 12 year old female presents to the clinic with complaints of fever, nausea, and vomiting and tenderness in the right groin. She says the pain started as an achy pain around her belly button that came and went but it is now constant and more crampy. The blood supply to the affected organ is most likely from which of the following arteries? A. Inferior mesenteric artery B. Superior rectal artery C. Splenic artery D. Superior mesenteric artery E. Celiac trunk

D. Superior mesenteric artery

The abdominal aorta passes through the abdominal cavity and has three anterior branches, the celiac trunk and the superior and inferior mesenteric arteries, that supply the gastrointestinal viscera. Which of the following correctly matches the anterior branch of the abdominal aorta with its corresponding area of distribution? A. Celiac trunk; hindgut B. Superior mesenteric artery; foregut C. Inferior mesenteric artery; foregut D. Superior mesenteric artery; midgut E. Celiac trunk; midgut

D. Superior mesenteric artery; midgut

A one-year old infant is brought to his family physician by his mother who has become concerned about chronic diarrhea and eczema the child has developed over the past month. The family physician, remembering that this is the second occurrence of the symptoms in this patient, treats the symptoms accordingly and orders genetic testing to rule out immunodeficiency. The results of the genetic testing confirm a mutation in the genes encoding the IL-2 receptor on the patient's T cells. What cellular processes will be inhibited by such a mutation? A. MHC restriction B. Immature T-cell development in the bone marrow C. Negative selection D. Survival and proliferation of T cells E. T-cell adhesion to the endothelium

D. Survival and proliferation of T cells

Marcus is a currently undergoing treatment for his moderate to severe RA. The medication Marcus is prescribed is an Anti-cytokine that shares the same action and target as Etanercept, Infliximab and Adalimumab. Which of the following is true regarding the cytokine targeted by the medication Marcus is prescribed? A. IL-6—plays a central role in the development of RA by stimulating Macrophages and synovial fibroblasts. B. IL-17—plays a central role in inflammation by stimulating the production of key inflammatory mediators, including IL-6 and IL-1β. C. TNF-α—involved in angiogenesis, neutrophil migration, differentiation of helper T cells, bone and cartilage metabolism, lipid metabolism, and cancer. D. TNF-α--activates endothelial cells, other monocytes, and synovial fibroblasts in the progression of RA. E. TNF-α—stimulates macrophages to produce cytotoxic metabolites thereby reduce phagocytic killing

D. TNF-α--activates endothelial cells, other monocytes, and synovial fibroblasts in the progression of RA.

A 35 year old woman was referred by her dentist because she recently began experiencing swelling and pain in her knees and hands while performing everyday tasks. This clinical presentation seems to be rheumatoid arthritis. Her dentist also noted that she had presented with recurrent bouts of gingivitis for the past 6 months. What is the most likely mechanism of action that caused her current symptoms? A. The gingival infection injured tissue and released antigens that are normally sequestered from the immune system B. The woman already had rheumatoid arthritis, the gingivitis just exacerbated the symptoms C. The gingival infection produced peptides that are similar to the self peptides in joints causing her lymphocytes to attack cells in her joints D. The inflammatory response to the infection lead to alterations in the chemical structure of self antigens, inducing an adaptive immune response

D. The inflammatory response to the infection lead to alterations in the chemical structure of self antigens, inducing an adaptive immune response

A 40 year old woman comes to the physician complaining of changes in bowel habits accompanied by bloating. A Rome criteria confirms for irritable bowel syndrome, and the doctor prescribes linaclotide for treatment. What are the most likely presentations of irritable bowel syndrome (IBS) in terms of weight and location of pain? A.) No weight loss and pain in the upper left quadrant B.) No weight loss and pain in the upper right quadrant C.) No weight loss and pain in the lower right quadrant D.) No weight loss and pain in the lower left quadrant E.) Significant weight loss and pain in the lower left quadrant

D.) No weight loss and pain in the lower left quadrant

The human microbiome in the gut serves many essential functions. The normal bacteria aid in digestion, absorption of food, and prevention of pathogen overgrowth. What is one mechanism behind why mature T lymphocytes do not react against these commensal organisms? A: Decreased IL-2 binding by regulatory T cells B: Gut dendritic cells lack toll-like receptors C: Normal gut dendritic cells lack ability to process antigens D: Abundance of IL-10 producing regulatory T cells in the intestines E: No physical separation between commensal bacteria and intestinal immune system

D: Abundance of IL-10 producing regulatory T cells in the intestines

Treatment options for Ulcerative colitis and Crohns disease for, Mild ulcerative colitis, moderate ulcerative colitis, severe ulcerative colitis, mild crohns disease, moderate crohns disease, severe crohns disease

DISEASE SEVERITY ULCERATIVE COLITIS CROHN'S DISEASE Mild Oral and topical 5-ASA compounds 5-ASA compounds Antibiotics Elemental diet Moderate Oral and topical 5-ASA compounds Oral steroids or budesonide MMX Azathioprine, 6-MP Infliximab, adalimumab, golimumab 5-ASA compounds Antibiotics Oral steroids or budesonide EC Azathioprine, 6-MP Methotrexate Infliximab, adalimumab, certolizumab natalizumab Severe Intravenous steroids Cyclosporine Infliximab, adalimumab, golimumab Vedolizumab Surgery Intravenous steroids Methotrexate Infliximab, adalimumab, certolizumab Natalizumab, vedolizumab Surgery

What is the principal mechanism of Central tolerance in T cells?

Death of immature T cells and the generation of CD4+ regulatory T cells

Since Guanosine is a precursor of Tetrahydrobiopterin (BH4) what does reduced levels of Guanosine lead to?

Decreased Nitric oxide (NO) production by neutrophils

What can the reduction of Guanosine by MPA lead to ?

Decreased expression of adhesion molecules that are required for recruitment of several immune cell types to sites of inflammation.

What happens if receptor editing fails?

Deletion (are killed) if receptor editing fails immature B cells that strongly recognize self antigens receive death signals and die by apoptosis. negative selection of B cells eliminates lymphocytes with high-affinity receptors for abundant, and usually widely expressed, cell membrane or soluble self antigens

What are the most important APCs for the initiation of adaptive immune responses?

Dendritic cells In the nonlymphoid tissues, dendritic cells engulf and process foreign antigens. Dendritic cells then migrate to lymphoid tissues, where they present these cognate antigens to T cells via specific molecular interactions

What is the most striking feature of Diversion colitis besides mucosal erythema and friability?

Development of numerous mucosal lymphoid follicles Increased numbers of lamina propria lymphocytes, monocytes, macrophages, and plasma cells may also be present In severe cases the histopathology may resemble IBD and include crypt abscesses, mucosal architectural distortion, or, rarely, granulomas.

What are the most significant adverse effects of Leflunomide?

Diarrhea and Reversible alopecia

What symptoms do patients initially exhibit in Ulcerative colitis?

Diarrhea, abdominal pain, urgency to defecate, rectal bleeding, and the passage of mucus per rectum At presentation, approximately 40% to 50% of patients have proctitis or proctosigmoiditis, 30% to 40% have left-sided colitis (disease extending to the splenic flexure), and the remaining 20% to 25% have pancolitis. Of the patients who initially present with proctitis or proctosig

symptomatic therapy

Dietary changes usually do not need to be drastic. The patient should be encouraged to take clear liquids and perhaps soft and low-fiber foods, which will aid in hydration and supply some calories for baseline energy requirements and enterocyte renewal. Milk should be avoided because of possible temporary lactose intolerance due to mucosal injury. Caffeine and ethanol should also be avoided because they stimulate intestinal motility In patients with acute diarrhea who do not have bloody diarrhea or fever, loperamide (Imodium), diphenoxylateatropine (Lomotil), or tincture of opium can decrease the frequency of watery stools. These agents also possess antisecretory properties, and they inhibit intestinal motility, thereby allowing more intestinal absorption. The usual dose of loperamide should not exceed 8 2-mg tablets per day and diphenoxylate should not exceed 8 5-mg tablets per day. Diphenoxylate and tincture of opium have central-acting opioid effects and can cause unwanted side effects, particularly in the elderly. These drugs also are associated with increased risk of hemolytic-uremic syndrome in patients with enterohemorrhagic E coli infections. The use of probiotics to repopulate intestinal flora in infectious diarrhea has not been well studied.

definition and absorption of fat

Dietary fat is composed predominantly of triglycerides (~95%) with long-chain fatty acids (16- and 18-carbon molecules). In animal fat, the constituent fatty acids are mostly saturated (e.g., palmitic acid, stearic acid), whereas those of vegetable origin are rich in unsaturated fatty acids (i.e., having one or more double bonds in the carbon chain, such as oleic and linoleic acids). Fats are insoluble in water (hydrophobic), and digestion begins with a process of emulsification, wherein larger fat droplets are dispersed in the aqueous medium of the lumen. In the proximal small intestine, bile salts from liver and pancreatic enzymes are released into the intestinal lumen; there, they mix with and bind to the surface of these globules, where colipase activity results in the release of fatty acids and a monoglyceride. These are taken up as mixed micelles with bile salts, and these hydrophobic particles cross the unstirred water layer that overlies the epithelial brush border. Within the cell, fatty acids are resynthesized into triglycerides, and, together with cholesterol and phospholipids, they are packaged into chylomicrons and very-low-density lipoproteins to be exported via lymphatic channels. Bile salts remain in the intestinal lumen, are recycled into new micelles, and are finally reabsorbed in the terminal ileum with 95% efficiency. Most dietary lipids are absorbed in the jejunum, together with the fat-soluble vitamins A, D, E, and K. It is recommended that dietary fat account for no more than 35% of calories because higher levels are associated with increased risk of cardiac disease, obesity, and some cancers.

What is the major source for amino acids and the only source for essential amino acids?

Dietary proteins

What do activated lymphocytes undergo which results in the conversion of naive T cells to ........ T cells?

Differentiation Effector T cells- function is to eliminate microbes

What can mucosal disease result from?

Diffuse small intestinal diseases such as celiac disease or Crohn's disease or from a decrease in surface area (e.g., after surgical resection for small bowel infarction Mucosal disease is a more common cause of malabsorption

What is the main purpose of the Gastrointestinal (GI) tract? What is Malabsorption?

Digest and absorb major nutrients (fat, carbohydrate, and protein), essential micronutrients (vitamins and trace minerals), water, and electrolytes Impaired absorption of these nutrients is defined as malabsorption

Luminal phase

Digestion is accomplished for the most part by pancreatic enzymes, particularly lipase, colipase, and trypsin; the gastric digestive enzymes do not play a major role. As a consequence, chronic pancreatitis can result in malabsorption, particularly for fat and protein. Deficiency in bile salts also contributes to fat malabsorption and may result from cholestatic liver disorders (impaired secretion of bile), bacterial overgrowth (resulting in luminal bile salt deconjugation), or ileal disease or resection with loss of effective enterohepatic circulation of the bile acids. The major part of the luminal phase of digestion occurs in the duodenum and the proximal jejunum.

Injury to a tissue causes the release of inflammatory mediators that what?

Dilate arterioles and postcapillary venules

What is the most definitive diagnostic test for Bacterial overgrowth? What other tests can be used?

Direct culture of jejunal aspirate C14-xylose breath test is an accurate and sensitive laboratory test;

diagnosis

Direct culture of jejunal aspirate is the most definitive diagnostic test, but it is invasive, uncomfortable, and costly. The 14C-xylose breath test is an accurate and sensitive laboratory test; measurement of breath hydrogen after an oral challenge with glucose is simpler but not as sensitive or as specific. An empirical therapeutic trial with antibiotics is an acceptable alternative to diagnostic testing.

Where does the large intestine extend from?

Distal end of the ileum to the anus a distance of approximately 1.5 m in adults

jejunal and ileal arteries

Distal to the inferior pancreaticoduodenal artery, the superior mesenteric artery gives off numerous branches. Arising on the left is a large number of jejunal and ileal arteries supplying the jejunum and most of the ileum (Fig. 4.127). These branches leave the main trunk of the artery, pass between two layers of the mesentery, and form anastomosing arches or arcades as they pass outward to supply the small intestine. The number of arterial arcades increases distally along the gut. There may be single and then double arcades in the area of the jejunum, with a continued increase in the number of arcades moving into and through the area of the ileum. Extending from the terminal arcade are vasa recta (straight arteries), which provide the final direct vascular supply to the walls of the small intestine. The vasa recta supplying the jejunum are usually long and close together, forming narrow windows visible in the mesentery. The vasa recta supplying the ileum are generally short and far apart, forming low broad windows

tolerance and costimulation

Diversity in the variable regions o immunoglobulins and T-cell receptors creates the potential or some o these molecules to recognize and attack native proteins, a circumstance termed autoimmunity. There are two primary mechanisms or avoiding autoimmunity. The rst is clonal deletion , in which T cells die during development when they express high-a nity receptors that recognize sel -antigen. In a second process re erred to as tolerance or anergy , cells o the immune system undergo a care ully regulated series o steps during development to ensure that mature immune cells do not recognize native proteins. Costimulation —the requirement or multiple simultaneous signals to initiate an immune response—ensures that stimulation o a single immune receptor does not activate a damaging immune reaction. Signal 1 provides speci city, while signal 2 is permissive, ensuring that an inf ammatory response is appropriate. Regulation o costimulatory molecules is a mechanism whereby the innate immune system regulates the extent o an immune response. I antigen is presented without a coincident costimulatory signal (i.e., without innate immune activation), then anergy may result, whereby a cell becomes unreactive and will not respond to urther antigenic stimuli. Drugs that induce anergy could be therapeutically attractive because such agents could allow long-term acceptance o an organ gra t or limit the extent o an autoimmune disease For T cells, signal 1 is mediated by the MHC:antigen:TCR interaction. Signal 2 is mediated predominantly by the interaction o CD28 on T cells with B7-1 (also called CD80) or B7-2 (CD86) on activated antigen-presenting cells (Fig. 42-4). Resting T cells present CD28, which can bind either B7-1 or B7-2. B7-1 and B7-2 are not normally present on antigen-presenting cells, but their expression is increased by the innate immune system during an immune response to a pathogen. The lack o expression o B7 molecules in the absence o an innate immune response may help to limit inappropriate adaptive immune responses. When a T cell receives both signal 1 and signal 2, the T cell is activated, IL-2 is expressed, and clonal expansion o T H cells speci c or that oreign epitope occurs. Activated T cells eventually down-regulate CD28 expression and up-regulate CTLA-4 expression. CTLA-4 , like CD28, binds B7-1 and B7-2 but with much higher a f nity than CD28. In contrast to the activating CD28 signal, interaction o CTLA-4 with B7-1 or B7-2 inhibits T cell proli eration. This appears to be a physiologic mechanism or sel -limitation o the immune response. CD40 ligand (CD40L) is another mediator o costimulation. Activated T cells express CD40L (CD154). CD40 is expressed on antigen-presenting cells, including B cells and macrophages ). Interaction o T H cell CD40L with B cell CD40 promotes B cell activation, isotype switching, clonal expansion, and a f nity maturation. Interaction o T H cell CD40L with macrophage CD40 promotes macrophage expression o B7-1 and B7-2. These molecules, as mentioned above, are crucial or costimulation o T cells. This pathway thus provides a positive eedback mechanism whereby activated T cells can promote urther expansion o activated T cells. In addition, the increased expression o B7-1 and B7-2 on macrophages is important or promoting CD8 T cell activation Because the CD40-CD40L interaction promotes multiple costimulation pathways, it has been hypothesized that blockade o CD40L could produce tolerance. Preliminary studies have demonstrated that blockade o CD40L with anti-CD40L antibody can indeed produce tolerance and long-term gra t survival in animal models o organ transplantation Increasing experimental evidence suggests that peripheral tolerance is maintained by a subset o T cells re erred to as regulatory T cells (T reg ) . These cells, the best characterized o which are CD4 CD25 , elaborate inhibitory cytokines in response to sel -antigens and thereby limit the immune response to these antigens. Pharmacologic induction o T reg cells may have applications in transplantation and autoimmune diseases, including type 1 diabetes.

sigmoid diverticular disease

Diverticular disease generally refers to acquired pseudodiverticular outpouchings of the colonic mucosa and submucosa. Unlike true diverticula, such as Meckel diverticulum, they are not invested by all three layers of the colonic wall. Colonic diverticula are rare in persons younger than age 30, but the prevalence approaches 50% in Western adult populations older than age 60. Diverticula are generally multiple and the condition is referred to as diverticulosis. This disease is much less common in Japan as well as developing countries, probably because of dietary differences. Moreover, most diverticula in Asia and Africa occur in the right colon, while right-sided diverticula are uncommon in Western countries. The reasons for this difference in distribution are not well-defined

Diverticular disease

Diverticular disease is the development of multiple colonic diverticula, predominantly throughout the sigmoid colon, though the whole colon may be affected (Fig. 4.99). The sigmoid colon has the smallest diameter of any portion of the colon and is therefore the site where intraluminal pressure is potentially the highest. Poor dietary fiber intake and obesity are also linked to diverticular disease. The presence of multiple diverticula does not necessarily mean the patient requires any treatment. Moreover, many patients have no other symptoms or signs. Patients tend to develop symptoms and signs when the neck of the diverticulum becomes obstructed by feces and becomes infected. Inflammation may spread along the wall, causing abdominal pain. When the sigmoid colon becomes inflamed (diverticulitis), abdominal pain and fever ensue Because of the anatomical position of the sigmoid colon there are a number of complications that may occur. The diverticula can perforate to form an abscess in the pelvis. The inflammation may produce an inflammatory mass, obstructing the left ureter. Inflammation may also spread to the bladder, producing a fistula between the sigmoid colon and the bladder. In these circumstances patients may develop a urinary tract infection and rarely have fecal material and gas passing per urethra. The diagnosis is based upon clinical examination and often CT scanning. In the first instance, patients will be treated with antibiotic therapy; however, a surgical resection may be necessary if symptoms persist.

What problem can arise as a result of transplantation? What is alloimmunity?

Donor and recipient tissues express different major histocompatibility complex (MHC) class I molecules, one class of alloantigens, and recipient immune cells there ore recognize the transplanted tissues as foreign this is alloimmunity - occurs when the recipient's immune system attacks a transplanted organ

What does the midgut include? What is it supplied by?

Duodenum (inferior to the major duodenal papilla), jejunum, ileum, cecum, appendix, ascending colon, and right two-thirds of the transverse colon Superior mesenteric artery

Where does the major part of the luminal phase of digestion occur?

Duodenum and proximal jejunum

What are the major mechanisms of Peripheral T-Cell Tolerance? A) Anergy B) Suppression by regulatory T-Cells C) Deletion / Apoptosis D) Receptor Editing E) Answers A, B, & C

E) Answers A, B, & C

A 35-year-old male comes into the ER with pain in his right lower quadrant. He states the pain started over his belly button and is now in the lower right side of his abdomen. He describes the pain as steady and crampy. What is the most likely diagnosis of this patient? A) Pancreatitis B) Peptic Ulcer C) Perforation D) Dyspepsia E) Appendicitis

E) Appendicitis

Ian presents with intermittent attacks of mild, bloody diarrhea and right lower quadrant pain. History is significant that he just started smoking cigarettes about 3 months ago. Colonoscopy, imaging, and biopsy reveal a continuous pattern of histologic disease, fissures, perianal lesions limited to only the colon and rectum. Which of the following statements are most likely true of Ian's disease? A) Based on the morphological features, Ian has Crohn disease B) Based on the location affected, Ian has Crohn disease C) Based on the morphological features, Ian has Ulcerative Colitis D) Based on the location affected, Ian has Ulcerative Colitis E) Based on both the presentation and location affected, Ian has Indeterminate Colitis

E) Based on both the presentation and location affected, Ian has Indeterminate Colitis

Of the following ways that infections can contribute to autoimmunity, which one is incorrect? A) An infection of a tissue may induce a local innate immune response B) Some infectious microbes may produce peptide antigens that are similar to, and cross-react with, self-antigens C) The innate response to infections may alter the chemical structure of self-antigens D) Infections may injure tissues and release antigens that are normally sequestered from the immune system E) Commensal microbes in the gut, skin, and other sites will not influence the health of the immune system and maintenance of self-tolerance

E) Commensal microbes in the gut, skin, and other sites will not influence the health of the immune system and maintenance of self-tolerance

A 52-year-old male with complaints of watery diarrhea came to see the physician three weeks after receiving a hematopoietic stem cell transplantation. Suspecting graft-versus-host disease, the physician orders a biopsy of colon. What is the most likely histological finding in this patient? A) Total gland destruction B) Metastatic glandular tissue C) Flask-shaped lesions D) Dense lymphocytic infiltrate in the lamina propria E) Epithelial apoptosis of the crypt cells

E) Epithelial apoptosis of the crypt cells

A 35-year-old male presents to his gastroenterologist for a follow up on previous testing done due to recurrent symptoms of abdominal pain, and rectal bleeding. With positive results, the gastroenterologist diagnoses him with ulcerative colitis and starts treatment with a biologic agent that is a chimeric monoclonal antibody to TNF-α. Which of the following was the patient prescribed? A) Prednisone B) Azathioprine C) Cyclosporin D) Methotrexate E) Infliximab

E) Infliximab

A 39-year-old female presents to the clinic for her normal check-up. Since she was a teenager, she has been taking an iron supplement, considering she tends to have heavy menstruation and that was causing an iron deficiency. 3 months ago, she had a kidney transplant, and was put on an immunosuppressant to allow for reduced chance of rejection. Since then, she has noticed that she is having some uncomfortable symptoms. Upon examination, her lungs sound rough and crackly. What drug could she have been prescribed that could be causing these symptoms? A) Azathioprine B) Cyclosporine C) Tacrolimus D) Ipilimumab E) Mycophenolate mofetil

E) Mycophenolate mofetil

The ascending colon receives arterial supply from multiple branches. Which of the following does not supply it? A) The colic branch from the ileocolic artery B) The anterior cecal artery from the ileocolic artery C) The right colic artery from the superior mesenteric artery D) The posterior cecal artery from the ileocolic artery E) The left colic artery from the inferior mesenteric artery

E) The left colic artery from the inferior mesenteric artery

David, a 65-year-old male, has come in with intermittent cramping, abdominal discomfort, distention, constipation and the sense of never being able to empty the rectum. He is diagnosed with sigmoid diverticulitis. This differs from most diverticula because of the layers involved. What layers are involved in sigmoid diverticulitis? A) The mucosal layer only B) The submucosal layer only C) All three layers D) The muscularis propria E) The submucosal and mucosal layers

E) The submucosal and mucosal layers

An 80 year old suffering from abdominal pain and presents with bright red blood in the stool. Testing indicates an extremely high level of white blood cells, and the patient is not taking any medication. Which of the following is most likely causing her symptoms A) angiodysplasia B) gastric cancer C) esophageal cancer D) gastric ulcer E) intestinal ischemia

E) intestinal ischemia

A 27 year old male comes to the family physician with complaints of bloody diarrhea and abdominal pain. On further evaluation, the patient shows no signs of fever. The physician orders a culture and the results show E coli 0157:h7. What is the best course of treatment for this patient? A) Metronidazole B) Clindamycin C) Vancomycin D) Doxycycline E)None of the above

E)None of the above

A 40 year old man recently had a kidney transplantation. After approximately 2 weeks, his B cells were noted to produce an antibody response against endothelial cells. He is most likely experiencing what phase of transplant rejection? A. Hyperacute B. Acute Cellular Rejection C. GVHD D. Chronic E. Acute Humoral Rejection

E. Acute Humoral Rejection

Madonna, a 61-year-old female, presents to the walk-in clinic with complaints that align with a possible upper respiratory infection. She asks the physician for an estimation of how long she will be feeling ill for. The physician then starts to explain to her the sequential steps that her T lymphocytes must go through in order to respond to the infection. One of the steps that is explained is how T lymphocytes must be activated by antigen and co-stimulation, resulting in a large pool of antigen specific clones from which effector cells can be generated to combat her infection. What is this step called? A. Differentiation B. Migration C. Adhesion D. Homeostasis E. Clonal Expansion

E. Clonal Expansion

A patient presents to an appointment with his oncologist after being referred by his dermatologist following his diagnosis of melanoma. The doctor orders a series of tests to stage the cancer. After the imaging results came in, the patient is informed that his melanoma has metastasized. The doctor tells the patient that he is going to start him on a drug that is given via an IV infusion every three weeks. What is the mechanism of action for this drug? A. Monoclonal antibody against alpha 4 integrin B. Chimeric human Monoclonal antibody against CD30 C. Recombinant IL-1 receptor antagonist D. Calcineurin inhibitor E. Humanized Monoclonal antibody against CTLA-4

E. Humanized Monoclonal antibody against CTLA-4

A patient presents to your office with complaints of pain in her lower right quadrant, diarrhea, fever, and weight loss. After several appointments she is later diagnosed with Crohn's disease. Which cytokine would the physician most expect to be elevated in this patient? A. IL-13 B. CRP C. IL-5 D. TH2 E. IL-2

E. IL-2

A 12-year-old was playing in the backyard and he got a cut. He did not tell his mother and is now experiencing a fever. What chemical mediator is important in inducing a fever? A. C5a B. C3a C. NO D. Bradykinin E. IL1

E. IL1

A patient's CT shows that they have enlarged lymph nodes in their abdomen. The nodes that are noted are directly draining the hindgut. Which lymph nodes are enlarged? A. Cervical nodes B. Axillary nodes C. Celiac nodes D. Superior mesenteric nodes E. Inferior mesenteric nodes

E. Inferior mesenteric nodes

A 60-year-old female with a history of chronic hypertension presents to her PCP with complaints of "wormy", distended bumps around her umbilicus. The physician explains her symptom to be caput medusae, a complication of portal hypertension. He explains that the portal vein is one of the most important vessels in the abdomen. Which of the following is false regarding the portal vein? A. It is the final common pathway for venous blood from the abdomen. B. It is formed by the union of the splenic vein and the superior mesenteric vein. C. The portal vein drains directly into the liver. D. Upon approaching the liver, it splits into right and left branches which enter liver parenchyma. E. It is formed by the union of the splenic vein and the inferior mesenteric vein.

E. It is formed by the union of the splenic vein and the inferior mesenteric vein.

1. Jim is a first year medical student attending LECOM and is learning about the three modes of solid organ rejection, hyperacute, acute, and chronic rejection. Which of the following best describes how hyperacute rejection is suppressed? A. Immunostimulation B. Immunosuppression C. Currently cannot be suppressed D. B and E E. Matching of donor and recipient blood types

E. Matching of donor and recipient blood types

Which of the following is NOT a mediator for vasodilation? A. Histamine B. Complement components C. Bradykinin D. Prostaglandins E. Oxygen radicals

E. Oxygen radicals

A 30-year-old male patient presents to his primary care provider with complaints of abdominal pain. He describes it as a gnawing pain in his epigastric region with additional pain that radiates to his back. What is the most likely diagnosis of the patient? A. Appendicitis B. Cholecystitis C. Pancreatitis D. Esophagitis E. Peptic Ulcer

E. Peptic Ulcer

A patient with a history of poorly controlled hypertension and hypercholesterolemia presents with rapid weight loss. The patient complains of severe abdominal pain after eating. Which area is the physician most likely to find atherosclerosis that is causing the patient's symptoms? A. Marginal Artery B. Inferior Mesenteric Artery C. Left Gastro-omental artery D. Left Gastric E. SMA

E. SMA

What polymorphism is linked to Ulcerative colitis but not Crohns disease?

ECM1 (extracellular matrix protein 1) which inhibits matrix metalloproteinase 9 inhibition of matrix metalloproteinase 9 reduces the severity of colitis in experimental models

How can obstruction lead to dehydration and electrolyte abnormalities?

Early diagnosis is important because considerable fluid and electrolytes enter the bowel lumen and fail to be reabsorbed,

Eicosanoids

Eicosanoids are metabolites o arachidonic acid, a atty acid component o phospholipids in the inner leaf et o the plasma membrane o many cell types. Inf ammatory mediators such as cytokines and complement stimulate the enzymatic release o arachidonic acid rom the plasma membrane. Multiple biochemical reactions ensue, resulting in the ormation o prostaglandins, leukotrienes, and other eicosanoids. Notably, certain arachidonic acid derivatives are proinf ammatory, whereas others serve to limit inf ammation. This underscores the act that acute inf ammation is a sel -limited process and that the process o pathogen destruction is intimately tied to the process o tissue repair. Chapter 43 is devoted to an in-depth discussion o eicosanoid physiology, pathophysiology, and pharmacology

What is the therapeutic goal of cytotoxic agents?

Elimination of pathogenic cells

What does Fat digestion begin with? What are the steps invovled?

Emulsification wherein larger fat droplets are dispersed in the aqueous medium of the lumen 1- In the proximal small intestine, bile salts from liver and pancreatic enzymes are released into the intestinal lumen; 2- there, they mix with and bind to the surface of these globules, where colipase activity results in the release of fatty acids and a monoglyceride. 3- These are taken up as mixed micelles with bile salts, and these hydrophobic particles cross the unstirred water layer that overlies the epithelial brush border.

What do MHC class II proteins display protein fragments derived from?

Endocytic vesicles In contrast to class I proteins, MHC class II proteins are expressed mostly on antigen-presenting cells (e.g., macrophages and dendritic cells),

What are protein fragments expressed on MHC II generally ?

Endocytic vesicles contain antigenic protein ragments derived from infectious agents after phagocytosis and proteolytic processing of those agents Therefore, the protein fragments expressed on MHC class II proteins generally identify extracellular foreign agents eg: bacteria

If the Radiographic images of the small intestine are unrevealing what tests can be performed?

Endoscopic evaluation of the small bowel may be attempted by capsule endoscopy or with push or balloon enteroscopy For the patient with persistent blood loss, no endoscopically identified source of bleeding in the upper GI tract or colon, and negative findings on radiologic studies, the entire small intestine may be examined at laparotomy with endoscopy in the operative suite. In addition, angiographic evaluation of the whole GI tract may reveal the source of chronic blood loss.

What does the interaction of a phagocytic cell with an opsonized particle initiate?

Engulfment and destruction of the offending agent.

What do effector T cells do after they leave lymphoid organs?

Enter the circulation, and migrate to any site of infection, where they can eradicate the infection Some effector T cells may remain in the lymph node, where they function to eradicate infected cells at that site or provide signals to B cells that promote antibody responses against the microbes.

What is Enterokinase secreted by? What does it split ?

Enterokinase (more properly, enteropeptidase) is secreted by the intestinal brush border; it splits trypsinogen to its active form, trypsin, which in turn converts the other proenzymes to their active forms. The products of luminal brush border peptidase digestion consist of amino acids and oligopeptides, which are transported across the epithelial cell

What is the most common cause of Acute traveler's diarrhea?

Entertoxigenic E Coli (ETEC)

What is the most common histologic finding of graft vs host disease?

Epithelial apoptosis, particularly of Crypt cells Rarely, total gland destruction occurs, although endocrine cells may persist. Intestinal graft-versus-host disease often presents as a watery diarrhea but may become bloody in severe cases.

What is Erythema nodosum characterized by? treatment?

Erythema nodosum occurs in 10% of IBD patients, usually with peripheral arthropathy produces raised, tender nodules, usually over the anterior surface of the tibia. Erythema nodosum responds to treatment for the underlying bowel disease.

What is increased intraluminal pressure in colonic diverticula due to?

Exaggerated peristaltic contractions, with spasmodic sequestration of bowel segments, and may be enhanced by diets low in fiber, which reduce stool bulk, particularly in the sigmoid colon

physical examination

Examination of the abdomen provides valuable clues to the diagnosis, but the examination should start with the general appearance of the patient. A patient who is writhing in bed and unable to find a comfortable position may be suffering from obstruction. In contrast, a patient lying with the lower extremities flexed and avoiding any motion may be suffering from peritonitis because movement makes peritoneal pain worse. Abdominal distention indicates obstruction or ascites. Visual inspection for peristalsis is helpful for the diagnosis of small bowel obstruction, but this sign is present only in the early stages. Focal areas of distention may indicate hernias; notice should also be taken of any scars from prior surgeries. Auscultation should be performed in several areas to evaluate the timbre and pattern of bowel sounds and to search for bruits or hums. Absence of bowel sounds suggests ileus, whereas the presence of hyperactive, high-pitched sounds may indicate obstruction. Multiple bruits alert the examiner to the possibility of significant vascular disease, suggesting ischemia. The abdomen should be palpated gently, starting in an area away from the pain. The examiner searches for areas of localized tenderness and rebound as well as for masses and enlarged organs. Percussion is performed to identify the size of organs or to determine the presence of ascites. Pain on percussion of the abdomen indicates peritoneal reaction, as does severe rebound tenderness. A rectal examination is important for identifying a rectal tumor in the case of colon obstruction or tenderness high in the rectum in acute appendicitis. A pelvic examination should be performed in women to rule out pelvic inflammatory disease.

What is there an excessive response of in Crohn's disease?

Excessive and persistent CD4 positive helper T lymphocyte subtype 1 (TH1) immune response to components of commensal bacterial flora

Rectum and anal canal

Extending from the sigmoid colon is the rectum (Fig. 4.91). The rectosigmoid junction is usually described as being at the level of vertebra SIII or at the end of the sigmoid mesocolon because the rectum is a retroperitoneal structure. The anal canal is the continuation of the large intestine inferior to the rectum The arterial supply to the rectum and anal canal includes: ■ the superior rectal artery from the inferior mesenteric artery, ■ the middle rectal artery from the internal iliac artery, and ■ the inferior rectal artery from the internal pudendal artery (from the internal iliac artery).

Utility of Small bowel biopsy specimens in malabsorption

FINDINGS OFTEN DIAGNOSTIC Whipple's disease Amyloidosis Eosinophilic enteritis Lymphangiectasia Primary intestinal lymphoma Giardiasis Abetalipoproteinemia Agammaglobulinemia Mastocytosis FINDINGS ABNORMAL BUT NOT DIAGNOSTIC Celiac disease Systemic sclerosis Radiation enteritis Bacterial overgrowth syndrome Tropical sprue Crohn's disease

Association of Autoimmune diseases with alleles of the major histocompatability complex (MHC) locus

Family and linkage studies show a greater likelihood of developing certain autoimmune diseases in persons who inherit particular human leukocyte antigen (HLA) alleles than in persons who lack these alleles (odds ratio or relative risk). Selected examples of HLA-disease associations are listed. For instance, in people who have the HLA-B27 allele, the risk of development of ankylosing spondylitis, an autoimmune diseases of the spine, is 90 to 100 times higher than in B27-negative people; other diseases show various degrees of association with other HLA alleles. Breeding studies in animals have also shown that the incidence of some autoimmune diseases correlates strongly with the inheritance of particular MHC alleles (e.g., type 1 diabetes mellitus with a mouse class II allele called I-Ag7).

What does deficiency in bile salts contribute to?

Fat malabsorption may result from cholestatic liver disorders (impaired secretion of bile), bacterial overgrowth (resulting in luminal bile salt deconjugation), or ileal disease or resection with loss of effective enterohepatic circulation of the bile acids

What are cells resynthesized and packaged with ?

Fatty acids are resynthesized into triglycerides, and, together with cholesterol and phospholipids, they are packaged into chylomicrons and very-low-density lipoproteins to be exported via lymphatic channels

What is included as Intrahepatic diseases?

Fatty liver, pericholangitis, and chronic active hepatitis. Pericholangitis, also known as small-duct sclerosing cholangitis, is the most common of these diseases. It usually is asymptomatic

Features that differ between Crohn disease and ulcerative colitis

Feature Crohn Disease Ulcerative Colitis Macroscopic Bowel region Ileum ± colon Colon only Distribution Skip lesions Diffuse Stricture Yes Rare Wall appearance Thick Thin Microscopic Inflammation Transmural Limited to mucosa Pseudopolyps Moderate Marked Ulcers Deep, knife-like Superficial, broad-based Lymphoid reaction Marked Moderate Fibrosis Marked Mild to none Serositis Marked Mild to none Granulomas Yes (~35%) No Fistulae/sinuses Yes No Clinical Perianal fistula Yes (in colonic disease) No Fat/vitamin malabsorption Yes No Malignant potential With colonic involvement Yes Recurrence after surgery Common No Toxic megacolon No Yes

Features of protein antigens that influence the choice between T cell tolerance and activation.

Feature of antigen Tolerogenic self antigens Immunogenic foreign antigens Location of antigens Duration of antigen exposure Presence in generative organs (some self antigens) induces negative selection and other mechanisms of central tolerance Deficiency of costimulators may lead to T cell anergy or apoptosis, development of Treg, or sensitivity to suppression by Treg Long-lived persistence (throughout life); prolonged TCR engagement may induce anergy and apoptosis Presence in blood and peripheral tissues (most microbial antigens) permits concentration in peripheral lymphoid organs Expression of costimulators, typically seen with microbes, promotes lymphocyte survival and activation Short exposure to microbial antigen reflects effective immune response, costimulation

What is ATP therapy often complicated by?

Fever and headache as prominent manifestations of the cytokine release syndrome

What tests can be used to diagnose Celiac disease?

Fiberoptic or capsule endoscopy may show the typical features of broad and flattened villi; with the former instrument, tissue can be sampled for histologic analysis. Serologic blood tests (antigliadin, antiendomysial, and reticulin antibodies) are helpful in screening of patients with atypical symptoms and asymptomatic relatives of patients with celiac disease.

What can chronic inflammation cause? What does in turn cause?

Fibrosis and stricture formation which in turn may result in partial or complete intestinal obstruction, with the patient complaining of abdominal pain, distention, nausea, and vomiting Strictures can also lead to stasis with subsequent small intestinal bacterial overgrowth

Obstruction

Fig. 4.95 This radiograph of the abdomen, anteroposterior view, demonstrates a number of dilated loops of small bowel. Small bowel can be identified by the plicae circulares that pass from wall to wall as indicated. The large bowel is not dilated. The cause of the small bowel dilatation is an adhesion after pelvic surgery. Dilation Fig. 4.96 Coronal CT demonstrates dilated and fluid-filled loops of small bowel in patient with small bowel obstruction

Where can dysplasia develop?

Flat areas of mucosa that are not grossly recognized as abnormal. Thus, advanced endoscopic imaging techniques including chromoendoscopy and confocal endoscopy are beginning to be used to increase the sensitivity of detection

What does the large intestine absorb and subsequently form?

Fluids and salts from the gut contents forms feces

What is Methotrexate?

Folate analogue drug used in the treatment of a wide variety of immune related diseases eg: Rheumatoid arthritis and psoriasis Methotrexate is also used for the prevention of Graft versus host disease

What do patients taking Sulfasalazine need to take in addition to it?

Folic acid supplementation

biochemical pathways of t cell activation

Following the recognition of antigens and costimulators, T cells express proteins that are involved in their proliferation, differentiation, and effector functions (Fig. 5-9). Naive T cells that have not encountered antigen have a low level of protein synthesis. Within minutes of antigen recognition, new gene transcription and protein synthesis are seen in the activated T cells. These newly expressed proteins mediate many of the subsequent responses of the T cells. Antigen recognition activates several biochemical mechanisms that lead to T cell responses, including the activation of enzymes such as kinases, recruitment of adaptor proteins, and production of active transcription factors (Fig. 5-10). These biochemical pathways are initiated when TCR complexes and the appropriate coreceptor are brought together by binding to MHC-peptide complexes on the surface of APCs. In addition, there is an orderly movement of proteins in both the APC and T cell membranes in the region of cell-to-cell contact, such that the TCR complex, CD4/CD8 coreceptors, and CD28 coalesce to the center and the integrins move to form a peripheral ring. This redistribution of signaling and adhesion molecules is required for optimal induction of activating signals in the T cell. The region of contact between the APC and T cell, including the redistributed membrane proteins, is called the immune synapse. Although the synapse was first described as the site of delivery of activating signals from membrane receptors to the cell's interior, it may serve other functions Some effector molecules and cytokines may be secreted through this region, ensuring that they do not diffuse away but are targeted to the APC. Enzymes that serve to degrade or inhibit signaling molecules are also recruited to the synapse, so it may be involved in terminating lymphocyte activation The CD4 and CD8 coreceptors facilitate signaling through a protein tyrosine kinase called Lck that is noncovalently attached to the cytoplasmic tails of these coreceptors. As discussed in Chapter 4, several transmembrane signaling proteins are associated with the TCR, including the CD3 and ζ chains. CD3 and ζ contain motifs, each with two tyrosine residues, called immunoreceptor tyrosine-based activation motifs (ITAMs), which are critical for signaling. Lck, which is brought near the TCR complex by the CD4 or CD8 molecules, phosphorylates tyrosine residues contained within the ITAMs of the CD3 and ζ proteins. The phosphorylated ITAMs of the ζ chain become docking sites for a tyrosine kinase called ZAP-70 (zeta-associated protein of 70 kD), which also is phosphorylated by Lck and thereby made enzymatically active. The active ZAP-70 then phosphorylates various adaptor proteins and enzymes, which assemble near the TCR complex and mediate additional signaling events.

What does the measurement of pancreatic enzymes (eg: fecal elastase 1) in the stool provide evidence for?

For the diagnosis of moderate to severe pancreatitic insufficiency

What is Abdominal pain?

Frequent manifestation of intra-abdominal disease abdominal pain is difficult to localize or grade because the sensation of pain often is colored by emotional and physical factors.

What is the most common adverse effect of Mycophenolate mofetil (MMF)?

Gastrointestinal discomfort is dose dependent and can include nausea, diarrhea, soft stools, anorexia and vomiting

Gastrostomy

Gastrostomy is performed when the stomach is attached to the anterior abdominal wall and a tube is placed through the skin into the stomach. Typically this is performed to feed the patient when it is impossible to take food and fluid orally (e.g., complex head and neck cancer). The procedure can be performed either surgically or through a direct needlestick puncture under sedation in the anterior abdominal wall, so patient gets proper nutrients

What can be done to prevent later complications associated with glucocorticoid useage?

Glucocorticoid dosage should be tapered slowly as therapy is terminated.

What are Glucocorticoids?

Glucocorticoids are steroid hormones that exert their physiologic actions by binding to the cytosolic glucocorticoid receptor The Glucocorticoid- glucocorticoid receptor complex translocates to the nucleus and binds to glucocorticoid response elements (GREs) in the promoter region of specific genes, either up-regulating or down-regulating gene expression.

inhibitors of gene expression- glucocorticoids

Glucocorticoids have broad anti-in ammatory e ects. The intimate relationship between cortisol and the immune system is discussed in Chapter 29, Pharmacology o the Adrenal Cortex. Brie y, glucocorticoids are steroid hormones that exert their physiologic actions by binding to the cytosolic glucocorticoid receptor. The glucocorticoid- glucocorticoid receptor complex translocates to the nucleus and binds to glucocorticoid response elements (GREs) in the promoter region o specif c genes, either up-regulating or down-regulating gene expression Glucocorticoids have important metabolic e ects on essentially all cells o the body and, in pharmacologic doses, suppress the activation and unction o innate and adaptive immune cells. Glucocorticoids down-regulate the expression o many in ammatory mediators, including key cytokines such as tumor necrosis actor (TNF)- , interleukin-1 (IL-1), and IL-4. The role o glucocorticoids in suppressing eicosanoid biosynthesis and signaling is discussed in Chapter 43, Pharmacology o Eicosanoids. The overall e ect o glucocorticoid administration is pro oundly anti-in ammatory and immunosuppressive, explaining the use o glucocorticoids or the treatment o numerous in ammatory diseases such as rheumatoid arthritis and transplant rejection. Long-term glucocorticoid administration has important adverse e ects. Diabetes, reduced resistance to in ection, osteoporosis, cataracts, increased appetite leading to weight gain, hypertension and its sequelae, and the masking o in ammation must all be closely monitored in patients receiving glucocorticoids. Abrupt cessation o glucocorticoid therapy can result in acute adrenal insu f ciency because the hypothalamus and pituitary gland require weeks to months to reestablish adequate ACTH production. During this time, the underlying disease can worsen because o disinhibition o the immune system. To prevent the latter complications, glucocorticoid dosage should be tapered slowly as therapy is terminated. This is why Mr. W's dose o glucocorticoids was tapered slowly a ter his pulsed steroid therapy or treatment o acute cellular rejection

What can result in the case of a bone marrow or stem cell transplant?

Graft versus host disease (GVHD) can result when donor lymphocytes mount an assault on recipient tissues

Graft versus host disease

Graft-versus-host disease occurs following hematopoietic stem cell transplantation. The small bowel and colon are involved in most cases. Although graft-versus-host disease is secondary to donor T cells targeting antigens on the recipient's GI epithelial cells, the lamina propria lymphocytic infiltrate is typically sparse. Epithelial apoptosis, particularly of crypt cells, is the most common histologic finding. Rarely, total gland destruction occurs, although endocrine cells may persist. Intestinal graft-versus-host disease often presents as a watery diarrhea but may become bloody in severe cases

What are found in 25-30% of histologic specimens of Crohn's disease but not in Ulcerative colitis?

Granulomas Granulomas are not diagnostic because they can be found in many other diseases, such as Beçhet's disease, tuberculosis, Yersinia infection, and lymphoma.

What is Toxic megacolon characterized by?

Gross dilation of the large bowel associated with fever, abdominal pain, dehydration, tachycardia, and bloody diarrhea.

Ulcerative colitis morhpology

Grossly, ulcerative colitis always involves the rectum and extends proximally in a continuous fashion to involve part or all of the colon. Disease of the entire colon is termed pancolitis (Fig. 17-36A), while left-sided disease extends no farther than the transverse colon. Limited distal disease may be referred to descriptively as ulcerative proctitis or ulcerative proctosigmoiditis. The small intestine is normal, although mild mucosal inflammation of the distal ileum, termed backwash ileitis, may be present in severe cases of pancolitis. Skip lesions are not seen (although focal appen diceal or cecal inflammation may occasionally be present in left-sided ulcerative colitis Grossly, involved colonic mucosa may be slightly red and granular or have extensive, broad-based ulcers. There can be an abrupt transition between diseased and uninvolved colon (Fig. 17-36B). Ulcers are aligned along the long axis of the colon but do not typically replicate the serpentine ulcers of Crohn disease. Isolated islands of regenerating mucosa often bulge into the lumen to create pseudopolyps (Fig. 17-36C), and the tips of these polyps may fuse to create mucosal bridges (Fig. 17-36D). Chronic disease may lead to mucosal atrophy with a flat and smooth mucosal surface that lacks normal folds. Unlike Crohn disease, mural thickening is not present, the serosal surface is normal, and strictures do not occur. However, inflammation and inflammatory mediators can damage the muscularis propria and disturb neuromuscular function leading to colonic dilation and toxic megacolon, which carries a significant risk of perforation Histologic features of mucosal disease in ulcerative colitis are similar to colonic Crohn disease and include inflammatory infiltrates, crypt abscesses (Fig. 17-37A), crypt distortion, and pseudopyloric epithelial metaplasia (Fig. 17-37B). However, the inflammatory process is diffuse and generally limited to the mucosa and superficial submucosa (Fig. 17-37C). In severe cases, extensive mucosal destruction may be accompanied by ulcers that extend more deeply into the submucosa, but the muscularis propria is rarely involved. Submucosal fibrosis, mucosal atrophy, and distorted mucosal architecture remain as residua of healed disease but histology may also revert to near normal after prolonged remission. Granulomas are not present in ulcerative colitis

What is the cascade of tissue repair mediated by the release of?

Growth factors and cytokines eg: Epidermal growth actor (EGF), platelet-derived growth factor (PDGF), basic fibroblast growth actor-2 (bFGF-2), transforming growth factor-beta1 (TGF-beta1), IL-1 and TNF-alpha

Where are HEVs located? What are they lined by?

HEVs are located in the T cell zones of lymphoid tissues are lined by specialized endothelial cells, which express carbohydrate ligands that bind to L-selectin

Modes of Immune rejection

HYPERACUTE REJECTION ACUTE REJECTION CHRONIC REJECTION Mechanism Preformed recipient antibodies react with donor antigen and activate complement Cellular—Donor antigen activates recipient T cells Humoral—Donor antigen stimulates recipient antibody response Unknown, but thought to be caused by chronic inf ammation resulting from activated T-cell responses to donor antigen Time course Minutes to hours Weeks to months Months to years How suppressed Matching of donor and recipient blood types Immunosuppression Currently cannot be suppressed

What are the uses of Anakinra?

Has modest effects on pain and swelling Significantly reduces bony erosions, possibly because it decreases osteoclast production and blocks IL-1-induced metalloproteinase release from synovial cells

What are poor sanitation, food contamination and crowded living conditions associated with? What can they lead to?

Helminthic infections leads to regulatory T-cell conditioning and stimulation of IL-10 and transforming growth factor-β (TGF-beta) production by mononuclear cells, thereby preventing intestinal inflammation.

Graft vs host disease occurs following what? What 2 regions are mostly involved?

Hematopoietic stem cell transplantation Small bowel and Colon Although graft-versus-host disease is secondary to donor T cells targeting antigens on the recipient's GI epithelial cells, the lamina propria lymphocytic infiltrate is typically sparse

Where does the Right gastric artery originate from?

Hepatic artery proper but it can also arise from the common hepatic artery or from the left hepatic, gastroduodenal, or supraduodenal arteries

What is an urgent condition for liver transplantation?

Hepatic encephalopathy if the condition is not reversed, it leads to irreversible neurological damage and death.

What may focal areas of distention indicate?

Hernia

What does any injury, from physical trauma to microbial invasion stimulates the mast cells to release?

Histamine

histamine

Histamine , one o the initiators o the inf ammatory response, is constitutively synthesized and stored in the granules o mast cells and basophils. These cells migrate through tissue on a continual basis. Any injury, rom physical trauma to microbial invasion, stimulates mast cells to release histamine into the interstitium. Histamine is re erred to as a vasoactive amine because its inf ammatory e ects occur mainly on the vasculature: histamine release stimulates dilation o arterioles and postcapillary venules, constriction o veins, and contraction o endothelial cells. These e ects are responsible or the early changes in hemodynamics and vascular permeability discussed below. Several classes o pharmacologic agents modi y histamine signaling; these agents are discussed in Chapter 44.

What gene has a strong association with Celiac disease?

Human leukocyte antigen (HLA) class II molecules particularly HLA-DQ2 and HLA-DQ8

What is the most important treatment for acute diarrhea?

Hydration Oral hydration solutions are effective because in many small bowel diarrheal illnesses, the intestine remains able to absorb water if glucose and salt are present to allow transport of water from the lumen

hyperacute rejection

Hyperacute rejection is mediated by pre ormed recipient antibodies against donor antigen. Because these antibodies are present at the time o organ implantation, hyperacute rejection occurs almost immediately a ter reper usion o the transplanted organ. In act, the surgeon can observe the changes in the organ minutes a ter restoration o blood ow. The normal, healthy, pink appearance o the transplanted organ rapidly becomes cyanotic, mottled, and f accid. This rapid change is the result o complement activation by antibody binding to endothelial cells o the transplanted organ, resulting in thrombosis and ischemia. Most commonly, hyperacute rejection is mediated by recipient antibodies that react with blood group antigens in donor organs (e.g., type AB donor in a type O recipient). Matching o blood types between donor and recipient prevents hyperacute rejection; there ore, drug therapy or hyperacute rejection is typically not necessary. Hyperacute rejection also occurs in xenotransplantation (i.e., organ transplantation between species, such as a pig heart transplanted into a human recipient), due to the presence o pre ormed human antibodies that react against antigenic proteins and carbohydrates expressed by the donor species. Mr

What does the Superior hypogastric plexus divide into inferiorly?

Hypogastric nerves which descend into the pelvis and contribute to the formation of the inferior hypogastric or pelvic plexus

evaluation of chronic diarrhea part 2

IBS can be diagnosed based on a typical history and clinical findings and does not usually require an extensive evaluation. If there are potentially serious symptoms that are not consistent with IBS, additional testing is undertaken. Functional diarrhea is defined by the Rome III criteria as continuous or recurrent passage of loose or watery stools without abdominal pain or discomfort, occurring in at least 75% of stools for at least 3 months. This criterion is somewhat ambiguous in guiding the evaluation because it does not help to exclude other causes of chronic diarrhea. Inflammatory bowel diseases, such as ulcerative colitis and Crohn's disease, are classic causes of chronic diarrhea. Radiation enterocolitis (usually based on the history) and ischemic colitis are more unusual causes. Crohn's disease can involve any portion of the GI tract, from the mouth to the anus. Patients with Crohn's disease typically have abdominal pain, diarrhea, weight loss, and fever; gross bleeding is not common with Crohn's disease. There is frequently a delay in presentation and diagnosis of several months from the onset of symptoms to the diagnosis of Crohn's disease. Anemia, leukocytosis, and elevated inflammatory markers are common laboratory findings. The presentation of ulcerative colitis varies a great deal because of variable involvement of the colon. Typically, if the inflammation is limited to the rectum or rectosigmoid region, the symptoms are relatively mild. The onset may be gradual, and there may be episodes of bloody mucus and intermittent diarrhea with fewer than four stools per day. Urgency, mild cramping, and a sensation of incomplete evacuation (tenesmus) are commonly reported, and constipation may also be a complaint. More serious symptoms are often associated with greater involvement of the colon (left-sided colitis or sometimes pancolitis). Patients have frequent loose, bloody stools (up to 10 per day) with mucus. Mild anemia and leukocytosis, mild-to-moderate cramping abdominal pain, and low-grade fever may be present. Weight loss is not common in mild disease. Severe disease with more than 10 bloody stools per day usually indicates pancolitis. There is severe cramping pain, fever, leukocytosis, and anemia, which often requires transfusion. There can be rapid weight loss leading to malnutrition. Overall, approximately one third of these patients have rectosigmoid disease, one third have left-sided disease, and the remainder have pancolitis. Fulminant disease is present in about 10% of patients at presentation. For information regarding the treatment of IBD, see Chapter 37. Microscopic colitis is most commonly seen in middle-aged women but can be found at all ages and in men as well. It usually manifests with chronic watery diarrhea. There can be mild cramping and weight loss, but dehydration and malnutrition are rare. The name implies that this is a histologic diagnosis, and indeed, the mucosal appearance of the colon at the time of endoscopy is frequently entirely normal, and biopsies are necessary to document its presence. Disease is present throughout the colon but is often more severe on the right side. There are two types of microscopic colitis: lymphocytic and collagenous. Although the underlying cause of the colitis is not known, it is associated with autoimmune disease in up to 50% of cases. Treatment starts with trials of symptomatic pharmacotherapy. Loperamide and cholestyramine may be useful in mild disease; budesonide, a poorly absorbed steroid, may be used, although relapses are frequent after weaning. Mesalamine and sulfasalazine have been used, but there is little information indicating effectiveness Malabsorption can result in diarrhea, most commonly from malabsorption of fat and nutrients. The bowel movements are classically described as greasy or oily, foul smelling, and large volume, although not usually watery. There may be associated weight loss. Malabsorption can be congenital, caused by membrane transport defects of small bowel enterocytes, or acquired due to extensive damage or resection of the small bowel resulting in decreased absorptive area. Celiac disease, Crohn's disease, and short bowel syndrome after resection can be causes. The jejunalileal bypass procedure, which was performed for patients with morbid obesity in the 1960s and 1970s but is no longer done due to extensive and serious complications, created severe malaborption resulting in weight loss. Maldigestion resulting from lack of pancreatic enzymes (chronic pancreatitis or, occasionally, pancreatic tumor) and lack of bile salts for fat absorption also can occur. Laboratory findings depend on the severity of malabsorption and which specific nutrient is deficient. The most common malabsorptive process is lactose intolerance that results in gas, bloating, and diarrhea. Lactose intolerance can be diagnosed by lactose breath testing or the more simple lactose avoidance trial. Celiac disease and tropical sprue can result in a wide spectrum and severity of symptoms, from iron deficiency to calcium and magnesium deficiency, fat-soluble vitamin deficiencies, and weight loss. The rare Whipple's disease can cause not only malabsorption but also systemic findings. Bacterial overgrowth, such as in some gut motility disorders and small bowel diverticula as well as surgically created blind loops, can also result in malabsorption. Breath testing is available for the evaluation of bacterial overgrowth. Other cases of chronic, but usually self-limited, diarrhea occur in about 10% of patients who have undergone gallbladder removal. The likely cause is the continuous presentation of bile to the intestine that results from the loss of storage capacity in the gallbladder; this continuous flow increases the level of bile salts in the colon, leading to diarrhea. Postcholecystectomy diarrhea usually responds to bile acid-binding agents such as cholestyramine and often resolves over time. Secretory diarrhea is uncommon and typically manifests with large-volume (>1 L/day), watery diarrhea that occurs both day and night and continues in spite of fasting. Although this diagnosis can usually be established by the history and a trial of stool monitoring while fasting, calculation of the stool osmolar gap from measured stool electrolytes can be helpful (see previous discussion). A secretory diarrhea will have an osmolar gap of less than 50 mOsm/kg, whereas the gap in an osmotic diarrhea will be greater than 125 mOsm/kg. Causes of secretory diarrhea are rarely infectious. They include fairly uncommon but often quite dramatic syndromes such as Zollinger-Ellison syndrome (gastrinoma), vasoactive intestinal peptide-producing tumor (VIPoma), and carcinoid syndrome. For the evaluation of chronic secretory diarrhea, stool cultures should be done. Imaging of the small bowel and colon should be considered, and appropriate testing for hormones and other secretagogues should be based on the history and findings.

What does the diagnosis of Irritable bowel syndrome require?

IBS is presently diagnosed using clinical criteria that require the occurrence of abdominal pain or discomfort at least 3 days per month over 3 months with improvement following defecation and a change in stool frequency or form Other causes, such as enteric infection or inflammatory bowel disease, must be excluded.

What are cytokines secreted primarily by the Hematopoietic lineage? What are they involved in?

IL-1 and TNF-alpha Acute inflammatory response

What immunoregulatory molecules play a role in in IBD pathogenesis?

IL-10 and TGF-β The role of IL-10 is supported by the observations that autosomal recessive mutations of the IL-10 and IL-10 receptor genes are linked to severe, early onset IBD.

Expression of what cytokine has been shown to be upregulated in active IBD- in both Crohn's disease and Ulcerative colitis?

IL-17

What is the survival and function of regulatory T cells (T-regs) dependent on? What role does it play?

IL-2 IL-2 is an example of a cytokine that serves two opposite roles: 1- It promotes immune responses by stimulating T cell proliferation 2- It inhibits immune responses by maintaining functional regulatory T cells

What cytokine is rapidly secreted by CD4 T cells?

IL-2 IL-2 is produced within 1 to 2 hours after activation of CD4+ T cells Cytokines are a large group of proteins that function as mediators of immunity and inflammation.

What does Cyclosporine (CsA) inhibit the production of?

IL-2 by activated T cells IL-2 is an important cytokine that acts in an autocrine and paracrine matter to cause activation and proliferation of T cells


Related study sets

Método A1. Unidad 5. Alimentos. Embutidos

View Set

Promulgated Contract - End of Section Questions

View Set

Psych 135 Social Psychology- Parkinson UCLA

View Set

Research Problems & Literature Reviews

View Set